home *** CD-ROM | disk | FTP | other *** search
/ Internet E-Mail Workshop / Internet E-Mail Workshop.iso / info / puzzfaq. < prev    next >
Text File  |  1993-11-24  |  897KB  |  23,215 lines

  1.  
  2. Xref: bloom-picayune.mit.edu rec.puzzles:18136 news.answers:3068
  3. Newsgroups: rec.puzzles,news.answers
  4. Path: bloom-picayune.mit.edu]snorkelwacker.mit.edu]usc]wupost]uunet]questrel]chris
  5. From: uunet]questrel]chris (Chris Cole)
  6. Subject: rec.puzzles FAQ, part 1 of 15
  7. Message-ID: <puzzles-faq-1_717034101@questrel.com>
  8. Followup-To: rec.puzzles
  9. Summary: This posting contains a list of
  10.      Frequently Asked Questions (and their answers).
  11.      It should be read by anyone who wishes to
  12.      post to the rec.puzzles newsgroup.
  13. Sender: chris@questrel.com (Chris Cole)
  14. Reply-To: uunet]questrel]faql-comment
  15. Organization: Questrel, Inc.
  16. Date: Mon, 21 Sep 1992 00:08:26 GMT
  17. Approved: news-answers-request@MIT.Edu
  18. Expires: Sat, 3 Apr 1993 00:08:21 GMT
  19. Lines: 1557
  20.  
  21. Archive-name: puzzles-faq/part01
  22. Last-modified: 1992/09/20
  23. Version: 3
  24.  
  25. Instructions for Accessing rec.puzzles Frequently Asked Questions List
  26.  
  27. INTRODUCTION
  28.  
  29. Below is a list of puzzles, categorized by subject area.  Each puzzle
  30. includes a solution, compiled from various sources, which is supposed
  31. to be definitive.
  32.  
  33. EMAIL
  34.  
  35. To request a puzzle, send a letter to uunet]questrel]faql-request
  36. containing one or more lines of the form:
  37.  
  38. send <puzzle_name>
  39.  
  40. For example, to request decision/allais.p, send the line:
  41.  
  42. send decision/allais.p
  43.  
  44. or just:
  45.  
  46. send allais
  47.  
  48. The puzzle will be mailed via return email to the address in your
  49. request's "From:" line.  If you are unsure of this address, and cannot
  50. edit this line, then include in your message BEFORE the first "send" line
  51. the line:
  52.  
  53. return_address <your_return_email_address>
  54.  
  55. FTP
  56.  
  57. The FAQL has been posted to news.answers.  News.answers is archived in
  58. the periodic posting archive on pit-manager.mit.edu ▌18.172.1.27¿.
  59. Postings are located in the anonymous ftp directory
  60. /pub/usenet/news.answers, and are archived by "Archive-name".  Other
  61. subdirectories of /pub/usenet contain periodic postings that may not
  62. appear in news.answers.
  63.  
  64. Other news.answers/FAQ archives (which carry some or all of the FAQs
  65. in the pit-manager archive) are:
  66.  
  67.         archive.cs.ruu.nl ▌131.211.80.5¿ in the anonymous ftp
  68.                 directory /pub/NEWS.ANSWERS (also accessible via mail
  69.                 server requests to mail-server@cs.ruu.nl)
  70.         cnam.cnam.fr ▌192.33.159.6¿ in the anonymous ftp directory /pub/FAQ
  71.         ftp.uu.net ▌137.39.1.9 or 192.48.96.9¿ in the anonymous ftp
  72.                 directory /usenet
  73.         ftp.win.tue.nl ▌131.155.70.100¿ in the anonymous ftp directory
  74.                 /pub/usenet/news.answers
  75.         grasp1.univ-lyon1.fr ▌134.214.100.25¿ in the anonymous ftp
  76.                 directory /pub/faq (also accessible via mail server
  77.                 requests to listserv@grasp1.univ-lyon1.fr), which is
  78.                 best used by EASInet sites and sites in France that do
  79.                 not have better connectivity to cnam.cnam.fr (e.g.
  80.                 Lyon, Grenoble)
  81.  
  82. Note that the periodic posting archives on pit-manager.mit.edu are
  83. also accessible via Prospero and WAIS (the database name is "usenet"
  84. on port 210).
  85.  
  86. CREDIT
  87.  
  88. The FAQL is NOT the original work of the editor (just in case you were
  89. wondering :^).
  90.  
  91. In keeping with the general net practice on FAQL's, I do not as a rule assign
  92. credit for FAQL solutions.  There are many reasons for this:
  93. 1.  The FAQL is about the answers to the questions, not about assigning credit.
  94. 2.  Many people, in providing free answers to the net, do not have the time
  95.     to cite their sources.
  96. 3.  I cut and paste freely from several people's solutions in most
  97.     cases to come up with as complete an answer as possible.
  98. 4.  I use sources other than postings.
  99. 5.  I am neither qualified nor motivated to assign credit.
  100.  
  101. However, I do whenever possible put bibliographies in FAQL entries, and
  102. I see the inclusion of the net addresses of interested parties as a
  103. logical extension of this practice.  In particular, if you wrote a
  104. program to solve a problem and posted the source code of the program,
  105. you are presumed to be interested in corresponding with others about
  106. the problem.  So, please let me know the entries you would like to be
  107. listed in and I will be happy to oblige.
  108.  
  109. Address corrections or comments to uunet]questrel]faql-comment.
  110.  
  111. INDEX
  112.  
  113. ==> analysis/bugs.p <==
  114. Four bugs are placed at the corners of a square. Each bug walks directly
  115. toward the next bug in the clockwise direction. The bugs walk with
  116. constant speed always directly toward their clockwise neighbor. Assuming
  117. the bugs make at least one full circuit around the center of the square
  118.  
  119. ==> analysis/c.infinity.p <==
  120. What function is zero at zero, strictly positive elsewhere, infinitely
  121. differentiable at zero and has all zero derivitives at zero?
  122.  
  123. ==> analysis/cache.p <==
  124. Cache and Ferry (How far can a truck go in a desert?)
  125. A pick-up truck is in the desert beside N 50-gallon gas drums, all full.
  126. The truck's gas tank holds 10 gallons and is empty.  The truck can carry
  127. one drum, whether full or empty, in its bed.  It gets 10 miles to the gallon.
  128.  
  129. ==> analysis/cats.and.rats.p <==
  130. If 6 cats can kill 6 rats in 6 minutes, how many cats does it take to
  131. kill one rat in one minute?
  132.  
  133. ==> analysis/e.and.pi.p <==
  134. Which is greater, e^(pi) or (pi)^e ?
  135.  
  136. ==> analysis/functional/distributed.p <==
  137.      Find all f: R -> R, f not identically zero, such that
  138. (*)     f( (x+y)/(x-y) ) = ( f(x)+f(y) )/( f(x)-f(y) ).
  139.  
  140. ==> analysis/functional/linear.p <==
  141. Suppose f is non-decreasing with
  142.   f(x+y) = f(x) + f(y) + C   for all real x, y.
  143. Prove: there is a constant A such that f(x) = Ax - C  for all x.
  144. (Note: continuity of f is not assumed in advance.)
  145.  
  146. ==> analysis/integral.p <==
  147. If f is integrable on (0,inf), and differentiable at 0, and a > 0, show:
  148.  
  149.  
  150.                   inf     ( f(x) - f(ax) )
  151.  
  152. ==> analysis/period.p <==
  153. What is the least possible integral period of the sum of functions
  154. of periods 3 and 6?
  155.  
  156. ==> analysis/rubberband.p <==
  157. A bug walks down a rubberband which is attached to a wall at one end and a car
  158. moving away from the wall at the other end. The car is moving at 1 m/sec while
  159. the bug is only moving at 1 cm/sec. Assuming the rubberband is uniformly and
  160. infinitely elastic, will the bug ever reach the car?
  161.  
  162. ==> analysis/series.p <==
  163. Show that in the series: x, 2x, 3x, .... (n-1)x (x can be any real number)
  164. there is at least one number which is within 1/n of an integer.
  165.  
  166. ==> analysis/snow.p <==
  167. Snow starts falling before noon on a cold December day.
  168. At noon a snowplow starts plowing a street.
  169. It travels 1 mile in the first hour, and 1/2 mile in the second hour.
  170. What time did the snow start falling??
  171.  
  172. ==> analysis/tower.p <==
  173. A number is raised to its own power. The same number is then raised to
  174. the power of this result. The same number is then raised to the power
  175. of this second result. This process is continued forever. What is the
  176. maximum number which will yield a finite result from this process?
  177.  
  178. ==> arithmetic/7-11.p <==
  179. A customer at a 7-11 store selected four items to buy, and was told
  180. that the cost was $7.11.  He was curious that the cost was the same
  181. as the store name, so he inquired as to how the figure was derived.
  182. The clerk said that he had simply multiplied the prices of the four
  183.  
  184. ==> arithmetic/clock/day.of.week.p <==
  185. It's restful sitting in Tom's cosy den, talking quietly and sipping
  186. a glass of his Madeira.
  187.  
  188. I was there one Sunday and we had the usual business of his clock.
  189.  
  190. ==> arithmetic/clock/thirds.p <==
  191. Do the 3 hands on a clock ever divide the face of the clock into 3
  192. equal segments, i.e. 120 degrees between each hand?
  193.  
  194. ==> arithmetic/consecutive.product.p <==
  195. Prove that the product of three or more consecutive natural numbers cannot be a
  196. perfect square.
  197.  
  198. ==> arithmetic/consecutive.sums.p <==
  199. Find all series of consecutive positive integers whose sum is exactly 10,000.
  200.  
  201. ==> arithmetic/digits/all.ones.p <==
  202. Prove that some multiple of any integer ending in 3 contains all 1s.
  203.  
  204. ==> arithmetic/digits/arabian.p <==
  205. What is the Arabian Nights factorial, the number x such that x] has 1001
  206. digits?  How about the prime x such that x] has exactly 1001 zeroes on
  207. the tail end.  (Bonus question, what is the 'rightmost' non-zero digit in x]?)
  208.  
  209. ==> arithmetic/digits/circular.p <==
  210. What 6 digit number, with 6 different digits, when multiplied by all integers
  211. up to 6, circulates its digits through all 6 possible positions, as follows:
  212. ABCDEF * 1 = ABCDEF
  213. ABCDEF * 3 = BCDEFA
  214.  
  215. ==> arithmetic/digits/divisible.p <==
  216. Find the least number using 0-9 exactly once that is evenly divisible by each
  217. of these digits?
  218.  
  219. ==> arithmetic/digits/equations/123456789.p <==
  220. In how many ways can "." be replaced with "+", "-", or "" (concatenate) in
  221. .1.2.3.4.5.6.7.8.9=1 to form a correct equation?
  222.  
  223. ==> arithmetic/digits/equations/1992.p <==
  224. 1 = -1+9-9+2.  Extend this list to 2 - 100 on the left side of the equals sign.
  225.  
  226. ==> arithmetic/digits/equations/383.p <==
  227. Make 383 out of 1,2,25,50,75,100 using +,-,*,/.
  228.  
  229. ==> arithmetic/digits/extreme.products.p <==
  230. What are the extremal products of three three-digit numbers using digits 1-9?
  231.  
  232. ==> arithmetic/digits/googol.p <==
  233. What digits does googol] start with?
  234.  
  235. ==> arithmetic/digits/labels.p <==
  236. You have an arbitrary number of model kits (which you assemble for
  237. fun and profit).  Each kit comes with twenty (20) stickers, two of which
  238. are labeled "0", two are labeled "1", ..., two are labeled "9".
  239. You decide to stick a serial number on each model you assemble starting
  240.  
  241. ==> arithmetic/digits/nine.digits.p <==
  242. Form a number using 0-9 once with its first n digits divisible by n.
  243.  
  244. ==> arithmetic/digits/palindrome.p <==
  245. Does the series formed by adding a number to its reversal always end in
  246. a palindrome?
  247.  
  248. ==> arithmetic/digits/palintiples.p <==
  249. Find all numbers that are multiples of their reversals.
  250.  
  251. ==> arithmetic/digits/power.two.p <==
  252. Prove that for any 9-digit number (base 10) there is an integral power
  253. of 2 whose first 9 digits are that number.
  254.  
  255. ==> arithmetic/digits/prime/101.p <==
  256. How many primes are in the sequence 101, 10101, 1010101, ...?
  257.  
  258. ==> arithmetic/digits/prime/all.prefix.p <==
  259. What is the longest prime whose every proper prefix is a prime?
  260.  
  261. ==> arithmetic/digits/prime/change.one.p <==
  262. What is the smallest number that cannot be made prime by changing a single
  263. digit?  Are there infinitely many such numbers?
  264.  
  265. ==> arithmetic/digits/prime/prefix.one.p <==
  266. 2 is prime, but 12, 22, ..., 92 are not.  Similarly, 5 is prime
  267. whereas 15, 25, ..., 95 are not.  What is the next prime number
  268. which is composite when any digit is prefixed?
  269.  
  270. ==> arithmetic/digits/reverse.p <==
  271. Is there an integer that has its digits reversed after dividing it by 2?
  272.  
  273. ==> arithmetic/digits/rotate.p <==
  274. Find integers where multiplying them by single digits rotates their digits.
  275.  
  276. ==> arithmetic/digits/sesqui.p <==
  277. Find the least number where moving the first digit to the end multiplies by 1.5.
  278.  
  279. ==> arithmetic/digits/squares/leading.7.to.8.p <==
  280. What is the smallest square with leading digit 7 which remains a square
  281. when leading 7 is replaced by an 8?
  282.  
  283. ==> arithmetic/digits/squares/length.22.p <==
  284. Is it possible to form two numbers A and B from 22 digits such that
  285. A = B^2?  Of course, leading digits must be non-zero.
  286.  
  287. ==> arithmetic/digits/squares/length.9.p <==
  288. Is it possible to make a number and its square, using the digits from 1 through
  289. 9 exactly once?
  290.  
  291. ==> arithmetic/digits/squares/three.digits.p <==
  292. What squares consist entirely of three digits (e.g., 1, 4, and 9)?
  293.  
  294. ==> arithmetic/digits/squares/twin.p <==
  295. Let a twin be a number formed by writing the same number twice,
  296. for instance, 81708170 or 132132.  What is the smallest square twin?
  297.  
  298. ==> arithmetic/digits/sum.of.digits.p <==
  299. Find sod ( sod ( sod (4444 ^ 4444 ) ) ).
  300.  
  301. ==> arithmetic/digits/zeros/factorial.p <==
  302. How many zeros are in the decimal expansion of n]?
  303.  
  304. ==> arithmetic/digits/zeros/lsd.factorial.p <==
  305. What is the least significant non-zero digit in the decimal expansion of n]?
  306.  
  307. ==> arithmetic/digits/zeros/million.p <==
  308. How many zeros occur in the numbers from 1 to 1,000,000?
  309.  
  310. ==> arithmetic/magic.squares.p <==
  311. Are there large squares, containing only consecutive integers, all of whose
  312. rows, columns and diagonals have the same sum?  How about cubes?
  313.  
  314. ==> arithmetic/pell.p <==
  315. Find integer solutions to x^2 - 92y^2 = 1.
  316.  
  317. ==> arithmetic/prime/arithmetic.progression.p <==
  318. Is there an arithmetic progression of 20 or more primes?
  319.  
  320. ==> arithmetic/prime/consecutive.composites.p <==
  321. Are there 10,000 consecutive non-prime numbers?
  322.  
  323. ==> arithmetic/sequence.p <==
  324. Prove that all sets of n integers contain a subset whose sum is divisible by n.
  325.  
  326. ==> arithmetic/sum.of.cubes.p <==
  327. Find two fractions whose cubes total 6.
  328.  
  329. ==> arithmetic/tests.for.divisibility/eleven.p <==
  330. What is the test to see if a number is divisible by eleven?
  331.  
  332.  
  333. ==> arithmetic/tests.for.divisibility/nine.p <==
  334. What is the test to see if a number is divisible by nine?
  335.  
  336. ==> arithmetic/tests.for.divisibility/seven.p <==
  337. What is the test to see if a number is divisible by 7?
  338.  
  339. ==> arithmetic/tests.for.divisibility/three.p <==
  340. Prove that if a number is divisible by 3, the sum of its digits is likewise.
  341.  
  342. ==> combinatorics/coinage/combinations.p <==
  343. How many ways are there to make change for a dollar?  Count
  344. combinations of coins, not permuations.
  345.  
  346. ==> combinatorics/coinage/dimes.p <==
  347. "Dad wants one-cent, two-cent, three-cent, five-cent, and ten-cent
  348. stamps.  He said to get four each of two sorts and three each of the
  349. others, but I've forgotten which.  He gave me exactly enough to buy
  350. them; just these dimes."  How many stamps of each type does Dad want?
  351.  
  352. ==> combinatorics/coinage/impossible.p <==
  353. What is the smallest number of coins that you can't make a dollar with?
  354. I.e., for what N does there not exist a set of N coins adding up to a dollar?
  355. It is possible to make a dollar with 1 current U.S. coin (a Susan B. Anthony),
  356. 2 coins (2 fifty cent pieces), 3 coins (2 quarters and a fifty cent piece),
  357.  
  358. ==> combinatorics/color.p <==
  359. An urn contains n balls of different colors.  Randomly select a pair, repaint
  360. the first to match the second, and replace the pair in the urn.  What is the
  361. expected time until the balls are all the same color?
  362.  
  363. ==> combinatorics/full.p <==
  364. Consider a string that contains all substrings of length n.  For example,
  365. for binary strings with n=2, a shortest string is 00110 -- it contains 00,
  366. 01, 10 and 11 as substrings.  Find the shortest such strings for all n.
  367.  
  368. ==> combinatorics/gossip.p <==
  369. n people each know a different piece of gossip.  They can telephone each other
  370. and exchange all the information they know (so that after the call they both
  371. know anything that either of them knew before the call).  What is the smallest
  372. number of calls needed so that everyone knows everything?
  373.  
  374. ==> combinatorics/grid.dissection.p <==
  375. How many (possibly overlapping) squares are in an mxn grid?
  376.  
  377. ==> combinatorics/subsets.p <==
  378. Out of the set of integers 1,...,100 you are given ten different
  379. integers.  From this set, A, of ten integers you can always find two
  380. disjoint subsets, S & T, such that the sum of elements in S equals the
  381. sum of elements in T.  Note: S union T need not be all ten elements of
  382.  
  383. ==> cryptology/Beale.p <==
  384. What are the Beale ciphers?
  385.  
  386. ==> cryptology/Feynman.p <==
  387. What are the Feynman ciphers?
  388.  
  389. ==> cryptology/Voynich.p <==
  390. What are the Voynich ciphers?
  391.  
  392. ==> cryptology/swiss.colony.p <==
  393. What are the 1987 Swiss Colony ciphers?
  394.  
  395. ==> decision/allais.p <==
  396. The Allais Paradox involves the choice between two alternatives:
  397.  
  398.         A. 89% chance of an unknown amount
  399.            10% chance of $1 million
  400.  
  401. ==> decision/division.p <==
  402. N-Person Fair Division
  403.  
  404. If two people want to divide a pie but do not trust each other, they can
  405. still ensure that each gets a fair share by using the technique that one
  406.  
  407. ==> decision/dowry.p <==
  408. Sultan's Dowry
  409.  
  410. A sultan has granted a commoner a chance to marry one of his hundred
  411. daughters. The commoner will be presented the daughters one at a time.
  412.  
  413. ==> decision/envelope.p <==
  414. Someone has prepared two envelopes containing money.  One contains twice as
  415. much money as the other.  You have decided to pick one envelope, but then the
  416. following argument occurs to you:  Suppose my chosen envelope contains $X,
  417. then the other envelope either contains $X/2 or $2X.  Both cases are
  418.  
  419. ==> decision/exchange.p <==
  420. At one time, the Mexican and American dollars were devalued by 10 cents on each
  421. side of the border (i.e. a Mexican dollar was 90 cents in the US, and a US
  422. dollar was worth 90 cents in Mexico).  A man walks into a bar on the American
  423. side of the border, orders 10 cents worth of beer, and tenders a Mexican dollar
  424.  
  425. ==> decision/newcomb.p <==
  426. Newcomb's Problem
  427.  
  428. A being put one thousand dollars in box A and either zero or one million
  429. dollars in box B and presents you with two choices:
  430.  
  431. ==> decision/prisoners.p <==
  432. Three prisoners on death row are told that one of them has been chosen
  433. at random for execution the next day, but the other two are to be
  434. freed.  One privately begs the warden to at least tell him the name of
  435. one other prisoner who will be freed.  The warden relents: 'Susie will
  436.  
  437. ==> decision/red.p <==
  438. I show you a shuffled deck of standard playing cards, one card at a
  439. time.  At any point before I run out of cards, you must say "RED]".
  440. If the next card I show is red (i.e. diamonds or hearts), you win.  We
  441. assume I the "dealer" don't have any control over what the order of
  442.  
  443. ==> decision/rotating.table.p <==
  444. Four glasses are placed upside down in the four corners of a square
  445. rotating table.  You wish to turn them all in the same direction,
  446. either all up or all down.  You may do so by grasping any two glasses
  447. and, optionally, turning either over.  There are two catches:  you are
  448.  
  449. ==> decision/stpetersburg.p <==
  450. What should you be willing to pay to play a game in which the payoff is
  451. calculated as follows:  a coin is flipped until in comes up heads on the
  452. nth toss and the payoff is set at 2^n dollars?
  453.  
  454. ==> decision/switch.p <==
  455. Switch? (The Monty Hall Problem)
  456.  
  457. Two black marbles and a red marble are in a bag. You choose one marble from the
  458. bag without looking at it. Another person chooses a marble from the bag and it
  459.  
  460. ==> decision/truel.p <==
  461. A, B, and C are to fight a three-cornered pistol duel.  All know that
  462. A's chance of hitting his target is 0.3, C's is 0.5, and B never misses.
  463. They are to fire at their choice of target in succession in the order
  464. A, B, C, cyclically (but a hit man loses further turns and is no longer
  465.  
  466. ==> english/acronym.p <==
  467. What acronyms have become common words?
  468.  
  469. ==> english/ambiguous.p <==
  470. What word in the English language is the most ambiguous?
  471. What is the greatest number of parts of speech that a single word
  472. can be used for?
  473.  
  474. ==> english/antonym.p <==
  475. What words, when a single letter is added, reverse their meanings?
  476.  
  477. Exclude words that are obtained by adding an "a-" to the beginning.
  478.  
  479. ==> english/behead.p <==
  480. Is there a sentence that remains a sentence when all its words are beheaded?
  481.  
  482. ==> english/capital.p <==
  483. What words change pronunciation when capitalized (e.g., polish -> Polish)?
  484.  
  485. ==> english/charades.p <==
  486. A ....... surgeon was ....... to operate because he had .......
  487.  
  488. ==> english/contradictory.proverbs.p <==
  489. What are some proverbs that contradict one another?
  490.  
  491. ==> english/contranym.p <==
  492. What words are their own antonym?
  493.  
  494. ==> english/element.p <==
  495. The name of what element ends in "h"?
  496.  
  497. ==> english/equations.p <==
  498. Each equation below contains the initials of words that will make the phrase
  499. correct.  Figure out the missing words.  Lower case is used only to help the
  500. initials stand out better.
  501.  
  502.  
  503. ==> english/fossil.p <==
  504. What are some examples of idioms that include obsolete words?
  505.  
  506. ==> english/frequency.p <==
  507. In the English language, what are the most frequently appearing:
  508.         1) letters overall?
  509.         2) letters BEGINNING words?
  510.         3) final letters?
  511.  
  512. ==> english/gry.p <==
  513. Find three completely different words ending in "gry."
  514.  
  515. ==> english/homographs.p <==
  516. List all homographs (words that are spelled the same but pronounced differently)
  517.  
  518. ==> english/homophones.p <==
  519. What words have four or more spellings that sound alike?
  520.  
  521. ==> english/j.ending.p <==
  522. What words and names end in j?
  523.  
  524. ==> english/ladder.p <==
  525. Find the shortest word ladders stretching between the following pairs:
  526. hit - ace
  527. pig - sty
  528. four - five
  529.  
  530. ==> english/less.ness.p <==
  531. Find a word that forms two other words, unrelated in meaning, when "less"
  532. and "ness" are added.
  533.  
  534. ==> english/letter.rebus.p <==
  535. Define the letters of the alphabet using self-referential common phrases (e.g.,
  536. "first of all" defines "a").
  537.  
  538. ==> english/lipograms.p <==
  539. What books have been written without specific letters, vowels, etc.?
  540.  
  541. ==> english/multi.lingual.p <==
  542. What words in multiple languages are related in interesting ways?
  543.  
  544. ==> english/near.palindrome.p <==
  545. What are some long near palindromes, i.e., words that except for one
  546. letter would be palindromes?
  547.  
  548. ==> english/palindromes.p <==
  549. What are some long palindromes?
  550.  
  551. ==> english/pangram.p <==
  552. A "pangram" is a sentence containing all 26 letters.
  553. What is the shortest pangram (measured by number of letters or words)?
  554. What is the shortest word list using all 26 letters in alphabetical order?
  555. In reverse alphabetical order?
  556.  
  557. ==> english/phonetic.letters.p <==
  558. What does "FUNEX" mean?
  559.  
  560. ==> english/piglatin.p <==
  561. What words in pig latin also are words?
  562.  
  563. ==> english/pleonasm.p <==
  564. What are some redundant terms that occur frequently (like "ABM missile")?
  565.  
  566. ==> english/plurals/collision.p <==
  567. Two words, spelled and pronounced differently, have plurals spelled
  568. the same but pronounced differently.
  569.  
  570. ==> english/plurals/doubtful.number.p <==
  571. A little word of doubtful number,
  572. a foe to rest and peaceful slumber.
  573. If you add an "s" to this,
  574. great is the metamorphosis.
  575.  
  576. ==> english/plurals/drop.s.p <==
  577. What plural is formed by DROPPING the terminal "s" in a word?
  578.  
  579. ==> english/plurals/endings.p <==
  580. List a plural ending with each letter of the alphabet.
  581.  
  582. ==> english/plurals/french.p <==
  583. What English word, when spelled backwards, is its French plural?
  584.  
  585. ==> english/plurals/man.p <==
  586. Words ending with "man" make their plurals by adding "s".
  587.  
  588. ==> english/plurals/switch.first.p <==
  589. What plural is formed by switching the first two letters?
  590.  
  591. ==> english/portmanteau.p <==
  592. What are some words formed by combining together parts of other words?
  593.  
  594. ==> english/potable.color.p <==
  595. Find words that are both beverages and colors.
  596.  
  597. ==> english/rare.trigraphs.p <==
  598. What trigraphs (three-letter combinations) occur in only one word?
  599.  
  600. ==> english/records/pronunciation/silent.p <==
  601. What words have an exceptional number of silent letters?
  602.  
  603. ==> english/records/pronunciation/spelling.p <==
  604. What words have exceptional ways to spell sounds?
  605.  
  606. ==> english/records/pronunciation/syllable.p <==
  607. What words have an exceptional number of letters per syllable?
  608.  
  609. ==> english/records/spelling/longest.p <==
  610. What is the longest word in the English language?
  611.  
  612. ==> english/records/spelling/most.p <==
  613. What word has the most variant spellings?
  614.  
  615. ==> english/records/spelling/operations.on.words/deletion.p <==
  616. What exceptional words turn into other words by deletion of letters?
  617.  
  618. ==> english/records/spelling/operations.on.words/insertion.and.deletion.p <==
  619. What exceptional words turn into other words by both insertion and
  620. deletion of letters?
  621.  
  622. ==> english/records/spelling/operations.on.words/insertion.p <==
  623. What exceptional words turn into other words by insertion of letters?
  624.  
  625. ==> english/records/spelling/operations.on.words/movement.p <==
  626. What exceptional words turn into other words by movement of letters?
  627.  
  628. ==> english/records/spelling/operations.on.words/substitution.p <==
  629. What exceptional words turn into other words by substitution of letters?
  630.  
  631. ==> english/records/spelling/operations.on.words/transposition.p <==
  632. What exceptional words turn into other words by transposition of letters?
  633.  
  634. ==> english/records/spelling/operations.on.words/words.within.words.p <==
  635. What exceptional words contain other words?
  636.  
  637. ==> english/records/spelling/sets.of.words/nots.and.crosses.p <==
  638. What is the most number of letters that can be fit into a three by three grid
  639. of words, such that no letter is repeated in any row, column or diagonal?
  640.  
  641. ==> english/records/spelling/sets.of.words/squares.p <==
  642. What are some exceptional word squares (square crosswords with no blanks)?
  643.  
  644. ==> english/records/spelling/single.words.p <==
  645. What words have exceptional lengths, patterns, etc.?
  646.  
  647. ==> english/repeat.p <==
  648. What is a sentence containing the most repeated words, without:
  649.         using quotation marks,
  650.         using proper names,
  651.         using a language other than English,
  652.  
  653. ==> english/repeated.words.p <==
  654. What is a sentence with the same word several times repeated?
  655.  
  656. ==> english/rhyme.p <==
  657. What English words are hard to rhyme?
  658.  
  659. "Rhyme is the identity in sound of an accented vowel in a word...and
  660. of all consonantal and vowel sounds following it; with a difference in
  661.  
  662. ==> english/self.ref.letters.p <==
  663. Construct a true sentence of the form: "This sentence contains _ a's, _ b's,
  664. _ c's, ...," where the numbers filling in the blanks are spelled out.
  665.  
  666. ==> english/self.ref.numbers.p <==
  667. What true sentence has the form: "There are _ 0's, _ 1's, _ 2's, ...,
  668. in this sentence"?
  669.  
  670. ==> english/self.ref.words.p <==
  671. What sentence describes its own word, syllable and letter count?
  672.  
  673. ==> english/sentence.p <==
  674. Find a sentence with words beginning with the letters of the alphabet, in order.
  675.  
  676. ==> english/snowball.p <==
  677. Construct the longest coherent sentence you can such that the nth
  678. word is n letters long.
  679.  
  680. ==> english/spoonerisms.p <==
  681. List some exceptional spoonerisms.
  682.  
  683. ==> english/states.p <==
  684. What long words have all bigrams either a postal state code or its reverse?
  685.  
  686. ==> english/telegrams.p <==
  687. Since telegrams cost by the word, phonetically similar messages can be cheaper.
  688. See if you can decipher these extreme cases:
  689.  
  690. UTICA CHANSON MIGRATE INVENTION ANNUAL KNOBBY SORRY IN FACTUAL BEEN CLOVER.
  691.  
  692. ==> english/trivial.p <==
  693. Consider the free non-abelian group on the twenty-six letters of the
  694. alphabet with all relations of the form <word1> = <word2>, where <word1>
  695. and <word2> are homophones (i.e. they sound alike but are spelled
  696. differently).  Show that every letter is trivial.
  697.  
  698. ==> english/weird.p <==
  699. Make a sentence containing only words that violate the "i before e" rule.
  700.  
  701. ==> english/word.boundaries.p <==
  702. List some sentences that can be radically altered by changing word boundaries
  703. and punctuation.
  704.  
  705. ==> english/word.torture.p <==
  706. What is the longest word all of whose contiguous subsequences are words?
  707.  
  708. ==> games/chess/knight.control.p <==
  709. How many knights does it take to attack or control the board?
  710.  
  711. ==> games/chess/mutual.check.p <==
  712. What position is a stalemate for both sides and is reachable in a legal game
  713. (including the requirement to prevent check)?
  714.  
  715. ==> games/chess/mutual.stalemate.p <==
  716. What's the minimal number of pieces in a legal mutual stalemate?
  717.  
  718. ==> games/chess/queens.p <==
  719. How many ways can eight queens be placed so that they control the board?
  720.  
  721. ==> games/chess/size.of.game.tree.p <==
  722. How many different positions are there in the game tree of chess?
  723.  
  724. ==> games/cigarettes.p <==
  725. The game of cigarettes is played as follows:
  726. Two players take turns placing a cigarette on a circular table.  The cigarettes
  727. can be placed upright (on end) or lying flat, but not so that it touches any
  728. other cigarette on the table.  This continues until one person looses by not
  729.  
  730. ==> games/connect.four.p <==
  731. Is there a winning strategy for Connect Four?
  732.  
  733. ==> games/craps.p <==
  734. What are the odds in craps?
  735.  
  736. ==> games/crosswords/cryptic/clues.p <==
  737. What are some clues (indicators) used in cryptics?
  738.  
  739. ==> games/crosswords/cryptic/double.p <==
  740. Each clue has two solutions, one for each diagram; one of the answers
  741. to 1ac. determines which solutions are for which diagram.
  742.  
  743. All solutions are in Chamber's and Webster's Third except for one solution
  744.  
  745. ==> games/crosswords/cryptic/intro.p <==
  746. What are the rules for cluing cryptic crosswords?
  747.  
  748. ==> games/go-moku.p <==
  749. For a game of k in a row on an n x n board,  for what values of k and n is
  750. there a win?  Is (the largest such) k eventually constant or does it increase
  751. with n?
  752.  
  753. ==> games/hi-q.p <==
  754. What is the quickest solution of the game Hi-Q (also called Solitair)?
  755.  
  756. For those of you who aren't sure what the game looks like:
  757.  
  758.  
  759. ==> games/jeopardy.p <==
  760. What are the highest, lowest, and most different scores contestants
  761. can achieve during a single game of Jeopardy?
  762.  
  763. ==> games/knight.tour.p <==
  764. For what board sizes is a knight's tour possible?
  765.  
  766. ==> games/nim.p <==
  767. Place 10 piles of 10 $1 bills in a row.  A valid move is to reduce
  768. the last i>0 piles by the same amount j>0 for some i and j; a pile
  769. reduced to nothing is considered to have been removed.  The loser
  770. is the player who picks up the last dollar, and they must forfeit
  771.  
  772. ==> games/othello.p <==
  773. How good are computers at Othello?
  774.  
  775. ==> games/risk.p <==
  776. What are the odds when tossing dice in Risk?
  777.  
  778. ==> games/rubiks.clock.p <==
  779. How do you quickly solve Rubik's clock?
  780.  
  781. ==> games/rubiks.cube.p <==
  782. What is known about bounds on solving Rubik's cube?
  783.  
  784. ==> games/rubiks.magic.p <==
  785. How do you solve Rubik's Magic?
  786.  
  787. ==> games/scrabble.p <==
  788. What are some exceptional scrabble games?
  789.  
  790. ==> games/square-1.p <==
  791. Does anyone have any hints on how to solve the Square-1 puzzle?
  792.  
  793. ==> games/think.and.jump.p <==
  794. THINK & JUMP:  FIRST THINK, THEN JUMP UNTIL YOU
  795.                ARE LEFT WITH ONE PEG]                      O - O   O - O
  796.                                                           / \ / \ / \ / \
  797.                                                          O---O---O---O---O
  798.  
  799. ==> games/tictactoe.p <==
  800. In random tic-tac-toe, what is the probability that the first mover wins?
  801.  
  802. ==> geometry/K3,3.p <==
  803. Can three houses be connected to three utilities without the pipes crossing?
  804.  
  805.             _______          _______          _______
  806.             ! oil !          !water!          ! gas !
  807.  
  808. ==> geometry/bear.p <==
  809. If a hunter goes out his front door, goes 50 miles south, then goes 50
  810. miles west, shoots a bear, goes 50 miles north and ends up in front of
  811. his house.  What color was the bear?
  812.  
  813. ==> geometry/bisector.p <==
  814. If two angle bisectors of a triangle are equal, then the triangle is
  815. isosceles (more specifically, the sides opposite to the two angles
  816. being bisected are equal).
  817.  
  818. ==> geometry/calendar.p <==
  819. Build a calendar from two sets of cubes.  On the first set,
  820. spell the months with a letter on each face of three cubes.
  821. Use lowercase three-letter abbreviations for the names of all
  822. twelve months (e.g., "jan", "feb", "mar").  On the second set,
  823.  
  824. ==> geometry/circles.and.triangles.p <==
  825. Find the radius of the inscribed and circumscribed circles for a triangle.
  826.  
  827. ==> geometry/coloring/cheese.cube.p <==
  828. A cube of cheese is divided into 27 subcubes.  A mouse starts at one
  829. corner and eats through every subcube.  Can it finish in the middle?
  830.  
  831. ==> geometry/coloring/dominoes.p <==
  832. There is a chess board (of course with 64 squares). You are given
  833. 21 dominoes of size 3-by-1 (the size of an individual square on
  834. a chess board is 1-by-1). Which square on the chess board can
  835. you cut out so that the 21 dominoes exactly cover the remaining
  836.  
  837. ==> geometry/construction/4.triangles.6.lines.p <==
  838. Can you construct 4 equilateral triangles with 6 toothpicks?
  839.  
  840. ==> geometry/construction/5.lines.with.4.points.p <==
  841. Arrange 10 points so that they form 5 rows of 4 each.
  842.  
  843. ==> geometry/construction/square.with.compass.p <==
  844. Construct a square with only a compass and a straight edge.
  845.  
  846. ==> geometry/cover.earth.p <==
  847. A thin membrane covers the surface of the earth.  One square meter is
  848. added to the area of this membrane.  How much is added to the radius and
  849. volume of this membrane?
  850.  
  851. ==> geometry/dissections/circle.p <==
  852. Can a circle be cut into similar pieces without point symmetry
  853. about the midpoint?  Can it be done with a finite number of pieces?
  854.  
  855. ==> geometry/dissections/hexagon.p <==
  856. Divide the hexagon into:
  857. 1) 3 indentical rhombuses.
  858. 2) 6 indentical kites(?).
  859. 3) 4 indentical trapezoids.
  860.  
  861. ==> geometry/dissections/square.70.p <==
  862. Since 1^2 + 2^2 + 3^2 + ... + 24^2 = 70^2, can a 70x70 sqaure be dissected into
  863. 24 squares of size 1x1, 2x2, 3x3, etc.?
  864.  
  865. ==> geometry/dissections/square.five.p <==
  866. Can you dissect a square into 5 parts of equal area with just a straight edge?
  867.  
  868. ==> geometry/duck.and.fox.p <==
  869. A duck is swimming about in a circular pond.  A ravenous fox (who cannot
  870. swim) is roaming the edges of the pond, waiting for the duck to come close.
  871. The fox can run faster than the duck can swim.  In order to escape,
  872. the duck must swim to the edge of the pond before flying away.  Assume that
  873.  
  874. ==> geometry/earth.band.p <==
  875. How much will a band around the equator rise above the surface if it
  876. is made one meter longer?
  877.  
  878. ==> geometry/ham.sandwich.p <==
  879. Consider a ham sandwich, consisting of two pieces of bread and one of
  880. ham.  Suppose the sandwich was dropped into a machine and spindled,
  881. torn and mutiliated.  Is it still possible to divide the ham sandwich
  882. with a straight knife cut such that both the ham and the bread are
  883.  
  884. ==> geometry/hike.p <==
  885. You are hiking in a half-planar woods, exactly 1 mile from the edge,
  886. when you suddenly trip and lose your sense of direction.  What's the
  887. shortest path that's guaranteed to take you out of the woods?  Assume
  888. that you can navigate perfectly relative to your current location and
  889.  
  890. ==> geometry/hole.in.sphere.p <==
  891. Old Boniface he took his cheer,
  892. Then he bored a hole through a solid sphere,
  893. Clear through the center, straight and strong,
  894. And the hole was just six inches long.
  895.  
  896. ==> geometry/ladders.p <==
  897. Two ladders form a rough X in an alley.  The ladders are 11 and 13 meters
  898. long and they cross 4 meters off the ground.  How wide is the alley?
  899.  
  900. ==> geometry/lattice/area.p <==
  901. Prove that the area of a triangle formed by three lattice points is integer/2.
  902.  
  903. ==> geometry/lattice/equilateral.p <==
  904. Can an equlateral triangle have vertices at integer lattice points?
  905.  
  906. ==> geometry/rotation.p <==
  907. What is the smallest rotation that returns an object to its original state?
  908.  
  909. ==> geometry/smuggler.p <==
  910. Somewhere on the high sees smuggler S is attempting, without much
  911. luck, to outspeed coast guard G, whose boat can go faster than S's. G
  912. is one mile east of S when a heavy fog descends. It's so heavy that
  913. nobody can see or hear anything further than a few feet. Immediately
  914.  
  915. ==> geometry/table.in.corner.p <==
  916. Put a round table into a (perpendicular) corner so that the table top
  917. touches both walls and the feet are firmly on the ground.  If there is
  918. a point on the perimeter of the table, in the quarter circle between
  919. the two points of contact, which is 10 cm from one wall and 5 cm from
  920.  
  921. ==> geometry/tesseract.p <==
  922. If you suspend a cube by one corner and slice it in half with a
  923. horizontal plane through its centre of gravity, the section face is a
  924. hexagon.  Now suspend a tesseract (a four dimensional hypercube) by one
  925. corner and slice it in half with a hyper-horizontal hyperplane through
  926.  
  927. ==> geometry/tetrahedron.p <==
  928. Suppose you have a sphere of radius R and you have four planes that are
  929. all tangent to the sphere such that they form an arbitrary tetrahedron
  930. (it can be irregular).  What is the ratio of the surface area of the
  931. tetrahedron to its volume?
  932.  
  933. ==> geometry/tiling/rational.sides.p <==
  934. A rectangular region R is divided into rectangular areas.  Show that if
  935. each of the rectangles in the region has at least one side with
  936. rational length then the same can be said of R.
  937.  
  938. ==> geometry/tiling/rectangles.with.squares.p <==
  939. Given two sorts of squares, (axa) and (bxb), what rectangles can be tiled?
  940.  
  941. ==> geometry/tiling/scaling.p <==
  942. A given rectangle can be entirely covered (i.e. concealed) by an
  943. appropriate arrangement of 25 disks of unit radius.
  944.  
  945. Can the same rectangle be covered by 100 disks of 1/2 unit radius?
  946.  
  947. ==> geometry/tiling/seven.cubes.p <==
  948. Consider 7 cubes of equal size arranged as follows. Place 5 cubes so
  949. that they form a Swiss cross or a + (plus). ( 4 cubes on the sides and
  950. 1 in the middle). Now place one cube on top of the middle cube and the
  951. seventh below the middle cube, to effectively form a 3-dimensional
  952.  
  953. ==> group/group.01.p <==
  954. AEFHIKLMNTVWXYZ BCDGJOPQRSU
  955.  
  956. ==> group/group.01a.p <==
  957. 147 0235689
  958.  
  959. ==> group/group.02.p <==
  960. ABEHIKMNOPTXZ CDFGJLQRSUVWY
  961.  
  962. ==> group/group.03.p <==
  963. BEJQXYZ DFGHLPRU KSTV CO AIW MN
  964.  
  965. ==> group/group.04.p <==
  966. BDO P ACGJLMNQRSUVWZ EFTY HIKX
  967.  
  968. ==> group/group.05.p <==
  969. CEFGHIJKLMNSTUVWXYZ ADOPQR B
  970.  
  971. ==> group/group.06.p <==
  972. BCEGKMQSW DFHIJLNOPRTUVXYZ
  973.  
  974. ==> induction/hanoi.p <==
  975. Is there an algorithom for solving the hanoi tower puzzle for any number
  976. of towers?  Is there an equation for determining the minimum number of
  977. moves required to solve it, given a variable number of disks and towers?
  978.  
  979. ==> induction/n-sphere.p <==
  980. With what odds do three random points on an n-sphere form an acute triangle?
  981.  
  982. ==> induction/paradox.p <==
  983. What simple property holds for the first 10,000 integers, then fails?
  984.  
  985. ==> induction/party.p <==
  986. You're at a party.  Any two (different) people at the party have exactly one
  987. friend in common (the friend is also at the party).  Prove that there is at
  988. least one person at the party who is a friend of everyone else.  Assume that
  989. the friendship relation is symmetric and not reflexive.
  990.  
  991. ==> induction/roll.p <==
  992. An ordinary die is thrown until the running total of the throws first
  993. exceeds 12.  What is the most likely final total that will be obtained?
  994.  
  995. ==> induction/takeover.p <==
  996. After graduating from college, you have taken an important managing position
  997. in the prestigious financial firm of "Mary and Lee".
  998. You are responsable for all the decisions concerning take-over bids.
  999. Your immediate concern is whether to take over "Financial Data".
  1000.  
  1001. ==> logic/29.p <==
  1002. Three people check into a hotel.  They pay $30 to the manager and go
  1003. to their room.  The manager finds out that the room rate is $25 and
  1004. gives $5 to the bellboy to return.  On the way to the room the bellboy
  1005. reasons that $5 would be difficult to share among three people so
  1006.  
  1007. ==> logic/ages.p <==
  1008. 1) Ten years from now Tim will be twice as old as Jane was when Mary was
  1009.    nine times as old as Tim.
  1010.  
  1011. 2) Eight years ago, Mary was half as old as Jane will be when Jane is one year
  1012.  
  1013. ==> logic/bookworm.p <==
  1014. A bookworm eats from the first page of an encyclopedia to the last page.
  1015. The bookworm eats in a straight line.  The encyclopedia consists of ten
  1016. 1000-page volumes.  Not counting covers, title pages, etc., how many pages
  1017. does the bookworm eat through?
  1018.  
  1019. ==> logic/boxes.p <==
  1020. Which Box Contains the Gold?
  1021.  
  1022. Two boxes are labeled "A" and "B".  A sign on box A says "The sign
  1023. on box B is true and the gold is in box A".  A sign on box B says
  1024.  
  1025. ==> logic/calibans.will.p <==
  1026.         ----------------------------------------------
  1027.         !       Caliban's Will by M.H. Newman        !
  1028.         ----------------------------------------------
  1029.  
  1030.  
  1031. ==> logic/camel.p <==
  1032. An Arab sheikh tells his two sons that are to race their camels to a
  1033. distant city to see who will inherit his fortune.  The one whose camel
  1034. is slower will win.  The brothers, after wandering aimlessly for days,
  1035. ask a wiseman for advise.  After hearing the advice they jump on the
  1036.  
  1037. ==> logic/centrifuge.p <==
  1038. You are a biochemist, working with a 12-slot centrifuge.  This is a gadget
  1039. that has 12 equally spaced slots around a central axis, in which you can
  1040. place chemical samples you want centrifuged.  When the machine is turned on,
  1041. the samples whirl around the central axis and do their thing.
  1042.  
  1043. ==> logic/children.p <==
  1044. A man walks into a bar, orders a drink, and starts chatting with the
  1045. bartender.  After a while, he learns that the bartender has three
  1046. children.  "How old are your children?" he asks.  "Well," replies the
  1047. bartender, "the product of their ages is 72."  The man thinks for a
  1048.  
  1049. ==> logic/condoms.p <==
  1050. How can you have mutually safe sex with three women with only two condoms?
  1051.  
  1052. ==> logic/dell.p <==
  1053. How can I solve logic puzzles (e.g., as published by Dell) automatically?
  1054.  
  1055. ==> logic/elimination.p <==
  1056. 97 baseball teams participate in an annual state tournament.
  1057. The way the champion is chosen for this tournament is by the same old
  1058. elimination schedule. That is, the 97 teams are to be divided into
  1059. pairs, and the two teams of each pair play against each other.
  1060.  
  1061. ==> logic/family.p <==
  1062. Suppose that it is equally likely for a pregnancy to deliver
  1063. a baby boy as it is to deliver a baby girl.  Suppose that for a
  1064. large society of people, every family continues to have children
  1065. until they have a boy, then they stop having children.
  1066.  
  1067. ==> logic/flip.p <==
  1068. How can a toss be called over the phone (without requiring trust)?
  1069.  
  1070. ==> logic/friends.p <==
  1071. Any group of 6 or more contains either 3 mutual friends or 3 mutual strangers.
  1072. Prove it.
  1073.  
  1074. ==> logic/hundred.p <==
  1075. A sheet of paper has statements numbered from 1 to 100.  Statement n says
  1076. "exactly n of the statements on this sheet are false."  Which statements are
  1077. true and which are false?  What if we replace "exactly" by "at least"?
  1078.  
  1079. ==> logic/inverter.p <==
  1080. Can a digital logic circuit with two inverters invert N independent inputs?
  1081. The circuit may contain any number of AND or OR gates.
  1082.  
  1083. ==> logic/josephine.p <==
  1084. The recent expedition to the lost city of Atlantis discovered scrolls
  1085. attributted to the great poet, scholar, philosopher Josephine. They
  1086. number eight in all, and here is the first.
  1087.  
  1088.  
  1089. ==> logic/locks.and.boxes.p <==
  1090. You want to send a valuable object to a friend.  You have a box which
  1091. is more than large enough to contain the object.  You have several
  1092. locks with keys.  The box has a locking ring which is more than large enough
  1093. to have a lock attached.  But your friend does not have the key to any
  1094.  
  1095. ==> logic/mixing.p <==
  1096. Start with a half cup of tea and a half cup of coffee. Take one tablespoon
  1097. of the tea and mix it in with the coffee. Take one tablespoon of this mixture
  1098. and mix it back in with the tea. Which of the two cups contains more of its
  1099. original contents?
  1100.  
  1101. ==> logic/number.p <==
  1102. Mr. S. and Mr. P. are both perfect logicians, being able to correctly deduce
  1103. any truth from any set of axioms.  Two integers (not necessarily unique) are
  1104. somehow chosen such that each is within some specified range.  Mr. S.
  1105. is given the sum of these two integers; Mr. P. is given the product of these
  1106.  
  1107. ==> logic/riddle.p <==
  1108. Who makes it, has no need of it.  Who buys it, has no use for it.  Who
  1109. uses it can neither see nor feel it.
  1110.  
  1111. Tell me what a dozen rubber trees with thirty boughs on each might be?
  1112.  
  1113. ==> logic/river.crossing.p <==
  1114. Three humans, one big monkey and two small monkeys are to cross a river:
  1115.         a) Only humans and the big monkey can row the boat.
  1116.         b) At all times, the number of human on either side of the
  1117.            river must be GREATER OR EQUAL to the number of monkeys
  1118.  
  1119. ==> logic/ropes.p <==
  1120. Two fifty foot ropes are suspended from a forty foot ceiling, about
  1121. twenty feet apart.  Armed with only a knife, how much of the rope can
  1122. you steal?
  1123.  
  1124. ==> logic/same.street.p <==
  1125. Sally and Sue have a strong desire to date Sam.  They all live on the
  1126. same street yet neither Sally or Sue know where Sam lives.  The houses
  1127. on this street are numbered 1 to 99.
  1128.  
  1129.  
  1130. ==> logic/self.ref.p <==
  1131. Find a number ABCDEFGHIJ such that A is the count of how many 0's are in the
  1132. number, B is the number of 1's, and so on.
  1133.  
  1134. ==> logic/situation.puzzles.outtakes.p <==
  1135. The following puzzles have been removed from my situation puzzles list,
  1136. or never made it onto the list in the first place.  There are a wide
  1137. variety of reasons for the non-inclusion: some I think are obvious,
  1138. some don't have enough of a story, some involve gimmicks that annoy me,
  1139.  
  1140. ==> logic/situation.puzzles.p <==
  1141.                         Jed's List of Situation Puzzles
  1142.  
  1143. History:
  1144.    original compilation            11/28/87
  1145.  
  1146. ==> logic/smullyan/black.hat.p <==
  1147. Three logicians, A, B, and C, are wearing hats, which they know are either
  1148. black or white but not all white. A can see the hats of B and C; B can see
  1149. the hats of A and C; C is blind.  Each is asked in turn if they know the color
  1150. of their own hat.  The answers are:
  1151.  
  1152. ==> logic/smullyan/fork.three.men.p <==
  1153. Three men stand at a fork in the road.  One fork leads to Someplaceorother;
  1154. the other fork leads to Nowheresville.  One of these people always answers
  1155. the truth to any yes/no question which is asked of him.  The other always
  1156. lies when asked any yes/no question.  The third person randomly lies and
  1157.  
  1158. ==> logic/smullyan/fork.two.men.p <==
  1159. Two men stand at a fork in the road.  One fork leads to Someplaceorother; the
  1160. other fork leads to Nowheresville.  One of these people always answers the
  1161. truth to any yes/no question which is asked of him.  The other always lies
  1162. when asked any yes/no question.  By asking one yes/no question, can you
  1163.  
  1164. ==> logic/smullyan/integers.p <==
  1165. Two logicians place cards on their foreheads so that what is written on the
  1166. card is visible only to the other logician.  Consecutive positive integers
  1167. have been written on the cards.  The following conversation ensues:
  1168.     A: "I don't know my number."
  1169.  
  1170. ==> logic/smullyan/liars.et.al.p <==
  1171. Of a group of n men, some always lie, some never lie, and the rest sometimes
  1172. lie.  They each know which is which.  You must determine the identity of each
  1173. man by asking the least number of yes-or-no questions.
  1174.  
  1175. ==> logic/smullyan/painted.heads.p <==
  1176. While three logicians were sleeping under a tree, a malicious child painted
  1177. their heads red.  Upon waking, each logician spies the child's handiwork as
  1178. it applied to the heads of the other two.  Naturally they start laughing.
  1179. Suddenly one falls silent.  Why?
  1180.  
  1181. ==> logic/smullyan/priest.p <==
  1182. A priest takes confession of all the inhabitants in a small town.  He
  1183. discovers that in N married pairs in the town, one of the pair has
  1184. committed adultery.  Assume that the spouse of each adulterer does not
  1185. know about the infidelity of his or her spouse, but that, since it is
  1186.  
  1187. ==> logic/smullyan/stamps.p <==
  1188. The moderator takes a set of 8 stamps, 4 red and 4 green, known to the
  1189. logicians, and loosely affixes two to the forehead of each logician so that
  1190. each logician can see all the other stamps except those 2 in the moderator's
  1191. pocket and the two on her own head.  He asks them in turn
  1192.  
  1193. ==> logic/timezone.p <==
  1194. Two people are talking long distance on the phone; one is in an East-
  1195. Coast state, the other is in a West-Coast state.  The first asks the other
  1196. "What time is it?", hears the answer, and says, "That's funny.  It's the
  1197. same time here]"
  1198.  
  1199. ==> logic/unexpected.p <==
  1200. Swedish civil defense authorities announced that a civil defense drill would
  1201. be held one day the following week, but the actual day would be a surprise.
  1202. However, we can prove by induction that the drill cannot be held.  Clearly,
  1203. they cannot wait until Friday, since everyone will know it will be held that
  1204.  
  1205. ==> logic/verger.p <==
  1206. A very bright and sunny Day
  1207. The Priest didst to the Verger say:
  1208. "Last Monday met I strangers three
  1209. None of which were known to Thee.
  1210.  
  1211. ==> logic/weighing/balance.p <==
  1212. You are given N balls and a balance scale and told that
  1213. one ball is slightly heavier or lighter than the other identical
  1214. ones.  The scale lets you put the same number of balls on each side
  1215. and observe which side (if either) is heavier.
  1216.  
  1217. ==> logic/weighing/box.p <==
  1218. You have ten boxes; each contains nine balls.  The balls in one box
  1219. weigh 0.9 kg; the rest weigh 1.0 kg.  You have one weighing on a
  1220. scale to find the box containing the light balls.  How do you do it?
  1221.  
  1222. ==> logic/weighing/gummy.bears.p <==
  1223. Real gummy drop bears have a mass of 10 grams, while imitation gummy
  1224. drop bears have a mass of 9 grams.  Spike has 7 cartons of gummy drop bears,
  1225. 4 of which contain real gummy drop bears, the others imitation.
  1226. Using a scale only once and the minimum number of gummy drop bears, how
  1227.  
  1228. ==> logic/weighing/weighings.p <==
  1229. Some of the supervisors of Scandalvania's n mints are producing bogus coins.
  1230. It would be easy to determine which mints are producing bogus coins but,
  1231. alas, the only scale in the known world is located in Nastyville,
  1232. which isn't on very friendly terms with Scandalville.  In fact, Nastyville's
  1233.  
  1234. ==> logic/zoo.p <==
  1235.  I took some nephews and nieces to the Zoo, and we halted at a cage marked
  1236.  
  1237.                     Tovus Slithius, male and female.
  1238.                   Beregovus Mimsius, male and female.
  1239.  
  1240. ==> physics/balloon.p <==
  1241. A helium-filled balloon is tied to the floor of a car that makes a
  1242. sharp right turn.  Does the balloon tilt while the turn is made?
  1243. If so, which way?  The windows are closed so there is no connection
  1244. with the outside air.
  1245.  
  1246. ==> physics/bicycle.p <==
  1247. A boy, a girl and a dog go for a 10 mile walk. The boy and girl can
  1248. walk 2 mph and the dog can trot at 4 mph. They also have bicycle
  1249. which only one of them can use at a time. When riding, the boy and
  1250. girl can travel at 12 mph while the dog can peddle at 16 mph.
  1251.  
  1252. ==> physics/boy.girl.dog.p <==
  1253. A boy, a girl and a dog are standing together on a long, straight road.
  1254. Simulataneously, they all start walking in the same direction:
  1255. The boy at 4 mph, the girl at 3 mph, and the dog trots back and forth
  1256. between them at 10 mph.  Assume all reversals of direction instantaneous.
  1257.  
  1258. ==> physics/brick.p <==
  1259. What is the maximum overhang you can create with an infinite supply of bricks?
  1260.  
  1261. ==> physics/cannonball.p <==
  1262. A person in a boat drops a cannonball overboard; does the water level change?
  1263.  
  1264. ==> physics/dog.p <==
  1265. A body of soldiers form a 50m-by-50m square ABCD on the parade ground.
  1266. In a unit of time, they march forward 50m in formation to take up the
  1267. position DCEF. The army's mascot, a small dog, is standing next to its
  1268.                                        handler at location A. When the
  1269.  
  1270. ==> physics/magnets.p <==
  1271. You have two bars of iron.  One is magnetic, the other is not.  Without
  1272. using any other instrument (thread, filings, other magnets, etc.), find
  1273. out which is which.
  1274.  
  1275. ==> physics/milk.and.coffee.p <==
  1276. You are just served a hot cup of coffee and want it to be as hot as possible
  1277. when you drink it some number of minutes later.  Do you add milk when you get
  1278. the cup or just before you drink it?
  1279.  
  1280. ==> physics/mirror.p <==
  1281. Why does a mirror appear to invert the left-right directions, but not up-down?
  1282.  
  1283. ==> physics/monkey.p <==
  1284. Hanging over a pulley, there is a rope, with a weight at one end.
  1285. At the other end hangs a monkey of equal weight.  The rope weighs
  1286. 4 ounces per foot.  The combined ages of the monkey and it's mother
  1287. is 4 years.  The weight of the monkey is as many pounds as the mother
  1288.  
  1289. ==> physics/particle.p <==
  1290. What is the longest time that a particle can take in travelling between two
  1291. points if it never increases its acceleration along the way and reaches the
  1292. second point with speed V?
  1293.  
  1294. ==> physics/pole.in.barn.p <==
  1295. Accelerate a pole of length l to a constant speed of 90% of the speed of
  1296. light (.9c).  Move this pole towards an open barn of length .9l (90%
  1297. the length of the pole).  Then, as soon as the pole is fully inside the
  1298. barn, close the door.  What do you see and what actually happens?
  1299.  
  1300. ==> physics/resistors.p <==
  1301. What are the resistances between lattices of resistors in the shape of a:
  1302.  
  1303. 1. Cube
  1304.  
  1305.  
  1306. ==> physics/sail.p <==
  1307. A sailor is in a sailboat on a river.  The water (current) is flowing
  1308. downriver at a velocity of 3 knots with respect to the land.  The wind
  1309. (air velocity) is zero, with respect to the land.  The sailor wants
  1310. to proceed downriver as quickly as possible, maximizing his downstream
  1311.  
  1312. ==> physics/skid.p <==
  1313. What is the fastest way to make a 90 degree turn on a slippery road?
  1314.  
  1315. ==> physics/spheres.p <==
  1316. Two spheres are the same size and weight, but one is hollow.  They are
  1317. made of uniform material, though of course not the same material.  Without
  1318. a minimum of apparatus, how can I tell which is hollow?
  1319.  
  1320. ==> physics/wind.p <==
  1321. Is a round-trip by airplane longer or shorter if there is wind blowing?
  1322.  
  1323. ==> probability/amoeba.p <==
  1324. A jar begins with one amoeba.  Every minute, every amoeba
  1325. turns into 0, 1, 2, or 3 amoebae with probability 25%
  1326. for each case ( dies, does nothing, splits into 2, or splits
  1327. into 3).  What is the probability that the amoeba population
  1328.  
  1329. ==> probability/apriori.p <==
  1330. An urn contains one hundred white and black balls.  You sample one hundred
  1331. balls with replacement and they are all white.  What is the probability
  1332. that all the balls are white?
  1333.  
  1334. ==> probability/cab.p <==
  1335. A cab was involved in a hit and run accident at night.  Two cab companies,
  1336. the Green and the Blue, operate in the city.  Here is some data:
  1337.  
  1338.         a)  Although the two companies are equal in size, 85% of cab
  1339.  
  1340. ==> probability/coincidence.p <==
  1341. Name some amazing coincidences.
  1342.  
  1343. ==> probability/coupon.p <==
  1344. There is a free gift in my breakfast cereal. The manufacturers say
  1345. that the gift comes in four different colours, and encourage one to
  1346. collect all four (& so eat lots of their cereal). Assuming there is
  1347. an equal chance of getting any one of the colours,  what is the
  1348.  
  1349. ==> probability/darts.p <==
  1350. Peter throws two darts at a dartboard, aiming for the center.  The
  1351. second dart lands farther from the center than the first.  If Peter now
  1352. throws another dart at the board, aiming for the center, what is the
  1353. probability that this third throw is also worse (i.e., farther from
  1354.  
  1355. ==> probability/flips.p <==
  1356. Consider a run of coin tosses: HHTHTTHTTTHTTTTHHHTHHHHHTHTTHT
  1357.  
  1358. Define a success as a run of one H or T (as in THT or HTH).  Use two
  1359. different methods of sampling.  The first method would consist of
  1360.  
  1361. ==> probability/flush.p <==
  1362. Which set contains more flushes than the set of all possible hands?
  1363. (1) Hands whose first card is an ace
  1364. (2) Hands whose first card is the ace of spades
  1365. (3) Hands with at least one ace
  1366.  
  1367. ==> probability/hospital.p <==
  1368. A town has two hospitals, one big and one small.  Every day the big
  1369. hospital delivers 1000 babies and the small hospital delivers 100
  1370. babies.  There's a 50/50 chance of male or female on each birth.
  1371. Which hospital has a better chance of having the same number of boys
  1372.  
  1373. ==> probability/icos.p <==
  1374. The "house" rolls two 20-sided dice and the "player" rolls one
  1375. 20-sided die.  If the player rolls a number on his die between the
  1376. two numbers the house rolled, then the player wins.  Otherwise, the
  1377. house wins (including ties).  What are the probabilities of the player
  1378.  
  1379. ==> probability/intervals.p <==
  1380. Given two random points x and y on the interval 0..1, what is the average
  1381. size of the smallest of the three resulting intervals?
  1382.  
  1383. ==> probability/lights.p <==
  1384. Waldo and Basil are exactly m blocks west and n blocks north from Central Park,
  1385. and always go with the green light until they run out of options.  Assuming
  1386. that the probability of the light being green is 1/2 in each direction and
  1387. that if the light is green in one direction it is red in the other, find the
  1388.  
  1389. ==> probability/lottery.p <==
  1390. There n tickets in the lottery, k winners and m allowing you to pick another
  1391. ticket. The problem is to determine the probability of winning the lottery
  1392. when you start by picking 1 (one) ticket.
  1393.  
  1394.  
  1395. ==> probability/particle.in.box.p <==
  1396. A particle is bouncing randomly in a two-dimensional box.  How far does it
  1397. travel between bounces, on avergae?
  1398.  
  1399. Suppose the particle is initially at some random position in the box and is
  1400.  
  1401. ==> probability/pi.p <==
  1402. Are the digits of pi random (i.e., can you make money betting on them)?
  1403.  
  1404. ==> probability/random.walk.p <==
  1405. Waldo has lost his car keys]  He's not using a very efficient search;
  1406. in fact, he's doing a random walk.  He starts at 0, and moves 1 unit
  1407. to the left or right, with equal probability.  On the next step, he
  1408. moves 2 units to the left or right, again with equal probability.  For
  1409.  
  1410. ==> probability/reactor.p <==
  1411. There is a reactor in which a reaction is to take place. This reaction
  1412. stops if an electron is present in the reactor. The reaction is started
  1413. with 18 positrons; the idea being that one of these positrons would
  1414. combine with any incoming electron (thus destroying both). Every second,
  1415.  
  1416. ==> probability/roulette.p <==
  1417. You are in a game of Russian roulette, but this time the gun (a 6
  1418. shooter revolver) has three bullets _in_a_row_ in three of the
  1419. chambers.  The barrel is spun only once.  Each player then points the
  1420. gun at his (her) head and pulls the trigger.  If he (she) is still
  1421.  
  1422. ==> probability/unfair.p <==
  1423. Generate even odds from an unfair coin.  For example, if you
  1424. thought a coin was biased toward heads, how could you get the
  1425. equivalent of a fair coin with several tosses of the unfair coin?
  1426.  
  1427. ==> series/series.01.p <==
  1428. M, N, B, D, P ?
  1429.  
  1430. ==> series/series.02.p <==
  1431. H, H, L, B, B, C, N, O, F ?
  1432.  
  1433. ==> series/series.03.p <==
  1434. W, A, J, M, M, A, J?
  1435.  
  1436. ==> series/series.03a.p <==
  1437. G, J, T, J, J, J, A, M, W, J, J, Z, M, F, J, ?
  1438.  
  1439.  
  1440. ==> series/series.03b.p <==
  1441. A, J, B, C, G, T, C, V, J, T, D, F, K, B, H, ?
  1442.  
  1443.  
  1444. ==> series/series.03c.p <==
  1445. M, A, M, D, E, L, R, H, ?
  1446.  
  1447.  
  1448. ==> series/series.04.p <==
  1449. A, E, H, I, K, L, ?
  1450.  
  1451. ==> series/series.05.p <==
  1452. A B C D E F G H?
  1453.  
  1454. ==> series/series.06.p <==
  1455. Z, O, T, T, F, F, S, S, E, N?
  1456.  
  1457. ==> series/series.06a.p <==
  1458. F, S, T, F, F, S, ?
  1459.  
  1460. ==> series/series.07.p <==
  1461. 1, 1 1, 2 1, 1 2 1 1, ...
  1462.  
  1463. What is the pattern and asymptotics of this series?
  1464.  
  1465. ==> series/series.08a.p <==
  1466. G, L, M, B, C, L, M, C, F, S, ?
  1467.  
  1468. ==> series/series.08b.p <==
  1469. A, V, R, R, C, C, L, L, L, E, ?
  1470.  
  1471. ==> series/series.09a.p <==
  1472. S, M, S, S, S, C, P, P, P, ?
  1473.  
  1474. ==> series/series.09b.p <==
  1475. M, S, C, P, P, P, S, S, S, ?
  1476.  
  1477. ==> series/series.10.p <==
  1478. D, P, N, G, C, M, M, S, ?
  1479.  
  1480. ==> series/series.11.p <==
  1481. R O Y G B ?
  1482.  
  1483. ==> series/series.12.p <==
  1484. A, T, G, C, L, ?
  1485.  
  1486. ==> series/series.13.p <==
  1487. M, V, E, M, J, S, ?
  1488.  
  1489. ==> series/series.14.p <==
  1490. A, B, D, O, P, ?
  1491.  
  1492. ==> series/series.14a.p <==
  1493. A, B, D, E, G, O, P, ?
  1494.  
  1495. ==> series/series.15.p <==
  1496. A, E, F, H, I, ?
  1497.  
  1498. ==> series/series.16.p <==
  1499. A, B, C, D, E, F, G, H, I, J, K, L, M, N, O, P, Q, R, S, T, U, V, X, Y?
  1500.  
  1501. ==> series/series.17.p <==
  1502. T, P, O, F, O, F, N, T, S, F, T, F, E, N, S, N?
  1503.  
  1504. ==> series/series.18.p <==
  1505. 10, 11, 12, 13, 14, 15, 16, 17, 20, 22, 24, ___ , 100, 121, 10000
  1506.  
  1507. ==> series/series.19.p <==
  1508. 1 01 01011 0101101011011 0101101011011010110101101101011011 etc.
  1509.  
  1510. Each string is formed from the previous string by substituting '01' for '1'
  1511. and '011' for '0' simultaneously at each occurance.
  1512.  
  1513. ==> series/series.20.p <==
  1514. 1 2 5 16 64 312 1812 12288
  1515.  
  1516. ==> series/series.21.p <==
  1517. 5, 6, 5, 6, 5, 5, 7, 5, ?
  1518.  
  1519. ==> series/series.22.p <==
  1520. 3 1 1 0 3 7 5 5 2 ?
  1521.  
  1522. ==> series/series.23.p <==
  1523. 22 22 30 13 13 16 16 28 28 11 ?
  1524.  
  1525. ==> series/series.24.p <==
  1526. What is the next letter in the sequence: W, I, T, N, L, I, T?
  1527.  
  1528. ==> series/series.25.p <==
  1529. 1 3 4 9 10 12 13 27 28 30 31 36 37 39 40 ?
  1530.  
  1531. ==> series/series.26.p <==
  1532. 1 3 2 6 7 5 4 12 13 15 14 10 11 9 8 24 25 27 26 ?
  1533.  
  1534. ==> series/series.27.p <==
  1535. 0 1 1 2 1 2 1 3 2 2 1 3 1 2 2 4 1 3 1 3 2 2 1 4 2 ?
  1536.  
  1537. ==> series/series.28.p <==
  1538. 0 2 3 4 5 5 7 6 6 7 11 7 13 9 8 8 17 8 19 9 10 13 23 9 10 ?
  1539.  
  1540. ==> series/series.29.p <==
  1541. 1 1 2 1 2 2 3 1 2 2 3 2 3 3 4 1 2 2 3 2 3 3 4 2 3 3 4 3 4 ?
  1542.  
  1543. ==> series/series.30.p <==
  1544. I I T Y W I M W Y B M A D
  1545.  
  1546. ==> series/series.31.p <==
  1547. 6 2 5 5 4 5 6 3 7
  1548.  
  1549. ==> series/series.32.p <==
  1550. 0 1 1 0 1 0 0 1 1 0 0 1 0 1 1 0 1
  1551.  
  1552. ==> series/series.33.p <==
  1553. 2 12 360 75600
  1554.  
  1555. ==> series/series.34.p <==
  1556. 3 5 4 4 3 5 5 4 3
  1557.  
  1558. ==> series/series.35.p <==
  1559. 1 2 3 2 1 2 3 4 2 1 2 3 4 2 2 3
  1560.  
  1561. ==> trivia/area.codes.p <==
  1562. When looking at a map of the distribution of telephone area codes
  1563. for North America, it appears that they are randomly distributed.
  1564. I am doubtful that this is the case, however.  Does anyone know
  1565. how the area codes were/are chosen?
  1566.  
  1567. ==> trivia/eskimo.snow.p <==
  1568. How many words do the Eskimo have for snow?
  1569.  
  1570. ==> trivia/federal.reserve.p <==
  1571. What is the pattern to this list:
  1572. Boston, MA
  1573. New York, NY
  1574. Philadelphia, PA
  1575.  
  1576. ==> trivia/jokes.self-referential.p <==
  1577. What are some self-referential jokes?
  1578. ==> analysis/bugs.p <==
  1579. Four bugs are placed at the corners of a square. Each bug walks directly
  1580. toward the next bug in the clockwise direction. The bugs walk with
  1581. constant speed always directly toward their clockwise neighbor. Assuming
  1582. the bugs make at least one full circuit around the center of the square
  1583. before meeting, how much closer to the center will a bug be at the end
  1584. of its first full circuit?
  1585.  
  1586. ==> analysis/bugs.s <==
  1587. Amorous Bugs
  1588.  
  1589. ANSWER: 1 - e^(-2*pi)
  1590.  
  1591.         Let O(t) be the angle at time t of bug 1 relative to its starting
  1592.         point and r(O(t)) be its distance from the center of the square.
  1593.         Bug 1's vector trajectory is (using a Cartesian coordinate system with
  1594.         the origin at the center of the square):
  1595.                 (1) X1 = ▌r(O) * cos(O), r(O) * sin(O)¿
  1596.         By symmetry, bug 2's trajectory is the same only rotated by pi/2, viz.:
  1597.                 (2) X2 = ▌-r(O) * sin(O), r(O) * cos(O)¿
  1598.         Since bug 1 walks directly toward bug 2, the velocity of bug 1 must be
  1599.         proportional to the vector from bug 1 to bug 2:
  1600.                 (3) d(X1)/d(t) = k * (X2 - X1)
  1601.         Equating each component of the vector equation (3) yields:
  1602.                 (4) (d(r)/d(O) * cos(O) - r * sin(O)) * d(O)/d(t) =
  1603.                         k * (-r * cos(O) - r * sin(O))
  1604.                 (5) (d(r)/d(O) * sin(O) + r * cos(O)) * d(O)/d(t) =
  1605.                         k * (-r * sin(O) + r * cos(O))
  1606.         These equations are solved by:
  1607.                 (6) k = d(O)/d(t)
  1608.         and:
  1609.                 (7) d(r)/d(O) = -r(O)
  1610.         (7) is solved by:
  1611.                 (8) r(O) = e^-O
  1612.         Constant speed gives:
  1613.                 (9) v^2 = constant = ((d(r)/d(O))^2+r^2)*(d(O)/d(t))^2
  1614.         Substituting (8) into (9) yields (let V = v/sqrt(2)):
  1615.                 (10) d(O)/d(t) = V * e^O
  1616.         Which is solved (using the boundary condition O(0) = 0) by:
  1617.                 (11) O(t) = -ln(1 - V * t)
  1618.         Substituting (11) into (8) yields:
  1619.                 (12) r(t) = r(0) - V * t
  1620.         The bug has made a full circle when O(T) = 2*pi; using (11):
  1621.                 (13) T = 1/V * (1 - e^(-2*pi))
  1622.         Substituting T into (12) yields the answer:
  1623.                 (14) r(T) - r(0) = 1 - e^(-2*pi)
  1624.  
  1625. ==> analysis/c.infinity.p <==
  1626. What function is zero at zero, strictly positive elsewhere, infinitely
  1627. differentiable at zero and has all zero derivitives at zero?
  1628.  
  1629. ==> analysis/c.infinity.s <==
  1630. exp(-1/x^2)
  1631.  
  1632. This tells us why Taylor Series are a more limited device than they might be.
  1633. We form a Taylor series by looking at the derivatives of a function at a given
  1634. point; but this example shows us that the derivatives at a point may tell us
  1635. almost nothing about its behavior away from that point.
  1636.  
  1637. ==> analysis/cache.p <==
  1638. Cache and Ferry (How far can a truck go in a desert?)
  1639. A pick-up truck is in the desert beside N 50-gallon gas drums, all full.
  1640. The truck's gas tank holds 10 gallons and is empty.  The truck can carry
  1641. one drum, whether full or empty, in its bed.  It gets 10 miles to the gallon.
  1642. How far away from the starting point can you drive the truck?
  1643.  
  1644. ==> analysis/cache.s <==
  1645. If the truck can siphon gas out of its tank and leave it in the cache,
  1646. the answer is:
  1647.         { 1/1 + 1/3 + ... + 1/(2 * N - 1) } x 500 miles.
  1648.  
  1649. Otherwise, the "Cache and Ferry" problem is the same as the "Desert Fox"
  1650. problem described, but not solved, by Dewdney, July '87 "Scientific American".
  1651.  
  1652. Dewdney's Oct. '87 Sci. Am. article gives for N=2, the optimal distance
  1653. of 733.33 miles.
  1654.  
  1655. In the Nov. issue, Dewdney lists the optimal distance of 860 miles for
  1656. N=3, and gives a better, but not optimal, general distance formula.
  1657.  
  1658. Westbrook, in Vol 74, #467, pp 49-50, March '90 "Mathematical Gazette",
  1659. gives an even better formula, for which he incorrectly claims optimality:
  1660.  
  1661.   For N = 2,3,4,5,6:
  1662.      Dist = (600/1 + 600/3 + ... + 600/(2N-3))  +  (600-100N)/(2N-1)
  1663.   For N > 6:
  1664.      Dist = (600/1 + 600/3 + ... + 600/9)  +  (500/11 + ... + 500/(2N-3))
  1665.  
  1666. The following shows that Westbrook's formula is not optimal for N=8:
  1667.  
  1668.    Ferry  7 drums forward   33.3333 miles   (356.6667 gallons remain)
  1669.    Ferry  6 drums forward   51.5151 miles   (300.0000 gallons remain)
  1670.    Ferry  5 drums forward   66.6667 miles   (240.0000 gallons remain)
  1671.    Ferry  4 drums forward   85.7143 miles   (180.0000 gallons remain)
  1672.    Ferry  3 drums forward  120.0000 miles   (120.0000 gallons remain)
  1673.    Ferry  2 drums forward  200.0000 miles   ( 60.0000 gallons remain)
  1674.    Ferry  1 drums forward  600.0000 miles
  1675.                           ---------------
  1676.          Total distance = 1157.2294 miles
  1677.    (Westbrook's formula = 1156.2970 miles)
  1678.  
  1679.        ▌"Ferrying n drums forward x miles" involves (2*n-1) trips,
  1680.          each of distance x.¿
  1681.  
  1682. Other attainable values I've found:
  1683.    N      Distance
  1684.   ---    ---------  (Ferry distances for each N are omitted for brevity.)
  1685.     5    1016.8254
  1686.     7    1117.8355
  1687.    11    1249.2749
  1688.    13    1296.8939
  1689.    17    1372.8577
  1690.    19    1404.1136  (The N <= 19 distances could be optimal.)
  1691.    31    1541.1550  (I doubt that this N = 31 distance is optimal.)
  1692.   139    1955.5509  (I'm sure that this N = 139 distance is not optimal.)
  1693.  
  1694. So...where's MY formula?
  1695. I haven't found one, and believe me, I've looked.
  1696.  
  1697. I would be most grateful if someone would end my misery by mailing me
  1698. a formula, a literature reference, or even an efficient algorithm that
  1699. computes the optimal distance.
  1700.  
  1701. If you do come up with the solution, you might want to first check it
  1702. against the attainable distances listed above, before sending it out.
  1703. (Not because you might be wrong, but just as a mere formality to check
  1704.  your work.)
  1705.  
  1706. ▌Warning:  the Mathematician General has determined that
  1707.            this problem is as addicting as Twinkies.¿
  1708.  
  1709. Myron P. Souris      ! "If you have anything to tell me of importance,
  1710. McDonnell Douglas    !  for God's sake begin at the end."
  1711. souris@mdcbbs.com    !                            Sara Jeanette Duncan
  1712.  
  1713.  
  1714. @@@@@@@@@@@@@@@@@@@@@@@@@@@@@@@@@@@@@@@@@@@@@@@@@@@@@@@@@@@@@@@@@@@@@@@
  1715.  
  1716. The following output comes from some hack programs that I've used to
  1717. empirically verify some proofs I've been working on.
  1718.  
  1719. Initial barrels:   12 (600 gallons)
  1720. Attainable distance= 1274.175211
  1721.                   Barrels  Distance      Gas
  1722.                    Moved    covered     left
  1723. >From depot   1:      10     63.1579   480.0000
  1724. >From depot   2:       8     50.0000   405.0000
  1725. >From depot   3:       7     37.5000   356.2500
  1726. >From depot   4:       6     51.1364   300.0000
  1727. >From depot   5:       5     66.6667   240.0000
  1728. >From depot   6:       4     85.7143   180.0000
  1729. >From depot   7:       3    120.0000   120.0000
  1730. >From depot   8:       2    200.0000    60.0000
  1731. >From depot   9:       1    600.0000     0.0000
  1732.  
  1733.  
  1734. Initial barrels:   40 (2000 gallons)
  1735. Attainable distance= 1611.591484
  1736.                   Barrels  Distance      Gas
  1737.                    Moved    covered     left
  1738. >From depot   1:      40      2.5316  1980.0000
  1739. >From depot   2:      33     50.0000  1655.0000
  1740. >From depot   3:      28     50.0000  1380.0000
  1741. >From depot   4:      23     53.3333  1140.0000
  1742. >From depot   5:      19     50.0000   955.0000
  1743. >From depot   6:      16     56.4516   780.0000
  1744. >From depot   7:      13     50.0000   655.0000
  1745. >From depot   8:      11     54.7619   540.0000
  1746. >From depot   9:       9     50.0000   455.0000
  1747. >From depot  10:       8     32.1429   406.7857
  1748. >From depot  11:       7     38.9881   356.1012
  1749. >From depot  12:       6     51.0011   300.0000
  1750. >From depot  13:       5     66.6667   240.0000
  1751. >From depot  14:       4     85.7143   180.0000
  1752. >From depot  15:       3    120.0000   120.0000
  1753. >From depot  16:       2    200.0000    60.0000
  1754. >From depot  17:       1    600.0000     0.0000
  1755.  
  1756. ==> analysis/cats.and.rats.p <==
  1757. If 6 cats can kill 6 rats in 6 minutes, how many cats does it take to
  1758. kill one rat in one minute?
  1759.  
  1760. ==> analysis/cats.and.rats.s <==
  1761. The following piece by Lewis Carroll first appeared in ``The Monthly
  1762. Packet'' of February 1880 and is reprinted in _The_Magic_of_Lewis_Carroll_,
  1763. edited by John Fisher, Bramhall House, 1973.
  1764.  
  1765. /Larry Denenberg
  1766. larry@bbn.com
  1767. larry@harvard.edu
  1768.  
  1769.  
  1770.  
  1771.  
  1772.  
  1773.                                  Cats and Rats
  1774.  
  1775.    If 6 cats kill 6 rats in 6 minutes, how many will be needed to kill 100
  1776.    rats in 50 minutes?
  1777.  
  1778.    This is a good example of a phenomenon that often occurs in working
  1779.    problems in double proportion; the answer looks all right at first, but,
  1780.    when we come to test it, we find that, owing to peculiar circumstances in
  1781.    the case, the solution is either impossible or else indefinite, and needing
  1782.    further data.  The 'peculiar circumstance' here is that fractional cats or
  1783.    rats are excluded from consideration, and in consequence of this the
  1784.    solution is, as we shall see, indefinite.
  1785.  
  1786.    The solution, by the ordinary rules of Double Proportion, is as follows:
  1787.            6 rats   :  100 rats  \
  1788.                                   >   :: 6 cats : ans.
  1789.           50 min.   :    6 min.  /
  1790.          .
  1791.         . .  ans. = (100)(6)(6)/(50)(6) = 12
  1792.  
  1793.    But when we come to trace the history of this sanguinary scene through all
  1794.    its horrid details, we find that at the end of 48 minutes 96 rats are dead,
  1795.    and that there remain 4 live rats and 2 minutes to kill them in: the
  1796.    question is, can this be done?
  1797.  
  1798.    Now there are at least *four* different ways in which the original feat,
  1799.    of 6 cats killing 6 rats in 6 minutes, may be achieved.  For the sake of
  1800.    clearness let us tabulate them:
  1801.       A.  All 6 cats are needed to kill a rat; and this they do in one minute,
  1802.           the other rats standing meekly by, waiting for their turn.
  1803.       B.  3 cats are needed to kill a rat, and they do it in 2 minutes.
  1804.       C.  2 cats are needed, and do it in 3 minutes.
  1805.       D.  Each cat kills a rat all by itself, and take 6 minutes to do it.
  1806.  
  1807.    In cases A and B it is clear that the 12 cats (who are assumed to come
  1808.    quite fresh from their 48 minutes of slaughter) can finish the affair in
  1809.    the required time; but, in case C, it can only be done by supposing that 2
  1810.    cats could kill two-thirds of a rat in 2 minutes; and in case D, by
  1811.    supposing that a cat could kill one-third of a rat in two minutes.  Neither
  1812.    supposition is warranted by the data; nor could the fractional rats (even
  1813.    if endowed with equal vitality) be fairly assigned to the different cats.
  1814.    For my part, if I were a cat in case D, and did not find my claws in good
  1815.    working order, I should certainly prefer to have my one-third-rat cut off
  1816.    from the tail end.
  1817.  
  1818.    In cases C and D, then, it is clear that we must provide extra cat-power.
  1819.    In case C *less* than 2 extra cats would be of no use.  If 2 were supplied,
  1820.    and if they began killing their 4 rats at the beginning of the time, they
  1821.    would finish them in 12 minutes, and have 36 minutes to spare, during which
  1822.    they might weep, like Alexander, because there were not 12 more rats to
  1823.    kill.  In case D, one extra cat would suffice; it would kill its 4 rats in
  1824.    24 minutes, and have 24 minutes to spare, during which it could have killed
  1825.    another 4.  But in neither case could any use be made of the last 2
  1826.    minutes, except to half-kill rats---a barbarity we need not take into
  1827.    consideration.
  1828.  
  1829.    To sum up our results.  If the 6 cats kill the 6 rats by method A or B,
  1830.    the answer is 12; if by method C, 14; if by method D, 13.
  1831.  
  1832.    This, then, is an instance of a solution made `indefinite' by the
  1833.    circumstances of the case.  If an instance of the `impossible' be desired,
  1834.    take the following: `If a cat can kill a rat in a minute, how many would be
  1835.    needed to kill it in the thousandth part of a second?'  The *mathematical*
  1836.    answer, of course, is `60,000,' and no doubt less than this would *not*
  1837.    suffice; but would 60,000 suffice?  I doubt it very much.  I fancy that at
  1838.    least 50,000 of the cats would never even see the rat, or have any idea of
  1839.    what was going on.
  1840.  
  1841.    Or take this: `If a cat can kill a rat in a minute, how long would it be
  1842.    killing 60,000 rats?'  Ah, how long, indeed]  My private opinion is that
  1843.    the rats would kill the cat.
  1844.  
  1845.  
  1846.  
  1847. ==> analysis/e.and.pi.p <==
  1848. Which is greater, e^(pi) or (pi)^e ?
  1849.  
  1850. ==> analysis/e.and.pi.s <==
  1851. Put x = pi/e - 1 in the inequality e^x > 1+x  (x>0).
  1852.  
  1853. ==> analysis/functional/distributed.p <==
  1854.      Find all f: R -> R, f not identically zero, such that
  1855. (*)     f( (x+y)/(x-y) ) = ( f(x)+f(y) )/( f(x)-f(y) ).
  1856.  
  1857. ==> analysis/functional/distributed.s <==
  1858. 1)  Assuming f finite everywhere, (*) ==> x<>y ==> f(x)<>f(y)
  1859.  
  1860. 2)  Exchanging x and y in (*) we see that f(-x) = -f(x).
  1861.  
  1862. 3)  a <> 0 ==> f((a-a)/(a+a)) = (f(a)-f(a))/(f(a)+f(a)) ==> f(0) = 0.
  1863.  
  1864. 4)  a <> 0 ==> f((a+0)/(a-0)) = f(a)/f(a) ==> f(1) = 1.
  1865.  
  1866. 5)  x<>y, y<>0 ==> f(x/y) =
  1867. f( ((x+y)/(x-y) + (x-y)/(x-y)) / ((x+y)/(x-y) - (x-y)/(x-y)) = f(x)/f(y)
  1868. ==> f(xy) = f(x)f(y) by replacing x with xy and by noting that
  1869. f(x*1) = f(x)*1 and f(x*0) = f(x)*f(0).
  1870.  
  1871. 6)  f(x*x) = f(x)*f(x) ==> f(x) > 0 if x>0.
  1872.  
  1873. 7)  Let a=1+\/2, b=1-\/2; a,b satisfy (x+1)/(x-1) = x ==>
  1874. f(x) = (f(x)+1)/(f(x)-1) ==> f(a)=a, f(b)=b.  f(1/\/2) = f((a+b)/(a-b))
  1875. = (a+b)/(a-b) = 1/\/2 ==> f(2) = 2.
  1876.  
  1877. 8)  By induction and the relation f((n+1)/(n-1)) = (f(n)+1)/(f(n)-1)
  1878. we get that f(n)=n for all integer n.  #5 now implies that f fixes
  1879. the rationals.
  1880.  
  1881. 9)  If x>y>0 (*) ==> f(x) - f(y) = f(x+y)/f((x+y)/(x-y)) > 0 by #6.
  1882. Thus f is order-preserving.
  1883.  
  1884. Since f fixes the rationals *and* f is order-preserving, f must be the
  1885. identity function.
  1886.  
  1887. This was E2176 in _The American Mathematical Monthly_ (the proposer was
  1888. R. S. Luthar).
  1889.  
  1890. ==> analysis/functional/linear.p <==
  1891. Suppose f is non-decreasing with
  1892.   f(x+y) = f(x) + f(y) + C   for all real x, y.
  1893. Prove: there is a constant A such that f(x) = Ax - C  for all x.
  1894. (Note: continuity of f is not assumed in advance.)
  1895.  
  1896. ==> analysis/functional/linear.s <==
  1897. By induction f(mx) = m(f(x)+C)-C.  Let x=1/n, m=n and find that
  1898. f(1/n) = (1/n)(f(1)+C)-C.  Now let x=1/n and find that f(m/n) =
  1899. (m/n)(f(1)+C)-C.  f(-x+x) = f(-x) + f(x) + C ==> f(-x) = -2C - f(x)
  1900. (since f(0) = -C) ==> f(-m/n) = -(m/n)(f(1)+C)-C.  Since f is
  1901. monotonic ==> f(x) = x*(f(1)+C)-C for all real x (Squeeze Theorem).
  1902.  
  1903. ==> analysis/integral.p <==
  1904. If f is integrable on (0,inf), and differentiable at 0, and a > 0, show:
  1905.  
  1906.  
  1907.                   inf     ( f(x) - f(ax) )
  1908.                Int        ----------------  dx   = f(0) ln(a)
  1909.                    0             x
  1910.  
  1911. ==> analysis/integral.s <==
  1912. First, note that if f(0) is 0, then by substituting u=ax in
  1913. the integral of f(x)/x, our integral is the difference of two
  1914. equal integrals and so is 0 (the integrals are finite because f is
  1915. 0 at 0 and differentiable there.  Note I make no requirement of
  1916. continuity).
  1917.  
  1918. Second, note that if f is the characteristic function of the
  1919. interval ▌0, 1¿--- i.e.
  1920.  
  1921.                 1, 0<=x<=1
  1922.         f (x) =
  1923.                 0 otherwise
  1924.  
  1925. then a little arithmetic reduces our integral to that of
  1926. 1/x from 1/a to 1 (assuming a>1; if a <= 1 the reasoning is similar),
  1927. which is ln(a) = f(0)ln(a) as required.  Call this function g.
  1928.  
  1929. Finally, note that the operator which takes the function f to the
  1930. value of our integral is linear, and that every function meeting the
  1931. hypotheses (incidentally, I should have said `differentiable from the right',
  1932. or else replaced the characteristic function of ▌0,1¿ above by that of
  1933. (-infinity, 1¿; but it really doesn't matter) is a linear combination of
  1934. one which is 0 at 0 and g, to wit
  1935.  
  1936.         f(x) = f(0)g(x) + (f(x) - g(x)f(0)).
  1937.  
  1938. ==> analysis/period.p <==
  1939. What is the least possible integral period of the sum of functions
  1940. of periods 3 and 6?
  1941.  
  1942. ==> analysis/period.s <==
  1943. Period 2.  Clearly, the sum of periodic functions of periods 2 and
  1944. three is 6.  So take the function which is the sum of that function of
  1945. period six and the negative of the function of period three and you
  1946. have a function of period 2.
  1947.  
  1948. ==> analysis/rubberband.p <==
  1949. A bug walks down a rubberband which is attached to a wall at one end and a car
  1950. moving away from the wall at the other end. The car is moving at 1 m/sec while
  1951. the bug is only moving at 1 cm/sec. Assuming the rubberband is uniformly and
  1952. infinitely elastic, will the bug ever reach the car?
  1953.  
  1954. ==> analysis/rubberband.s <==
  1955. Let w = speed of bug and N = ratio of car speed/bug speed = 100.  Paint N+1
  1956. equally spaced stripes on the rubberband.  When the bug is standing on one
  1957. stripe, the next stripe is moving away from him at a speed slightly < w
  1958. (relative to him).  Since he is walking at w, clearly the bug can reach
  1959. the next stripe.  But once he reaches that stripe, the next one is only
  1960. receeding at < w.  So he walks on down to the car, one stripe at a time.
  1961.  
  1962. The bug starts gaining on the car when he is at the next to last stripe.
  1963.  
  1964. ==> analysis/series.p <==
  1965. Show that in the series: x, 2x, 3x, .... (n-1)x (x can be any real number)
  1966. there is at least one number which is within 1/n of an integer.
  1967.  
  1968. ==> analysis/series.s <==
  1969. Throw 0 into the sequence; there are now n numbers, so some pair must
  1970. have fractional parts within 1/n of each other; their difference is
  1971. then within 1/n of an integer.
  1972.  
  1973. ==> analysis/snow.p <==
  1974. Snow starts falling before noon on a cold December day.
  1975. At noon a snowplow starts plowing a street.
  1976. It travels 1 mile in the first hour, and 1/2 mile in the second hour.
  1977. What time did the snow start falling??
  1978.  
  1979. You may assume that the plow's rate of travel is inversely proportioned
  1980. to the height of the snow, and that the snow falls at a uniform rate.
  1981.  
  1982. ==> analysis/snow.s <==
  1983. 11:22:55.077 am.
  1984.  
  1985. Method:
  1986.  
  1987. Let b = the depth of the snow at noon, a = the rate of increase in the
  1988. depth.  Then the depth at time t (where noon is t=0) is at+b, the
  1989. snowfall started at t_0=-b/a, and the snowplow's rate of progress is
  1990. ds/dt = k/(at+b).
  1991.  
  1992. If the snowplow starts at s=0 then s(t) = (k/a) log(1+at/b).  Note that
  1993. s(2 hours) = 1.5 s(1 hour), or  log(1+2A/b) = 1.5 log(1+A/b), where
  1994. A = (1 hour)*a.  Letting x = A/b we have (1+2x)^2 = (1+x)^3.  Solve for
  1995. x and t_0 = -(1 hour)/x.
  1996.  
  1997. The exact answer is 11:(90-30 Sqrt▌5¿).
  1998.  
  1999. _American Mathematics Monthly_, April 1937, page 245
  2000. E 275.  Proposed by J. A. Benner, Lafayette College, Easton. Pa.
  2001.  
  2002. The solution appears, appropriately, in the December 1937 issue,
  2003. pp. 666-667.  Also solved by William Douglas, C. E. Springer,
  2004. E. P. Starke, W. J.  Taylor, and the proposer.
  2005.  
  2006. See R.P. Agnew, "Differential Equations," 2nd edition, p. 39 ff.
  2007.  
  2008. ==> analysis/tower.p <==
  2009. A number is raised to its own power. The same number is then raised to
  2010. the power of this result. The same number is then raised to the power
  2011. of this second result. This process is continued forever. What is the
  2012. maximum number which will yield a finite result from this process?
  2013.  
  2014. ==> analysis/tower.s <==
  2015. Tower of Exponentials
  2016.  
  2017. ANSWER: e^(1/e)
  2018.  
  2019.         Let N be the number in question and R the result of the process. Then
  2020.         R can be defined recursively by the equation:
  2021.                 (1) R = N^R
  2022.         Taking the logarithm of both sides of (1):
  2023.                 (2) ln(R) = R * ln(N)
  2024.         Dividing (2) by R and rearranging:
  2025.                 (3) ln(N) = ln(R) / R
  2026.         Exponentiating (3):
  2027.                 (4) N = R^(1/R)
  2028.         We wish to find the maximum value of N with respect to R. Find the
  2029.         derivative of N with respect to R and set it equal to zero:
  2030.                 (5) d(N)/d(R) = (1 - ln(R)) / R^2 = 0
  2031.         For finite values of R, (5) is satisfied by R = e. This is a maximum of
  2032.         N if the second derivative of N at R = e is less than zero.
  2033.                 (6) d2(N)/d2(R) ! R=e = (2 * ln(R) - 3) / R^3 ! R=e = -1 / e^3 < 0
  2034.         The solution therefore is (4) at R = e:
  2035.                 (7) Nmax = e^(1/e)
  2036.  
  2037. ==> arithmetic/7-11.p <==
  2038. A customer at a 7-11 store selected four items to buy, and was told
  2039. that the cost was $7.11.  He was curious that the cost was the same
  2040. as the store name, so he inquired as to how the figure was derived.
  2041. The clerk said that he had simply multiplied the prices of the four
  2042. individual  items.  The customer  protested  that  the four  prices
  2043. should have been ADDED,  not MULTIPLIED.  The  clerk said that that
  2044. was OK with him, but, the result was still the same: exactly $7.11.
  2045.  
  2046. What were the prices of the four items?
  2047.  
  2048. ==> arithmetic/7-11.s <==
  2049. The prices are: $1.20, $1.25, $1.50, and $3.16
  2050.  
  2051. $7.11 is not the only number which works.  Here are the first 160 such
  2052. numbers, preceded by a count of distinct solutions for that price.
  2053. Note that $7.11 has a single, unique solution.
  2054.  
  2055.        1 -  $6.44      1 -  $7.83      2 -  $9.20      3 - $10.89
  2056.        1 -  $6.51      1 -  $7.86      1 -  $9.23      1 - $10.95
  2057.        1 -  $6.60      3 -  $7.92      1 -  $9.24      2 - $11.00
  2058.        1 -  $6.63      1 -  $8.00      1 -  $9.27      1 - $11.07
  2059.        1 -  $6.65      1 -  $8.01      2 -  $9.35      1 - $11.13
  2060.        1 -  $6.72      1 -  $8.03      3 -  $9.36      1 - $11.16
  2061.        2 -  $6.75      5 -  $8.10      1 -  $9.38      1 - $11.22
  2062.        1 -  $6.78      1 -  $8.12      5 -  $9.45      2 - $11.25
  2063.        1 -  $6.80      1 -  $8.16      2 -  $9.48      2 - $11.27
  2064.        2 -  $6.84      2 -  $8.19      1 -  $9.54      1 - $11.30
  2065.        1 -  $6.86      1 -  $8.22      1 -  $9.57      1 - $11.36
  2066.        1 -  $6.89      1 -  $8.25      1 -  $9.59      1 - $11.40
  2067.        2 -  $6.93      3 -  $8.28      2 -  $9.60      2 - $11.43
  2068.        1 -  $7.02      3 -  $8.33      1 -  $9.62      2 - $11.52
  2069.        1 -  $7.05      1 -  $8.36      2 -  $9.63      2 - $11.55
  2070.        2 -  $7.07      1 -  $8.37      1 -  $9.66      2 - $11.61
  2071.        1 -  $7.08      2 -  $8.40      1 -  $9.68      1 - $11.69
  2072.        1 -  $7.11      1 -  $8.45      2 -  $9.69      1 - $11.70
  2073.        1 -  $7.13      2 -  $8.46      1 -  $9.78      1 - $11.88
  2074.        2 -  $7.14      1 -  $8.52      2 -  $9.80      1 - $11.90
  2075.        3 -  $7.20      5 -  $8.55      1 -  $9.81      1 - $11.99
  2076.        1 -  $7.25      1 -  $8.60      1 -  $9.87      1 - $12.06
  2077.        1 -  $7.26      4 -  $8.64      4 -  $9.90      1 - $12.15
  2078.        2 -  $7.28      1 -  $8.67      1 -  $9.92      1 - $12.18
  2079.        1 -  $7.29      1 -  $8.69      2 -  $9.99      1 - $12.24
  2080.        3 -  $7.35      1 -  $8.73      1 - $10.01      1 - $12.30
  2081.        1 -  $7.37      2 -  $8.75      1 - $10.05      1 - $12.32
  2082.        1 -  $7.47      1 -  $8.76      2 - $10.08      1 - $12.35
  2083.        1 -  $7.50      1 -  $8.78      1 - $10.17      2 - $12.42
  2084.        1 -  $7.52      5 -  $8.82      1 - $10.20      1 - $12.51
  2085.        4 -  $7.56      1 -  $8.85      2 - $10.26      1 - $12.65
  2086.        1 -  $7.62      1 -  $8.88      3 - $10.29      2 - $12.69
  2087.        4 -  $7.65      2 -  $8.91      3 - $10.35      1 - $12.75
  2088.        1 -  $7.67      1 -  $8.94      2 - $10.44      1 - $12.92
  2089.        2 -  $7.70      1 -  $8.96      1 - $10.53      1 - $12.96
  2090.        3 -  $7.74      3 -  $9.00      1 - $10.56      1 - $13.23
  2091.        1 -  $7.77      1 -  $9.02      1 - $10.64      1 - $13.41
  2092.        1 -  $7.79      2 -  $9.03      2 - $10.71      1 - $13.56
  2093.        2 -  $7.80      1 -  $9.12      3 - $10.80      1 - $14.49
  2094.        1 -  $7.82      2 -  $9.18      1 - $10.85      1 - $15.18
  2095.  
  2096.  
  2097. There are plenty of solutions for five summands.  Here are a few:
  2098.  
  2099.          $8.28  -- at least two solutions
  2100.          $8.47  -- at least two solutions
  2101.          $8.82  -- at least two solutions
  2102. --
  2103.      Mark Johnson       mark@microunity.com       (408) 734-8100
  2104.  
  2105. There may be many approximate solutions, for example: $1.01, $1.15, $2.41,
  2106. and $2.54.  These sum to $7.11 but the product is 7.1100061.
  2107.  
  2108. ==> arithmetic/clock/day.of.week.p <==
  2109. It's restful sitting in Tom's cosy den, talking quietly and sipping
  2110. a glass of his Madeira.
  2111.  
  2112. I was there one Sunday and we had the usual business of his clock.
  2113. When the radio gave the time at the hour, the Ormolu antique was
  2114. exactly 3 minutes slow.
  2115.  
  2116. "It loses 7 minutes every hour", my old friend told me, as he had done
  2117. so many times before.  "No more and no less, but I've gotten used to
  2118. it that way."
  2119.  
  2120. When I spent a second evening with him later that same month, I remarked
  2121. on the fact that the clock was dead right by radio time at the hour.
  2122. It was rather late in the evening, but Tom assured me that his treasure
  2123. had not been adjusted nor fixed since my last visit.
  2124.  
  2125. What day of the week was the second visit?
  2126.  
  2127. From "Mathematical Diversions" by Hunter + Madachy
  2128.  
  2129. ==> arithmetic/clock/day.of.week.s <==
  2130. The answer is 17 days and 3 hours later, which would have been a Wednesday.
  2131. This is the only other time in the same month when the two would agree at all.
  2132.  
  2133. In 17 days the slow clock loses 17*24*7 minutes = 2856 minutes,
  2134. or 47 hours and 36 minutes.  In 3 hours more it loses 21 minutes, so
  2135. it has lost a total of 47 hours and 57 minutes.  Modulo 12 hours, it
  2136. has *gained* 3 minutes so as to make up the 3 minutes it was slow on
  2137. Sunday.  It is now (fortnight plus 3 days) exactly accurate.
  2138.  
  2139. ==> arithmetic/clock/thirds.p <==
  2140. Do the 3 hands on a clock ever divide the face of the clock into 3
  2141. equal segments, i.e. 120 degrees between each hand?
  2142.  
  2143. ==> arithmetic/clock/thirds.s <==
  2144. First let us assume that our clock has 60 divisions.  We will show that
  2145. any time the hour hand and the minute hand are 20 divisions (120 degrees)
  2146. apart, the second hand cannot be an integral number of divisions from the
  2147. other hands, unless it is straight up (on the minute).
  2148.  
  2149. Let us use h for hours, m for minutes, and s for seconds.
  2150. We will use =n to mean congruent mod n, thus 12 =5 7.
  2151.  
  2152. We know that m =60 12h, that is, the minute hand moves 12 times as fast
  2153. as the hour hand, and wraps around at 60.
  2154. We also have s =60 60m. This simplifies to s/60 =1 m, which goes to
  2155. s/60 = frac(m) (assuming s is in the range 0 <= s < 60), which goes to
  2156. s = 60 frac(m).  Thus, if m is 5.5, s is 30.
  2157.  
  2158. Now let us assume the minute hand is 20 divisions ahead of the hour hand.
  2159. So m =60 h + 20, thus 12h =60 h + 20, 11h =60 20, and, finally,
  2160. h =60/11 20/11 (read 'h is congruent mod 60/11 to 20/11').
  2161. So all values of m are k + n/11 for some integral k and integral n,
  2162. 0 <= n < 11.  s is therefore 60n/11.  If s is to be an integral number of
  2163. units from m and h, we must have 60n =11 n.  But 60 and 11 are relatively
  2164. prime, so this holds only for n = 0.  But if n = 0, m is integral, so
  2165. s is 0.
  2166.  
  2167. Now assume, instead, that the minute hand is 20 divisions behind the hour hand.
  2168. So m =60 h - 20, 12h =60 h - 20, 11h =60 -20, h =60/11 -20/11.
  2169. So m is still k + n/11.  Thus s must be 0.
  2170.  
  2171. But if s is 0, h must be 20 or 40.  But this translates to 4 o'clock or
  2172. 8 o'clock, at both of which the minute hand is at 0, along with the second
  2173. hand.
  2174.  
  2175. Thus the 3 hands can never be 120 degrees apart, Q.E.D.
  2176.  
  2177. ==> arithmetic/consecutive.product.p <==
  2178. Prove that the product of three or more consecutive natural numbers cannot be a
  2179. perfect square.
  2180.  
  2181. ==> arithmetic/consecutive.product.s <==
  2182. Three consecutive numbers:
  2183.   If a and b are relatively prime, and ab is a square,
  2184.   then a and b are squares. (This is left as an exercise.)
  2185.  
  2186.   Suppose (n - 1)n(n + 1) = k^2, where n > 1.
  2187.   Then n(n^2 - 1) = k^2.  But n and (n^2 - 1) are relatively prime.
  2188.   Therefore n^2 - 1 is a perfect square, which is a contradiction.
  2189.  
  2190. Four consecutive numbers:
  2191.   n(n + 1)(n + 2)(n + 3) = (n^2 + 3n + 1)^2 - 1
  2192.  
  2193. Five consecutive numbers:
  2194.   Assume the product is a integer square, call it m.
  2195.  
  2196.   The prime factorization of m must have even numbers of each prime factor.
  2197.  
  2198.   For each prime factor, p, of m, p >= 5, p^2k must divide one of the
  2199. consecutive naturals in the product.  (Otherwise, the difference between two
  2200. of the naturals in the product would be a positive multiple of a prime >= 5.
  2201. But in this problem, the greatest difference is 4.) So we need only consider
  2202. the primes 2 and 3.
  2203.  
  2204.   Each of the consecutive naturals is one of:
  2205.         1)      a perfect square
  2206.         2)      2 times a perfect square
  2207.         3)      3 times a perfect square
  2208.         4)      6 times a perfect square.
  2209.  
  2210.   By the shoe box principle, two of the five consecutive numbers must fall into
  2211. the same category.
  2212.  
  2213.   If there are two perfect squares, then their difference being less than five
  2214. limits their values to be 1 and 4.  (0 is not a natural number, so 0 and 1
  2215. and 0 and 4 cannot be the perfect squares.)  But 1*2*3*4*5=120]=x*x where x
  2216. is an integer.
  2217.  
  2218.   If there are two numbers that are 2 times a perfect square, then their
  2219. difference being less than five implies that the perfect squares (which are
  2220. multiplied by 2) are less than 3 apart, and no two natural squares differ by
  2221. only 1 or 2.
  2222.  
  2223.   A similar argument holds for two numbers which are 3 times a perfect square.
  2224.  
  2225.   We cannot have the case that two of the 5 consecutive numbers are multiples
  2226. (much less square multiples) of 6, since their difference would be >= 6, and
  2227. our span of five consecutive numbers is only 4.
  2228.  
  2229.   Therefore the assumption that m is a perfect square does not hold.
  2230.  
  2231.   QED.
  2232.  
  2233. In general the equation:
  2234.  
  2235. y^2 = x(x+1)(x+2)...(x+n),    n > 3
  2236.  
  2237. has only the solution corresponding to y = 0.
  2238.  
  2239. This is a theorem of Rigge ▌O. Rigge, ``Uber ein diophantisches Problem'',
  2240. IX Skan. Math. Kong. Helsingfors (1938)¿ and Erdos ▌P. Erdos, ``Note on
  2241. products of consecutive integers,'' J. London Math. Soc. #14 (1939),
  2242. pages 194-198¿.
  2243.  
  2244. A proof can be found on page 276 of ▌L. Mordell, ``Diophantine
  2245. Equations'', Academic Press 1969¿.
  2246.  
  2247. ==> arithmetic/consecutive.sums.p <==
  2248. Find all series of consecutive positive integers whose sum is exactly 10,000.
  2249.  
  2250. ==> arithmetic/consecutive.sums.s <==
  2251. Generalize to find X (and I) such that
  2252.     (X + X+1 + X+2 + ... + X+I) = T
  2253. for any integer T.
  2254.  
  2255. You are asking for all (X,I) s.t. (2X+I)(I+1) = 2T.  The problem is
  2256. (very) slightly easier if we don't restrict X to being positive, so
  2257. we'll solve this first.
  2258.  
  2259. Note that 2X+I and I+1 must have different parities, so the answer
  2260. to the relaxed question is N = 2*(o_1+1)*(o_2+1)*...*(o_n+1), where
  2261. 2T = 2^o_0*3^o_1*...*p_n^o_n (the prime factorization); this is easily
  2262. seen to be the number of ways we can break 2T up into two positive
  2263. factors of differing parity (with order).
  2264.  
  2265. In particular, 20000 = 2^5*5^4, hence there are 2*(4+1) = 10 solutions
  2266. for T = 10000.  These are (2X+I,I+1):
  2267.  
  2268. (32*1,5^4)   (32*5,5^3)   (32*5^2,5^2)   (32*5^3,5)   (32*5^4,1)
  2269. (5^4,32*1)   (5^3,32*5)   (5^2,32*5^2)   (5,32*5^3)   (1,32*5^4)
  2270.  
  2271. And they give rise to the solutions (X,I):
  2272.  
  2273. (-296,624)   (28,124)   (388,24)   (1998,4)     (10000,0)
  2274. (297,31)     (-27,179)  (-387,799) (-1997,3999) (-9999,19999)
  2275.  
  2276. If you require that X>0 note that this is true iff 2X+I > I+1 and
  2277. hence the number of solutions to this problem is N/2 (due to the
  2278. symmetry of the above ordered pairs).
  2279.  
  2280. ==> arithmetic/digits/all.ones.p <==
  2281. Prove that some multiple of any integer ending in 3 contains all 1s.
  2282.  
  2283. ==> arithmetic/digits/all.ones.s <==
  2284. Let n be our integer; one such desired multiple is then
  2285. ( 10^(phi(n))-1 )/9.  All we need is that (n,10) = 1, and
  2286. if the last digit is 3 this must be the case.  A different
  2287. proof using the pigeonhole principle is to consider the sequence
  2288. 1, 11, 111, ..., (10^n - 1)/9.  By previous reasoning we must
  2289. have at some point that either some member of our sequence = 0 (mod n)
  2290. or else some value (mod n) is duplicated.  Assume the latter, with
  2291. x_a and x_b, x_b>x_a,  possesing the duplicated remainders.  We then
  2292. have that x_b - x-a = 0 (mod n).  Let m be the highest power of 10
  2293. dividing x_b - x_a.  Now since (10,n) = 1, we can divide by 10^m and
  2294. get that (x_b - x_a)/10^m = 0 (n).  But (x_b - x_a)/10^m is a number
  2295. containing only the digit 1.
  2296.  
  2297. Q.E.D.
  2298.  
  2299. ==> arithmetic/digits/arabian.p <==
  2300. What is the Arabian Nights factorial, the number x such that x] has 1001
  2301. digits?  How about the prime x such that x] has exactly 1001 zeroes on
  2302. the tail end.  (Bonus question, what is the 'rightmost' non-zero digit in x]?)
  2303.  
  2304. ==> arithmetic/digits/arabian.s <==
  2305. The first answer is 450].
  2306.  
  2307. Determining the number of zeroes at the end of x] is relatively easy once
  2308. you realize that each such zero comes from a factor of 10 in the product
  2309.  
  2310.    1 * 2 * 3 * ... * x
  2311.  
  2312. Each factor of 10, in turn, comes from a factor of 5 and a factor of 2.
  2313. Since there are many more factors of 2 than factors of 5, the number of 5s
  2314. determines the number of zeroes at the end of the factorial.
  2315.  
  2316. The number of 5s in the set of numbers 1 .. x (and therefore the number
  2317. of zeroes at the end of x]) is:
  2318.  
  2319.   z(x) = int(x/5) + int(x/25) + int(x/125) + int(x/625) + ...
  2320.  
  2321. This series terminates when the powers of 5 exceed x.
  2322.  
  2323. I know of no simple way to invert the above formula (i.e., to find x for
  2324. a given z(x)), but I can approximate it by noting that, except for the "int"
  2325. function,
  2326.  
  2327.    5*z(x) - x = z(x)
  2328.  
  2329. which gives:
  2330.  
  2331.    x = 4*z(x) (approximately).
  2332.  
  2333. The given problem asked, "For what prime x is z(x)=1001".  By the above forumla,
  2334. this is approximately 4*1001=4004.  However, 4004] has only
  2335.  
  2336.   800 + 160 + 32 + 6 + 1 = 999 zeroes at the end of it.
  2337.  
  2338. The numbers 4005] through 4009] all have 1000 zeroes at their end and
  2339. the numbers 4010] through 4014] all have 1001 zeroes at their end.
  2340.  
  2341. Since the problem asked for a prime x, and 4011 = 3*7*191, the only solution
  2342. is x=4013.
  2343.  
  2344. The problem of determining the rightmost nonzero digit in x] is somewhat more
  2345. difficult.  If we took the numbers 1, 2, ... , x and removed all factors of 5
  2346. (and an equal number of factors of 2), the remaining numbers multiplied
  2347. together modulo 10 would be the answer.  Note that since there are still many
  2348. factors of 2 left, the rightmost nonzero digit must be 2, 4, 6, or 8 (x > 1).
  2349.  
  2350. Letting r(x) be the rightmost nonzero digit in x], an expression for r(x) is:
  2351.  
  2352.   r(x) = (r(int(x/5)) * w * r(x mod 10)) mod 10, x >= 10.
  2353.  
  2354. where w is 4 if int(x/10) is odd and 6 if it is even.
  2355.  
  2356. The values of r(x) for 0 <= x <= 9 are 1, 1, 2, 6, 4, 2, 2, 4, 2, and 8.
  2357.  
  2358. The way to see this is true is to take the numbers 1, 2, ..., x in groups
  2359. of 10.  In each group, remove 2 factors of 10.  For example, from the
  2360. set 1, 2, ..., 10, choose a factor of 2 from 2 and 6 and a factor of 5 from
  2361. 5 and 10.  This leaves 1, 1, 3, 4, 1, 3, 7, 8, 9, 2.  Next, separate all the
  2362. factors that came from multiples of 5.  The rightmost nonzero digit of x]
  2363. can now (hopefully) be seen to be:
  2364.  
  2365.   r(x) = (r(int(x/5)) * w * r(x mod 10)) mod 10
  2366.  
  2367. where w is the rightmost digit in the number formed by multiplying the numbers
  2368. 1, 2, 3, ..., 10*int(x/10) after the factors of 10 and the factors left over
  2369. by the multiples of 5 have been removed.  In the example with x = 10, this
  2370. would be (1 * 1 * 3 * 4 * 3 * 7 * 8 * 9) mod 10 = 4.  The "r(x mod 10)" term
  2371. takes care of the numbers from 10*int(x/10)+1 up to x.
  2372.  
  2373. The "w" term can be seen to be 4 or 6 depending on whether int(x/10) is odd or
  2374. even since, after removing 10*n+5 and 10*n+10 and a factor of 2 each from
  2375. 10*n+2 and 10*n+6 from the group of numbers 10*n+1 through 10*n+10, the
  2376. remaining factors (mod 10) always equals 4 and 4^t mod 10 = 4 if t is odd and
  2377. 6 when t is even (t ]= 0).
  2378.  
  2379. So, finally, the rightmost nonzero digit in 4013] is found as follows:
  2380.  
  2381.   r(4013) = (r(802) * 4 * 6) mod 10
  2382.   r(802)  = (r(160) * 6 * 2) mod 10
  2383.   r(160)  = (r(32)  * 6 * 1) mod 10
  2384.   r(32)   = (r(6)   * 4 * 2) mod 10
  2385.  
  2386. Using a table of r(x) for 0 <= x <= 9, r(6) = 2.  Then,
  2387.  
  2388.   r(32)   = (2 * 4 * 2) mod 10 = 6
  2389.   r(160)  = (6 * 6 * 1) mod 10 = 6
  2390.   r(802)  = (6 * 6 * 2) mod 10 = 2
  2391.   r(4013) = (2 * 4 * 6) mod 10 = 8
  2392.  
  2393. Thus, the rightmost nonzero digit in 4013] is 8.
  2394.  
  2395. ==> arithmetic/digits/circular.p <==
  2396. What 6 digit number, with 6 different digits, when multiplied by all integers
  2397. up to 6, circulates its digits through all 6 possible positions, as follows:
  2398. ABCDEF * 1 = ABCDEF
  2399. ABCDEF * 3 = BCDEFA
  2400. ABCDEF * 2 = CDEFAB
  2401. ABCDEF * 6 = DEFABC
  2402. ABCDEF * 4 = EFABCD
  2403. ABCDEF * 5 = FABCDE
  2404.  
  2405. ==> arithmetic/digits/circular.s <==
  2406. ABCDEF=142857 (the digits of the expansion of 1/7).
  2407.  
  2408. ==> arithmetic/digits/divisible.p <==
  2409. Find the least number using 0-9 exactly once that is evenly divisible by each
  2410. of these digits?
  2411.  
  2412. ==> arithmetic/digits/divisible.s <==
  2413. Since the sum of the digits is 45, any permutation of the digits gives a
  2414. multiple of 9.  To get a multiple of both 2 and 5, the last digit must
  2415. be 0, and thus to get a multiple of 8 (and 4), the tens digit must be
  2416. even, and the hundreds digit must be odd if the tens digit is 2 or 6,
  2417. and even otherwise.  The number will also be divisible by 6, since it is
  2418. divisible by 2 and 3, so 7 is all we need to check.  First, we will look
  2419. for a number whose first five digits are 12345; now, 1234500000 has a
  2420. remainder of 6 when divided by 7, so we have to arrange the remaining
  2421. digits to get a remainder of 1.  The possible arrangements, in
  2422. increasing order, are
  2423.  
  2424. 78960, remainder 0
  2425. 79680, remainder 6
  2426. 87960, remainder 5
  2427. 89760, remainder 6
  2428. 97680, remainder 2
  2429. 98760, remainder 4
  2430.  
  2431. That didn't work, so try numbers starting with 12346; this is impossible
  2432. because the tens digit must be 8, and the hundreds digit cannot be even.
  2433. Now try 12347, and 1234700000 has remainder 2.  The last five digits can
  2434. be
  2435.  
  2436. 58960, remainder 6
  2437. 59680, remainder 5, so this works, and the number is
  2438.  
  2439. 1234759680.
  2440.  
  2441. ==> arithmetic/digits/equations/123456789.p <==
  2442. In how many ways can "." be replaced with "+", "-", or "" (concatenate) in
  2443. .1.2.3.4.5.6.7.8.9=1 to form a correct equation?
  2444.  
  2445. ==> arithmetic/digits/equations/123456789.s <==
  2446.  1-2 3+4 5+6 7-8 9 = 1
  2447. +1-2 3+4 5+6 7-8 9 = 1
  2448.  1+2 3+4-5+6 7-8 9 = 1
  2449. +1+2 3+4-5+6 7-8 9 = 1
  2450. -1+2 3-4+5+6 7-8 9 = 1
  2451.  1+2 3-4 5-6 7+8 9 = 1
  2452. +1+2 3-4 5-6 7+8 9 = 1
  2453.  1-2 3-4+5-6 7+8 9 = 1
  2454. +1-2 3-4+5-6 7+8 9 = 1
  2455.  1-2-3-4 5+6 7-8-9 = 1
  2456. +1-2-3-4 5+6 7-8-9 = 1
  2457.  1+2-3 4+5 6-7-8-9 = 1
  2458. +1+2-3 4+5 6-7-8-9 = 1
  2459. -1+2 3+4+5-6-7-8-9 = 1
  2460. -1 2+3 4-5-6+7-8-9 = 1
  2461.  1+2+3+4-5+6+7-8-9 = 1
  2462. +1+2+3+4-5+6+7-8-9 = 1
  2463. -1+2+3-4+5+6+7-8-9 = 1
  2464.  1-2-3+4+5+6+7-8-9 = 1
  2465. +1-2-3+4+5+6+7-8-9 = 1
  2466.  1+2 3+4 5-6 7+8-9 = 1
  2467. +1+2 3+4 5-6 7+8-9 = 1
  2468.  1+2 3-4-5-6-7+8-9 = 1
  2469. +1+2 3-4-5-6-7+8-9 = 1
  2470.  1+2+3+4+5-6-7+8-9 = 1
  2471. +1+2+3+4+5-6-7+8-9 = 1
  2472. -1+2+3+4-5+6-7+8-9 = 1
  2473.  1-2+3-4+5+6-7+8-9 = 1
  2474. +1-2+3-4+5+6-7+8-9 = 1
  2475. -1-2-3+4+5+6-7+8-9 = 1
  2476.  1-2+3+4-5-6+7+8-9 = 1
  2477. +1-2+3+4-5-6+7+8-9 = 1
  2478.  1+2-3-4+5-6+7+8-9 = 1
  2479. +1+2-3-4+5-6+7+8-9 = 1
  2480. -1-2+3-4+5-6+7+8-9 = 1
  2481. -1+2-3-4-5+6+7+8-9 = 1
  2482. -1+2 3+4 5-6 7-8+9 = 1
  2483.  1-2 3-4 5+6 7-8+9 = 1
  2484. +1-2 3-4 5+6 7-8+9 = 1
  2485. -1+2 3-4-5-6-7-8+9 = 1
  2486. -1+2+3+4+5-6-7-8+9 = 1
  2487.  1-2+3+4-5+6-7-8+9 = 1
  2488. +1-2+3+4-5+6-7-8+9 = 1
  2489.  1+2-3-4+5+6-7-8+9 = 1
  2490. +1+2-3-4+5+6-7-8+9 = 1
  2491. -1-2+3-4+5+6-7-8+9 = 1
  2492.  1+2-3+4-5-6+7-8+9 = 1
  2493. +1+2-3+4-5-6+7-8+9 = 1
  2494. -1-2+3+4-5-6+7-8+9 = 1
  2495. -1+2-3-4+5-6+7-8+9 = 1
  2496.  1-2-3-4-5+6+7-8+9 = 1
  2497. +1-2-3-4-5+6+7-8+9 = 1
  2498.  1-2 3+4+5+6+7-8+9 = 1
  2499. +1-2 3+4+5+6+7-8+9 = 1
  2500.  1+2+3+4 5-6 7+8+9 = 1
  2501. +1+2+3+4 5-6 7+8+9 = 1
  2502.  1 2+3 4+5-6 7+8+9 = 1
  2503. +1 2+3 4+5-6 7+8+9 = 1
  2504.  1+2+3-4-5-6-7+8+9 = 1
  2505. +1+2+3-4-5-6-7+8+9 = 1
  2506. -1+2-3+4-5-6-7+8+9 = 1
  2507.  1-2-3-4+5-6-7+8+9 = 1
  2508. +1-2-3-4+5-6-7+8+9 = 1
  2509. -1-2-3-4-5+6-7+8+9 = 1
  2510. -1-2 3+4+5+6-7+8+9 = 1
  2511.  1-2+3 4-5 6+7+8+9 = 1
  2512. +1-2+3 4-5 6+7+8+9 = 1
  2513.  1 2-3 4+5-6+7+8+9 = 1
  2514. +1 2-3 4+5-6+7+8+9 = 1
  2515. Total solutions  = 69
  2516.  
  2517. 69/19683 = 0.35 %
  2518.  
  2519. for those who care (it's not very elegant but it did the trick):
  2520.  
  2521. #include <stdio.h>
  2522. #include <math.h>
  2523.  
  2524. main (argc,argv)
  2525.      int argc;
  2526.      char *argv▌¿;
  2527. {
  2528.   int sresult, result, operator▌10¿,thisop;
  2529.   char buf▌256¿,ops▌3¿;
  2530.   int i,j,tot=0,temp;
  2531.  
  2532.   ops▌0¿ = ' ';
  2533.   ops▌1¿ = '-';
  2534.   ops▌2¿ = '+';
  2535.  
  2536.   for (i=1; i<10; i++) operator▌i¿ = 0;
  2537.  
  2538.   for (j=0; j < 19683; j++) {
  2539.     result = 0;
  2540.     sresult = 0;
  2541.     thisop = 1;
  2542.     for (i=1; i<10; i++) {
  2543.       switch (operator▌i¿) {
  2544.       case 0:
  2545.         sresult = sresult * 10 + i;
  2546.         break;
  2547.       case 1:
  2548.         result = result + sresult * thisop;
  2549.         sresult = i;
  2550.         thisop = -1;
  2551.         break;
  2552.       case 2:
  2553.         result = result + sresult * thisop;
  2554.         sresult = i;
  2555.         thisop = 1;
  2556.         break;
  2557.       }
  2558.     }
  2559.  
  2560.     result  = result + sresult * thisop;
  2561.     if (result == 1) {
  2562.       tot++;
  2563.       for  (i=1;i<10;i++)
  2564.         printf("%c%d",ops▌operator▌i¿¿,i);
  2565.       printf(" = %d\n",result);
  2566.     }
  2567.     temp = 0;
  2568.     operator▌1¿ += 1;
  2569.     for (i=1;i<10;i++) {
  2570.       operator▌i¿ += temp;
  2571.       if (operator▌i¿ > 2) { operator▌i¿ = 0; temp = 1;}
  2572.       else temp = 0;
  2573.     }
  2574.  
  2575.   }
  2576.  
  2577.   printf("Total solutions  = %d\n" , tot);
  2578. }
  2579.  
  2580. cwren@media.mit.edu (Christopher Wren)
  2581.  
  2582. ==> arithmetic/digits/equations/1992.p <==
  2583. 1 = -1+9-9+2.  Extend this list to 2 - 100 on the left side of the equals sign.
  2584.  
  2585. ==> arithmetic/digits/equations/1992.s <==
  2586. 1 = -1+9-9+2
  2587. 2 = 1*9-9+2
  2588. 3 = 1+9-9+2
  2589. 4 = 1+9/9+2
  2590. 5 = 1+9-sqrt(9)-2
  2591. 6 = 1^9+sqrt(9)+2
  2592. 7 = -1+sqrt(9)+sqrt(9)+2
  2593. 8 = 19-9-2
  2594. 9 = (1/9)*9^2
  2595. 10= 1+(9+9)/2
  2596. 11= 1+9+sqrt(9)-2
  2597. 12= 19-9+2
  2598. 13= (1+sqrt(9))]-9-2
  2599. 14= 1+9+sqrt(9)]-2
  2600. 15= -1+9+9-2
  2601. 16= -1+9+sqrt(9)]+2
  2602. 17= 1+9+9-2
  2603. 18= 1+9+sqrt(9)]+2
  2604. 19= -1+9+9+2
  2605. 20= (19-9)*2
  2606. 21= 1+9+9+2
  2607. 22= (-1+sqrt(9))*(9-2)
  2608. 23= (1+sqrt(9))]-sqrt(9)+2
  2609. 24= -1+9*sqrt(9)-2
  2610. 25= 1*9*sqrt(9)-2
  2611. 26= 19+9-2
  2612. 27= 1*9+9*2
  2613. 28= 1+9+9*2
  2614. 29= 1*9*sqrt(9)+2
  2615. 30= 19+9+2
  2616. 31= (1+sqrt(9))]+9-2
  2617. 32= -1+sqrt(9)*(9+2)
  2618. 33= 1*sqrt(9)*(9+2)
  2619. 34= (-1+9+9)*2
  2620. 35= -1+(9+9)*2
  2621. 36= 1^9*sqrt(9)]^2
  2622. 37= 19+9*2
  2623. 38= 1*sqrt(9)]*sqrt(9)]+2
  2624. 39= 1+sqrt(9)]*sqrt(9)]+2
  2625. 40= (1+sqrt(9)])*sqrt(9)]-2
  2626. 41= -1+sqrt(9)]*(9-2)
  2627. 42= (1+sqrt(9))]+9*2
  2628. 43= 1+sqrt(9)]*(9-2)
  2629. 44= -1+9*(sqrt(9)+2)
  2630. 45= 1*9*(sqrt(9)+2)
  2631. 46= 1+9*(sqrt(9)+2)
  2632. 47= (-1+sqrt(9)])*9+2
  2633. 48= 1*sqrt(9)]*(sqrt(9)]+2)
  2634. 49= (1+sqrt(9)])*(9-2)
  2635. 50= (-1+9)*sqrt(9)]+2
  2636. 51= -1+9*sqrt(9)]-2
  2637. 52= 1*9*sqrt(9)]-2
  2638. 53= -1+9*sqrt(9)*2
  2639. 54= 1*9*sqrt(9)*2
  2640. 55= 1+9*sqrt(9)*2
  2641. 56= 1*9*sqrt(9)]+2
  2642. 57= 1+9*sqrt(9)]+2
  2643. 58= (1+9)*sqrt(9)]-2
  2644. 59= 19*sqrt(9)+2
  2645. 60= (1+9)*sqrt(9)*2
  2646. 61= (1+sqrt(9)])*9-2
  2647. 62= -1+9*(9-2)
  2648. 63= 1*9*(9-2)
  2649. 64= 1+9*(9-2)
  2650. 65= (1+sqrt(9)])*9+2
  2651. 66= 1*sqrt(9)]*(9+2)
  2652. 67= 1+sqrt(9)]*(9+2)
  2653. 68= -(1+sqrt(9))]+92
  2654. 69= (1+sqrt(9))]+(9/.2)
  2655. 70= (1+9)*(9-2)
  2656. 71= -1-9+9^2
  2657. 72= (1+sqrt(9))*9*2
  2658. 73= -19+92
  2659. 74= (-1+9)*9+2
  2660. 75= -1*sqrt(9)]+9^2
  2661. 76= 1-sqrt(9)]+9^2
  2662. 77= (1+sqrt(9)])*(9+2)
  2663. 78= -1+9*9-2
  2664. 79= 1*9*9-2
  2665. 80= 1+9*9-2
  2666. 81= 1*9*sqrt(9)^2
  2667. 82= -1+9*9+2
  2668. 83= 1*9*9+2
  2669. 84= 1+9*9+2
  2670. 85= -1-sqrt(9)]+92
  2671. 86= -1*sqrt(9)]+92
  2672. 87= 1-sqrt(9)]+92
  2673. 88= (1+9)*9-2
  2674. 89= -1*sqrt(9)+92
  2675. 90= 1-sqrt(9)+92
  2676. 91= -1^9+92
  2677. 92= (1+9)*9+2
  2678. 93= 1^9+92
  2679. 94= -1+sqrt(9)+92
  2680. 95= 19*(sqrt(9)+2)
  2681. 96= -1+99-2
  2682. 97= 1*99-2
  2683. 98= 1+99-2
  2684. 99= 1*9*(9+2)
  2685. 100= -1+99+2
  2686.  
  2687. ==> arithmetic/digits/equations/383.p <==
  2688. Make 383 out of 1,2,25,50,75,100 using +,-,*,/.
  2689.  
  2690. ==> arithmetic/digits/equations/383.s <==
  2691. You can get 383 with ((2+50)/25+1)*100+75.
  2692.  
  2693. Of course, if you expect / as in C, the above expression is just 375.
  2694. But then you can get 383 with (25*50-100)/(1+2).  Pity there's no way
  2695. to use the 75.
  2696.  
  2697. If we had a language that rounded instead of truncating, we could use
  2698. ((1+75+100)*50)/(25-2) or (2*75*(25+100))/(50-1).
  2699.  
  2700. I imagine your problem lies in not dividing things that aren't
  2701. divisible.
  2702.  
  2703. Dan Hoey
  2704. Hoey@AIC.NRL.Navy.Mil
  2705.  
  2706. ==> arithmetic/digits/extreme.products.p <==
  2707. What are the extremal products of three three-digit numbers using digits 1-9?
  2708.  
  2709. ==> arithmetic/digits/extreme.products.s <==
  2710. There is a simple procedure which applies to these types of problems (and
  2711. which can be proven with help from the arithmetic-geometric inequality).
  2712.  
  2713. For the first part we use the "first large then equal" procedure.
  2714. This means that are three numbers will be 7**, 8**, and 9**.  Now
  2715. the digits 4,5,6 get distributed so as to make our three number as
  2716. close to each other as possible, i.e. 76*, 85*, 94*.  The same goes
  2717. for the remaining three digits, and we get 763, 852, 941.
  2718.  
  2719. For the second part we use the "first small then different" procedure.
  2720. Our three numbers will be of the form 1**, 2**, 3**.  We now place
  2721. the three digits so as to make our three numbers as unequal as possible;
  2722. this gives 14*, 25*, 36*.  Finishing, we get 147, 258, 369.
  2723.  
  2724. Now, *prove* that these procedures work for generalizations of this
  2725. problem.
  2726.  
  2727. ==> arithmetic/digits/googol.p <==
  2728. What digits does googol] start with?
  2729.  
  2730. ==> arithmetic/digits/googol.s <==
  2731. I'm not sure how to calculate the first googol of digits of log10(e), but
  2732. here's the first 150(approximately) of them...
  2733.  
  2734. 0.43429448190325182765112891891660508229439700580366656611445378316586464920
  2735. 8870774729224949338431748318706106744766303733641679287158963906569221064663
  2736.  
  2737. We need to deal with the digits immediately after the decimal point in
  2738. googol*log10(e), which are  .187061
  2739.  
  2740. frac▌log(googol])¿ = frac▌halflog2pi + 50 + googol(100-log10(e))¿
  2741.  = frac{halflog2pi + frac▌googol(100-log10(e))¿}
  2742.  = frac▌.399090 + (1- .187061)¿
  2743.  = .212029
  2744.  
  2745. 10 ** .212029 = 1.629405
  2746.  
  2747. Which means that googol] starts with 1629
  2748.  
  2749. ==> arithmetic/digits/labels.p <==
  2750. You have an arbitrary number of model kits (which you assemble for
  2751. fun and profit).  Each kit comes with twenty (20) stickers, two of which
  2752. are labeled "0", two are labeled "1", ..., two are labeled "9".
  2753. You decide to stick a serial number on each model you assemble starting
  2754. with one.  What is the first number you cannot stick.  You may stockpile
  2755. unused numbers on already assembled models, but you may not crack open
  2756. a new model to get at its stickers.  You complete assembling the current
  2757. model before starting the next.
  2758.  
  2759. ==> arithmetic/digits/labels.s <==
  2760. The method I used for this problem involved first coming up with a
  2761. formula that says how many times a digit has been used in all n models.
  2762.  
  2763. n = k*10^i + m for some k,m with 0 <k <10, m < 10^i
  2764. f(d,n) = (number of d's used getting to k*10^i from digits 0 to (i-1)) +
  2765.         (number of d's used by #'s 10^i to n from digit i) + f(d,m)
  2766. f(d,n) = i*k*10^(i-1) + (if (d < k) 10^i else if (d == k) m+1 else 0) + f(d,m)
  2767.  
  2768. This doesn't count 0's, which should be ok as they are not used as often
  2769. as other digits.  From the formula, it is clear that f(1,n) is never
  2770. less than f(d,n) for 1<d<10.
  2771. So I just calculated f(1,n) for various n (with some help from bc).
  2772.  
  2773. I quickly discovered that for n = 2*10^15, f(1,n) = 2*n.  After further
  2774. trials I determined that for n = 1999919999999981, f(1,n) = 2*n + 1.
  2775. This appears to be the smallest n with f(1,n) > 2*n.
  2776.  
  2777. ==> arithmetic/digits/nine.digits.p <==
  2778. Form a number using 0-9 once with its first n digits divisible by n.
  2779.  
  2780. ==> arithmetic/digits/nine.digits.s <==
  2781. First, reduce the sample set. For each digit of ABCDEFGHI, such that the last
  2782. digit, (current digit), is the same as a multiple of N :
  2783.  
  2784. A: Any number 1-9
  2785. B: Even numbers 2,4,6,8 (divisible by 2).
  2786. C: Any number 1-9 (21,12,3,24,15,6,27,18,9).
  2787. D: Even numbers 2,4,6,8 (divisible by 4, every other even).
  2788. E: 5 (divisible by 5 and 0 not allowed).
  2789. F: Even numbers (12,24,6,18)
  2790. G: Any number 1-9 (21,42,63,14,35,56,7,28,49).
  2791. H: Even numbers (32,24,16,8)
  2792. I: Any number 1-9 (81,72,63,54,45,36,27,18,9)
  2793.  
  2794. Since E must be 5, I can eliminate it everywhere else.
  2795. Since I will use up all the even digits, (2,4,6,8) filling in those spots
  2796.    that must be even. Any number becomes all odds, except 5.
  2797.  
  2798. A: 1,3,7,9
  2799. B: 2,4,6,8
  2800. C: 1,3,7,9
  2801. D: 2,4,6,8
  2802. E: 5
  2803. F: 2,4,6,8
  2804. G: 1,3,7,9
  2805. H: 2,4,6,8
  2806. I: 1,3,7,9
  2807.  
  2808. We have that 2C+D=0 (mod 4), and since C is odd,
  2809. this implies that D is 2 or 6; similarly we find that H is 2 or 6 ==>
  2810. {B,F} = {4,8}.  D+5+F=0 (mod 3) ==> if D=2 then F=8, if D=6 then F=4.
  2811.  
  2812. We have two cases.
  2813.  
  2814. Assume our number is of the form A4C258G6I0.  Now the case n=8 ==>
  2815. G=1,9; case n=3 ==> A+1+C=0 (mod 3) ==> {A,C}={1,7} ==> G=9, I=3.
  2816. The two numbers remaining fail for n=7.
  2817.  
  2818. Assume our number is of the form A8C654G2I0.  The case n=8 ==> G=3,7.
  2819. If G=3, we need to check to see which of 1896543, 9816543, 7896543,
  2820. and 9876543 are divisible by 7; none are.
  2821.  
  2822. If G=7, we need to check to see which of 1896547, 9816547, 1836547,
  2823. and 3816547 are divisible by 7; only the last one is, which yields
  2824. the solution 3816547290.
  2825.  
  2826. ==> arithmetic/digits/palindrome.p <==
  2827. Does the series formed by adding a number to its reversal always end in
  2828. a palindrome?
  2829.  
  2830. ==> arithmetic/digits/palindrome.s <==
  2831. This is not known.
  2832.  
  2833. If you start with 196, after 9480000 iterations you get a 3924257-digit
  2834. non-palindromic number.  However, there is no known proof that you will
  2835. never get a palindrome.
  2836.  
  2837. The statement is provably false for binary numbers. Roland Sprague has
  2838. shown that 10110 starts a series that never goes palindromic.
  2839.  
  2840. ==> arithmetic/digits/palintiples.p <==
  2841. Find all numbers that are multiples of their reversals.
  2842.  
  2843. ==> arithmetic/digits/palintiples.s <==
  2844. We are asked to find numbers that are integer multiples of their
  2845. reversals, which I call palintiples.  Of course, all the palindromic
  2846. numbers are a trivial example, but if we disregard the unit multiples,
  2847. the field is narrowed considerably.
  2848.  
  2849. Rouse Ball (_Mathematical_recreations_and_essays_) originated the
  2850. problem, and G. H. Hardy (_A_mathematician's_apology_) used the result
  2851. that 9801 and 8712 are the only four-digit palintiples as an example
  2852. of a theorem that is not ``serious''.  Martin Beech (_The_mathema-
  2853. tical_gazette_, Vol 74, #467, pp 50-51, March '90) observed that
  2854. 989*01 and 879*12 are palintiples, an observation he ``confirmed'' on
  2855. a hand calculator, and conjectured that these are all that exist.
  2856.  
  2857. I confirm that Beech's numbers are palintiples, I will show that they
  2858. are not all of the palintiples.  I will show that the palintiples do
  2859. not form a regular language.  And then I will prove that I have found
  2860. all the palintiples, by describing the them with a generalized form
  2861. of regular expression.  The results become more interesting in other
  2862. bases.
  2863.  
  2864. First, I have a more reasonable method of confirming that these
  2865. numbers are palintiples:
  2866.  
  2867.     Proof:  First, letting "9*" and "0*" refer an arbitrary string of
  2868.     nines and a string of zeroes of the same length, I note that
  2869.  
  2870.         879*12 = 879*00 + 12 = (880*00 - 100) + 12 = 880*00 - 88
  2871.         219*78 = 219*00 + 78 = (220*00 - 100) + 78 = 220*00 - 22
  2872.  
  2873.         989*01 = 989*00 +  1 = (990*00 - 100) +  1 = 990*00 - 99
  2874.         109*89 = 109*00 + 89 = (110*00 - 100) + 89 = 110*00 - 11
  2875.  
  2876.     It is obvious that 4x(220*00 - 22) = 880*00 - 88 and that
  2877.     9x(110*00 - 11) = 990*00 - 99.  QED.
  2878.  
  2879. Now, to show that these palintiples are not all that exist, let us
  2880. take the (infinite) language L▌4¿ = (879*12 + 0*), and let Pal(L▌4¿)
  2881. refer to the set of palindromes over the alphabet L▌4¿.  It is
  2882. immediate that the numbers in Pal(L▌4¿) are palintiples.  For
  2883. instance,
  2884.  
  2885.           8712 000 87912 879999912 879999912 87912 000 8712
  2886.     = 4 x 2178 000 21978 219999978 219999978 21978 000 2178
  2887.  
  2888. (where I have inserted spaces to enhance readability) is a palintiple.
  2889. Similarly, taking L▌9¿ = (989*01 + 0*), the numbers in Pal(L▌9¿) are
  2890. palintiples.  We exclude numbers starting with zeroes.
  2891.  
  2892. The reason these do not form a regular language is that the
  2893. sub-palintiples on the left end of the number must be the same (in
  2894. reverse order) as the sub-palintiples on the right end of the number:
  2895.  
  2896.          8712 8712 87999912 = 4 x 2178 2178 21999978
  2897.  
  2898. is not a palintiple, because 8712 8712 87999912 is not the reverse of
  2899. 2178 2178 21999978.  The pumping lemma can be used to prove that
  2900. Pal(L▌4¿)+Pal(L▌9¿) is not a regular language, just as in the familiar
  2901. proof that the palindromes over a non-singleton alphabet do not form a
  2902. regular language.
  2903.  
  2904. Now to characterize all the palintiples, let N be a palintiple,
  2905. N=CxR(N), where R(.) signifies reversal, and C>1 is an integer.  (I
  2906. use "x" for multiplication, to avoid confusion with the Kleene star
  2907. "*", which signifies the concatenated closure.)  If D is a digit of N,
  2908. let D' refer to the corresponding digit of R(N).  Since N=CxR(N),
  2909. D+10T = CxD'+S, where S is the carry in to the position occupied by D'
  2910. when R(N) is multiplied by C, and T is the carry out of that position.
  2911. Similarly, D'+10T'=CxD+S', where S', T' are carries in and out of the
  2912. position occupied by D when R(N) is multiplied by C.
  2913.  
  2914. Since D and D' are so closely related, I will use the symbol D:D' to
  2915. refer to a digit D on the left side of a string with a corresponding
  2916. digit D' on the right side of the string.  More formally, an
  2917. expression "x▌1¿:y▌1¿ x▌2¿:y▌2¿ ... x▌n¿:y▌n¿ w" will refer to a
  2918. string "x▌1¿ x▌2¿ ... x▌n¿ w y▌n¿ ... y▌2¿ y▌1¿", where the x▌i¿ and
  2919. y▌i¿ are digits and w is a string of zero or one digits.  So 989901
  2920. may be written as 9:1 8:0 9:9 and 87912 may be written as 8:2 7:1 9.
  2921. Thus Pal(L▌4¿)+Pal(L▌9¿) (omitting numbers with leading zeroes) can be
  2922. represented as
  2923.  
  2924.             (8:2 7:1 9:9* 1:7 2:8 0:0*)*
  2925.               (0:0* + 0 + 8:2 7:1 ( 9:9* + 9:9* 9))
  2926.           + (9:1 8:0 9:9* 0:8 1:9 0:0*)*
  2927.               (0:0* + 0 + 9:1 8:0 ( 9:9* + 9:9* 9)).      (1)
  2928.  
  2929. For each pair of digits D:D', there are a very limited--and often
  2930. empty--set of quadruples S,T,S',T' of digits that satisfy the
  2931. equations
  2932.  
  2933.                     D +10T =CxD'+S
  2934.                     D'+10T'=CxD +S',                      (2)
  2935.  
  2936. yet such a quadruple must exist for "D:D'" to appear in a palintiple
  2937. with multiplier C.  Furthermore, the S and T' of one D:D' must be T
  2938. and S', respectively, of the next pair of digits that appear.  This
  2939. enables us to construct a finite state machine to recognize those
  2940. palintiples.  The states ▌X#Y¿ refer to a pair of carries in D and D',
  2941. and we allow a transition from state ▌T#S'¿ to state ▌S#T'¿ on input
  2942. symbol D:D' exactly when equations (2) are satisfied.  Special
  2943. transitions for a single-digit input symbol (the central digit of
  2944. odd-length palintiples) and the criteria for the initial and the
  2945. accepting states are left as exercises.  The finite state machines
  2946. thus formed are
  2947.  
  2948.    State         Symbol  New     Symbol  New      Symbol   New
  2949.         Accept?         State           State             State
  2950.  
  2951. --> ▌0#0¿  Y       8:2  ▌0#3¿      0:0  ▌0#0¿         0   ▌A¿
  2952.     ▌0#3¿  N       7:1  ▌3#3¿
  2953.     ▌3#3¿  Y       1:7  ▌3#0¿      9:9  ▌3#3¿         9   ▌A¿
  2954.     ▌3#0¿  N       2:8  ▌0#0¿
  2955.      ▌A¿   Y
  2956.  
  2957. for constant C=4, and
  2958.  
  2959.    State         Symbol  New     Symbol  New      Symbol   New
  2960.         Accept?         State           State             State
  2961.  
  2962. --> ▌0#0¿  Y       1:9  ▌0#8¿      0:0  ▌0#0¿         0   ▌A¿
  2963.     ▌0#8¿  N       8:0  ▌8#8¿
  2964.     ▌8#8¿  Y       0:8  ▌8#0¿      9:9  ▌8#8¿         9   ▌A¿
  2965.     ▌8#0¿  N       9:1  ▌0#0¿
  2966.      ▌A¿   Y
  2967.  
  2968. for constant C=9, and the finite state machines for other constants
  2969. accept only strings of zeroes.  It is not hard to verify that the
  2970. proposed regular expression (1) represents the union of the languages
  2971. accepted by these machines, omitting the empty string and strings
  2972. beginning with zero.
  2973.  
  2974. I have written a computer program that constructs finite state
  2975. machines for recognizing palintiples for various bases and constants.
  2976. I found that base 10 is actually an unusually boring base for this
  2977. problem.  For instance, the machine for base 8, constant C=5 is
  2978.  
  2979.    State         Symbol  New     Symbol  New      Symbol  New
  2980.         Accept?         State           State            State
  2981.  
  2982. --> ▌0#0¿  Y       0:0  ▌0#0¿      5:1  ▌0#3¿         0  ▌A¿
  2983.     ▌0#3¿  N       1:0  ▌1#1¿      6:1  ▌1#4¿
  2984.     ▌1#1¿  Y       0:1  ▌3#0¿      5:2  ▌3#3¿
  2985.     ▌3#0¿  N       1:5  ▌0#0¿      6:6  ▌0#3¿         6  ▌A¿
  2986.     ▌3#3¿  Y       2:5  ▌1#1¿      7:6  ▌1#4¿
  2987.     ▌1#4¿  N       1:1  ▌4#1¿      6:2  ▌4#4¿         1  ▌A¿
  2988.     ▌4#4¿  Y       2:6  ▌4#1¿      7:7  ▌4#4¿         7  ▌A¿
  2989.     ▌4#1¿  N       1:6  ▌3#0¿      6:7  ▌3#3¿
  2990.      ▌A¿   Y
  2991.  
  2992. for which I invite masochists to write the regular expression.  If
  2993. anyone wants more, I should remark that the base 29 machine for
  2994. constant C=18 has 71 states]
  2995.  
  2996. By the way, I did not find any way of predicting the size or form of
  2997. the machines for the various bases, except that the machines for C=B-1
  2998. all seem to be isomorphic to each other.  If anyone investigates the
  2999. general behavior, I would be most happy to hear about it.
  3000.  
  3001. Dan Hoey
  3002. Hoey@AIC.NRL.Navy.Mil
  3003. May, 1992
  3004. ▌ A preliminary version of this message appeared in April, 1991. ¿
  3005. ================================================================
  3006. Dan
  3007.  
  3008.  
  3009.  
  3010. ==> arithmetic/digits/power.two.p <==
  3011. Prove that for any 9-digit number (base 10) there is an integral power
  3012. of 2 whose first 9 digits are that number.
  3013.  
  3014. ==> arithmetic/digits/power.two.s <==
  3015. Let v = log to base 10 of 2.
  3016. Then v is irrational.
  3017.  
  3018. Let w = log to base 10 of these 9 digits.
  3019.  
  3020. Since v is irrational, given epsilon > 0, there exists some natural number
  3021. n such that
  3022.  
  3023.    {w} < {nv} < {w} + epsilon
  3024.  
  3025. ({x} is the fractional part of x.)  Let us pick n for when
  3026.  
  3027.    epsilon = log 1.00000000000000000000001.
  3028.  
  3029. Then 2^n does the job.
  3030.  
  3031. ==> arithmetic/digits/prime/101.p <==
  3032. How many primes are in the sequence 101, 10101, 1010101, ...?
  3033.  
  3034. ==> arithmetic/digits/prime/101.s <==
  3035. Note that the sequence
  3036. 101 , 10101, 1010101, ....
  3037. can be viewed as
  3038. 100**1 +1, 100**2 + 100**1 + 1, 100**3 + 100**2 + 100**1 +1 ....
  3039. that is,
  3040. the k-th term in the sequence is
  3041. 100**k + 100**(k-1) + 100**(k-2) + ...+ 100**(1) + 1
  3042. = (100)**(k+1) - 1
  3043.   ----------------
  3044.     11 * 9
  3045. = (10)**(2k+2) - 1
  3046.   ----------------
  3047.     11 * 9
  3048. = ((10)**(k+1) - 1)*((10)**(k+1) +1)
  3049.    ---------------------------------
  3050.        11*9
  3051. thus either 11 and 9 divide the numerator. Either they both divide the
  3052. same factor in the numerator or different factors in the numerator. In
  3053. any case, after dividing, they leave the numerators as a product of two
  3054. integers.  Only in the case of k = 1, one of the integers is 1. Thus
  3055. there is exactly one prime in the above sequence: 101.
  3056.  
  3057. ==> arithmetic/digits/prime/all.prefix.p <==
  3058. What is the longest prime whose every proper prefix is a prime?
  3059.  
  3060. ==> arithmetic/digits/prime/all.prefix.s <==
  3061. 23399339, 29399999, 37337999, 59393339, 73939133
  3062.  
  3063. ==> arithmetic/digits/prime/change.one.p <==
  3064. What is the smallest number that cannot be made prime by changing a single
  3065. digit?  Are there infinitely many such numbers?
  3066.  
  3067. ==> arithmetic/digits/prime/change.one.s <==
  3068. 200.  Obviously, you would have to change the last digit, but 201, 203,
  3069. 207, and 209 are all composite.  For any smaller number, you can change
  3070. the last digit, and get
  3071. 2,11,23,31,41,53,61,71,83,97,101,113,127,131,149,151,163,173,181, or 191.
  3072.  
  3073. 200+2310n gives an infinite family, because changing the last
  3074. digit to 1 or 7 gives a number divisible by 3; to 3, a number divisible
  3075. by 7; to 9, a number divisible by 11.
  3076.  
  3077. ==> arithmetic/digits/prime/prefix.one.p <==
  3078. 2 is prime, but 12, 22, ..., 92 are not.  Similarly, 5 is prime
  3079. whereas 15, 25, ..., 95 are not.  What is the next prime number
  3080. which is composite when any digit is prefixed?
  3081.  
  3082. ==> arithmetic/digits/prime/prefix.one.s <==
  3083. 149
  3084.  
  3085. ==> arithmetic/digits/reverse.p <==
  3086. Is there an integer that has its digits reversed after dividing it by 2?
  3087.  
  3088. ==> arithmetic/digits/reverse.s <==
  3089. Assume there's such a positive integer x such that x/2=y and y is the
  3090. reverse of x.
  3091.  
  3092. Then x=2y.  Let x = a...b, then y = b...a, and:
  3093.  
  3094.                  b...a   (y)
  3095.                x     2
  3096.                --------
  3097.                  a...b   (x)
  3098.  
  3099. From the last digit b of x, we have b = 2a (mod 10), the possible
  3100. values for b are 2, 4, 6, 8 and hence possible values for (a, b) are
  3101. (1,2), (6,2), (2,4), (7,4), (3,6), (8,6), (4,8), (9,8).
  3102.  
  3103. From the first digit a of x, we have a = 2b or a = 2b+1.  None of the
  3104. above pairs satisfy this condition.  A contradiction.
  3105.  
  3106. Hence there's no such integer.
  3107.  
  3108. ==> arithmetic/digits/rotate.p <==
  3109. Find integers where multiplying them by single digits rotates their digits.
  3110.  
  3111. ==> arithmetic/digits/rotate.s <==
  3112. 2 105263157894736842
  3113. 3 1034482758620689655172413793
  3114. 4 102564 153846 179487 205128 230769
  3115. 5 142857 102040816326530612244897959183673469387755
  3116. 6 1016949152542372881355932203389830508474576271186440677966
  3117.   1186440677966101694915254237288135593220338983050847457627
  3118.   1355932203389830508474576271186440677966101694915254237288
  3119.   1525423728813559322033898305084745762711864406779661016949
  3120. 7 1014492753623188405797 1159420289855072463768 1304347826086956521739
  3121. 8 1012658227848 1139240506329
  3122. 9 10112359550561797752808988764044943820224719
  3123.  
  3124. In base B, suppose you have an N-digit answer A whose digits are
  3125. rotated when multiplied by K.  If D is the low-order digit of A, we
  3126. have
  3127.  
  3128.     (A-D)/B + D B^(N-1) = K A .
  3129.  
  3130. Solving this for A we have
  3131.  
  3132.         D (B^N - 1)
  3133.     A = ----------- .
  3134.           B K - 1
  3135.  
  3136. In order for A >= B^(N-1) we must have D >= K.  Now we have to find N
  3137. such that B^N-1 is divisible by R=(BK-1)/gcd(BK-1,D).  This always has
  3138. a minimal solution N0(R,B)<R, and the set of all solutions is the set
  3139. of multiples of N0(R,B).  N0(R,B) is the length of the repeating part
  3140. of the fraction 1/R in base B.
  3141.  
  3142. N0(ST,B)=N0(S,B)N0(T,B) when (S,T)=1, and for prime powers, N0(P^X,B)
  3143. divides (P-1)P^(X-1). Determining which divisor is a little more
  3144. complicated but well-known (cf. Hardy & Wright).
  3145.  
  3146. So given B and K, there is one minimal solution for each
  3147. D=K,K+1,...,B-1, and you get all the solutions by taking repetitions
  3148. of the minimal solutions.
  3149.  
  3150. ==> arithmetic/digits/sesqui.p <==
  3151. Find the least number where moving the first digit
  3152.  
  3153. ==> arithmetic/digits/sesqui.s <==
  3154. Let's represent this number as  a*10^n+b,  where 1<=a<=9 and
  3155. b < 10^n.  Then the condition to be satisfied is:
  3156.  
  3157. 3/2(a*10^n+b) = 10b+a
  3158.  
  3159.   3(a*10^n+b) = 20b+2a
  3160.  
  3161.    3a*10^n+3b = 20b+2a
  3162.  
  3163.   (3*10^n-2)a = 17b
  3164.  
  3165.             b = a*(3*10^n-2)/17
  3166.  
  3167. So we must have 3*10^n-2 = 0 (mod 17) (since a is less than 10, it
  3168. cannot contribute the needed prime 17 to the factorization of 17b).
  3169. (Also, assuming large n, we must have a at most 5 so that b < 10^n will
  3170. be satisfied, but note that we can choose a=1).  Now,
  3171.  
  3172. 3*10^n-2 = 0 (mod 17)
  3173.  
  3174. 3*10^n = 2 (mod 17)
  3175.  
  3176. 10^n = 12 (mod 17)
  3177.  
  3178. A quick check shows that the smallest n which satisfies this is 15
  3179. (the fact that one exists was assured to us because 17 is prime).  So,
  3180. setting n=15 and a=1 (obviously) gives us b=176470588235294, so the
  3181. number we are looking for is
  3182.  
  3183.                         1176470588235294
  3184.  
  3185. and, by the way, we can set a=2 to give us the second smallest such
  3186. number,
  3187.                         2352941176470588
  3188.  
  3189. Other things we can infer about these numbers is that there are 5 of
  3190. them less than 10^16, 5 more less than 10^33, etc.
  3191.  
  3192. ==> arithmetic/digits/squares/leading.7.to.8.p <==
  3193. What is the smallest square with leading digit 7 which remains a square
  3194. when leading 7 is replaced by an 8?
  3195.  
  3196. ==> arithmetic/digits/squares/leading.7.to.8.s <==
  3197. x=2996282391593370361328125
  3198. y=2824483699753370361328125
  3199. x^2=8977708170172487211329625006796419620513916015625
  3200. y^2=7977708170172487211329625006796419620513916015625
  3201.  
  3202. ==> arithmetic/digits/squares/length.22.p <==
  3203. Is it possible to form two numbers A and B from 22 digits such that
  3204. A = B^2?  Of course, leading digits must be non-zero.
  3205.  
  3206. ==> arithmetic/digits/squares/length.22.s <==
  3207. No, the number of digits of A^2 must be of the form 3n or 3n-1.
  3208.  
  3209. ==> arithmetic/digits/squares/length.9.p <==
  3210. Is it possible to make a number and its square, using the digits from 1 through
  3211. 9 exactly once?
  3212.  
  3213. ==> arithmetic/digits/squares/length.9.s <==
  3214. 567 and 854.
  3215.  
  3216. ==> arithmetic/digits/squares/three.digits.p <==
  3217. What squares consist entirely of three digits (e.g., 1, 4, and 9)?
  3218.  
  3219. ==> arithmetic/digits/squares/three.digits.s <==
  3220. The full set of solutions up to 10**12 is
  3221.               1 ->                            1
  3222.               2 ->                            4
  3223.               3 ->                            9
  3224.               7 ->                           49
  3225.              12 ->                          144
  3226.              21 ->                          441
  3227.              38 ->                         1444
  3228.             107 ->                        11449
  3229.             212 ->                        44944
  3230.           31488 ->                   9914 94144
  3231.           70107 ->                  49149 91449
  3232.         3 87288 ->               14 99919 94944
  3233.       956 10729 ->          9 14141 14499 11441
  3234.      4466 53271 ->        199 49914 44949 99441
  3235.     31487 17107 ->       9914 41941 99144 49449
  3236.   2 10810 79479 ->    4 44411 91199 99149 11441
  3237.  
  3238. If the algorithm is used in the form I presented it before, generating
  3239. the whole set P_n before starting on P_{n+1}, the store requirements
  3240. begin to become embarassing. For n>8 I switched to a depth-first
  3241. strategy, generating all the elements in P_i (i=9..12) congruent to
  3242. a particular x in P_8 for each x in turn. This means the solutions
  3243. don't come out in any particular order, of course. CPU time was 16.2
  3244. seconds (IBM 3084).
  3245.  
  3246. In article <1990Feb6.025205.28153@sun.soe.clarkson.edu>, Steven
  3247. Stadnicki suggests alternate triples of digits, in particular {1,4,6}
  3248. (with many solutions) and {2,4,8} (with few). I ran my program on
  3249. these as well, up to 10**12 again:
  3250.               1 ->                            1
  3251.               2 ->                            4
  3252.               4 ->                           16
  3253.               8 ->                           64
  3254.              12 ->                          144
  3255.              21 ->                          441
  3256.              38 ->                         1444
  3257.             108 ->                        11664
  3258.             119 ->                        14161
  3259.             121 ->                        14641
  3260.             129 ->                        16641
  3261.             204 ->                        41616
  3262.             408 ->                      1 66464
  3263.             804 ->                      6 46416
  3264.            2538 ->                     64 41444
  3265.            3408 ->                    116 14464
  3266.            6642 ->                    441 16164
  3267.           12908 ->                   1666 16464
  3268.           25771 ->                   6641 44441
  3269.           78196 ->                  61146 14416
  3270.           81619 ->                  66616 61161
  3271.         3 33858 ->               11 14611 64164
  3272.      2040 00408 ->         41 61616 64641 66464
  3273.      6681 64962 ->        446 44441 64444 61444
  3274.      8131 18358 ->        661 16146 41166 16164
  3275.     40182 85038 ->      16146 61464 66146 61444  (Steven's last soln.)
  3276.   1 20068 50738 ->    1 44164 46464 46111 44644
  3277.   1 26941 38988 ->    1 61141 16464 66616 64144
  3278.   1 27069 43631 ->    1 61466 41644 14114 64161
  3279.   4 01822 24262 ->   16 14611 14664 16614 44644
  3280.   4 05784 63021 ->   16 46611 66114 66644 46441
  3281.  78 51539 12392 -> 6164 66666 14446 44111 61664
  3282. and
  3283.               2 ->                            4
  3284.              22 ->                          484
  3285.             168 ->                        28224
  3286.             478 ->                      2 28484
  3287.            2878 ->                     82 82884 (Steven's last soln.)
  3288.      2109 12978 ->         44 48428 42888 28484
  3289. (so the answer to Steven's "Are there any more at all?" is "Yes".)
  3290.  
  3291. The CPU times were 42.9 seconds for {1,4,6}, 18.7 for {2,4,8}. This
  3292. corresponds to an interesting point: the abundance of solutions for
  3293. {1,4,6} is associated with abnormally large sets P_n (!P_8! = 16088
  3294. for {1,4,6} compared to !P_8! = 5904 for {1,4,9}) but the deficiency
  3295. of solutions for {2,4,8} is *not* associated with small P_n's (!P_8!
  3296. = 6816 for {2,4,8}). Can anyone wave a hand convincingly to explain
  3297. why the solutions for {2,4,8} are so sparse?
  3298.  
  3299. I suspect we are now getting to the point where an improved algorithm
  3300. is called for. The time to determine all the n-digit solutions (i.e.
  3301. 2n-digit squares) using this last-significant-digit-first is essentially
  3302. constant * 3**n. Dean Hickerson in <90036.134503HUL@PSUVM.BITNET>, and
  3303. Ilan Vardi in <1990Feb5.214249.22811@Neon.Stanford.EDU>, suggest using
  3304. a most-significant-digit-first strategy, based on the fact that the
  3305. first n digits of the square determine the (integral) square root; this
  3306. also has a running time constant * 3**n. Can one attack both ends at
  3307. once and do better?
  3308.  
  3309. Chris Thompson
  3310. JANET:    cet1@uk.ac.cam.phx
  3311. Internet: cet1%phx.cam.ac.uk@nsfnet-relay.ac.uk
  3312.  
  3313. Hey guys, what about
  3314.  
  3315. 648070211589107021 ^ 2 = 419994999149149944149149944191494441
  3316.  
  3317. This was found by David Applegate and myself (about 5 minutes on a DEC 3100,
  3318. program in C).
  3319.  
  3320. This is the largest square less than 10^42 with the 149-property; checking
  3321. took a bit more than an hour of CPU time.
  3322.  
  3323. As somebody suggested, we used a combined most-significant/least-significant
  3324. digits attack.  First we make a table of p-digit prefixes (most significant
  3325. p digits) that could begin a root whose square has the 149 property in its
  3326. first p digits.  We organize this table into buckets by the least
  3327. significant q digits of the prefixes.  Then we enumerate the s digit
  3328. suffixes whose squares have the 149 property in their last s digits.  For
  3329. each such suffix, we look in the table for those prefixes whose last q
  3330. digits match the first q of the suffix.  For each match, we consider the p +
  3331. s - q digit number formed by overlapping the prefix and the suffix by q
  3332. digits.  The squares of these overlap numbers must contain all the squares
  3333. with the 149 property.
  3334.  
  3335. The time expended is O(3^p) to generate the prefix table, O(3^s) to
  3336. enumerate the suffixes, and O(3^(p+s) / 10^q) to check the overlaps (being
  3337. very rough and ignoring the polynomial factors) By judiciously chosing p, q,
  3338. and s, we can fix things so that each bucket of the table has around O(1)
  3339. entries: set q = p log10(3).  Setting p = s, we end up looking for squares
  3340. whose roots have n = 2 - log10(3) digits, with an algorithm that takes time
  3341. O( 3 ^ ▌n / (2 - log10(3)¿) ), roughly time O(3^▌.66n¿).  Compared to the
  3342. O(3^n) performance of either single-ended algorithm, this lets us check 50%
  3343. more digits in the same amount of time (ignoring polynomial factors).  Of
  3344. course, the space cost of the combined-ends method is high.
  3345.  
  3346. -- Guy and Dave
  3347. --
  3348. Guy Jacobson                      School of Computer Science
  3349. Carnegie Mellon         arpanet : guy@cs.cmu.edu
  3350. Pittsburgh, PA  15213   csnet   : Guy.Jacobson%a.cs.cmu.edu@csnet-relay
  3351. (412) 268-3056          uucp    : ...]{seismo, ucbvax, harvard}]cs.cmu.edu]guy
  3352.  
  3353. Here is an algorithm which takes O(sqrt(n)log(n)) steps to find all perfect
  3354. squares < n whose only digits are 1, 4 and 9.
  3355.  
  3356. This doesn't sound too great *but* it doesn't use a lot of memory and only
  3357. requires addition and <.  Also, the actual run time will depend on where the
  3358. first non-{1,4,9} digit appears in each square.
  3359.  
  3360.         set n = 1
  3361.         set odd = 1
  3362.  
  3363.         while(n < MAXVAL)
  3364.         {
  3365.                 if(all digits of n are in {1,4,9})
  3366.                 {
  3367.                         print n
  3368.                 }
  3369.  
  3370.                 add 2 to odd
  3371.                 add odd to n
  3372.         }
  3373.  
  3374. This works because (X+1)^2 - x^2 = 2x+1.
  3375. That is, if you start with 0 and add successive odd
  3376. numbers to it you get 0+1=1, 1+3=4, 4+5=9, 9+7=16 etc.
  3377. I've started the algorithm at 1 for convenience.
  3378.  
  3379. The "O" value comes from looking at at most all digits
  3380. (log(n)) of all perfect squares < n (sqrt(n) of them)
  3381. at most a constant number of times.
  3382.  
  3383. I didn't save the articles with algorithms claiming to be
  3384. O(3^log(n)) so I don't know if their calculations needed
  3385. to (or did) account for multiplication or sqrt() of large
  3386. numbers.  O(3^log(n)) sounds reasonable so I'm going to
  3387. assume they did unless I hear otherwise.
  3388.  
  3389. Any comments? Please email if you just want to refresh my memory
  3390. on the other algorithms.
  3391.  
  3392. Andrew Charles
  3393. acgd@ihuxy.ATT.COMM
  3394.  
  3395. ==> arithmetic/digits/squares/twin.p <==
  3396. Let a twin be a number formed by writing the same number twice,
  3397. for instance, 81708170 or 132132.  What is the smallest square twin?
  3398.  
  3399. ==> arithmetic/digits/squares/twin.s <==
  3400. 1322314049613223140496 = 36363636364 ^ 2.
  3401.  
  3402. The key to solving this puzzle is looking at the basic form of these
  3403. "twin" numbers, which is some number k = 1 + 10^n multiplied by some number
  3404. a < 10^n. If ak is a perfect square, k must have some repeated factor,
  3405. since a<k. Searching the possible values of k for one with a repeated factor
  3406. eventually turns up the number 1 + 10^11 = 11^2 * 826446281.
  3407. So, we set a=826446281 and ak = 9090909091^2 = 82644628100826446281,
  3408. but this needs leading zeros to fit the pattern. So, we multiply by a suitable
  3409. small square (in this case 16) to get the above answer.
  3410.  
  3411. ==> arithmetic/digits/sum.of.digits.p <==
  3412. Find sod ( sod ( sod (4444 ^ 4444 ) ) ).
  3413.  
  3414. ==> arithmetic/digits/sum.of.digits.s <==
  3415. let X = 4444^4444
  3416.  
  3417. sod(X) <= 9 * (# of digits) < 145900
  3418. sod(sod(X)) <= sod(99999) = 45
  3419. sod(sod(sod(X))) <= sod(39) = 12
  3420.  
  3421. but sod(sod(sod(X))) = 7 (mod 9)
  3422.  
  3423. thus sod(sod(sod(X))) = 7
  3424.  
  3425. ==> arithmetic/digits/zeros/factorial.p <==
  3426. How many zeros are in the decimal expansion of n]?
  3427.  
  3428. ==> arithmetic/digits/zeros/factorial.s <==
  3429. The general answer to the question
  3430. "what power of p divides x]" where p is prime
  3431. is (x-d)/(p-1) where d is the sum of the digits of (x written in base p).
  3432.  
  3433. So where p=5, 10 is written as 20 and is divisible by 5^2 (2 = (10-2)/4);
  3434. x to base 10:     100    1000    10000    100000     1000000
  3435. x to base 5:      400   13000   310000  11200000   224000000
  3436. d          :        4       4        4         4           8
  3437. trailing 0s in x]  24     249     2499     24999      249998
  3438.  
  3439. ==> arithmetic/digits/zeros/lsd.factorial.p <==
  3440. What is the least significant non-zero digit in the decimal expansion of n]?
  3441.  
  3442. ==> arithmetic/digits/zeros/lsd.factorial.s <==
  3443. Reduce mod 10 the numbers 2..n and then cancel out the
  3444. required factors of 10. The final step then involves
  3445. computing 2^i*3^j*7^k mod 10 for suitable i,j and k.
  3446.  
  3447. A small program that performs this calculation is appended. Like the
  3448. other solutions, it takes O(log n) arithmetic operations.
  3449.  
  3450. -kym
  3451. ===
  3452.  
  3453. #include<stdio.h>
  3454. #include<assert.h>
  3455.  
  3456. int     p▌6¿▌4¿={
  3457.         /*2*/   2,      4,      8,      6,
  3458.         /*3*/   3,      9,      7,      1,
  3459.         /*4*/   4,      6,      4,      6,
  3460.         /*5*/   5,      5,      5,      5,
  3461.         /*6*/   6,      6,      6,      6,
  3462.         /*7*/   7,      9,      3,      1,
  3463.         };
  3464.  
  3465. main(){
  3466.         int     i;
  3467.         int n;
  3468.  
  3469.         for(n=2;n<1000;n++){
  3470.                 i=lsdfact(n);
  3471.                 printf("%d\n",i);
  3472.                 }
  3473.  
  3474.         exit(0);
  3475.         }
  3476.  
  3477. lsdfact(n){
  3478.         int     a▌10¿;
  3479.         int     i;
  3480.         int     n5;
  3481.         int     tmp;
  3482.  
  3483.         for(i=0;i<=9;i++)a▌i¿=alpha(i,n);
  3484.  
  3485.         n5=0;
  3486. /* NOTE: order is important in following */
  3487. l5:;
  3488.         while(tmp=a▌5¿){        /* cancel factors of 5 */
  3489.                 n5+=tmp;
  3490.                 a▌1¿+=(tmp+4)/5;
  3491.                 a▌3¿+=(tmp+3)/5;
  3492.                 a▌5¿=(tmp+2)/5;
  3493.                 a▌7¿+=(tmp+1)/5;
  3494.                 a▌9¿+=(tmp+0)/5;
  3495.                 }
  3496. l10:;
  3497.         if(tmp=a▌0¿){
  3498.                 a▌0¿=0; /* cancel all factors of 10 */
  3499.                 for(i=0;i<=9;i++)a▌i¿+=alpha(i,tmp);
  3500.                 }
  3501.         if(a▌5¿) goto l5;
  3502.         if(a▌0¿) goto l10;
  3503.  
  3504. /* n5 == number of 5's cancelled;
  3505.    must now cancel same number of factors of 2 */
  3506.         i=ipow(2,a▌2¿+2*a▌4¿+a▌6¿+3*a▌8¿-n5)*
  3507.                 ipow(3,a▌3¿+a▌6¿+2*a▌9¿)*
  3508.                 ipow(7,a▌7¿);
  3509.         assert(i%10);   /* must not be zero */
  3510.         return  i%10;
  3511.         }
  3512.  
  3513. alpha(d,n){
  3514. /* number of decimal numbers in ▌1,n¿ ending in digit d */
  3515.         int tmp;
  3516.         tmp=(n+10-d)/10;
  3517.         if(d==0)tmp--;  /* forget 0 */
  3518.         return tmp;
  3519.         }
  3520.  
  3521. ipow(x,y){
  3522. /* x^y mod 10 */
  3523.         if(y==0) return 1;
  3524.         if(y==1) return x;
  3525.         return p▌x-2¿▌(y-1)%4¿;
  3526.         }
  3527.  
  3528.  
  3529.  
  3530.  
  3531. ==> arithmetic/digits/zeros/million.p <==
  3532. How many zeros occur in the numbers from 1 to 1,000,000?
  3533.  
  3534. ==> arithmetic/digits/zeros/million.s <==
  3535. In the numbers from 10^(n-1) through 10^n - 1, there are 9 * 10^(n-1)
  3536. numbers of n digits each, so 9(n-1)10^(n-1) non-leading digits, of
  3537. which one tenth, or 9(n-1)10^(n-2), are zeroes.  When we change the
  3538. range to 10^(n-1) + 1 through 10^n, we remove 10^(n-1) and put in
  3539. 10^n, gaining one zero, so
  3540.  
  3541.     p(n) = p(n-1) + 9(n-1)10^(n-2) + 1 with p(1)=1.
  3542.  
  3543. Solving the recurrence yields the closed form
  3544.  
  3545.     p(n) = n(10^(n-1)+1) - (10^n-1)/9.
  3546.  
  3547. For n=6, there are 488,895 zeroes, 600,001 ones, and 600,000 of all other
  3548. digits.
  3549.  
  3550. ==> arithmetic/magic.squares.p <==
  3551. Are there large squares, containing only consecutive integers, all of whose
  3552. rows, columns and diagonals have the same sum?  How about cubes?
  3553.  
  3554. ==> arithmetic/magic.squares.s <==
  3555. Here is an 8x8 example:
  3556.  
  3557. 01 56 48 25 33 24 16 57
  3558. 63 10 18 39 31 42 50 07
  3559. 62 11 19 38 30 43 51 06
  3560. 04 53 45 28 36 21 13 60
  3561. 05 52 44 29 37 20 12 61
  3562. 59 14 22 35 27 46 54 03
  3563. 58 15 23 34 26 47 55 02
  3564. 08 49 41 32 40 17 09 64
  3565.  
  3566. References:
  3567.     "Magic Squares and Cubes"
  3568.     W.S. Andrews
  3569.     The Open Court Publishing Co.
  3570.     Chicago, 1908
  3571.  
  3572.     "Mathematical Recreations"
  3573.     M. Kraitchik
  3574.     Dover
  3575.     New York, 1953
  3576.  
  3577.  
  3578.  
  3579.  
  3580. ==> arithmetic/pell.p <==
  3581. Find integer solutions to x^2 - 92y^2 = 1.
  3582.  
  3583. ==> arithmetic/pell.s <==
  3584. x=1        y=0
  3585. x=1151     y=120
  3586. x=2649601  y=276240
  3587. etc.
  3588.  
  3589. Each successive solution is about 2300 times the previous
  3590. solution;  they are every 8th partial fraction (x=numerator,
  3591. y=denominator) of the continued fraction for sqrt(92) =
  3592. ▌9,  1,1,2,4,2,1,1,18,  1,1,2,4,2,1,1,18,  1,1,2,4,2,1,1,18, ...¿
  3593.  
  3594. Once you have the smallest positive solution (x1,y1) you
  3595. don't need to "search" for the rest.  You can obtain the nth positive
  3596. solution (xn,yn) by the formula
  3597.  
  3598. (x1 + y1 sqrt(92))^n = xn + yn sqrt(92).
  3599.  
  3600. See Niven & Zuckerman's An Introduction to the Theory of Numbers.
  3601. Look in the index under Pell's equation.
  3602.  
  3603. ==> arithmetic/prime/arithmetic.progression.p <==
  3604. Is there an arithmetic progression of 20 or more primes?
  3605.  
  3606. ==> arithmetic/prime/arithmetic.progression.s <==
  3607. There is an arithmetic progression of 21 primes:
  3608.     142072321123 + 1419763024680 i, 0 <= i < 21.
  3609.  
  3610. It was discovered on 30 November 1990, by programs running in the background
  3611. on a network of Sun 3 workstations in the Department of Computer Science,
  3612. University of Queensland, Australia.
  3613.  
  3614. See: Andrew Moran and Paul Pritchard, The design of a background job
  3615. on a local area network, Procs. 14th Australian Computer Science Conference,
  3616. 1991, to appear.
  3617.  
  3618. ==> arithmetic/prime/consecutive.composites.p <==
  3619. Are there 10,000 consecutive non-prime numbers?
  3620.  
  3621. ==> arithmetic/prime/consecutive.composites.s <==
  3622. 9973]+2 through 9973]+10006 are composite.
  3623.  
  3624. ==> arithmetic/sequence.p <==
  3625. Prove that all sets of n integers contain a subset whose sum is divisible by n.
  3626.  
  3627. ==> arithmetic/sequence.s <==
  3628. Consider the set of remainders of the partial sums a(1) + ... + a(i).
  3629. Since there are n such sums, either one has remainder zero (and we're
  3630. thru) or 2 coincide, say the i'th and j'th.  In this case, a(i+1) +
  3631. ... + a(j) is divisible by n.  (note this is a stronger result since
  3632. the subsequence constructed is of *adjacent* terms.)  Consider a(1)
  3633. (mod n), a(1)+a(2) (mod n), ..., a(1)+...+a(n) (mod n).  Either at
  3634. some point we have a(1)+...+a(m) = 0 (mod n) or else by the pigeonhole
  3635. principle some value (mod n) will have been duplicated.  We win either
  3636. way.
  3637.  
  3638. ==> arithmetic/sum.of.cubes.p <==
  3639. Find two fractions whose cubes total 6.
  3640.  
  3641. ==> arithmetic/sum.of.cubes.s <==
  3642. Restated:
  3643. Find X, Y, minimum Z (all positive integers) where
  3644. (X/Z)^3 + (Y/Z)^3 = 6
  3645.  
  3646. Again, a generalized solution would be nice.
  3647.  
  3648. You are asking for the smallest z s.t. x^3 + y^3 = 6*z^3 and x,y,z in Z+.
  3649. In general, questions like these are extremely difficult; if you're
  3650. interested take a look at books covering Diophantine equations
  3651. (especially Baker's work on effective methods of computing solutions).
  3652.  
  3653. Dudeney mentions this problem in connection with #20 in _The Canterbury
  3654. Puzzles_; the smallest answer is (17/21)^3 + (37/21)^3 = 6.
  3655.  
  3656. For the interest of the readers of this group I'll quote:
  3657.  
  3658.    "Given a known case for the expression of a number as the sum or
  3659. difference of two cubes, we can, by formula, derive from it an infinite
  3660. number of other cases alternately positive and negative.  Thus Fermat,
  3661. starting from the known case 1^3 + 2^3 = 9 (which we will call a
  3662. fundamental case), first obtained a negative solution in bigger
  3663. figures, and from this his positive solution in bigger figures still.
  3664. But there is an infinite number of fundamentals, and I found by trial
  3665. a negative fundamental solution in smaller figures than his derived
  3666. negative solution, from which I obtained the result shown above.  That
  3667. is the simple explanation."
  3668.  
  3669. In the above paragraph Dudeney is explaining how he derived (*by hand*)
  3670. that (415280564497/348671682660)^3 + (676702467503/348671682660)^3 = 9.
  3671.  
  3672. He continues:
  3673.  
  3674. "We can say of any number up to 100 whether it is possible or not to
  3675. express it as the sum of two cubes, except 66.  Students should read
  3676. the Introduction to Lucas's _Theorie des Nombres_, p. xxx."
  3677.  
  3678. "Some years ago I published a solution for the case 6 = (17/21)^3 +
  3679. (37/21)^3, of which Legendre gave at some length a 'proof' of
  3680. impossibility; but I have since found that Lucas anticipated me in
  3681. a communication to Sylvester."
  3682.  
  3683. ==> arithmetic/tests.for.divisibility/eleven.p <==
  3684. What is the test to see if a number is divisible by eleven?
  3685.  
  3686.  
  3687. ==> arithmetic/tests.for.divisibility/eleven.s <==
  3688. If the alternating sum of the digits is divisible by eleven, so is the number.
  3689.  
  3690. For example, 1639 leads to 9 - 3 + 6 - 1 = 11, so 1639 is divisible by 11.
  3691.  
  3692. Proof:
  3693. Every integer n can be expressed as
  3694. n = a1*(10^k) + a2*(10^k-1)+ .....+ a_k+1
  3695. where a1, a2, a3, ...a_k+1 are integers between 0 and 9.
  3696. 10 is congruent to -1 mod(11).
  3697. Thus if (-1^k)*a1 + (-1^k-1)*a2 + ...+ (a_k+1) is congruent to 0mod(11) then
  3698. n is divisible by 11.
  3699.  
  3700. ==> arithmetic/tests.for.divisibility/nine.p <==
  3701. What is the test to see if a number is divisible by nine?
  3702.  
  3703. ==> arithmetic/tests.for.divisibility/nine.s <==
  3704. If the sum of the digits is divisible by nine, so is the number.
  3705.  
  3706. Proof:
  3707. Every integer n can be expressed as
  3708. n = a1*(10^k) + a2*(10^k-1)+ .....+ a_k+1
  3709. where a1, a2, a3, ...a_k+1 are integers between 0 and 9.
  3710. Note that 10 is congruent to 1 (mod 9). Thus 10^k is congruent to 1 (mod 9) for
  3711. every k >= 0.
  3712. Thus n is congruent to (a1+a2+a3+....+a_k+1) mod(9).
  3713. Hence (a1+a2+...+a_k+1) is divisible by 9 iff n is divisible by 9.
  3714.  
  3715. ==> arithmetic/tests.for.divisibility/seven.p <==
  3716. What is the test to see if a number is divisible by 7?
  3717.  
  3718. ==> arithmetic/tests.for.divisibility/seven.s <==
  3719. Take the last digit (n mod 10) and double it.
  3720. Take the rest of the digits (n div 10) and subtract the doubled last
  3721.     digit from it.
  3722. The resulting number is divisible by 7 iff the original number
  3723.     is divisible by 7.
  3724.  
  3725. Example:  Take 2009.
  3726.           Subtract (2009 mod 10) * 2 from (2009 div 10)
  3727.                            -  9  * 2   +   200
  3728.                             = 182
  3729.           Subtract (182 mod 10)  * 2 from (182 div 10)
  3730.                            -  2  * 2   +   18
  3731.                             = 14
  3732.           so 2009 is divisible by 7.
  3733.  
  3734. ==> arithmetic/tests.for.divisibility/three.p <==
  3735. Prove that if a number is divisible by 3, the sum of its digits is likewise.
  3736.  
  3737. ==> arithmetic/tests.for.divisibility/three.s <==
  3738. First, prove 10^N = 1 mod 3 for all integers N >= 0.
  3739. 1 = 1 mod 3. 10 = 1 mod 3. 10^N = 10^(N-1) * 10 = 10^(N-1) mod 3.
  3740. QED by induction.
  3741. Now let D▌0¿ be the units digit of N, D▌1¿ the tens digit, etc.
  3742. Now N = Summation From k=0 to k=inf of D▌k¿*10^k.
  3743. Therefore N mod 3 = Summation from k=0 to k=inf of D▌k¿ mod 3. QED
  3744.  
  3745. ==> combinatorics/coinage/combinations.p <==
  3746. How many ways are there to make change for a dollar?  Count
  3747. combinations of coins, not permuations.
  3748.  
  3749. ==> combinatorics/coinage/combinations.s <==
  3750. Assuming that you had coins of one cent, five cents, ten cents, 25 cents,
  3751. 50 cents, and 100 cents, there are 293 ways to make change for a dollar.
  3752. This can be calculated by determining the number of ways to make change
  3753. using only a penny and then a penny and nickel, then penny, nickel, and
  3754. dime, etc.
  3755.  
  3756. The table is shown below:
  3757.  
  3758. Amount  00 05 10 15 20 25 30 35 40 45 50 55 60 65  70  75  80  85  90  95  100
  3759. Coins
  3760. .01      1  1  1  1  1  1  1  1  1  1  1  1  1  1   1   1   1   1   1   1   1
  3761. .05      1  2  3  4  5  6  7  8  9 10 11 12 13 14  15  16  17  18  19  20  21
  3762. .10      1  2  4  6  9 12 16 20 25 30 36 42 49 56  64  72  81  90 100 110 121
  3763. .25      1  2  4  6  9 13 18 24 31 39 49 60 73 87 103 121 141 163 187 214 242
  3764. .50      1  2  4  6  9 13 18 24 31 39 49 62 77 93 112 134 159 187 218 253 292
  3765. 1.0      1  2  4  6  9 13 18 24 31 39 49 62 77 93 112 134 159 187 218 253 293
  3766.  
  3767. The meaning of each entry is as follows:
  3768. If you wish to make change for 50 cents using only pennies, nickels and dimes,
  3769. go to the .10 row and the 50 column to obtain 36 ways to do this.
  3770.  
  3771. To calculate each entry, you start with the pennies.  There is exactly one
  3772. way to make change for every amount.  Then calculate the .05 row by adding
  3773. the number of ways to make change for the amount using pennies plus the number
  3774. of ways to make change for five cents less using nickels and pennies.  This
  3775. continues on for all denominations of coins.
  3776.  
  3777. An example, to get change for 75 cents using all coins up to a .50, add the
  3778. number of ways to make change using only .25 and down (121) and the number of
  3779. ways to make change for 25 cents using coins up to .50 (13).  This yields the
  3780. answer of 134.
  3781.  
  3782. ==> combinatorics/coinage/dimes.p <==
  3783. "Dad wants one-cent, two-cent, three-cent, five-cent, and ten-cent
  3784. stamps.  He said to get four each of two sorts and three each of the
  3785. others, but I've forgotten which.  He gave me exactly enough to buy
  3786. them; just these dimes."  How many stamps of each type does Dad want?
  3787. ▌J.A.H. Hunter¿
  3788.  
  3789. ==> combinatorics/coinage/dimes.s <==
  3790. The easy way to solve this is to sell her three each, for
  3791. 3x(1+2+3+5+10) = 63 cents.  Two more stamps must be bought, and they
  3792. must make seven cents (since 17 is too much), so the fourth stamps are
  3793. a two and a five.
  3794.  
  3795. ==> combinatorics/coinage/impossible.p <==
  3796. What is the smallest number of coins that you can't make a dollar with?
  3797. I.e., for what N does there not exist a set of N coins adding up to a dollar?
  3798. It is possible to make a dollar with 1 current U.S. coin (a Susan B. Anthony),
  3799. 2 coins (2 fifty cent pieces), 3 coins (2 quarters and a fifty cent piece),
  3800. etc.  It is not possible to make exactly a dollar with 101 coins.
  3801.  
  3802. ==> combinatorics/coinage/impossible.s <==
  3803. The answer is 77:
  3804.  
  3805. a) 5c  = 1 or 5;
  3806. b) 10c = 1 or 2 a's (1,2,6,10)
  3807. c) 25c = 1 or 2 b's + 1 a
  3808. d) 50c = 1 or 2 c's
  3809. e) $1  = 1 or 2 d's
  3810.  
  3811. total   penny   nickle  dime    quarter half
  3812. 5               1       2       1       1
  3813. 6               3       1       1       1
  3814. 7               5               1       1
  3815. 8               4       3               1
  3816. 9               6       2               1
  3817. 10              8       1               1
  3818. 11              10                      1
  3819. 12              7       4       1
  3820. 13              9       3       1
  3821. 14              11      2       1
  3822. 15              13      1       1
  3823. 16              15              1
  3824. 17              14      3
  3825. 18              16      2
  3826. 19              18      1
  3827. 20              20
  3828. 21      5       13      3
  3829. 22      5       15      2
  3830. 23      5       17      1
  3831. 24      5       19
  3832. 25      10      12      3
  3833. 26      10      14      2
  3834. 27      10      16      1
  3835. 28      10      18
  3836. 29      15      11      3
  3837. 30      15      13      2
  3838. 31      15      15      1
  3839. 32      15      17
  3840. 33      20      10      3
  3841. 34      20      12      2
  3842. 35      20      14      1
  3843. 36      20      16
  3844. 37      25      9       3
  3845. 38      25      11      2
  3846. 39      25      13      1
  3847. 40      25      15
  3848. 41      30      8       3
  3849. 42      30      10      2
  3850. 43      30      12      1
  3851. 44      30      14
  3852. 45      35      7       3
  3853. 46      35      9       2
  3854. 47      35      11      1
  3855. 48      35      13
  3856. 49      40      6       3
  3857. 50      40      8       2
  3858. 51      40      10      1
  3859. 52      40      12
  3860. 53      45      5       3
  3861. 54      45      7       2
  3862. 55      45      9       1
  3863. 56      45      11
  3864. 57      50      4       3
  3865. 58      50      6       2
  3866. 59      50      8       1
  3867. 60      50      10
  3868. 61      55      3       3
  3869. 62      55      5       2
  3870. 63      55      7       1
  3871. 64      55      9
  3872. 65      60      2       3
  3873. 66      60      4       2
  3874. 67      60      6       1
  3875. 68      60      8
  3876. 69      65      1       3
  3877. 70      65      3       2
  3878. 71      65      5       1
  3879. 72      65      7
  3880. 73      70              3
  3881. 74      70      2       2
  3882. 75      70      4       1
  3883. 76      70      6
  3884. 77      can't be done
  3885. 78      75      1       2
  3886. 79      75      3       1
  3887. 80      75      5
  3888. 81      can't be done
  3889. 82      80              2
  3890. 83      80      2       1
  3891. 84      80      4
  3892. 85      can't be done
  3893. 86      can't be done
  3894. 87      85      1       1
  3895. 88      85      3
  3896. 89      can't be done
  3897. 90      can't be done
  3898. 91      90              1
  3899. 92      90      2
  3900. 93-95   can't be done
  3901. 96      95      1
  3902. 97-99   can't be done
  3903. 100     100
  3904.  
  3905. ==> combinatorics/color.p <==
  3906. An urn contains n balls of different colors.  Randomly select a pair, repaint
  3907. the first to match the second, and replace the pair in the urn.  What is the
  3908. expected time until the balls are all the same color?
  3909.  
  3910. ==> combinatorics/color.s <==
  3911. (n-1)^2.
  3912.  
  3913. If the color classes have sizes k1, k2, ..., km, then the expected number of
  3914. steps from here is (dropping the subscript on k):
  3915.  
  3916.      2               k(k-1)              (j-1) (k-j)
  3917. (n-1)  -  SUM      ( ------  +   SUM   --------------- )
  3918.          classes,      2        1<j<k      (n-j)
  3919.       class.size=k
  3920.  
  3921. The verification goes roughly as follows.  Defining  phi(k) as  (k(k-1)/2 +
  3922. sum▌j¿...), we first show that  phi(k+1) + phi(k-1) - 2*phi(k) = (n-1)/(n-k)
  3923. except when k=n; the k(k-1)/2 contributes 1, the term j=k contributes
  3924. (j-1)/(n-j)=(k-1)/(n-k), and the other summands j<k contribute nothing.
  3925. Then we say that the expected change in phi(k) on a given color class is
  3926. k*(n-k)/(n*(n-1)) times (phi(k+1) + phi(k-1) - 2*phi(k)), since with
  3927. probability k*(n-k)/(n*(n-1)) the class goes to size k+1 and with the same
  3928. probability it goes to size k-1.  This expected change comes out to k/n.
  3929. Summing over the color classes (and remembering the minus sign), the
  3930. expected change in the "cost from here" on one step is -1, except when we're
  3931. already monochromatic, where the handy exception k=n kicks in.
  3932.  
  3933. One can rewrite the contribution from k as
  3934.  
  3935.    (n-1) SUM (k-j)/(n-j)
  3936.         0<j<k
  3937.  
  3938. which incorporates both the k(k-1)/2 and the previous sum over j.
  3939. That makes the proof a little cleaner.
  3940.  
  3941. ==> combinatorics/full.p <==
  3942. Consider a string that contains all substrings of length n.  For example,
  3943. for binary strings with n=2, a shortest string is 00110 -- it contains 00,
  3944. 01, 10 and 11 as substrings.  Find the shortest such strings for all n.
  3945.  
  3946. ==> combinatorics/full.s <==
  3947. Knuth, Volume 2 Seminumerical Algorithms, section 3.2.2 discusses this problem.
  3948. He cites the following results:
  3949. Shortest length: m^n + n - 1, where m = number of symbols in the language.
  3950. Algorithms:
  3951. ▌Exercise 7, W. Mantel, 1897¿
  3952. The binary sequence is the LSB of X computed by the MIX program:
  3953.     LDA X
  3954.     JANZ *+2
  3955.     LDA A
  3956.     ADD X
  3957.     JNOV *+3
  3958.     JAZ *+2
  3959.     XOR A
  3960.     STA X
  3961. ▌Exercise 10, M. H. Martin, 1934¿
  3962. Set x▌1¿ = x▌2¿ = ... = x▌n¿ = 0.  Set x▌i+1¿ = largest value < n such that
  3963. substring of n digits ending at x▌i+1¿ does not occur earlier in string.
  3964. Terminate when this is not possible.
  3965.  
  3966. If we instead consider the strings as circular, we have a well known
  3967. problem whose solution is given by any hamiltonian cycle in the de
  3968. Bruijn (or Good) graph of dimension K.  (Or equivalently an eulerian
  3969. circuit in the de Bruijn graph of dimension K-1) As a string of length
  3970. 2^K is produced, it must be optimal, and any shortest sequence must be
  3971. an eulerian circuit in a dB graph.
  3972.  
  3973. The de Bruijn graph Tn has as its vertex set the binary n-strings.
  3974. Directed edges join n-strings that may be derived by deleting the left
  3975. most digit and appending a 0 or 1 to the right end.  de Bruijn + van
  3976. Ardenne-Ehrenfest (in 1951) counted the number of eulerian circuits in
  3977. Tn. There are 2^(2^(n-1)-n) of them.
  3978.  
  3979. Some examples:
  3980. K=2 1100
  3981. K=3 11101000
  3982. K=4 1111001011010000
  3983.  
  3984. The solution to the above problem (non-circular strings) can be found
  3985. by duplicating the first K-1 digits of the solution string at the end
  3986. of the string.  These are not the only solutions, but they
  3987. are of minimum length: 2^K + K-1.
  3988.  
  3989. We can obtain a lower bound for the optimal sequence for the general case as
  3990. follows:
  3991.  
  3992. Consider first the simpler case of breaking into an answer machine which
  3993. accepts d+1 digits, values 0 to n-1.  We wish to find the minimal universal
  3994. code that will allow us access to any such answering machine.
  3995.  
  3996. Let us construct a digraph G = (V,E), where the n^d vertices are labelled
  3997. with a d sequence of digits.  Notation: let ▌v_{i,1},v_{i,2},...,v_{i,d}¿
  3998. denote the labelling on node v_i.  An edge e = (v_i, v_j) is in E iff for k
  3999. in 1, ..., d-1: v_{i,k+1} = v_{j,k}, i.e., the last d-1 digits in the
  4000. labelling of the initial vertex of e is identical with the first d-1 digits
  4001. in the labelling of the terminal vertex of e.  We associate with each edge a
  4002. value, t(e) = v_{j,d}, the last digit in the labelling of the terminal
  4003. vertex.
  4004.  
  4005. The intuition goes as follows:  we are going to perform a Euler circuit of
  4006. the digraph, where the label on the current vertex gives the last d digits
  4007. in the output sequence so far.  If we make a transition on edge e, we output
  4008. the tone/digit t(e) as the next output value, thus preserving the invariant
  4009. on the labelling.
  4010.  
  4011. How do we know that a Euler circuit exists?  Simple:  a connected digraph
  4012. has an Euler circuit iff for all vertices v: indegree(v) = outdegree(v).
  4013. This property is trivially true for this digraph.
  4014.  
  4015. So, in order to generate a universal code for the AM, we simply output 0^d
  4016. (to satisfy the precondition for being in vertex ▌0,...,0¿), and perform an
  4017. Euler circuit starting at node ▌0,...,0¿.
  4018.  
  4019. Now, the total length of the universal sequence is just the number of edges
  4020. traversed in the Euler circuit plus the initial precondition sequence, or
  4021. n^d * n + d (number of vertices times the out-degree) or n^{d+1} + d.  That
  4022. this is a minimal sequence is obvious.
  4023.  
  4024. Next, let us consider the machine AM' where the security code is of the form
  4025. ▌0...n-1¿^d ▌0...m-1¿, i.e., d digits ranging from 0 to n-1, followed by a
  4026. terminal digit ranging from 0 to m-1, m < n.
  4027.  
  4028. We build a digraph G = (V, E) similar to the construction above, except for
  4029. the following:  an edge e is in E iff t(e) in 0 to m-1.  This digraph is
  4030. clearly non-Eulerian.  In particular, there are two classes of vertices:
  4031.  
  4032. (1) v is of the form ▌0...n-1¿^{d-1} ▌0...m-1¿  (``fat'' vertices)
  4033.  
  4034.         and
  4035.  
  4036. (2) v is of the form ▌0...n-1¿^{d-1} ▌m...n-1¿  (``thin'' vertices)
  4037.  
  4038. Observations:  there are (n^{d-1} * m) fat vertices, and (n^{d-1} * (n-m))
  4039. thin vertices.  All vertices have out-degree of m.  Fat vertices have
  4040. in-degrees of n, and thin vertices have in-degrees of 0.  Color all the
  4041. edges blue.
  4042.  
  4043. The question now becomes:  can we put a bound on how many new (red) edges
  4044. must we add to G in order to make a blue edge covering path possible?
  4045. (Instead of thinking of edges being traversed multiple times in the blue
  4046. edge covering path, we allow multiple edges between vertices and allow each
  4047. edge to be traversed once.)  Note that, in this procedure, we add edges only
  4048. if it is allowed (the vertex labelling constraint).  We will first obtain a
  4049. lower bound on the length of a blue covering circuit, and then transform it
  4050. into a bound for arbitrary blue covering paths.
  4051.  
  4052. Clearly, we must add at least (n-m)*(n^{d-1}*m) edges incident from the fat
  4053. vertices.  ▌ We need (n-m) new out-going edges for each of (n^{d-1}*m)
  4054. vertices to bring the out-degree up to the in-degree. ¿
  4055.  
  4056. Let us partition our vertices into sets.  Denote the range ▌0..m-1¿ by S,
  4057. the range ▌m..n-1¿ by L, and the range ▌0..n-1¿ by X.
  4058.  
  4059. Let S_0 = { v: v = ▌X^{d-1}S¿ }.  S_0 is just the set of fat vertices.
  4060. Define in(S_0) = number of edges from vertices not in S to vertices in S.
  4061. Define out(S_0) in the corresponding fashion, and let excess(S_0) =
  4062. in(S_0)-out(S_0).  Clearly, excess(S_0) = n^{d-1}m(n-m) from the argument
  4063. above.  Generalizing the requirement for Eulerian digraphs, we see that we
  4064. must add excess(S_0) edges from S_0 if the blue edges connected to/within
  4065. S_0 are to be covered by some circuit (edges may not be travered multiple
  4066. times -- we add parallel edges to handle that case).  In particular, edges
  4067. from S_0 will be incident on vertices of the form ▌X^{d-2}SX¿.  Furthermore,
  4068. they can not be ▌X^{d-2}SS¿ since that is a subset of S_0 and adding those
  4069. edges will not help excess(S_0).  ▌Now, these edges may be needed if we are
  4070. to have a circuit, but we do not consider them since they do not help
  4071. excess(S_0).¿ So, we are forced to add excess(S_0) edges from S_0 to S_1 = {
  4072. v: v = ▌X^{d-2}SL¿ }.  Color these newly added edges red.
  4073.  
  4074. Let us define in(S_1), out(S_1) and excess(S_1) as above for the modified
  4075. digraph, i.e., including the red excess(S_0) edges that we just added.
  4076. Clearly, in(S_1) = out(S_0) = n^{d-1}m(n-m), and out(S_1) = m*!S_1! =
  4077. m*n^{d-2}m(n-m), so excess(S_1) = n^{d-2}m(n-m)^2.  Consider S_0 union S_1.
  4078. We must add excess(S_1) edges to S_0 union S_1 to make it possible for the
  4079. digraph to be covered by a circuit, and these edges must go from {S_0 union
  4080. S_1} to S_2 = { v: v = ▌X^{d-3}SL^2¿ } by a similar argument as before.
  4081. Repeating this partitioning process, eventually we get to S_{d-1} = { v: v =
  4082. ▌SL^{d-1}¿ }, where union of S_0 to S_{d-1} will need edges to S_d = { v: v
  4083. = ▌L^d¿ }, where this process terminates.  Note that at this time,
  4084. excess(union of S_0 to S_{d-1}) = m(n-m)^d, but in(S_d) = 0 and out(S_d) =
  4085. m(n-m)^d, and the process terminates.
  4086.  
  4087. What have we shown?  Adding up blue edges and the red edges gives us a lower
  4088. bound on the total number of edges in a blue-edges covering circuit (not
  4089. necessarily Eulerian) in the complete digraph.  This comes out to be
  4090. n^{d+1}-(n-m)^{d+1} edges.
  4091.  
  4092. Next, we note that if we had an optimal path covering all the blue edges, we
  4093. can transform it into a circuit by adding d edges.  So, a minimal path can
  4094. be no more than d edges shorter than the minimal circuit covering all blue
  4095. edges.  ▌Otherwise, we add d extra edges to make it into a shorter circuit.¿
  4096.  
  4097. So the shortest blue covering path through the digraph is at least
  4098. n^{d+1}-{n-m}^{d+1}-d.  With an initial pre-condition sequence of length d
  4099. (to establish the transition invariant), the shortest universal answering
  4100. machine sequence is of length at least n^{d+1}-(n-m)^{d+1}.
  4101.  
  4102. While this has not been that constructive, it is easy to see that we can
  4103. achieve this bound.  If we looked at the vertices in each of the S_i's, we
  4104. just add exactly the edges to S_{i+1} and no more.  The resultant digraph
  4105. would be Eulerian, and to find the minimal path we need only start at the
  4106. vertex labelled ▌{n-1}^d¿, find the Euler circuit, and omit the last d edges
  4107. from the tour.
  4108.  
  4109. ==> combinatorics/gossip.p <==
  4110. n people each know a different piece of gossip.  They can telephone each other
  4111. and exchange all the information they know (so that after the call they both
  4112. know anything that either of them knew before the call).  What is the smallest
  4113. number of calls needed so that everyone knows everything?
  4114.  
  4115. ==> combinatorics/gossip.s <==
  4116. 1 for n=2
  4117. 3 for n=3
  4118. 2n-4 for n>=4
  4119.  
  4120. This can be achieved as follows: choose four professors (A, B, C, and D) as
  4121. the "core group".  Each professor outside the core group phones a member of
  4122. the core group (it doesn't matter which); this takes n-4 calls.  Now the
  4123. core group makes 4 calls: A-B, C-D, A-C, and B-D.  At this point, each
  4124. member of the core group knows everything.  Now, each person outside the
  4125. core group calls anybody who knows everything; this again requires n-4
  4126. calls, for a total of 2n-4.
  4127.  
  4128. The solution to the "gossip problem" has been published several times:
  4129.  
  4130.         1.  R. Tidjeman, "On a telephone problem", Nieuw Arch. Wisk. 3
  4131.                 (1971), 188-192.
  4132.  
  4133.         2.  B. Baker and R. Shostak, "Gossips and telephones", Discrete
  4134.                 Math. 2 (1972), 191-193.
  4135.  
  4136.         3.  A. Hajnal, E. C. Milner, and E. Szemeredi, "A cure for the
  4137.                 telephone disease", Canad Math. Bull 15 (1976), 447-450.
  4138.  
  4139.         4. Kleitman and Shearer, Disc. Math. 30 (1980), 151-156.
  4140.  
  4141.         5.  R. T. Bumby, "A problem with telephones", Siam J. Disc. Meth. 2
  4142.                 (1981), 13-18.
  4143.  
  4144. ==> combinatorics/grid.dissection.p <==
  4145. How many (possibly overlapping) squares are in an mxn grid?
  4146.  
  4147. ==> combinatorics/grid.dissection.s <==
  4148. Given an n*m grid with n > m.
  4149.  
  4150. Orient the grid so n is its width.  Divide the grid into two portions,
  4151. an m*m square on the left and an (n-m)*m rectangle on the right.
  4152. Count the squares that have their upper right-hand corners in the
  4153. m*m square.  There are m^2 of size 1*1, (m-1)^2 of size 2*2, ...
  4154. up to 1^2 of size m*m.  Now look at the n-m columns of lattice points
  4155. in the rectangle on the right, in which we find upper right-hand
  4156. corners of squares not yet counted.  For each column we count m new
  4157. 1*1 squares, m-1 new 2*2 squares, ... up to 1 new m*m square.
  4158.  
  4159. Combining all these counts in summations:
  4160.  
  4161.          m                 m
  4162. total = sum i^2 + (n - m) sum i
  4163.         i=1               i=1
  4164.  
  4165.         (2m + 1)(m + 1)m   (n - m)(m + 1)m
  4166.       = ---------------- + ---------------
  4167.                 6                 2
  4168.  
  4169.       = (3n - m + 1)(m + 1)m/6
  4170.  
  4171. -- David Karr
  4172.  
  4173. ==> combinatorics/subsets.p <==
  4174. Out of the set of integers 1,...,100 you are given ten different
  4175. integers.  From this set, A, of ten integers you can always find two
  4176. disjoint subsets, S & T, such that the sum of elements in S equals the
  4177. sum of elements in T.  Note: S union T need not be all ten elements of
  4178. A.  Prove this.
  4179.  
  4180. ==> combinatorics/subsets.s <==
  4181. First, a couple of points:
  4182.  
  4183. (1) All empty subsets of the 10 integers are disjoint and have the same sum.
  4184.     This doesn't make for a very interesting problem.  Thus, we impose the
  4185.     additional restriction that S and T be non-empty.
  4186. (2) The 10 integers must be pairwise distinct.  Consider, e.g., the 10
  4187.     integers 1, 1, 1, 1, 1, 1, 1, 1, 1, and 1.  There are no non-empty
  4188.     disjoint subsets with equal sums.
  4189.  
  4190. Proof of puzzle:
  4191.  
  4192. There are 2^10 = 1,024 subsets of the 10 integers, but there can be only 901
  4193. possible sums, the number of integers between the minimum and maximum sums.
  4194. With more subsets than possible sums, there must exist at least one sum that
  4195. corresponds to at least two subsets.  Call two subsets with equal sums A and B.
  4196. Let C = A intersect B; define S = A - C, T = B - C.  Then S is disjoint from T,
  4197. and sum(S) = sum(A-C) = sum(A) - sub(C) = sum(B) - sum(C) = sum(B-C) = sum(T).
  4198. QED
  4199.  
  4200. ==> cryptology/Beale.p <==
  4201. What are the Beale ciphers?
  4202.  
  4203. ==> cryptology/Beale.s <==
  4204. The Beale ciphers are one of the greatest unsolved puzzles of all time.
  4205. About 100 years ago, a fellow by the name of Beale supposedly buried two
  4206. wagons-full of silver-coin filled pots in Bedford County, near Roanoke.
  4207. There are local rumors about the treasure being buried near Bedford Lake.
  4208.  
  4209. He wrote three encoded letters telling what was buried, where it was buried,
  4210. and who it belonged to.  He entrusted these three letters to a friend and went
  4211. west.  He was never heard from again.
  4212.  
  4213. Several years later, someone examined the letters and was able to break the
  4214. code used in the second letter.  The code used either the text from the
  4215. Declaration of Independence.  A number in the letter indicated which word
  4216. in the document was to be used.  The first letter of that word replaced the
  4217. number.  For example, if the first three words of the document were "We
  4218. hold these truths", the number 3 in the letter would represent the letter t.
  4219.  
  4220. One of the remaining letters supposedly contains directions on how to find
  4221. the treasure.  To date, no one has solved the code.  It is believed that
  4222. both of the remaining letters are encoded using either the same document
  4223. in a different way, or another very public document.
  4224.  
  4225. For those interested, write to:
  4226.         The Beale Cypher Association
  4227.         P.O. Box 975
  4228.         Beaver Falls, PA 15010
  4229.  
  4230. Item #904 is the 1885 pamphlet version ($5.00).  #152 is the
  4231. Cryptologia article by Gillogly that argues the hoax side ($2.00).
  4232. A year's membership is $25, and includes 4 newsletters.
  4233.  
  4234. TEXT for part 1
  4235.  
  4236.                The Locality of the Vault.
  4237.  
  4238. 71,194,38,1701,89,76,11,83,1629,48,94,63,132,16,111,95,84,341
  4239. 975,14,40,64,27,81,139,213,63,90,1120,8,15,3,126,2018,40,74
  4240. 758,485,604,230,436,664,582,150,251,284,308,231,124,211,486,225
  4241. 401,370,11,101,305,139,189,17,33,88,208,193,145,1,94,73,416
  4242. 918,263,28,500,538,356,117,136,219,27,176,130,10,460,25,485,18
  4243. 436,65,84,200,283,118,320,138,36,416,280,15,71,224,961,44,16,401
  4244. 39,88,61,304,12,21,24,283,134,92,63,246,486,682,7,219,184,360,780
  4245. 18,64,463,474,131,160,79,73,440,95,18,64,581,34,69,128,367,460,17
  4246. 81,12,103,820,62,110,97,103,862,70,60,1317,471,540,208,121,890
  4247. 346,36,150,59,568,614,13,120,63,219,812,2160,1780,99,35,18,21,136
  4248. 872,15,28,170,88,4,30,44,112,18,147,436,195,320,37,122,113,6,140
  4249. 8,120,305,42,58,461,44,106,301,13,408,680,93,86,116,530,82,568,9
  4250. 102,38,416,89,71,216,728,965,818,2,38,121,195,14,326,148,234,18
  4251. 55,131,234,361,824,5,81,623,48,961,19,26,33,10,1101,365,92,88,181
  4252. 275,346,201,206,86,36,219,324,829,840,64,326,19,48,122,85,216,284
  4253. 919,861,326,985,233,64,68,232,431,960,50,29,81,216,321,603,14,612
  4254. 81,360,36,51,62,194,78,60,200,314,676,112,4,28,18,61,136,247,819
  4255. 921,1060,464,895,10,6,66,119,38,41,49,602,423,962,302,294,875,78
  4256. 14,23,111,109,62,31,501,823,216,280,34,24,150,1000,162,286,19,21
  4257. 17,340,19,242,31,86,234,140,607,115,33,191,67,104,86,52,88,16,80
  4258. 121,67,95,122,216,548,96,11,201,77,364,218,65,667,890,236,154,211
  4259. 10,98,34,119,56,216,119,71,218,1164,1496,1817,51,39,210,36,3,19
  4260. 540,232,22,141,617,84,290,80,46,207,411,150,29,38,46,172,85,194
  4261. 39,261,543,897,624,18,212,416,127,931,19,4,63,96,12,101,418,16,140
  4262. 230,460,538,19,27,88,612,1431,90,716,275,74,83,11,426,89,72,84
  4263. 1300,1706,814,221,132,40,102,34,868,975,1101,84,16,79,23,16,81,122
  4264. 324,403,912,227,936,447,55,86,34,43,212,107,96,314,264,1065,323
  4265. 428,601,203,124,95,216,814,2906,654,820,2,301,112,176,213,71,87,96
  4266. 202,35,10,2,41,17,84,221,736,820,214,11,60,760
  4267.  
  4268.  
  4269.  
  4270. TEXT for part 2
  4271.  
  4272.                 (no title exists for this part)
  4273.  
  4274. 115,73,24,807,37,52,49,17,31,62,647,22,7,15,140,47,29,107,79,84
  4275. 56,239,10,26,811,5,196,308,85,52,160,136,59,211,36,9,46,316,554
  4276. 122,106,95,53,58,2,42,7,35,122,53,31,82,77,250,196,56,96,118,71
  4277. 140,287,28,353,37,1005,65,147,807,24,3,8,12,47,43,59,807,45,316
  4278. 101,41,78,154,1005,122,138,191,16,77,49,102,57,72,34,73,85,35,371
  4279. 59,196,81,92,191,106,273,60,394,620,270,220,106,388,287,63,3,6
  4280. 191,122,43,234,400,106,290,314,47,48,81,96,26,115,92,158,191,110
  4281. 77,85,197,46,10,113,140,353,48,120,106,2,607,61,420,811,29,125,14
  4282. 20,37,105,28,248,16,159,7,35,19,301,125,110,486,287,98,117,511,62
  4283. 51,220,37,113,140,807,138,540,8,44,287,388,117,18,79,344,34,20,59
  4284. 511,548,107,603,220,7,66,154,41,20,50,6,575,122,154,248,110,61,52,33
  4285. 30,5,38,8,14,84,57,540,217,115,71,29,84,63,43,131,29,138,47,73,239
  4286. 540,52,53,79,118,51,44,63,196,12,239,112,3,49,79,353,105,56,371,557
  4287. 211,505,125,360,133,143,101,15,284,540,252,14,205,140,344,26,811,138
  4288. 115,48,73,34,205,316,607,63,220,7,52,150,44,52,16,40,37,158,807,37
  4289. 121,12,95,10,15,35,12,131,62,115,102,807,49,53,135,138,30,31,62,67,41
  4290. 85,63,10,106,807,138,8,113,20,32,33,37,353,287,140,47,85,50,37,49,47
  4291. 64,6,7,71,33,4,43,47,63,1,27,600,208,230,15,191,246,85,94,511,2,270
  4292. 20,39,7,33,44,22,40,7,10,3,811,106,44,486,230,353,211,200,31,10,38
  4293. 140,297,61,603,320,302,666,287,2,44,33,32,511,548,10,6,250,557,246
  4294. 53,37,52,83,47,320,38,33,807,7,44,30,31,250,10,15,35,106,160,113,31
  4295. 102,406,230,540,320,29,66,33,101,807,138,301,316,353,320,220,37,52
  4296. 28,540,320,33,8,48,107,50,811,7,2,113,73,16,125,11,110,67,102,807,33
  4297. 59,81,158,38,43,581,138,19,85,400,38,43,77,14,27,8,47,138,63,140,44
  4298. 35,22,177,106,250,314,217,2,10,7,1005,4,20,25,44,48,7,26,46,110,230
  4299. 807,191,34,112,147,44,110,121,125,96,41,51,50,140,56,47,152,540
  4300. 63,807,28,42,250,138,582,98,643,32,107,140,112,26,85,138,540,53,20
  4301. 125,371,38,36,10,52,118,136,102,420,150,112,71,14,20,7,24,18,12,807
  4302. 37,67,110,62,33,21,95,220,511,102,811,30,83,84,305,620,15,2,108,220
  4303. 106,353,105,106,60,275,72,8,50,205,185,112,125,540,65,106,807,188,96,110
  4304. 16,73,32,807,150,409,400,50,154,285,96,106,316,270,205,101,811,400,8
  4305. 44,37,52,40,241,34,205,38,16,46,47,85,24,44,15,64,73,138,807,85,78,110
  4306. 33,420,505,53,37,38,22,31,10,110,106,101,140,15,38,3,5,44,7,98,287
  4307. 135,150,96,33,84,125,807,191,96,511,118,440,370,643,466,106,41,107
  4308. 603,220,275,30,150,105,49,53,287,250,208,134,7,53,12,47,85,63,138,110
  4309. 21,112,140,485,486,505,14,73,84,575,1005,150,200,16,42,5,4,25,42
  4310. 8,16,811,125,160,32,205,603,807,81,96,405,41,600,136,14,20,28,26
  4311. 353,302,246,8,131,160,140,84,440,42,16,811,40,67,101,102,194,138
  4312. 205,51,63,241,540,122,8,10,63,140,47,48,140,288
  4313.  
  4314. CLEAR for part 2, made human readable.
  4315.  
  4316. I have deposited in the county of Bedford about four miles from
  4317. Bufords in an excavation or vault six feet below the surface
  4318. of the ground the following articles belonging jointly to
  4319. the parties whose names are given in number three herewith.
  4320. The first deposit consisted of ten hundred and fourteen pounds
  4321. of gold and thirty eight hundred and twelve pounds of silver
  4322. deposited Nov eighteen nineteen.  The second was made Dec
  4323. eighteen twenty one and consisted of nineteen hundred and seven
  4324. pounds of gold and twelve hundred and eighty eight of silver,
  4325. also jewels obtained in St. Louis in exchange to save transportation
  4326. and valued at thirteen ▌t¿housand dollars.  The above
  4327. is securely packed i▌n¿ ▌i¿ron pots with iron cov▌e¿rs.  Th▌e¿ vault
  4328. is roughly lined with stone and the vessels rest on solid stone
  4329. and are covered ▌w¿ith others.  Paper number one describes th▌e¿
  4330. exact locality of the va▌u¿lt so that no difficulty will be had
  4331. in finding it.
  4332.  
  4333. CLEAR for part 2, using only the first 480 words of the
  4334. Declaration of Independence, then blanks filled in by
  4335. inspection.  ALL mistakes shown were caused by sloppy
  4336. encryption.
  4337.     0----5----10---15---20---25---30---35---40---45---
  4338.   0 ihavedepositedinthecountyofbedfordaboutfourmilesfr
  4339.  50 ombufordsinanexcavationorvaultsixfeetbelowthesurfa
  4340. 100 ceofthegroundthefollowingarticlesbelongingjointlyt
  4341. 150 othepartieswhosenamesaregiveninnumberthreeherewith
  4342. 200 thefirstdepositconsistcdoftenhundredandfourteenpou
  4343. 250 ndsofgoldandthirtyeighthundredandtwelvepoundsofsil
  4344. 300 verdepositednoveighteennineteenthesecondwasmadedec
  4345. 350 eighteentwentyoneandconsistedofnineteenhundredands
  4346. 400 evenpoundsofgoldandtwelvehundredandeightyeightofsi
  4347. 450 lveralsojewelsobtainedinstlouisinexchangetosavetra
  4348. 500 nsportationandvaluedatthirteenrhousanddollarstheab
  4349. 550 oveissecurelypackeditronpotswithironcovtrsthtvault
  4350. 600 isroughlylinedwithstoneandthevesselsrestonsolidsto
  4351. 650 neandarecovereduithotherspapernumberonedescribesth
  4352. 700 cexactlocalityofthevarltsothatnodifficultywillbeha
  4353. 750 dinfindingit
  4354.  
  4355.  
  4356. TEXT for part 3
  4357.  
  4358.                 Names and Residences.
  4359.  
  4360. 317,8,92,73,112,89,67,318,28,96,107,41,631,78,146,397,118,98
  4361. 114,246,348,116,74,88,12,65,32,14,81,19,76,121,216,85,33,66,15
  4362. 108,68,77,43,24,122,96,117,36,211,301,15,44,11,46,89,18,136,68
  4363. 317,28,90,82,304,71,43,221,198,176,310,319,81,99,264,380,56,37
  4364. 319,2,44,53,28,44,75,98,102,37,85,107,117,64,88,136,48,154,99,175
  4365. 89,315,326,78,96,214,218,311,43,89,51,90,75,128,96,33,28,103,84
  4366. 65,26,41,246,84,270,98,116,32,59,74,66,69,240,15,8,121,20,77,80
  4367. 31,11,106,81,191,224,328,18,75,52,82,117,201,39,23,217,27,21,84
  4368. 35,54,109,128,49,77,88,1,81,217,64,55,83,116,251,269,311,96,54,32
  4369. 120,18,132,102,219,211,84,150,219,275,312,64,10,106,87,75,47,21
  4370. 29,37,81,44,18,126,115,132,160,181,203,76,81,299,314,337,351,96,11
  4371. 28,97,318,238,106,24,93,3,19,17,26,60,73,88,14,126,138,234,286
  4372. 297,321,365,264,19,22,84,56,107,98,123,111,214,136,7,33,45,40,13
  4373. 28,46,42,107,196,227,344,198,203,247,116,19,8,212,230,31,6,328
  4374. 65,48,52,59,41,122,33,117,11,18,25,71,36,45,83,76,89,92,31,65,70
  4375. 83,96,27,33,44,50,61,24,112,136,149,176,180,194,143,171,205,296
  4376. 87,12,44,51,89,98,34,41,208,173,66,9,35,16,95,8,113,175,90,56
  4377. 203,19,177,183,206,157,200,218,260,291,305,618,951,320,18,124,78
  4378. 65,19,32,124,48,53,57,84,96,207,244,66,82,119,71,11,86,77,213,54
  4379. 82,316,245,303,86,97,106,212,18,37,15,81,89,16,7,81,39,96,14,43
  4380. 216,118,29,55,109,136,172,213,64,8,227,304,611,221,364,819,375
  4381. 128,296,1,18,53,76,10,15,23,19,71,84,120,134,66,73,89,96,230,48
  4382. 77,26,101,127,936,218,439,178,171,61,226,313,215,102,18,167,262
  4383. 114,218,66,59,48,27,19,13,82,48,162,119,34,127,139,34,128,129,74
  4384. 63,120,11,54,61,73,92,180,66,75,101,124,265,89,96,126,274,896,917
  4385. 434,461,235,890,312,413,328,381,96,105,217,66,118,22,77,64,42,12
  4386. 7,55,24,83,67,97,109,121,135,181,203,219,228,256,21,34,77,319,374
  4387. 382,675,684,717,864,203,4,18,92,16,63,82,22,46,55,69,74,112,134
  4388. 186,175,119,213,416,312,343,264,119,186,218,343,417,845,951,124
  4389. 209,49,617,856,924,936,72,19,28,11,35,42,40,66,85,94,112,65,82
  4390. 115,119,233,244,186,172,112,85,6,56,38,44,85,72,32,47,63,96,124
  4391. 217,314,319,221,644,817,821,934,922,416,975,10,22,18,46,137,181
  4392. 101,39,86,103,116,138,164,212,218,296,815,380,412,460,495,675,820
  4393. 952
  4394.  
  4395.  
  4396.  
  4397.  
  4398. Evidence in favor of a hoax-
  4399.    . Too many players.
  4400.    . Inflated quantities of treasure.
  4401.    . Many discrepancies exist in all documents.
  4402.    . The Declaration of Independence is too hokey a key.
  4403.    . Part 3 (list of 30 names) considered too little text.
  4404.    . W.F. Friedman couldn't crack it.
  4405.    . Why even encrypt parts 1 & 3?
  4406.    . Why use multi-part text, and why different keys for each part?
  4407.    . Difficult to keep treasure in ground if 30 men know where it was buried.
  4408.    . Who'd leave it with other than your own family?
  4409.    . The Inn Keeper waited an extra 10 years before opening box with
  4410.       ciphers in it?  Who would do this, curiousity runs too deep in
  4411.       humans?
  4412.    . Why did anybody waste time deciphering paper 2, which had no title?
  4413.       1 & 3 had titles] These should have been deciphered first?
  4414.    . Why not just one single letter?
  4415.    . Statistical analysis show 1&3 similar in very obscure ways, that
  4416.       2 differs.  Did somebody else encipher it?  And why?
  4417.       Check length of keytexts, and forward/backward next word
  4418.       displacement selections.
  4419.    . Who could cross the entire country with that much gold and only
  4420.       10 men and survive back then?
  4421.    . Practically everybody who visited New Mexico before 1821, left
  4422.       by way of the Pearly Gates, as the Spanish got almost every
  4423.       tourist:-)
  4424.  
  4425.  
  4426. References:
  4427.  
  4428.    "The Beale Treasure: A History of a Mystery", by Peter Viemeister,
  4429.        Bedord, VA: Hamilton's, 1987.  ISBN: 0-9608598-3-7.  230 pages.
  4430.    "The Codebreakers", by David Kahn, pg 771, CCN 63-16109.
  4431.       1967.
  4432.    "Gold in the Blue Ridge, The True Story of the Beale Treasure",
  4433.       by P.B. Innis & Walter Dean Innis, Devon Publ. Co., Wash, D.C.
  4434.       1973.
  4435.     "Signature Simulation and Certain Cryptographic Codes", Hammer,
  4436.         Communications of the ACM, 14 (1), January 1971, pp. 3-14.
  4437.     "How did TJB Encode B2?", Hammer, Cryptologia, 3 (1), Jan. 1979, pp. 9-15.
  4438.     "Second Order Homophonic Ciphers", Hammer, Cryptologia, 12 (1) Jan. 1988,
  4439.         pp 11-20.
  4440.  
  4441. ==> cryptology/Feynman.p <==
  4442. What are the Feynman ciphers?
  4443.  
  4444. ==> cryptology/Feynman.s <==
  4445. When I was a graduate student at Caltech, Professor Feynman showed me three
  4446. samples of code that he had been challenged with by a fellow scientist at
  4447. Los Alamos and which he had not been able to crack.  I also was unable to
  4448. crack them.  I posted them to Usenet and Jack C. Morrison of JPL cracked
  4449. the first one.  It is a simple transposition cipher: split the text into
  4450. 5-column pieces, then read from lower right upward.  What results are the
  4451. opening lines of Chaucer's Canterbury Tales in Middle English.
  4452.  
  4453. 1. Easier
  4454. MEOTAIHSIBRTEWDGLGKNLANEA
  4455. INOEEPEYSTNPEUOOEHRONLTIR
  4456. OSDHEOTNPHGAAETOHSZOTTENT
  4457. KEPADLYPHEODOWCFORRRNLCUE
  4458. EEEOPGMRLHNNDFTOENEALKEHH
  4459. EATTHNMESCNSHIRAETDAHLHEM
  4460. TETRFSWEDOEOENEGFHETAEDGH
  4461. RLNNGOAAEOCMTURRSLTDIDORE
  4462. HNHEHNAYVTIERHEENECTRNVIO
  4463. UOEHOTRNWSAYIFSNSHOEMRTRR
  4464. EUAUUHOHOOHCDCHTEEISEVRLS
  4465. KLIHIIAPCHRHSIHPSNWTOIISI
  4466. SHHNWEMTIEYAFELNRENLEERYI
  4467. PHBEROTEVPHNTYATIERTIHEEA
  4468. WTWVHTASETHHSDNGEIEAYNHHH
  4469. NNHTW
  4470.  
  4471. 2. Harder
  4472. XUKEXWSLZJUAXUNKIGWFSOZRAWURO
  4473. RKXAOSLHROBXBTKCMUWDVPTFBLMKE
  4474. FVWMUXTVTWUIDDJVZKBRMCWOIWYDX
  4475. MLUFPVSHAGSVWUFWORCWUIDUJCNVT
  4476. TBERTUNOJUZHVTWKORSVRZSVVFSQX
  4477. OCMUWPYTRLGBMCYPOJCLRIYTVFCCM
  4478. UWUFPOXCNMCIWMSKPXEDLYIQKDJWI
  4479. WCJUMVRCJUMVRKXWURKPSEEIWZVXU
  4480. LEIOETOOFWKBIUXPXUGOWLFPWUSCH
  4481.  
  4482. 3. New Message
  4483. WURVFXGJYTHEIZXSQXOBGSV
  4484. RUDOOJXATBKTARVIXPYTMYA
  4485. BMVUFXPXKUJVPLSDVTGNGOS
  4486. IGLWURPKFCVGELLRNNGLPYT
  4487. FVTPXAJOSCWRODORWNWSICL
  4488. FKEMOTGJYCRRAOJVNTODVMN
  4489. SQIVICRBICRUDCSKXYPDMDR
  4490. OJUZICRVFWXIFPXIVVIEPYT
  4491. DOIAVRBOOXWRAKPSZXTZKVR
  4492. OSWCRCFVEESOLWKTOBXAUXV
  4493. B
  4494.  
  4495. Chris Cole
  4496. Peregrine Systems
  4497. uunet]peregrine]chris
  4498.  
  4499. ==> cryptology/Voynich.p <==
  4500. What are the
  4501. ==> cryptology/Voynich.s <==
  4502. The Voynich Manuscript is a manuscript that first surfaced in the court of
  4503. Rudolf II (Holy Roman Emperor), who bought it for some large number of
  4504. gold pieces (600?).  Rudolf was interested in the occult, and the strange
  4505. characters and bizarre illustrations suggested that it had some deep
  4506. mystical/magical significance.  After Rudolf's court broke up, the
  4507. manuscript was sent to (if memory serves) Athanasius Kircher, with nobody
  4508. on the list having been able to read it.  It ended up in a chest of other
  4509. manuscripts in the Villa Mondragone ▌?¿ in Italy, and was discovered there
  4510. by Wilfred Voynich, a collector, in about 1910 or so.  He took it to a
  4511. linguist who wasn't a cryptanalyst, who identified it as a work by the
  4512. 12th century monk Roger Bacon and produced extended bogus decryptions based
  4513. on shorthand characters he saw in it.  A great deal of effort by the best
  4514. cryptanalysts in the country hasn't resulted in any breakthrough.  William
  4515. F. Friedman (arguably the best) thought it was written in an artificial
  4516. language.  I believe the manuscript is currently in the Beinecke Rare
  4517. Book Collection at ▌Harvard?¿.
  4518.  
  4519. Mary D'Imperio's paper is scholarly and detailed, and provides an
  4520. excellent starting point for anyone who is interested in the subject.
  4521. David Kahn's "The Codebreakers" has enough detail to tell you if you're
  4522. interested; it also has one or more plates showing the script and some
  4523. illustrations.  I believe D'Imperio's monograph has been reprinted by
  4524. Aegean Park Press.  A number of people have published their own ideas
  4525. about it, including Brumbaugh, without anybody agreeing.  A recent
  4526. publication from Aegean Park Press offers another decryption; I haven't
  4527. seen that one.
  4528.  
  4529. If you want *my* guess, it's a hoax made up by Edmund Kelley and an
  4530. unnamed co-conspirator and sold to Rudolf through the reputation of John
  4531. Dee (Queen Elizabeth I's astrologer).
  4532. --
  4533.         Jim Gillogly
  4534.         {hplabs, ihnp4}]sdcrdcf]randvax]jim
  4535.         jim@rand-unix.arpa
  4536.  
  4537. I read "Labyrinths of Reason" by William Poundstone recently.  I'm
  4538. posting this to so many newsgroups in part to recommend this book, which,
  4539. while of a popular nature, gives a good analysis of a wide variety of
  4540. paradoxes and philosophical quandaries, and is a great read.
  4541.  
  4542. Anyway, it mentions something called the Voynich manuscript, which is
  4543. now at Yale University's Beinecke Rare Book and Manuscript Library.
  4544. It's a real pity that I didn't know about this manuscript and go see it
  4545. when I was at Yale.
  4546.  
  4547. The Voynich manuscript is apparently very old.  It is a 232-page illuminated
  4548. manuscript written in a cipher that has never been cracked.  (That's
  4549. what Poundstone says - but see my hypothesis below.)  If I may quote
  4550. Poundstone's charming description, "Its author, subject matter, and
  4551. meaning are unfathomed mysteries.  No one even knows what language the
  4552. text would be in if you deciphered it.  Fanciful picutres of nude women,
  4553. peculiar inventions, and nonexistent flora and fauna tantalize the
  4554. would-be decipherer.   Color sketches in the exacting style of a
  4555. medieval herbal depict blossoms and spices that never spring from earth
  4556. and constellations found in no sky.  Plans for weird, otherworldly
  4557. plumbing show nymphets frolicking in sitz baths connected with
  4558. elbow-macaroni pipes.  The manuscript has the eerie quality of a
  4559. perfectly sensible book from an alternate universe."
  4560.  
  4561. There is a picture of one page in Poundstone's book.  It's written in a
  4562. flowing script using "approximately 21 curlicued symbols," some of which
  4563. are close to the Roman alphabet, but others of which supposedly resemble
  4564. Cyrillic, Glagolitic, and Ethiopian.  There is one tiny note in Middle
  4565. High German, not necessarily by the original author, talking about the
  4566. Herbal of Matthiolaus.  Some astrology charts in the manuscript have the
  4567. months labeled in Spanish.  "What appears to be a cipher table on the
  4568. first page has long faded into illegibility," and on the other hand, some
  4569. scholars have guessed that a barely legible inscription on the *last*
  4570. page is a key]
  4571.  
  4572. It is said to have "languished for a long time at the Jesuit College of
  4573. Mondragone in Frascati, Italy.  Then in 1912 it was purchased by Wilfred
  4574. M. Voynich, a Polish-born scientist and bibliophile...  Voynich was the
  4575. son-in-law of George Boole, the logician..."  A letter written in 1666
  4576. claims that Holy Roman Emperor Rudolf II of Bohemia (1552-1612) bought
  4577. the manuscript for 600 gold ducats.  He may have bought it from Dr.
  4578. John Dee, the famous astrologer.  Rudolf thought the manuscript was
  4579. written by Roger Bacon]   ▌Wouldn't it more likely have been written by
  4580. Dee, out to make a fast ducat?¿
  4581.  
  4582. "Many of the most talented military code breakers of this century have
  4583. tried to decipher it as a show of prowess.  Herbert Yardley, the
  4584. American code expert who solved the German cipher in WW1 and who cracked
  4585. a Japanese diplomatic cipher without knowing the Japanese language,
  4586. failed with the Voynich manuscript.  So did John Manly, who unscrambled
  4587. the Waberski cipher, and William Friedman, who defeated the Japanese
  4588. "purple code" of the 1940's.  Computers have been drafted into the
  4589. effort in recent years, to no avail."
  4590.  
  4591. Poundstone goes on to describe a kook, Newbold, who was apparently driven
  4592. batty in his attempt to crack the manuscript.  He then mentions that one
  4593. Leo Levitov also claimed in 1987 to crack the cipher, saying that it was
  4594. the text of a 12th-century cult of Isis worshipers, and that it
  4595. describes a method of euthanasia by opening a vein in a warm bathtub,
  4596. among other morbid matters.  According to Levitov's translation the text
  4597. begins:
  4598.  
  4599. "ones treat the dying each the man lying deathly ill the one person who
  4600. aches Isis each that dies treats the person"
  4601.  
  4602. Poundstone rejects this translation.
  4603.  
  4604. According to Poundstone, a William Bennett (see below) has analysed the
  4605. text with a computer and finds that its entropy is less than any known
  4606. European language, and closer to those of Polynesian languages.
  4607.  
  4608. My wild hypothesis, on the basis solely of the evidence above, is this.
  4609. Perhaps the text was meant to be RANDOM.  Of course humans are lousy at
  4610. generating random sequences.  So I'm wondering how attempted random
  4611. sequences (written in a weird alphabet) would compare statistically with
  4612. the Voynich manuscript.
  4613.  
  4614. Anyway, the only source Poundstone seems to cite, other than the
  4615. manuscript itself, is Leo Levitov's "Solution of the Voynich Manuscript,
  4616. A Liturgical Manual for the Endura Rite of the Cathari Heresy, the Cult
  4617. of Isis," Laguna Hills, Calif., Aegean Park Press, 1987, and William
  4618. Ralph Bennett Jr.'s "Scientific and Engineering Problem-Solving with the
  4619. Computer," Englewood Cliffs, New Jersey, Prentice-Hall 1976.
  4620.  
  4621. I will check the Bennett book; the other sounds hard to get ahold of]  I
  4622. would LOVE any further information about this bizarre puzzle.  If anyone
  4623. knows Bennett and can get samples of the Voynich manuscript in
  4624. electronic form, I would LOVE to get my hands on it.
  4625.  
  4626. Also, I would appreciate any information on:
  4627.  
  4628. Voynich
  4629. The Jesuit College of Mondragone
  4630. Rudolf II
  4631. The letter by Rudolf II (where is it? what does it say?)
  4632. The attempts of Yardley, Friedman and Manly
  4633. The Herbal of Matthiolaus
  4634.  
  4635. and, just for the heck of it, the "Waberski cipher" and the "purple
  4636. code"]
  4637.  
  4638. This whole business sounds like a quagmire into which angels would fear
  4639. to tread, but a fool like me finds it fascinating.
  4640.  
  4641.   -- sender's name lost (]?)
  4642.  
  4643. To counter a few hypotheses that were suggested here:
  4644.  
  4645. The Voynich Manuscript is certainly not strictly a polyalphabetic cipher
  4646. like Vigenere or Beaufort or (the one usually called) Porta, because of
  4647. the frequent repetitions of "words" at intervals that couldn't be
  4648. multiples of any key length.  I suppose one could imagine that it's an
  4649. interrupted key Vig or something, but common elements appearing at places
  4650. other than the beginnings of words would seem to rule that out.  The I.C.
  4651. is too high for a digraphic system like (an anachronistic) Playfair in any
  4652. European language.
  4653.  
  4654. One of the most interesting Voynich discoveries was made by Prescott Currier,
  4655. who discovered that the two different "hands" (visually distinct handwriting)
  4656. used different "dialects": that is, the frequencies for pages written in
  4657. one hand are different from those written in the other.  I confirmed this
  4658. observation by running some correlation coefficients on the digraph matrices
  4659. for the two kinds of pages.
  4660.  
  4661. W. F. Friedman ("The Man Who Broke Purple") thought the Voynich was
  4662. written in some artificial language.  If it's not a hoax, I don't see any
  4663. evidence to suggest he's wrong.  My personal theory (yeah, I've offered
  4664. too many of those lately) is that it was constructed by Edward Kelley,
  4665. John Dee's scryer, with somebody else's help (to explain the second
  4666. handwriting) -- perhaps Dee himself, although he's always struck me as a
  4667. credulous dupe of Kelley rather than a co-conspirator (cf the Angelic
  4668. language stuff).
  4669.  
  4670. The best source I know for the Voynich is Mary D'Imperio's monograph
  4671. "The Voynich Manuscript: An Elegant Enigma", which is available from
  4672. Aegean Park Press.
  4673.  
  4674. --
  4675.  Jim Gillogly
  4676.  jim@rand.org
  4677.  
  4678.  
  4679. Here's an update on the Voynich manuscript.  This will concentrate on
  4680. sources for information on the Voynich; later I will write a survey of
  4681. what I have found out so far.  I begin with some references to the
  4682. case, kindly sent to me by Karl Kluge (the first three) and Micheal Roe
  4683. <M.Roe@cs.ucl.ac.uk> (the rest).
  4684.  
  4685.  TITLE     Thirty-five manuscripts : including the St. Blasien psalter, the
  4686.            Llangattock hours, the Gotha missal, the Roger Bacon (Voynich)
  4687.            cipher ms.
  4688.            Catalogue ; 100
  4689.            35 manuscripts.
  4690.  CITATION  New York, N.Y. : H.P. Kraus, ▌1962¿ 86 p., lxvii p. of plates, ▌1¿
  4691.            leaf of plates : ill. (some col.), facsims. ; 36 cm.
  4692.  NOTES     "30 years, 1932-1962" (▌28¿ p.) in pocket. Includes indexes.
  4693.  SUBJECT   Manuscripts Catalogs.
  4694.            Illumination of books and manuscripts Catalogs.
  4695.  
  4696.  AUTHOR    Brumbaugh, Robert Sherrick, 1918-
  4697.  TITLE     The most mysterious manuscript : the Voynich "Roger Bacon" cipher
  4698.            manuscript / edited by Robert S. Brumbaugh.
  4699.  CITATION  Carbondale : Southern Illinois University Press, c1978. xii, 175 p.
  4700.            : ill. ; 22 cm.
  4701.  SUBJECT   Bacon, Roger, 1214?-1294.
  4702.            Ciphers.
  4703.  
  4704.  AUTHOR    D'Imperio, M. E.
  4705.  TITLE     The Voynich manuscript : an elegant enigma / M. E. D'Imperio.
  4706.  CITATION  Fort George E. Mead, Md. : National Security Agency/Central Security
  4707.            Service, 1978. ix, 140 p. : ill. ; 27 cm.
  4708.  NOTES     Includes index. Bibliography: p. 124-131.
  4709.  SUBJECT   Voynich manuscript.       ▌NOTE: see alternate publisher below]¿
  4710.  
  4711. @book{Bennett76,
  4712. author = "Bennett, William Ralph",
  4713. title = "Scientific and Engineering Problem Solving with the Computer",
  4714. address = "Englewood Cliffs, NJ",
  4715. publisher =  "Prentice-Hall",
  4716. year = 1976}
  4717.  
  4718. @book{dImperio78,
  4719. author = "D'Imperio, M E",
  4720. title = "The Voynich manuscript: An Elegant Enigma",
  4721. publisher= "Aegean Park Press",
  4722. year = 1978}
  4723.  
  4724. @article{Friedman62,
  4725. author = "Friedman, Elizebeth Smith",
  4726. title = "``The Most Mysterious Manuscript'' Still Mysterious",
  4727. booktitle = "Washington Post",
  4728. month = "August 5",
  4729. notes = "Section E",
  4730. pages = "1,5",
  4731. year = 1962}
  4732.  
  4733. @book{Kahn67,
  4734. author = "Kahn, David",
  4735. title = "The Codebreakers",
  4736. publisher = "Macmillan",
  4737. year = "1967"}
  4738.  
  4739. @article{Manly31,
  4740. author = "Manly, John Matthews",
  4741. title = "Roger Bacon and the Voynich MS",
  4742. boooktitle = "Speculum VI",
  4743. pages = "345--91",
  4744. year = 1931}
  4745.  
  4746. @article{ONeill44,
  4747. author = "O'Neill, Hugh",
  4748. title = "Botanical Remarks on the Voynich MS",
  4749. journal = "Speculum XIX",
  4750. pages = "p.126",
  4751. year = 1944}
  4752.  
  4753. @book{Poundstone88,
  4754. author = "Poundstone, W.",
  4755. title = "Labyrinths of Reason",
  4756. publisher = "Doubleday",
  4757. address = "New York",
  4758. month = "November",
  4759. year = 1988}
  4760.  
  4761. @article{Zimanski70,
  4762. author = "Zimanski, C.",
  4763. title = "William Friedman and the Voynich Manuscript",
  4764. journal = "Philological Quarterly",
  4765. year = "1970"}
  4766.  
  4767. @article{Guy91b,
  4768. author = "Guy, J. B. M.",
  4769. title = "Statistical Properties of Two Folios of the Voynich Manuscript",
  4770. journal = "Cryptologia",
  4771. volume = "XV",
  4772. number = "4",
  4773. pages = "pp. 207--218",
  4774. month = "July",
  4775. year = 1991}
  4776.  
  4777. @article{Guy91a,
  4778. author = "Guy, J. B. M.",
  4779. title = "Letter to the Editor Re Voynich Manuscript",
  4780. journal = "Cryptologia",
  4781. volume = "XV",
  4782. number = "3",
  4783. pages = "pp. 161--166",
  4784. year = 1991}
  4785.  
  4786. This is by no means a complete list.  It doesn't include Newbold's
  4787. (largely discredited) work, nor work by Feely and Stong.
  4788. In addition, there is the proposed decryption by Leo Levitov (also
  4789. largely discredited):
  4790.  
  4791. "Solution of the Voynich Manuscript: A Liturgical Manual for the
  4792. Endura Rite of the Cathari Heresy, the Cult of Isis_, available from
  4793. Aegean Park Press, P. O. Box 2837, Laguna Hills CA 92654-0837."
  4794.  
  4795. According to Earl Boebert, this book is reviewed in
  4796. Cryptologia XII, 1 (January 1988).  I should add that Brumbaugh's book
  4797. above gives a third, also largely discredited, decryption of the Voynich.
  4798.  
  4799. According to smb@att.ulysses.com, Aegean Park Press does mail-order
  4800. business and can be reached at the above address or at 714-586-8811
  4801. (an answering machine).
  4802.  
  4803. Micheal Roe has explained how one get microfilms of the whole
  4804. manuscript:
  4805.  
  4806. "The Beinecke Rare Book Library, Yale University sells a microfilm of the
  4807. manuscript. Their catalog number for the original is MS 408, ``The Voynich
  4808. `Roger Bacon' Cipher MS''. You should write to them.
  4809.  
  4810. The British Library ▌sic - should be Museum¿ has a photocopy of the MS
  4811. donated to them by John Manly circa 1931. They apparently lost it until
  4812. 12 March 1947, when it was entered in the catalogue (without
  4813. cross-references under Voynich, Manly, Roger Bacon or any other useful
  4814. keywords...)
  4815.  
  4816. It appears as ``MS Facs 461: Positive rotographs of a Cipher MS (folios 1-56)
  4817. acquired in 1912 by Wilfred M. Voynich in Southern Europe.'
  4818. Correspondance between Newbold, Manly and various British Museum experts
  4819. appears under ``MS Facs 439: Leaves of the Voynich MS, alleged to be in
  4820. Roger Bacon's cypher, with correspondence and other pertinent material''
  4821. See John Manly's 1931 article in Speculum and Newbold's book for what the
  4822. correspondance was about] There are also a number of press cuttings.
  4823.  
  4824. Both of these in are in the manuscript collection, for which special
  4825. permission is needed in addition to a normal British Library reader's pass."
  4826.  
  4827. Also, Jim Gillogly has been extremely kind in making available
  4828. part of the manuscript that was transcribed and keyed in by Mary
  4829. D'Imperio (see above), using Prescott Currier's notation.  It appears to
  4830. consist of 166 of the total 232 pages.    I hope to do some statistical
  4831. studies on this, and I encourage others to do the same and let me know
  4832. what they find]  As Jim notes, the file is pub/jim/voynich.tar.Z and is
  4833. available by anonymous ftp at rand.org.   I've had a little trouble with
  4834. this file at page 165, where I read "1650voynich   664" etc., with page
  4835. 166 missing.  If anyone else notes this let Jim or I know.
  4836.  
  4837. Jim says he has confirmed by correlations between digraph matrices the
  4838. discovery by Prescott Crurrier that the manuscript is written in two
  4839. visibly distinct hands.  These are marked "A" and "B" in the file
  4840. voynich.tar.Z.
  4841.  
  4842. Because of the possibility that the Voynich is nonsense, it would be
  4843. interesting to compare the Voynich to the Codex Seraphinianus, which
  4844. Kevin McCarty kindly reminded me of.  He writes:
  4845.  
  4846. "This is very odd.  I know nothing of the Voynich manuscript, but
  4847. I know of something which sounds very much like it and was created
  4848. by an Italian artist, who it now seems was probably influenced
  4849. by this work.   It a book titled "Codex Seraphinianus", written in
  4850. a very strange script.  The title page contains only the book's title
  4851. and the publisher's name: Abbeville Press, New York.  The only clues
  4852. in English (in *any* recognizable language) are some blurbs on the
  4853. dust jacket that identify it as a modern work of art, and the copyright
  4854. notice, in fine print, which reads
  4855.  
  4856. "Library of Congress Cataloging in Publication Data
  4857.  
  4858. Serafini, Luigi.
  4859.         Codex Seraphinianus.
  4860.  
  4861.  1. Imaginary Languages. 2. Imaginary societies.
  4862.  3. Encyclopedias and Dictionaries-- Miscellanea.
  4863.  
  4864. I. Title.
  4865. PN6381.S4       1983    818'.5407       83.-7076
  4866. ISBN 0-89659-428-9
  4867.  
  4868. First American Edition, 1983.
  4869. Copyright (c) 1981 by Franco Maria Ricci.  All rights reserved
  4870. by Abbeville Press.  No part of this book may be reproduced...
  4871. without permission in writing from the publisher. Inquiries should
  4872. be addressed to Abbeville Press, Inc., 505 Park Avenue, New York
  4873. 10022. Printed and bound in Italy."
  4874.  
  4875. The book is remarkable and bizarre.  It *looks* like an encyclopedia
  4876. for an imaginary world.  Page after page of beautiful pictures
  4877. of imaginary flora and fauna, with annotations and captions in
  4878. a completely strange script.  Machines, architecture, umm, 'situations',
  4879. arcane diagrams, implements, an archeologist pointing at a Rosetta stone
  4880. (with phony hieroglyphics), an article on penmanship (with unorthodox
  4881. pens), and much more, finally ending with a brief index.
  4882.  
  4883. The script in this work looks vaguely similar to the Voynich orthography
  4884. shown in Poundstone's book (I just compared them); the alphabets
  4885. look quite similar, but the Codex script is more cursive and less
  4886. bookish than Voynich.  It runs to about 200 pages, and probably
  4887. ought to provide someone two things:
  4888. - a possible explanation of what the Voynich manuscript is
  4889.   (a highly imaginative work of art)
  4890. - a textual work which looks like it was inspired by it and might
  4891.   provide an interesting comparison for statistical study."
  4892.  
  4893. I suppose it would be too much to hope that someone has already
  4894. transcribed parts of the Codex, but nonetheless, if anyone has any in
  4895. electronic form, I would love to have a copy for comparative statistics.
  4896.  
  4897. Jacques Guy kindly summarized his analysis (in Cryptologia, see above)
  4898. of the Voynich as follows:
  4899.  
  4900. "I transcribed the two folios in Bennett's book and submitted them to
  4901. letter-frequency counts, distinguishing word-initial, word-medial,
  4902. word-final, isolated, line-initial, and line-final positions. I also
  4903. submitted that transcription to Sukhotin's algorithm which, given a text
  4904. written in an alphabetical system, identifies which symbols are vowels and
  4905. which are consonants. The letter transcribed CT in Bennett's system came
  4906. out as a consonant, the one transcribed CC as vowel. Now it so happens
  4907. that CT is exactly the shape of the letter "t" in the Beneventan script
  4908. (used in medieval Spain and Northern Italy), and CC is exactly the shape
  4909. of "a" in that same script. I concluded that the author had a knowledge
  4910. of that script, and that the values of CT and CC probably were "t" and
  4911. "a". There's a lot more, but more shaky."
  4912.  
  4913. By popular demand I've put a machine-readable copy of the Voynich Manuscript
  4914. up for anonymous ftp:
  4915.  
  4916.         Host: rand.org
  4917.         File: pub/jim/voynich.tar.Z
  4918.  
  4919. It uses Prescott Currier's notation, and was transcribed by Mary D'Imperio.
  4920. If you use it in any analysis, be sure to give credit to D'Imperio, who put
  4921. in a lot of effort to get it right.
  4922.  
  4923. --
  4924.  Jim Gillogly
  4925.  jim@rand.org
  4926.  
  4927.  
  4928.  
  4929. This post is essentially a summary of the fruit of a short research
  4930. quest at the local library.
  4931.  
  4932. Brief description of the Voynich manuscript:
  4933.  
  4934. The Voynich manuscript was bought (in about 1586) by the Holy Roman
  4935. Emperor Rudolf II.  He believed it to be the work of Roger Bacon
  4936. an english 13th century philosopher.  The manuscript consisted of about
  4937. 200 pages with many illustrations.  It is believed that the manuscript
  4938. contains some secret scientific or magical knowledge since it is entirely
  4939. written in secret writing (presumably in cipher).
  4940.  
  4941. The Voynich Manuscript is often abbreviated "Voynich MS" in all of the
  4942. books I have read on Voynich.  This is done without explanation.  I
  4943. suppose it is just a convention started by the founding analysts of
  4944. the manuscript to call it that.
  4945.  
  4946. William R. Newbold, one of the original analysts of the Voynich MS after
  4947. Voynich, claims to have arrived at a partial decipherment of the entire
  4948. manuscript.   His book The Cipher of Roger Bacon ▌2¿ contains a history
  4949. of the unravelment of the cipher *and* keys to the cipher itself.  As well
  4950. as translations of several pages of the manuscript.
  4951.  
  4952. Newbold derives his decipherment rules through a study of the medeival
  4953. mind (which he is a leading scholar in) as well as the other writings
  4954. of Roger Bacon.  Says Newbold, ciphers in Roger Bacon's writings are not
  4955. new, as Bacon discusses in other works the need for monks to use
  4956. encipherment to protect their knowlege.
  4957.  
  4958. Newbold includes many partial decipherments from the Voynich MS but most of
  4959. them are presented in Latin only.
  4960.  
  4961. Newbolds deciphering rules (from The Cipher of Roger Bacon ▌1¿)
  4962. ---------------------------------------------------------------
  4963. 1. Syllabification: ▌double all but the first and last letters of each
  4964. word, and divide the product into biliteral groups or symbols.¿
  4965. 2. Translation: ▌translate these symbols into the alphabetic values¿
  4966. 3. Reversion: ▌change the alphabetic values to the phonetic values, by use
  4967. of the reversion alphabet¿
  4968. 4. Recomposition: ▌ rearrange the letters in order, and thus recompose the
  4969. true text¿.
  4970.  
  4971. The text I copied this from failed to note step 0 which was:
  4972. 0. Ignore. ▌ignore the actual shape of every symbol and analyze only the
  4973. (random?) properties of the direction of swirl and crosshatch patterns
  4974. of the characters when viewed under a microscope.  14 distinct contruction
  4975. patterns can be identified among the (much larger) set of symbols¿
  4976.  
  4977. John M. Manly in The Most Mysterious Manuscript ▌3¿, suggests that Newbold's
  4978. method of decipherment is totally invalid.  Manly goes on to show that it
  4979. is not difficult to obtain *ANY DESIRABLE* message from the Voynich MS
  4980. using Newbold's rules.  He shows that after fifteen minutes deciphering
  4981. a short sequence of letters he arrives at the plaintext message
  4982.         "Paris is lured into loving vestals..."
  4983. and quips that he will furnish a continuation of the translation upon
  4984. request]
  4985.  
  4986. The reason I have spent so much time explaining Newbold's method is that
  4987. Newbold presents the most convincing argument for how he arrived at his
  4988. conclusions.  Notwithstanding the fact that he invented the oija board of
  4989. deciphering systems.
  4990.  
  4991. Joseph Martin Feely, in his book on the Voynich MS ▌2¿ , claims to have
  4992. found the key to deciphering at least one page of the Voynich MS.  His entire
  4993. book on the topic of the Voynich manuscript is devoted to the deciphering of
  4994. the single page 78.  Feely presents full tables of translation of the page 78
  4995. from its written form into latin (and english).  It seems that Feely was using
  4996. the exhaustive analysis method to determine the key.
  4997.  
  4998. Feely suggests the following translation of (the first fiew lines of) page
  4999. 78 of the Voynich MS:
  5000.  
  5001. "the combined stream when well humidified, ramifies; afterward it is broken
  5002. down smaller; afterward, at a distance, into the fore-bladder it comes ▌1¿.
  5003. Then vesselled, it is after-a-while ruminated: well humidified it is
  5004. clothed with veinlets ▌2¿.  Thence after-a-bit they move down; tiny
  5005. teats they provide (or live upon) in the outpimpling of the veinlets.
  5006. They are impermiated; are thrown down below; they are ruminated; they are
  5007. feminized with the tiny teats.  .... "
  5008.  
  5009.         ... and so on for three more pages of "english plaintext".
  5010.  
  5011. The descriptions by Feely say that this text is accompanied in the Voynich MS
  5012. by an illustration that (he says) is unmistakably the internal female
  5013. reproductive organs (I saw the plate myself and they DO look like fallopian
  5014. tubes *AFTER* I read the explanation).
  5015.  
  5016. The most informative work that I found (I feel) was "The Most Mysterious
  5017. Manuscript".   Of the five books on Voynich that I found, this was the only
  5018. one that didn't claim to have found the key but was, rather, a collection
  5019. of essays on the history of the Voynich MS and criticisms of various attempts
  5020. by earlier scientists.  It was also the *latest* book that I was able to
  5021. consult, being published in 1978.
  5022.  
  5023. My impression from the black and white plates of the Voynich MS I've seen, are
  5024. that the illustrations are very weird when compared to other 'illuminated'
  5025. manuscripts of this time.  Particularly I would say that there is emphasis
  5026. on the female nude that is unusual for the art of this period.  I can't say
  5027. that I myself believe the images to have ANYTHING to do with the text.
  5028. My own conjecture is that the manuscript is a one-way encipherment.  A
  5029. cipher so clever that the inventor didn't even think of how it could be
  5030. deciphered.  Sorta like an /etc/passwd file.
  5031.  
  5032. Bibliography
  5033. ------------
  5034. 1. William R. Newbold. _The Cipher of Roger Bacon_Roland G Kent, ed.
  5035. University of Pennsylvania Press, 1928.
  5036. 2. Joseph Martin Feely. _Roger Bacon's Cipher: The Right Key Found_
  5037. Rochester N.Y.:Joseph Martin Feely, pub., 1943.
  5038. 3. _The Most Mysterious Manuscript_ Robert S. Brumbaugh, ed.  Southern Illinois
  5039. Press, 1978
  5040.  
  5041.  
  5042. Unix filters are so wonderful. Massaging the machine readable file, we find:
  5043.  
  5044. 4182 "words", of which 1284 are used more than once, 308 used 8+ times,
  5045. 184 used 15+ times, 23 used 100+ times.
  5046.  
  5047. Does this tell us anything about the language (if any) the text is written
  5048. in?
  5049.  
  5050. For those who may be interested, here are the 23 words used 100+ times:
  5051.  121 2
  5052.  115 4OFAE
  5053.  114 4OFAM
  5054.  155 4OFAN
  5055.  195 4OFC89
  5056.  162 4OFCC89
  5057.  101 4OFCC9
  5058.  189 89
  5059.  111 8AE
  5060.  492 8AM
  5061.  134 8AN
  5062.  156 8AR
  5063.  248 OE
  5064.  148 OR
  5065.  111 S9
  5066.  251 SC89
  5067.  142 SC9
  5068.  238 SOE
  5069.  150 SOR
  5070.  244 ZC89
  5071.  116 ZC9
  5072.  116 ZOE
  5073.  
  5074.  
  5075.  
  5076. Could someone email the Voynich Ms. ref list that appeared here not
  5077. very long ago? Thanks in advance...
  5078.  
  5079. Also... I came across the following ref that is fun(?):
  5080.  
  5081. The Voynich manuscript: an elegant enigma / M. E. D'Imperio
  5082. Fort George E. Mead, Md. : National Security Agency(])
  5083. Central Security Service(?), 1978. ix, 140 p. : ill. ; 27 cm.
  5084.  
  5085. The (?]) are mine... Sorry if this was already on the list, but the
  5086. mention of the NSA (and what's the CSS?) made it jump out at me...
  5087.  
  5088. --
  5089. Ron Carter ! rcarter@nyx.cs.du.edu  rcarter GEnie  70707.3047 CIS
  5090.   Director ! Center for the Study of Creative Intelligence
  5091. Denver, CO ! Knowledge is power. Knowledge to the people. Just say know.
  5092.  
  5093.  
  5094.  
  5095. Distribution: na
  5096. Organization: Wetware Diversions, San Francisco
  5097. Keywords:
  5098.  
  5099.  
  5100. From sci.archaeology:
  5101. >From: jamie@cs.sfu.ca (Jamie Andrews)
  5102. >Date: 16 Nov 91 00:49:08 GMT
  5103. >
  5104. >     It seems like the person who would be most likely to solve
  5105. >this Voynich manuscript cipher would have
  5106. >(a) knowledge of the modern techniques for solving more complex
  5107. >    ciphers such as Playfairs and Vigineres; and
  5108. >(b) knowledge of the possible contemporary and archaic languages
  5109. >    in which the plaintext could have been written.
  5110.  
  5111. An extended discussion of the Voynich Manuscript may be found in the
  5112. tape of the same name by Terence McKenna.  I'm not sure who is currently
  5113. publishing this particular McKenna tape but probably one of:
  5114. Dolphin Tapes, POB 71, Big Sur, CA 93920
  5115. Sounds True, 1825 Pearl St., Boulder, CO 80302
  5116. Sound Photosynthesis, POB 2111, Mill Valley, CA 94942
  5117.  
  5118. The Spring 1988 issue of Gnosis magazine contained an article by McKenna
  5119. giving some background of the Voynich Manuscipt and attempts to decipher
  5120. it, and  reviewing Leo Levitov's "Solution of the Voynich Manuscript"
  5121. (published in 1987 by Aegean Park Press, POB 2837 Laguna Hills, CA 92654).
  5122. Levitov's thesis is that the manuscript is the only surviving primary
  5123. document of the Cathar faith (exterminated on the orders of the Pope in
  5124. the Albigensian Crusade in the 1230s) and that it is in fact not
  5125. encrypted material but rather is a highly polyglot form of Medieval
  5126. Flemish with a large number of Old French and Old High German loan
  5127. words, written in a special script.
  5128.  
  5129. As far as I know Levitov's there has been no challenge to Levitov's
  5130. claims so far.
  5131.  
  5132.  
  5133.  
  5134. Michael Barlow, who had reviewed Levitov's book in Cryptologia, had sent me
  5135. photocopies of the pages where much of the language was described
  5136. (pp.21-31). I have just found them, and am looking at them now as I am
  5137. typing this. Incidentally, I do not believe this has anything to do with
  5138. cryptology proper, but the decipherment of texts in unknown languages. So
  5139. if you are into cryptography proper, skip this.
  5140.  
  5141. Looking at the "Voynich alphabet" pp.25-27, I made a list of the letters of
  5142. the Voynich language as Levitov interprets them, and I added phonetic
  5143. descriptions of the sounds I *think* Levitov meant to describe. Here it is:
  5144.  
  5145. Letter#  Phonetic              Phonetic descriptions
  5146.          (IPA)         in linguists' jargon:          in plain English:
  5147.  
  5148. 1       a           low open, central unrounded       a as in father
  5149.         e           mid close, front, unrounded       ay as in May
  5150.         O           mid open, back, rounded           aw as in law
  5151.                                                       or o as in got
  5152.                                                       (British
  5153.                                                       pronunciation)
  5154.  
  5155. 2       s           unvoiced dental fricative         s as in so
  5156. 3       d           voiced dental stop                d
  5157. 4       E           mid, front, unrounded             e as in wet
  5158. 5       f           unvoiced labiodental fricative    f
  5159. 6       i           short, high open, front,          i as in dim
  5160.                       unrounded
  5161. 7       i:          long, high, front, unrounded      ea as in weak
  5162.  
  5163. 8       i:E (?)     I can't make head nor tail of Levitov's
  5164.                     explanations. Probably like "ei" in "weird"
  5165.                     dragging along the "e": "weeeird"] (British
  5166.                     pronunciation, with a silent "r")
  5167. 9      C            unvoiced palatal fricative       ch in German ich
  5168. 10     k            unvoived velar stop              k
  5169.  
  5170. 11     l            lateral, can't be more precise from
  5171.                     description, probably like l in "loony"
  5172.  
  5173. 12     m            voiced bilabial nasal             m
  5174. 13     n            voiced dental nasal               n
  5175. 14     r (?)        cannot tell precisely from        Scottish r?
  5176.                     description                       Dutch r?
  5177. 15     t            no description; dental stop?      t
  5178. 16     t            another form for #15              t
  5179. 17     T (?)        no description                    th as in this?
  5180.                                                       th as in thick?
  5181. 18     TE (?)       again, no description
  5182.     or ET (?)
  5183. 19     v            voiced labiodental fricative      v as in rave
  5184. 20     v            ditto, same as #19                  ditto
  5185.  
  5186. (By now, you will have guessed what my conclusion about Levitov's
  5187. decipherment was)
  5188.  
  5189. In the column headed "Phonetic (IPA)" I have used capital letters for lack
  5190. of the special international phonetic symbols:
  5191.  
  5192. E for the Greek letter "epsilon"
  5193. O for the letter that looks like a mirror-image of "c"
  5194. C for c-cedilla
  5195. T for the Greek letter "theta"
  5196.  
  5197. The colon (:) means that the sound represented by the preceding letter is
  5198. long, e.g. "i:" is a long "i".
  5199.  
  5200. The rest, #21 to 25, are not "letters" proper, but represent groups
  5201. of two or more letters, just like #18 does. They are:
  5202.  
  5203. 21    av
  5204. 22a   Ev
  5205. 22b   vE
  5206. 23    CET
  5207. 24    kET
  5208. 25    sET
  5209.  
  5210. That gives us a language with 6 vowels: a (#1), e (#1 again), O (#1 again),
  5211. E (#4), i (#6), and i: (#7). Letter #8 is not a vowel, but a combination
  5212. of two vowels: i: (#7) and probably E (#4). Levitov writes that the
  5213. language is derived from Dutch. If so, it has lost the "oo" sound (English
  5214. spelling; "oe" in Dutch spelling), and the three front rounded vowels of
  5215. Dutch: u as in U ("you", polite), eu as in deur ("door"), u as in vlug
  5216. ("quick"). Note that out of six vowels, three are confused under the same
  5217. letter (#1), even though they sound very different from one another: a, e,
  5218. O. Just imagine that you had no way of distinguishing between "last",
  5219. "lest" and "lost" when writing in English, and you'll have a fair idea of
  5220. the consequences.
  5221.  
  5222. Let us look at the consonants now. I will put them in a matrix, with the
  5223. points of articulation in one dimension, and the manner of articulation in
  5224. the other (it's all standard procedure when analyzing a language). Brackets
  5225. around a letter will mean that I could not tell where to place it exactly,
  5226. and just took a guess.
  5227.  
  5228.                      labial     dental  palatal   velar
  5229.              nasal     m          n
  5230.        voiced stop                d
  5231.      unvoiced stop                t                 k
  5232.   voiced fricative     v         (T)
  5233. unvoiced fricative     f          s        C
  5234.            lateral                l
  5235.          trill (?)               (r)
  5236.  
  5237. Note that there are only twelve consonant sounds. That is unheard of for a
  5238. European language. No European language has so few consonant sounds.
  5239. Spanish, which has very few sounds (only five vowels), has seventeen
  5240. distinct consonants sounds, plus two semi-consonants. Dutch has from 18 to
  5241. 20 consonants (depending on speakers, and how you analyze the sounds.
  5242. Warning: I just counted them on the back of an envelope; I might have
  5243. missed one or two). What is also extraordinary in Levitov's language is
  5244. that it lacks a "g", and *BOTH* "b" and "p". I cannot think of one single
  5245. language in the world that lacks both "b" and "p". Levitov also says that
  5246. "m" occurs only word-finally, never at the beginning, nor in the middle of
  5247. a word. That's true: the letter he says is an "m" is always word-final in
  5248. the reproductions I have seen of the Voynich MS. But no language I know of
  5249. behaves like that. All have an "m" (except one American Indian language,
  5250. which is very famous for that, and the name of which escapes me right now),
  5251. but, if there is a position where "m" never appears in some languages, that
  5252. position is word-finally. Exactly the reverse of Levitov's language.
  5253.  
  5254. What does Levitov say about the origin of the language?
  5255.  
  5256. "The language was very much standardized. It was an application of a
  5257. polyglot oral tongue into a literary language which would be understandable
  5258. to people who did not understand Latin and to whom this language could be
  5259. read."
  5260.  
  5261. At first reading, I would dismiss it all as nonsense: "polyglot oral
  5262. tongue" means nothing in linguistics terms. But Levitov is a medical
  5263. doctor, so allowances must be made. The best meaning I can read into
  5264. "polyglot oral tongue" is "a language that had never been written before
  5265. and which had taken words from many different languages". That is perfectly
  5266. reasonable: English for one, has done that. Half its vocabulary is Norman
  5267. French, and some of the commonest words have non-Anglo-Saxon origins.
  5268. "Sky", for instance, is a Danish word. So far, so good.
  5269.  
  5270. Levitov continues: "The Voynich is actually a simple language because it
  5271. follows set rules and has a very limited vocabulary.... There is a
  5272. deliberate duality and plurality of words in the Voynich and much use of
  5273. apostrophism".
  5274.  
  5275. By "duality and plurality of words" Levitov means that the words are highly
  5276. ambiguous, most words having two or more different meanings. I can only
  5277. guess at what he means by apostrophism: running words together, leaving
  5278. bits out, as we do in English: can not --> cannot --> can't, is not -->
  5279. ain't.
  5280.  
  5281. Time for a tutorial in the Voynich language as I could piece it together
  5282. from Levitov's description. Because, according to Levitov, letter #1
  5283. represent 3 vowels sounds, I will represent it by just "a", but remember:
  5284. it can be pronounced a, e, or o. But I will distinguish, as does Levitov,
  5285. between the two letters which he says were both pronounced "v", using "v"
  5286. for letter #20 and "w" for letter #21.
  5287.  
  5288. Some vocabulary now. Some verbs first, which Levitov gives in the
  5289. infinitive. In the Voynich language the infinitive of verbs ends in -en,
  5290. just like in Dutch and in German. I have removed that grammatical ending in
  5291. the list which follows, and given probable etymologies in parentheses
  5292. (Levitov gives doesn't give any):
  5293.  
  5294. ad   = to aid, help  ("aid")
  5295. ak   = to ache, pain ("ache")
  5296. al   = to ail ("ail")
  5297. and  = to undergo the "Endura" rite ("End▌ura¿", probably)
  5298. d    = to die ("d▌ie¿")
  5299. fad  = to be for help (from  f= for and  ad=aid)
  5300. fal  = to fail ("fail")
  5301. fil  = to be for illness (from: f=for and il=ill)
  5302. il   = to be ill ("ill")
  5303. k    = to understand ("ken", Dutch and German "kennen" meaning "to know")
  5304. l    = to lie deathly ill, in extremis ("lie", "lay")
  5305. s    = to see ("see", Dutch "zien")
  5306. t    = to do, treat (German "tun" = to do)
  5307. v    = to will ("will" or Latin "volo" perhaps)
  5308. vid  = to be with death (from vi=with and d=die)
  5309. vil  = to want, wish, desire (German "willen")
  5310. vis  = to know  ("wit", German "wissen", Dutch "weten")
  5311. vit  = to know  (ditto)
  5312. viT  = to use   (no idea, Latin "uti" perhaps?)
  5313. vi   = to be the way (Latin "via")
  5314. eC   = to be each ("each")
  5315. ai:a = to eye, look at ("eye", "oog" in Dutch)
  5316. en   = to do (no idea)
  5317.        Example given by Levitov: enden "to do to death" made up of "en"
  5318.        (to do), "d" (to die) and "en" (infinitive ending). Well, to me,
  5319.        that's doing it the hard way. What's wrong with just "enden" = to
  5320.        end (German "enden", too])
  5321.  
  5322. More vocabulary:
  5323.  
  5324. em = he or they (masculine) ("him")
  5325. er = her or they (feminine) ("her")
  5326. eT = it or they ("it" or perhaps "they" or Dutch "het")
  5327. an = one ("one", Dutch "een")
  5328.  
  5329. "There are no declensions of nouns or conjugation of verbs. Only the
  5330. present tense is used" says Levitov.
  5331.  
  5332. Examples:
  5333.  
  5334. den  = to die (infinitive) (d = die, -en = infinitive)
  5335. deT = it/they die (d = die, eT = it/they)
  5336. diteT = it does die (d = die, t = do, eT = it/they, with an "i" added to
  5337.         make it easier to pronounce, which is quite common and natural
  5338.         in languages)
  5339.  
  5340. But Levitov contradicts himself immediately, giving another tense (known
  5341. as present progressive in English grammar):
  5342.  
  5343. dieT = it is dying
  5344.  
  5345. But I may be unfair there, perhaps it is a compound: d = die, i = is
  5346. ...-ing, eT = it/they.
  5347.  
  5348. Plurals are formed by suffixing "s" in one part of the MS, "eT" in another:
  5349. "ans" or "aneT" = ones.
  5350.  
  5351. More:
  5352.  
  5353. wians = we ones (wi = we, wie in Dutch, an = one, s = plural)
  5354. vian  = one way (vi = way, an = one)
  5355. wia   = one who (wi = who, a = one)
  5356. va    = one will (v = will, a = one)
  5357. wa    = who
  5358. wi    = who
  5359. wieT  = who, it (wi = who, eT = it)
  5360. witeT = who does it (wi = who, t = do, eT = it/they)
  5361. weT   = who it is (wi = who, eT = it, then loss of "i", giving "weT")
  5362. ker   = she understands (k = understand, er =she)
  5363.  
  5364. At this stage I would like to comment that we are here in the presence of a
  5365. Germanic language which behaves very, very strangely in the way of the
  5366. meanings of its compound words. For instance, "viden" (to be with death) is
  5367. made up of the words for "with", "die" and the infinitive suffix. I am sure
  5368. that Levitov here was thinking of a construction like German "mitkommen"
  5369. which means "to come along" (to "withcome"). I suppose I could say "Bitte,
  5370. sterben Sie mit" on the same model as "Bitte, kommen Sie mit" ("Come with
  5371. me/us, please), thereby making up a verb "mitsterben", but that would mean
  5372. "to die together with someone else", not "to be with death".
  5373.  
  5374. Let us see how Levitov translates a whole sentence. Since he does not
  5375. explain how he breaks up those compound words I have tried to do it using
  5376. the vocabulary and grammar he provides in those pages. My tentative
  5377. explanations are in parenthesis.
  5378.  
  5379. TanvieT faditeT wan aTviteT anTviteT atwiteT aneT
  5380.  
  5381. TanvieT = the one way (T = the (?), an = one, vi =way, eT = it)
  5382. faditeT = doing for help (f = for, ad = aid, i = -ing, t = do, eT = it)
  5383. wan     = person (wi/wa = who, an = one)
  5384. aTviteT = one that one knows (a = one, T = that, vit = know, eT = it.
  5385.           Here, Levitov adds one extra letter which is not in the text,
  5386.           getting "aTaviteT", which provide the second "one" of his
  5387.           translation)
  5388. anTviteT = one that knows (an =one, T = that, vit = know, eT = it)
  5389. atwiteT  = one treats one who does it (a = one, t = do, wi = who,
  5390.            t = do, eT = it. Literally: "one does ▌one¿ who does it".
  5391.            The first "do" is translated as "treat", the second "one" is
  5392.            added in by Levitov: he added one letter, which gives him
  5393.            "atawiteT")
  5394. aneT    =  ones (an = one, -eT = the plural ending)
  5395.  
  5396. Levitov's translation of the above in better English: "the one way for
  5397. helping a person who needs it, is to know one of the ones who do treat
  5398. one".
  5399.  
  5400. Need I say more? Does anyone still believe that Levitov's translations are
  5401. worth anything?
  5402.  
  5403. As an exercise, here is the last sentence on p.31, with its word-for-word
  5404. translation by Levitov. I leave you to work it out, and to figure out what
  5405. it might possibly mean. Good luck]
  5406.  
  5407. tvieT nwn anvit fadan van aleC
  5408.  
  5409. tvieT    = do the ways
  5410. nwn      = not who does (but Levitov adds a letter to make it "nwen")
  5411. anvit    = one knows
  5412. fadan    = one for help
  5413. van      = one will
  5414. aleC     = each ail
  5415.  
  5416. ==> cryptology/swiss.colony.p <==
  5417. What are the 1987 Swiss Colony ciphers?
  5418.  
  5419. ==> cryptology/swiss.colony.s <==
  5420. Did anyone solve the 1987 'Crypto-gift' contest that was run by
  5421. Swiss Colony?  My friend and I worked on it for 4 months, but
  5422. didn't get anywhere.  My friend solved the 1986 puzzle in
  5423. about a week and won $1000.  I fear that we missed some clue that
  5424. makes it incredibly easy to solve.  I'm including the code, clues
  5425. and a few notes for those of you so inclined to give it a shot.
  5426.  
  5427.                     197,333,318,511,824,
  5428.                     864,864,457,197,333,
  5429.                     824,769,372,769,864,
  5430.         865,457,153,824,511,223,845,318,
  5431.         489,953,234,769,703,489,845,703,
  5432.         372,216,457,509,333,153,845,333,
  5433.         511,864,621,611,769,707,153,333,
  5434.         703,197,845,769,372,621,223,333,
  5435.         197,845,489,953,223,769,216,223,
  5436.         769,769,457,153,824,511,372,223,
  5437.         769,824,824,216,865,845,153,769,
  5438.         333,704,511,457,153,333,824,333,
  5439.         953,372,621,234,953,234,865,703,
  5440.         318,223,333,489,944,153,824,769,
  5441.         318,457,234,845,318,223,372,769,
  5442.         216,894,153,333,511,611,
  5443.         769,704,511,153,372,621,
  5444.         197,894,894,153,333,953,
  5445.                 234,845,318,223
  5446.  
  5447.         CHRIS IS BACK WITH GOLD FOR YOU
  5448.           HIS RHYMES CONTAIN THE SECRET.
  5449.         YOU SCOUTS WHO'VE EARNED YOUR MERIT BADGE
  5450.           WILL QUICKLY LEARN TO READ IT.
  5451.         SO WHEN YOUR CHRISTMAS HAM'S ALL GONE
  5452.           AND YOU'RE READY FOR THE TUSSLE,
  5453.         BALL UP YOUR HAND INTO A FIST
  5454.           AND SHOW OUR MOUSE YOUR MUSCLE.
  5455.         PLEASE READ THESE CLUES WE LEAVE TO YOU
  5456.           BOTH FINE ONES AND THE COARSE;
  5457.         IF CARE IS USED TO HEED THEM ALL
  5458.           YOU'LL SUFFER NO REMORSE.
  5459.  
  5460. Notes:
  5461. The puzzle comes as a jigsaw that when assembled has the list of
  5462. numbers.  They are arranged as indicated on the puzzle, with commas.
  5463. The lower right corner has a drawing of 'Secret Agent Chris Mouse'.
  5464. He holds a box under his arm which looks like the box
  5465. the puzzle comes in.  The upper left
  5466. corner has the words 'NEW 1987 $50,000 Puzzle'.  The lower
  5467. left corner is empty.  The clues are printed on the
  5468. entry form in upper case, with the punctuation as shown.
  5469.  
  5470.                                         Ed Rupp
  5471.                                         ...]ut-sally]oakhill]ed
  5472.                                         Motorola, Inc., Austin Tx.
  5473.  
  5474.  
  5475.  
  5476. ==> decision/allais.p <==
  5477. The Allais Paradox involves the choice between two alternatives:
  5478.  
  5479.         A. 89% chance of an unknown amount
  5480.            10% chance of $1 million
  5481.            1% chance of $1 million
  5482.         B. 89% chance of an unknown amount (the same amount as in A)
  5483.            10% chance of $2.5 million
  5484.            1% chance of nothing
  5485.  
  5486. What is the rational choice?  Does this choice remain the same if the
  5487. unknown amount is $1 million?  If it is nothing?
  5488.  
  5489. ==> decision/allais.s <==
  5490. This is "Allais' Paradox".
  5491.  
  5492. Which choice is rational depends upon the subjective value of money.
  5493. Many people are risk averse, and prefer the better chance of $1
  5494. million of option A.  This choice is firm when the unknown amount is
  5495. $1 million, but seems to waver as the amount falls to nothing.  In the
  5496. latter case, the risk averse person favors B because there is not much
  5497. difference between 10% and 11%, but there is a big difference between
  5498. $1 million and $2.5 million.
  5499.  
  5500. Thus the choice between A and B depends upon the unknown amount, even
  5501. though it is the same unknown amount independent of the choice.  This
  5502. violates the "independence axiom" that rational choice between two
  5503. alternatives should depend only upon how those two alternatives
  5504. differ.
  5505.  
  5506. However, if the amounts involved in the problem are reduced to tens of
  5507. dollars instead of millions of dollars, people's behavior tends to
  5508. fall back in line with the axioms of rational choice.  People tend to
  5509. choose option B regardless of the unknown amount.  Perhaps when
  5510. presented with such huge numbers, people begin to calculate
  5511. qualitatively.  For example, if the unknown amount is $1 million the
  5512. options are:
  5513.  
  5514.         A. a fortune, guaranteed
  5515.         B. a fortune, almost guaranteed
  5516.            a tiny chance of nothing
  5517.  
  5518. Then the choice of A is rational.  However, if the unknown amount is
  5519. nothing, the options are:
  5520.  
  5521.         A. small chance of a fortune ($1 million)
  5522.            large chance of nothing
  5523.         B. small chance of a larger fortune ($2.5 million)
  5524.            large chance of nothing
  5525.  
  5526. In this case, the choice of B is rational.  The Allais Paradox then
  5527. results from the limited ability to rationally calculate with such
  5528. unusual quantities.  The brain is not a calculator and rational
  5529. calculations may rely on things like training, experience, and
  5530. analogy, none of which would be help in this case.  This hypothesis
  5531. could be tested by studying the correlation between paradoxical
  5532. behavior and "unusualness" of the amounts involved.
  5533.  
  5534. If this explanation is correct, then the Paradox amounts to little
  5535. more than the observation that the brain is an imperfect rational
  5536. engine.
  5537.  
  5538. ==> decision/division.p <==
  5539. N-Person Fair Division
  5540.  
  5541. If two people want to divide a pie but do not trust each other, they can
  5542. still ensure that each gets a fair share by using the technique that one
  5543. person cuts and the other person chooses. Generalize this technique
  5544. to more than two people. Take care to ensure that no one can be cheated
  5545. by a coalition of the others.
  5546.  
  5547.  
  5548. ==> decision/division.s <==
  5549. N-Person Fair Division
  5550.  
  5551. Number the people from 1 to N. Person 1 cuts off a piece of the pie.
  5552. Person 2 can either diminish the size of the cut off piece or pass.
  5553. The same for persons 3 through N. The last person to touch the piece
  5554. must take it and is removed from the process. Repeat this procedure
  5555. with the remaining N - 1 people, until everyone has a piece.
  5556. (cf. Luce and Raiffa, "Games and Decisions", Wiley, 1957, p. 366)
  5557.  
  5558. There is a cute result in combinatorics called the Marriage Theorem.
  5559. A village has n men and n women, such that for all 0 < k <= n and for any
  5560. set of k men there are at least k women, each of whom is in love with at least
  5561. one of the k men.  All of the men are in love with all of the women :-}.
  5562. The theorem asserts that there is a way to arrange the village into n
  5563. monogamous couplings.
  5564.  
  5565. The Marriage Theorem can be applied to the Fair Pie-Cutting Problem.
  5566.  
  5567. One player cuts the pie into n pieces.  Each of the players labels
  5568. some non-null subset of the pieces as acceptable to him.  For reasons
  5569. given below he should "accept" each piece of size > 1/n, not just the
  5570. best piece(s).  The pie-cutter is required to "accept" all of the pieces.
  5571.  
  5572. Given a set S of players let S' denote the set of pie-pieces
  5573. acceptable to at least one player in S.  Let t be the size of the largest
  5574. set (T) of players satisfying  !T! > !T'!.  If there is no such set, the
  5575. Marriage Theorem can be applied directly.  Since the pie-cutter accepts
  5576. every piece we know that  t < n.
  5577.  
  5578. Choose  !T! - !T'!  pieces at random from outside T', glue them
  5579. together with the pieces in T' and let the players in T repeat the game
  5580. with this smaller (t/n)-size pie.  This is fair since they all rejected
  5581. the other n-t pieces, so they believe this pie is larger than t/n.
  5582.  
  5583. The remaining n-t players can each be assigned one of the remaining
  5584. n-t pie-pieces without further ado due to the Marriage Theorem.  (Otherwise
  5585. the set T above was not maximal.)
  5586.  
  5587. ==> decision/dowry.p <==
  5588. Sultan's Dowry
  5589.  
  5590. A sultan has granted a commoner a chance to marry one of his hundred
  5591. daughters. The commoner will be presented the daughters one at a time.
  5592. When a daughter is presented, the commoner will be told the daughter's
  5593. dowry. The commoner has only one chance to accept or reject each
  5594. daughter; he cannot return to a previously rejected daughter.
  5595. The sultan's catch is that the commoner may only marry the daughter with
  5596. the highest dowry. What is the commoner's best strategy assuming
  5597. he knows nothing about the distribution of dowries?
  5598.  
  5599.  
  5600. ==> decision/dowry.s <==
  5601. Solution
  5602.  
  5603. Since the commoner knows nothing about the distribution of the dowries,
  5604. the best strategy is to wait until a certain number of daughters have
  5605. been presented then pick the highest dowry thereafter. The exact number to
  5606. skip is determined by the condition that the odds that the highest dowry
  5607. has already been seen is just greater than the odds that it remains to be
  5608. seen AND THAT IF IT IS SEEN IT WILL BE PICKED. This amounts to finding the
  5609. smallest x such that:
  5610.         x/n > x/n * (1/(x+1) + ... + 1/(n-1)).
  5611. Working out the math for n=100 and calculating the probability gives:
  5612. The commoner should wait until he has seen 37 of the daughters,
  5613. then pick the first daughter with a dowry that is bigger than any
  5614. preceding dowry. With this strategy, his odds of choosing the daughter
  5615. with the highest dowry are surprisingly high: about 37%.
  5616. (cf. F. Mosteller, "Fifty Challenging Problems in Probability with Solutions",
  5617. Addison-Wesley, 1965, #47; "Mathematical Plums", edited by Ross Honsberger,
  5618. pp. 104-110)
  5619.  
  5620. ==> decision/envelope.p <==
  5621. Someone has prepared two envelopes containing money.  One contains twice as
  5622. much money as the other.  You have decided to pick one envelope, but then the
  5623. following argument occurs to you:  Suppose my chosen envelope contains $X,
  5624. then the other envelope either contains $X/2 or $2X.  Both cases are
  5625. equally likely, so my expectation if I take the other envelope is
  5626. .5 * $X/2 + .5 * $2X = $1.25X, which is higher than my current $X, so I
  5627. should change my mind and take the other envelope.  But then I can apply the
  5628. argument all over again.  Something is wrong here]  Where did I go wrong?
  5629.  
  5630. In a variant of this problem, you are allowed to peek into the envelope
  5631. you chose before finally settling on it.  Suppose that when you peek you
  5632. see $100.  Should you switch now?
  5633.  
  5634. ==> decision/envelope.s <==
  5635. Let's follow the argument carefully, substituting real numbers for
  5636. variables, to see where we went wrong.  In the following, we will assume
  5637. the envelopes contain $100 and $200.  We will consider the two equally
  5638. likely cases separately, then average the results.
  5639.  
  5640. First, take the case that X=$100.
  5641.  
  5642. "I have $100 in my hand.  If I exchange I get $200.  The value of the exchange
  5643. is $200.  The value from not exchanging is $100.  Therefore, I gain $100
  5644. by exchanging."
  5645.  
  5646. Second, take the case that X=$200.
  5647.  
  5648. "I have $200 in my hand.  If I exchange I get $100.  The value of the exchange
  5649. is $100.  The value from not exchanging is $200.  Therefore, I lose $100
  5650. by exchanging."
  5651.  
  5652. Now, averaging the two cases, I see that the expected gain is zero.
  5653.  
  5654. So where is the slip up?  In one case, switching gets X/2 ($100), in the
  5655. other case, switching gets 2X ($200), but X is different in the two
  5656. cases, and I can't simply average the two different X's to get 1.25X.
  5657. I can average the two numbers ($100 and $200) to get $150, the expected
  5658. value of switching, which is also the expected value of not switching,
  5659. but I cannot under any circumstances average X/2 and 2X.
  5660.  
  5661. This is a classic case of confusing variables with constants.
  5662.  
  5663. OK, so let's consider the case in which I looked into the envelope and
  5664. found that it contained $100.  This pins down what X is: a constant.
  5665.  
  5666. Now the argument is that the odds of $50 is .5 and the odds of $200
  5667. is .5, so the expected value of switching is $125, so we should switch.
  5668. However, the only way the odds of $50 could be .5 and the odds of $200
  5669. could be .5 is if all integer values are equally likely.  But any
  5670. probability distribution that is finite and equal for all integers
  5671. would sum to infinity, not one as it must to be a probability distribution.
  5672. Thus, the assumption of equal likelihood for all integer values is
  5673. self-contradictory, and leads to the invalid proof that you should
  5674. always switch.  This is reminiscent of the plethora of proofs that 0=1;
  5675. they always involve some illegitimate assumption, such as the validity
  5676. of division by zero.
  5677.  
  5678. Limiting the maximum value in the envelopes removes the self-contradiction
  5679. and the argument for switching.  Let's see how this works.
  5680.  
  5681. Suppose all amounts up to $1 trillion were equally likely to be
  5682. found in the first envelope, and all amounts beyond that would never
  5683. appear.  Then for small amounts one should indeed switch, but not for
  5684. amounts above $500 billion.  The strategy of always switching would pay
  5685. off for most reasonable amounts but would lead to disastrous losses for
  5686. large amounts, and the two would balance each other out.
  5687.  
  5688. For those who would prefer to see this worked out in detail:
  5689. Assume the smaller envelope is uniform on ▌$0,$M¿, for some value
  5690. of $M.  What is the expectation value of always switching?  A quarter of
  5691. the time $100 >= $M (i.e. 50% chance $X is in ▌$M/2,$M¿ and 50% chance
  5692. the larger envelope is chosen).  In this case the expected switching
  5693. gain is -$50 (a loss).  Thus overall the always switch policy has an
  5694. expected (relative to $100) gain of (3/4)*$50 + (1/4)*(-$50) = $25.
  5695. However the expected absolute gain (in terms of M) is:
  5696.   / M
  5697.   !    g f(g) dg,   ▌ where f(g) = (1/2)*Uniform▌0,M)(g) +
  5698.   /-M                              (1/2)*Uniform(-M,0¿(g). ¿
  5699.  
  5700.   = 0.  QED.
  5701.  
  5702. OK, so always switching is not the optimal switching strategy.  Surely
  5703. there must be some strategy that takes advantage of the fact that we
  5704. looked into the envelope and we know something we did not know before
  5705. we looked.
  5706.  
  5707. Well, if we know the maximum value $M that can be in the smaller envelope,
  5708. then the optimal decision criterion is to switch if $100 < $M, otherwise stick.
  5709. The reason for the stick case is straightforward. The reason for the
  5710. switch case is due to the pdf of the smaller envelope being twice as
  5711. high as that of the larger envelope over the range ▌0,$M). That is, the
  5712. expected gain in switching is (2/3)*$100 + (1/3)*(-$50) = $50.
  5713.  
  5714. What if we do not know the maximum value of the pdf?  You can exploit
  5715. the "test value" technique to improve your chances.  The trick here is
  5716. to pick a test value T.  If the amount in the envelope is less than the
  5717. test value, switch; if it is more, do not.  This works in that if T happens
  5718. to be in the range ▌M,2M¿ you will make the correct decision.  Therefore,
  5719. assuming the unknown pdf is uniform on ▌0,M¿, you are slightly better off
  5720. with this technique.
  5721.  
  5722. Of course, the pdf may not even be uniform, so the "test value" technique
  5723. may not offer much of an advantage.  If you are allowed to play the game
  5724. repeatedly, you can estimate the pdf, but that is another story...
  5725.  
  5726. ==> decision/exchange.p <==
  5727. At one time, the Mexican and American dollars were devalued by 10 cents on each
  5728. side of the border (i.e. a Mexican dollar was 90 cents in the US, and a US
  5729. dollar was worth 90 cents in Mexico).  A man walks into a bar on the American
  5730. side of the border, orders 10 cents worth of beer, and tenders a Mexican dollar
  5731. in change.  He then walks across the border to Mexico, orders 10 cents worth of
  5732. beer and tenders a US dollar in change.  He continues this throughout the day,
  5733. and ends up dead drunk with the original dollar in his pocket.
  5734.  
  5735. Who pays for the drinks?
  5736.  
  5737. ==> decision/exchange.s <==
  5738. The man paid for all the drinks.  But, you say, he ended up with the same
  5739. amount of money that he started with]  However, as he transported Mexican
  5740. dollars into Mexico and US dollars into the US, he performed "economic work"
  5741. by moving the currency to a location where it was in greater demand (and thus
  5742. valued higher).  The earnings from this work were spent on the drinks.
  5743.  
  5744. Note that he can only continue to do this until the Mexican bar runs out
  5745. of US dollars, or the US bar runs out of Mexican dollars, i.e., until
  5746. he runs out of "work" to do.
  5747.  
  5748. ==> decision/newcomb.p <==
  5749. Newcomb's Problem
  5750.  
  5751. A being put one thousand dollars in box A and either zero or one million
  5752. dollars in box B and presents you with two choices:
  5753.         (1) Open box B only.
  5754.         (2) Open both box A and B.
  5755. The being put money in box B only if it predicted you will choose option (1).
  5756. The being put nothing in box B if it predicted you will do anything other than
  5757. choose option (1) (including choosing option (2), flipping a coin, etc.).
  5758.  
  5759. Assuming that you have never known the being to be wrong in predicting your
  5760. actions, which option should you choose to maximize the amount of money you
  5761. get?
  5762.  
  5763.  
  5764. ==> decision/newcomb.s <==
  5765. This is "Newcomb's Paradox".
  5766.  
  5767. You are presented with two boxes: one certainly contains $1000 and the
  5768. other might contain $1 million.  You can either take one box or both.
  5769. You cannot change what is in the boxes.  Therefore, to maximize your
  5770. gain you should take both boxes.
  5771.  
  5772. However, it might be argued that you can change the probability that
  5773. the $1 million is there.  Since there is no way to change whether the
  5774. million is in the box or not, what does it mean that you can change
  5775. the probability that the million is in the box?  It means that your
  5776. choice is correlated with the state of the box.
  5777.  
  5778. Events which proceed from a common cause are correlated.  My mental
  5779. states lead to my choice and, very probably, to the state of the box.
  5780. Therefore my choice and the state of the box are highly correlated.
  5781. In this sense, my choice changes the "probability" that the money is
  5782. in the box.  However, since your choice cannot change the state of the
  5783. box, this correlation is irrelevant.
  5784.  
  5785. The following argument might be made: your expected gain if you take
  5786. both boxes is (nearly) $1000, whereas your expected gain if you take
  5787. one box is (nearly) $1 million, therefore you should take one box.
  5788. However, this argument is fallacious.  In order to compute the
  5789. expected gain, one would use the formulas:
  5790.  
  5791.         E(take one) =   $0 * P(predict take both ! take one) +
  5792.                                 $1,000,000 * P(predict take one ! take one)
  5793.         E(take both) =  $1,000 * P(predict take both ! take both) +
  5794.                                 $1,001,000 * P(predict take one ! take both)
  5795.  
  5796. While you are given that P(do X ! predict X) is high, it is not given
  5797. that P(predict X ! do X) is high.  Indeed, specifying that P(predict X
  5798. ! do X) is high would be equivalent to specifying that the being could
  5799. use magic (or reverse causality) to fill the boxes.  Therefore, the
  5800. expected gain from either action cannot be determined from the
  5801. information given.
  5802.  
  5803.  
  5804. ==> decision/prisoners.p <==
  5805. Three prisoners on death row are told that one of them has been chosen
  5806. at random for execution the next day, but the other two are to be
  5807. freed.  One privately begs the warden to at least tell him the name of
  5808. one other prisoner who will be freed.  The warden relents: 'Susie will
  5809. go free.'  Horrified, the first prisoner says that because he is now
  5810. one of only two remaining prisoners at risk, his chances of execution
  5811. have risen from one-third to one-half]  Should the warden have kept his
  5812. mouth shut?
  5813.  
  5814. ==> decision/prisoners.s <==
  5815. Each prisoner had an equal chance of being the one chosen to be
  5816. executed.  So we have three cases:
  5817.  
  5818. Prisoner executed:         A    B    C
  5819. Probability of this case: 1/3  1/3  1/3
  5820.  
  5821. Now, if A is to be executed, the warden will randomly choose either B or C,
  5822. and tell A that name.  When B or C is the one to be executed, there is only
  5823. one prisoner other than A who will not be executed, and the warden will always
  5824. give that name.  So now we have:
  5825.  
  5826. Prisoner executed:  A    A    B    C
  5827. Name given to A:    B    C    C    B
  5828. Probability:       1/6  1/6  1/3  1/3
  5829.  
  5830. We can calculate all this without knowing the warden's answer.
  5831. When he tells us B will not be executed, we eliminate the middle two
  5832. choices above.  Now, among the two remaining cases, C is twice
  5833. as likely as A to be the one executed.  Thus, the probability that
  5834. A will be executed is still 1/3, and C's chances are
  5835.  
  5836. ==> decision/red.p <==
  5837. I show you a shuffled deck of standard playing cards, one card at a
  5838. time.  At any point before I run out of cards, you must say "RED]".
  5839. If the next card I show is red (i.e. diamonds or hearts), you win.  We
  5840. assume I the "dealer" don't have any control over what the order of
  5841. cards is.
  5842.  
  5843. The question is, what's the best strategy, and what is your
  5844. probability of winning ?
  5845.  
  5846. ==> decision/red.s <==
  5847. If a deck has n cards, r red and b black, the best strategy wins
  5848. with a probability of r/n.  Thus, you can say "red" on the first card,
  5849. the last card, or any other card you wish.
  5850. Proof by induction on n.  The statement is clearly true for one-card decks.
  5851. Suppose it is true for n-card decks, and add a red card.
  5852. I will even allow a nondeterministic strategy, meaning you say "red"
  5853. on the first card with probability p.  With probability 1-p,
  5854. you watch the first card go by, and then apply the "optimal" strategy
  5855. to the remaining n-card deck, since you now know its composition.
  5856. The odds of winning are therefore: p * (r+1)/(n+1)  +
  5857.         (1-p) * ((r+1)/(n+1) * r/n  +  b/(n+1) * (r+1)/n).
  5858. After some algebra, this becomes (r+1)/(n+1) as expected.
  5859. Adding a black card yields: p * r/(n+1)  +
  5860.         (1-p) * (r/(n+1) * (r-1)/n  +  (b+1)/(n+1) * r/n).
  5861. This becomes r/(n+1) as expected.
  5862.  
  5863. ==> decision/rotating.table.p <==
  5864. Four glasses are placed upside down in the four corners of a square
  5865. rotating table.  You wish to turn them all in the same direction,
  5866. either all up or all down.  You may do so by grasping any two glasses
  5867. and, optionally, turning either over.  There are two catches:  you are
  5868. blindfolded and the table is spun after each time you touch the
  5869. glasses.  How do you do it?
  5870. ==> decision/rotating.table.s <==
  5871. 1.  Turn two adjacent glasses up.
  5872. 2.  Turn two diagonal glasses up.
  5873. 3.  Pull out two diagonal glasses.  If one is down, turn it up and you're done.
  5874.     If not, turn one down and replace.
  5875. 4.  Take two adjacent glasses.  Invert them both.
  5876. 5.  Take two diagonal glasses.  Invert them both.
  5877.  
  5878. References
  5879.     Probing the Rotating Table"
  5880.     W. T. Laaser and L. Ramshaw
  5881.     _The Mathematical Gardner_,
  5882.     Wadsworth International, Belmont CA 1981.
  5883.  
  5884.         ... we will see that such a procedure exists if and
  5885.         only if the parameters k and n satisfy the inequality
  5886.         k >= (1-1/p)n, where p is the largest prime factor
  5887.         of n.
  5888.  
  5889. The paper mentions (without discussing) two other generalizations:
  5890. more than two orientations of the glasses (Graham and Diaconis)
  5891. and more symmetries in the table, e.g. those of a cube (Kim).
  5892.  
  5893. ==> decision/stpetersburg.p <==
  5894. What should you be willing to pay to play a game in which the payoff is
  5895. calculated as follows:  a coin is flipped until in comes up heads on the
  5896. nth toss and the payoff is set at 2^n dollars?
  5897.  
  5898. ==> decision/stpetersburg.s <==
  5899. Classical decison theory says that you should be willing to pay any
  5900. amount up to the expected value of the wager.  Let's calculate the
  5901. expected value:  The probability of winning at step n is 2^-n, and the
  5902. payoff at step n is 2^n, so the sum of the products of the
  5903. probabilities and the payoffs is:
  5904.  
  5905.     E = sum over n (2^-n * 2^n) = sum over n (1) = infinity
  5906.  
  5907. So you should be willing to pay any amount to play this game.  This is
  5908. called the "St. Petersburg Paradox."
  5909.  
  5910. The classical solution to this problem was given by Bernoulli.  He
  5911. noted that people's desire for money is not linear in the amount of
  5912. money involved.  In other words, people do not desire $2 million twice
  5913. as much as they desire $1 million.  Suppose, for example, that people's
  5914. desire for money is a logarithmic function of the amount of money.
  5915. Then the expected VALUE of the game is:
  5916.  
  5917.     E = sum over n (2^-n * C * log(2^n)) = sum over n (2^-n * C' * n) =  C''
  5918.  
  5919. Here the C's are constants that depend upon the risk aversion of the
  5920. player, but at least the expected value is finite.  However, it turns
  5921. out that these constants are usually much higher than people are really
  5922. willing to pay to play, and in fact it can be shown that any
  5923. non-bounded utility function (map from amount of money to value of
  5924. money) is prey to a generalization of the St. Petersburg paradox.  So
  5925. the classical solution of Bernoulli is only part of the story.
  5926.  
  5927. The rest of the story lies in the observation that bankrolls are always
  5928. finite, and this dramatically reduces the amount you are willing to bet
  5929. in the St. Petersburg game.
  5930.  
  5931. To figure out what would be a fair value to charge for playing the game
  5932. we must know the bank's resources.  Assume that the bank has 1 million
  5933. dollars (1*K*K = 2^20).  I cannot possibly win more than $1 million
  5934. whether I toss 20 tails in a row or 2000.
  5935.  
  5936. Therefore my expected amount of winning is
  5937.  
  5938.     E = sum n up to 20 (2^-n * 2^n) = sum n up to 20 (1) = $20
  5939.  
  5940. and my expected value of winning is
  5941.  
  5942.     E = sum n up to 20 (2^-n * C * log(2^n)) = some small number
  5943.  
  5944. This is much more in keeping with what people would really pay to
  5945. play the game.
  5946.  
  5947. Incidentally, T.C. Fry suggested this change to the problem in 1928
  5948. (see W.W.R. Ball, Mathematical Recreations and Essays, N.Y.: Macmillan,
  5949. 1960, pp.  44-45).
  5950.  
  5951. The problem remains interesting when modified in this way,
  5952. for the following reason. For a particular value of the bank's
  5953. resources, let
  5954.  
  5955.       e denote the expected value of the player's winnings; and let
  5956.       p denote the probability that the player profits from the game, assuming
  5957.         the price of getting into the game is 0.8e (20% discount).
  5958.  
  5959. Note that the expected value of the player's profit is 0.2e.  Now
  5960. let's vary the bank's resources and observe how e and p change.  It
  5961. will be seen that as e (and hence the expected value of the profit)
  5962. increases, p diminishes.  The more the game is to the player's
  5963. advantage in terms of expected value of profit, the less likely it is
  5964. that the player will come away with any profit at all.  This
  5965. is mildly counterintuitive.
  5966.  
  5967. ==> decision/switch.p <==
  5968. Switch? (The Monty Hall Problem)
  5969.  
  5970. Two black marbles and a red marble are in a bag. You choose one marble from the
  5971. bag without looking at it. Another person chooses a marble from the bag and it
  5972. is black. You are given a chance to keep the marble you have or switch it with
  5973. the one in the bag. If you want to end up with the red marble, is there an
  5974. advantage to switching? What if the other person looked at the marbles remaining
  5975. in the bag and purposefully selected a black one?
  5976.  
  5977. ==> decision/switch.s <==
  5978. Generalize the problem from three marbles to n marbles.
  5979.  
  5980. If there are n marbles, your odds of having selected the red one are 1/n. After
  5981. the other person selected a black one at random, your odds go up to 1/(n-1).
  5982. There are n-2 marbles left in the bag, so your odds of selecting the red one
  5983. by switching are 1/(n-2) times the odds that you did not already select it
  5984. (n-2)/(n-1) or 1/(n-1), the same as the odds of already selecting it. Therefore
  5985. there is no advantage to switching.
  5986.  
  5987. If the person looked into the bag and selected a black one on purpose, then
  5988. your odds of having selected the red one are not improved, so the odds of
  5989. selecting the red one by switching are 1/(n-2) times (n-1)/n or (n-1)/n(n-2).
  5990. This is (n-1)/(n-2) times better than the odds without switching, so you
  5991. should switch.
  5992.  
  5993. This is a clarified version of the Monty Hall "paradox":
  5994.  
  5995. You are a participant on "Let's Make a Deal." Monty Hall shows you
  5996. three closed doors.  He tells you that two of the closed doors have a
  5997. goat behind them and that one of the doors has a new car behind it.
  5998. You pick one door, but before you open it, Monty opens one of the two
  5999. remaining doors and shows that it hides a goat.  He then offers you a
  6000. chance to switch doors with the remaining closed door.  Is it to your
  6001. advantage to do so?
  6002.  
  6003. The original Monty Hall problem (and solution) appears to be due to
  6004. Steve Selvin, and appears in American Statistician, Feb 1975, V. 29,
  6005. No. 1, p. 67 under the title ``A Problem in Probability.''  It should
  6006. be of no surprise to readers of this group that he received several
  6007. letters contesting the accuracy of his solution, so he responded two
  6008. issues later (American Statistician, Aug 1975, V. 29, No. 3, p. 134).
  6009. I extract a few words of interest, including a response from Monty
  6010. Hall himself:
  6011.  
  6012.    ...  The basis to my solution is that Monty Hall knows which box
  6013.    contains the prize and when he can open either of two boxes without
  6014.    exposing the prize, he chooses between them at random ...
  6015.  
  6016.    Benjamin King pointed out the critical assumptions about Monty
  6017.    Hall's behavior that are necessary to solve the problem, and
  6018.    emphasized that ``the prior distribution is not the only part of
  6019.    the probabilistic side of a decision problem that is subjective.''
  6020.  
  6021.    Monty Hall wrote and expressed that he was not ``a student of
  6022.    statistics problems'' but ``the big hole in your argument is that
  6023.    once the first box is seen to be empty, the contestant cannot
  6024.    exchange his box.''  He continues to say, ``Oh, and incidentally,
  6025.    after one ▌box¿ is seen to be empty, his chances are not 50/50 but
  6026.    remain what they were in the first place, one out of three.  It
  6027.    just seems to the contestant that one box having been eliminated,
  6028.    he stands a better chance.  Not so.''  I could not have said it
  6029.    better myself.
  6030.  
  6031. The basic idea is that the Monty Hall problem is confusing for two
  6032. reasons:  first,  there are hidden assumptions about Monty's motivation
  6033. that cloud the issue in some peoples' minds; and second, novice probability
  6034. students do not see that the opening of the door gave them any new
  6035. information.
  6036.  
  6037. Monty can have one of three basic motives:
  6038. 1.  He randomly opens doors.
  6039. 2.  He always opens the door he knows contains nothing.
  6040. 3.  He only opens a door when the contestant has picked the grand prize.
  6041.  
  6042. These result in very different strategies:
  6043. 1.  No improvement when switching.
  6044. 2.  Double your odds by switching.
  6045. 3.  Don't switch]
  6046.  
  6047. Most people, myself included, think that (2) is the intended
  6048. interpretation of Monty's motive.
  6049.  
  6050. A good way to see that Monty is giving you information by opening doors is to
  6051. increase the number of doors from three to 100.  If there are 100 doors,
  6052. and Monty shows that 98 of them are empty, isn't it pretty clear that
  6053. the chance the prize is behind the remaining door is 99/100?
  6054.  
  6055. Reference (too numerous to mention, but this one should do):
  6056.     Leonard Gillman
  6057.     "The Car and the Goats"
  6058.     The American Mathematical Monthly, 99:1 (Jan 1992), pp. 3-7.
  6059.  
  6060. ==> decision/truel.p <==
  6061. A, B, and C are to fight a three-cornered pistol duel.  All know that
  6062. A's chance of hitting his target is 0.3, C's is 0.5, and B never misses.
  6063. They are to fire at their choice of target in succession in the order
  6064. A, B, C, cyclically (but a hit man loses further turns and is no longer
  6065. shot at) until only one man is left.  What should A's strategy be?
  6066.  
  6067. ==> decision/truel.s <==
  6068. This is problem 20 in Mosteller _Fifty Challenging Problems in Probability_
  6069. and it also appears (with an almost identical solution) on page 82 in
  6070. Larsen & Marx _An Introduction to Probability and Its Applications_.
  6071.  
  6072. Here's Mosteller's solution:
  6073.  
  6074.   A is naturally not feeling cheery about this enterprise.  Having the
  6075. first shot he sees that, if he hits C, B will then surely hit him, and
  6076. so he is not going to shoot at C.  If he shoots at B and misses him,
  6077. then B clearly {I disagree; this is not at all clear]} shoots the more
  6078. dangerous C first, and A gets one shot at B with probability 0.3 of
  6079. succeeding.  If he misses this time, the less said the better.  On the
  6080. other hand, suppose A hits B.  Then C and A shoot alternately until one
  6081. hits.  A's chance of winning is (.5)(.3) + (.5)^2(.7)(.3) +
  6082. (.5)^3(.7)^2(.3) + ... .  Each term cooresponds to a sequence of misses
  6083. by both C and A ending with a final hit by A.  Summing the geometric
  6084. series we get ... 3/13 < 3/10.  Thus hitting B and finishing off with
  6085. C has less probability of winning for A than just missing the first shot.
  6086. So A fires his first shot into the ground and then tries to hit B with
  6087. his next shot.  C is out of luck.
  6088.  
  6089. As much as I respect Mosteller, I have some serious problems with this
  6090. solution.  If we allow the option of firing into the ground, then if
  6091. all fire into the ground with every shot, each will survive with
  6092. probability 1.  Now, the argument could be made that a certain
  6093. strategy for X that both allows them to survive with probability 1
  6094. *and* gives less than a probability of survival of less than 1 for
  6095. at least one of their foes would be preferred by X.  However, if
  6096. X pulls the trigger and actually hits someone what would the remaining
  6097. person, say Y, do?  If P(X hits)=1, clearly Y must try to hit X, since
  6098. X firing at Y with intent to hit dominates any other strategy for X.
  6099. If P(X hits)<1 and X fires at Y with intent to hit, then
  6100. P(Y survives)<1 (since X could have hit Y).  Thus, Y must insure that
  6101. X can not follow this strategy by shooting back at X (thus insuring
  6102. that P(X survives)<1).  Therefore, I would conclude that the ideal
  6103. strategy for all three players, assuming that they are rational and
  6104. value survival above killing their enemies, would be to keep firing
  6105. into the ground.  If they don't value survival above killing their
  6106. enemies (which is the only a priori assumption that I feel can be
  6107. safely made in the absence of more information), then the problem
  6108. can't be solved unless the function each player is trying to maximize
  6109. is explicitly given.
  6110. --
  6111.     -- clong@remus.rutgers.edu (Chris Long)
  6112.  
  6113. OK - I'll have a go at this.
  6114.  
  6115. How about the payoff function being 1 if you win the "duel" (i.e. if at some
  6116. point you are still standing and both the others have been shot) and 0
  6117. otherwise? This should ensure that an infinite sequence of deliberate misses
  6118. is not to anyone's advantage. Furthermore, I don't think simple survival
  6119. makes a realistic payoff function, since people with such a payoff function
  6120. would not get involved in the fight in the first place]
  6121.  
  6122. ▌ I.e. I am presupposing a form of irrationality on the part of the
  6123.   fighters: they're only interested in survival if they win the duel. Come
  6124.   to think of it, this may be quite rational - spending the rest of my life
  6125.   firing a gun into the ground would be a very unattractive proposition to
  6126.   me :-)
  6127. ¿
  6128.  
  6129. Now, denote each position in the game by the list of people left standing,
  6130. in the order in which they get their turns (so the initial position is
  6131. (A,B,C), and the position after A misses the first shot (B,C,A)). We need to
  6132. know the value of each possible position for each person.
  6133.  
  6134. By definition:
  6135.  
  6136.     valA(A) = 1            valB(A) = 0            valC(A) = 0
  6137.     valA(B) = 0            valB(B) = 1            valC(B) = 0
  6138.     valA(C) = 0            valB(C) = 0            valC(C) = 1
  6139.  
  6140. Consider the two player position (X,Y). An infinite sequence of misses has
  6141. value zero to both players, and each player can ensure a positive payoff by
  6142. trying to shoot the other player. So both players deliberately missing is a
  6143. sub-optimal result for both players. The question is then whether both
  6144. players should try to shoot the other first, or whether one should let the
  6145. other take the first shot. Since having the first shot is always an
  6146. advantage, given that some real shots are going to be fired, both players
  6147. should try to shoot the other first. It is then easy to establish that:
  6148.  
  6149.     valA(A,B) = 3/10       valB(A,B) = 7/10       valC(A,B) = 0
  6150.     valA(B,A) = 0          valB(B,A) = 1          valC(B,A) = 0
  6151.     valA(B,C) = 0          valB(B,C) = 1          valC(B,C) = 0
  6152.     valA(C,B) = 0          valB(C,B) = 5/10       valC(C,B) = 5/10
  6153.     valA(C,A) = 3/13       valB(C,A) = 0          valC(C,A) = 10/13
  6154.     valA(A,C) = 6/13       valB(A,C) = 0          valC(A,C) = 7/13
  6155.  
  6156. Now for the three player positions (A,B,C), (B,C,A) and (C,A,B). Again, the
  6157. fact that an infinite sequence of misses is sub-optimal for all three
  6158. players means that at least one player is going to decide to fire. However,
  6159. it is less clear than in the 2 player case that any particular player is
  6160. going to fire. In the 2 player case, each player knew that *if* it was
  6161. sub-optimal for him to fire, then it was optimal for the other player to
  6162. fire *at him* and that he would be at a disadvantage in the ensuing duel
  6163. because of not having got the first shot. This is not necessarily true in
  6164. the 3 player case.
  6165.  
  6166. Consider the payoff to A in the position (A,B,C). If he shoots at B, his
  6167. expected payoff is:
  6168.  
  6169.     0.3*valA(C,A) + 0.7*valA(B,C,A) = 9/130 + 0.7*valA(B,C,A)
  6170.  
  6171. If he shoots at C, his expected payoff is:
  6172.  
  6173.     0.3*valA(B,A) + 0.7*valA(B,C,A) = 0.7*valA(B,C,A)
  6174.  
  6175. And if he deliberately misses, his expected payoff is:
  6176.  
  6177.     valA(B,C,A)
  6178.  
  6179. Since he tries to maximise his payoff, we can immediately eliminate shooting
  6180. at C as a strategy - it is strictly dominated by shooting at B. So A's
  6181. expected payoff is:
  6182.  
  6183.     valA(A,B,C) = MAX(valA(B,C,A), 9/130 + 0.7*valA(B,C,A))
  6184.  
  6185. A similar argument shows that C's expected payoffs in the (C,A,B) position are:
  6186.  
  6187.     For shooting at A: 0.5*valC(A,B,C)
  6188.     For shooting at B: 35/130 + 0.5*valC(A,B,C)
  6189.     For missing:       valC(A,B,C)
  6190.  
  6191. So C either shoots at B or deliberately misses, and:
  6192.  
  6193.     valC(C,A,B) = MAX(valC(A,B,C), 35/130 + 0.5*valC(A,B,C))
  6194.  
  6195. Each player can obtain a positive expected payoff by shooting at one of the
  6196. other players, and it is known that an infinite sequence of misses will
  6197. result in a zero payoff for all players. So it is known that some player's
  6198. strategy must involve shooting at another player rather than deliberately
  6199. missing.
  6200.  
  6201. Now look at this from the point of view of player B. He knows that *if* it
  6202. is sub-optimal for him to shoot at another player, then it is optimal for at
  6203. least one of the other players to shoot. He also knows that if the other
  6204. players choose to shoot, they will shoot *at him*. If he deliberately
  6205. misses, therefore, the best that he can hope for is that they miss him and
  6206. he is presented with the same situation again. This is clearly less good for
  6207. him than getting his shot in first. So in position (B,C,A), he must shoot at
  6208. another player rather than deliberately miss.
  6209.  
  6210. B's expected payoffs are:
  6211.  
  6212.     For shooting at A: valB(C,B) = 5/10
  6213.     For shooting at C: valB(A,B) = 7/10
  6214.  
  6215. So in position (B,C,A), B shoots at C for an expected payoff of 7/10. This
  6216. gives us:
  6217.  
  6218.     valA(B,C,A) = 3/10     valB(B,C,A) = 7/10     valC(B,C,A) = 0
  6219.  
  6220. So valA(A,B,C) = MAX(3/10, 9/130 + 21/100) = 3/10, and A's best strategy is
  6221. position (A,B,C) is to deliberately miss, giving us:
  6222.  
  6223.     valA(A,B,C) = 3/10     valB(A,B,C) = 7/10     valC(A,B,C) = 0
  6224.  
  6225. And finally, valC(C,A,B) = MAX(0, 35/130 + 0) = 7/26, and C's best strategy
  6226. in position (C,A,B) is to shoot at B, giving us:
  6227.  
  6228.     valA(C,A,B) = 57/260   valB(C,A,B) = 133/260  valC(C,A,B) = 7/26
  6229.  
  6230. I suspect that, with this payoff function, all positions with 3 players can
  6231. be resolved. For each player, we can establish that if their correct
  6232. strategy is to fire at another player, then it is to fire at whichever of
  6233. the other players is more dangerous. The most dangerous of the three players
  6234. then finds that he has nothing to lose by firing at the second most
  6235. dangerous.
  6236.  
  6237. Questions:
  6238.  
  6239. (a) In the general case, what are the optimal strategies for the other two
  6240.     players, possibly as functions of the hit probabilities and the cyclic
  6241.     order of the three players?
  6242.  
  6243. (b) What happens in the 4 or more player case?
  6244.  
  6245.     -- David Seal <dseal@armltd.co.uk>
  6246.  
  6247. ==> english/acronym.p <==
  6248. What acronyms have become common words?
  6249.  
  6250. ==> english/acronym.s <==
  6251. The following is the list of acronyms which have become common nouns.
  6252. An acronym is "a word formed from the initial letter or letters of each
  6253. of the successive parts or major parts of a compound term" (Webster's Ninth).
  6254. A common noun will occur uncapitalized in Webster's Ninth.
  6255.  
  6256. Entries in the following table include the year in which they first
  6257. entered the language (according to the Ninth), and the Merriam-Webster
  6258. dictionary that first contains them.  The following symbols are used:
  6259.  
  6260. NI1     New International (1909)
  6261. NI1+    New Words section of the New International (1931)
  6262. NI2     New International Second Edition (1934)
  6263. NI2+    Addendum section of the Second (1959, same as 1954)
  6264. NI3     Third New International (1961)
  6265. 9C      Ninth New Collegiate (1983)
  6266. 12W     12,000 Words (separately published addendum to the Third, 1986)
  6267.  
  6268. asdic   Anti-Submarine Detection Investigation Committee (1940, NI2+)
  6269. dew     Distant Early Warning (1953, 9C)
  6270. dopa    DihydrOxyPhenylAlanine (1917, NI3)
  6271. fido    Freaks + Irregulars + Defects + Oddities (1966, 9C)
  6272. jato    Jet-Assisted TakeOff (1947, NI2+)
  6273. laser   Light Amplification by Stimulated Emission of Radiation (1957, NI3)
  6274. lidar   LIght Detection And Ranging (1963, 9C)
  6275. maser   Microwave Amplification by Stimulated Emission of Radiation (1955, NI3)
  6276. nitinol NIckel + TIn + Naval Ordinance Laboratory (1968, 9C)
  6277. rad     Radiation Absorbed Dose (1918, NI3)
  6278. radar   RAdio Detection And Ranging (ca. 1941, NI2+)
  6279. rem     Roentgen Equivalent Man (1947, NI3)
  6280. rep     Roentgen Equivalent Physical (1947, NI3)
  6281. scuba   Self-Contained Underwater Breathing Apparatus (1952, NI3)
  6282. snafu   Situation Normal -- All Fucked (Fouled) Up (ca. 1940, NI2+)
  6283. sofar   SOund Fixing And Ranging (1946, NI2+)
  6284. sonar   SOund NAvigation Ranging (1945, NI2+)
  6285. tepa    Tri-Ethylene Phosphor-Amide (1953, 9C)
  6286. zip     Zone Improvement Plan (1963, 9C)
  6287.  
  6288. Below are blends that technically are also acronyms:
  6289.  
  6290. alnico  ALuminum + NIckel + CObalt (1935, NI2+)
  6291. avgas   AViation GASoline (1943, NI3)
  6292. boff    Box OFFice (1946, NI3)
  6293. ceramal CERAMic ALloy (ca. 1948, NI2+)
  6294. cermet  CERamic METal (1948, NI2+)
  6295. comsymp COMmunist SYMPathizer (ca. 1961, 9C)
  6296. cyborg  CYBernetic ORGanism (ca. 1962, 9C)
  6297. dorper  DORset horn + blackhead PERsian (1949, NI3)
  6298. elhi    ELementary school + HIgh school (1948, 9C)
  6299. gox     Gaseous OXygen (1959, 9C)
  6300. hela    HEnrietta LAcks (1953, 9C)
  6301. kip     KIlo- + Pound (1914, NI2)
  6302. linac   LINear ACcelerator (1950, 9C)
  6303. loran   LOng-RAnge Navigation (ca. 1932, NI2+)
  6304. lox     Liquid OXygen (1923, 9C)
  6305. mascon  MASs CONcentration (1968, 9C)
  6306. maximin MAXImum + MINimum (1951, 9C)
  6307. minimax MINImum + MAXimum (1918, 9C)
  6308. modem   MOdulator + DEModulator (ca. 1952, 9C)
  6309. motocross MOTOr + CROSS-country (1951, 9C)
  6310. napalm  NAphthenic and PALMitic acids (1942, NI2+)
  6311. parsec  PARallax SECond (ca. 1913, NI1+)
  6312. redox   REDuction + OXidation (1828, NI2)
  6313. selsyn  SELf-SYNchronizing (1936, NI2+)
  6314. shoran  SHOrt-RAnge Navigation (ca. 1932, NI2+)
  6315. silvex  SILVa + EXterminator (1961, 9C)
  6316. sitcom  SITuation COMedy (1965, 9C)
  6317. teleran TELEvision-RAdar Navigation (1946, NI2+)
  6318. telex   TELeprinter EXchange (ca. 1943, 9C)
  6319. vidicon VIDeo + ICONoscope (1950, NI3)
  6320. wilco   WILl COmply (ca. 1938, NI3)
  6321.  
  6322. Acronyms from other languages:
  6323.  
  6324. agitprop AGITatsiya + PROPaganda (Russian, ca. 1926, NI2+)
  6325. flak    FLiegerAbwehrKanonen (German, 1938, NI2+)
  6326. gestapo GEheime STAatsPOlizei (German, 1934, NI2+)
  6327. gulag   Glavnoe Upravlenie ispravitel'notrudovykh LAGerei (Russian, 1974, 9C)
  6328. kolkhoz KOLlektivnoe KHOZyaistvo (Russian, 1921, NI2)
  6329. moped   MOtor + PEDal (Swedish, ca. 1955, 9C)
  6330. sambo   SAMozashchita Bez Oruzhiya (Russian, 1972, 9C)
  6331.  
  6332. Selected near misses:
  6333.  
  6334. athodyd Aero-THermODYnamic Duct (1945, NI2+) -- blend
  6335. awol    Absent WithOut Leave (1919, NI2+) -- usually capitalized
  6336. benday  BENjamin DAY (1903, NI1+) -- blend
  6337. deet    Di-Ethyl Tolumide (1962, 9C) -- pronunciation of D. E. T.
  6338. echovirus Enteric Cytopathogenic Human Orphan VIRUS (1955, 9C) -- blend
  6339. hi-fi   HIgh FIdelity (1948, NI2+) -- hyphenated
  6340. ibuprofen Iso-BUtyl PROpionic PHENyl (1969, 12W) -- PH pronounced f
  6341. jaygee  Junior Grade (1943, NI3) -- pronunciation of J. G.
  6342. jayvee  Junior Varsity (1937, NI3) -- pronunciation of J. V.
  6343. jeep    General Purpose (1940, NI2+) -- pronunciation of G. P.
  6344. op-ed   OPposite EDitorial (1970, 9C) -- hyphenated
  6345. pj's    PaJamas (1951, NI3) -- punctuated
  6346. nazi    NAtionalsoZIalist (German, 1930, NI2) -- shorten & alter
  6347. nystatin New York STATe + -IN (1952, NI3) -- extraneous suffix
  6348. reovirus Respiratory Enteric Orphan VIRUS (1959, 9C) -- blend
  6349. sci-fi  SCIence FIction (1955, 9C) -- hyphenated
  6350. siloxane SILicon + OXygen + methANE (1922, NI3) -- blend
  6351. tokamak TOroidskaja KAmera MAGneticheskaja (Russian, 1965, 9C) -- G pron. k
  6352. tradevman TRAining DEVices MAN  (ca. 1947, NI3) -- blend
  6353. updo    UPswept hairDO (1946, NI2+) -- blend
  6354. veep    Vice President (1940, NI2+) -- pronunciation of V. P.
  6355. warfarin Wisconsin Alumni Research Foundation + coumARIN (ca. 1950, NI3) - blend
  6356. yuppie  Young Urban Professional + -PIE (1983, 9C) -- extraneous suffix
  6357.  
  6358. Acronyms that should be in Webster's Ninth:
  6359.  
  6360. biopic  BIOgraphical PICture (12W)
  6361. fifo    First In, First Out (NI2+)
  6362. lifo    Last In, First Out (NI2+)
  6363. nomic   NO Metal In Composition (NI3) (John Bulten)
  6364. quango  QUAsi-Non Governmental Organization (12W)
  6365. shazam  Solomon Hercules Atlas Zeus Achilles Mercury (12W)
  6366. tacan   TACtical Air Navigation (12W)
  6367.  
  6368. Supposed acronyms:
  6369.  
  6370. posh    Port Out, Starboard Home
  6371. spiff   Sales Productivity Incentive Fund
  6372. tip     To Insure (should be Ensure) Politeness (or Promptness)
  6373.  
  6374. ==> english/ambiguous.p <==
  6375. What word in the English language is the most ambiguous?
  6376. What is the greatest number of parts of speech that a single word
  6377. can be used for?
  6378.  
  6379. ==> english/ambiguous.s <==
  6380. In Webster's Ninth, "set" occupies 1.2 columns, has 25 vb entries, 11 vi
  6381. entries, 23 noun entries, 7 adjective entries; "take" occupies 1.3 columns,
  6382. has 19 vb entries, 8 vi entries, 4 noun entries.
  6383.  
  6384. The word "like" occupies eight parts of speech:
  6385.         verb                    "Fruit flies like a banana."
  6386.         noun                    "He has his likes and dislikes."
  6387.         adjective               "People of like tastes agree."
  6388.         adverb                  "The truth is more like this."
  6389.         conjunction             "Time flies like an arrow."
  6390.         preposition             "She cries like a woman."
  6391.         interjection            "Like, man, that was far out."
  6392.         verbal auxiliary        "So loud I like to fell out of bed."
  6393.  
  6394. ==> english/antonym.p <==
  6395. What words, when a single letter is added, reverse their meanings?
  6396.  
  6397. Exclude words that are obtained by adding an "a-" to the beginning.
  6398.  
  6399. ==> english/antonym.s <==
  6400. e: fast -> feast, fiancee -> fiance
  6401. h: treat -> threat
  6402. r: fiend -> friend
  6403. s: he -> she
  6404. t: here -> there
  6405.  
  6406. ==> english/behead.p <==
  6407. Is there a sentence that remains a sentence when all its words are beheaded?
  6408.  
  6409. ==> english/behead.s <==
  6410. Show this bold Prussian that praises slaughter, slaughter brings rout.
  6411.  
  6412. ==> english/capital.p <==
  6413. What words change pronunciation when capitalized (e.g., polish -> Polish)?
  6414.  
  6415. ==> english/capital.s <==
  6416. A partial list is:
  6417. askew
  6418. august
  6419. begin
  6420. chile
  6421. colon
  6422. concord
  6423. degas
  6424. ewe     (African language)
  6425. herb
  6426. job
  6427. levy
  6428. lima
  6429. messier
  6430. mobile
  6431. natal
  6432. nice
  6433. polish
  6434. rainier
  6435. ravel
  6436. reading
  6437. tang    (Chinese dynasty)
  6438. tangier
  6439. worms   (Germany city)
  6440.  
  6441. ==> english/charades.p <==
  6442. A ....... surgeon was ....... to operate because he had .......
  6443.  
  6444. ==> english/charades.s <==
  6445. A notable surgeon was not able to operate because he had no table.
  6446.  
  6447. ==> english/contradictory.proverbs.p <==
  6448. What are some proverbs that contradict one another?
  6449.  
  6450. ==> english/contradictory.proverbs.s <==
  6451. Beware of Greeks bearing gifts.
  6452. Never look a gift horse in the mouth.
  6453.  
  6454. Look before you leap.
  6455. He who hesitates is lost.
  6456.  
  6457. Nothing venture, nothing gain.
  6458. Fools rush in where angels fear to tread.
  6459.  
  6460. Seek and ye shall find.
  6461. Curiosity killed the cat.
  6462.  
  6463. Save for a rainy day.
  6464. Tomorrow will take care of itself.
  6465.  
  6466. Life is what we make it.
  6467. What is to be will be.
  6468.  
  6469. Too many cooks spoil the broth.
  6470. Many hands make light work.
  6471.  
  6472. One man's meat is another man's poison.
  6473. Sauce for the goose is sauce for the gander.
  6474.  
  6475. With age comes wisdom.
  6476. Out of the mouths of babes and sucklings come all wise sayings.
  6477.  
  6478. Bear ye one another's burdens. (Gal. 6:2)
  6479. For every man shall bear his own burden. (Gal. 6:5)
  6480.  
  6481. Great minds run in the same channel.
  6482. Fools think alike.
  6483.  
  6484. A rolling stone gathers no moss.
  6485. A setting hen never lays.
  6486.  
  6487. ==> english/contranym.p <==
  6488. What words are their own antonym?
  6489.  
  6490. ==> english/contranym.s <==
  6491. In his 1989 book _Crazy English_, Richard Lederer calls such words contranyms
  6492. and lists more than 35, although some are phrases instead of words.
  6493. These can be divided into homographs (same spelling) and homophones (same pronunciation).
  6494.  
  6495. A partial list of homographs:
  6496. aught = all, nothing
  6497. bill = invoice, money
  6498. cleave = to separate, to join
  6499. clip = cut apart, fasten together
  6500. comprise = contain, compose
  6501. dust = to remove, add fine particles
  6502. fast = rapid, unmoving
  6503. literally = actually, figuratively
  6504. moot = debatable, not needing to be debated (already decided)
  6505. note = promise to pay, money
  6506. oversight = care, error
  6507. peep = look quietly, beep
  6508. peer = noble, companion
  6509. put = lay, throw
  6510. puzzle = pose problem, solve problem
  6511. quantum = very small, very large (quantum leap)
  6512. ravel = entangle, disentangle
  6513. resign = to quit, to sign up again
  6514. sanction = to approve of, to punish
  6515. sanguine = murderous, optimistic
  6516. scan = to examine closely, to glance at quickly
  6517. set = fix, flow
  6518. skin = to cover with, remove outer covering
  6519. speak = express verbally, express nonverbally
  6520. table = propose ▌British¿, set aside
  6521. temper = calmness, passion
  6522. trim = cut things off, put things on
  6523.  
  6524. A very short list of homophones:
  6525. aural, oral = heard, spoken
  6526. fiance, fiancee = female betrothed, male betrothed
  6527. raise, raze = erect, tear down
  6528.  
  6529. A pair of French words which can be very confusing:
  6530. La symetrie (symmetry) and L'asymetrie (asymmetry).
  6531.  
  6532. Latin:
  6533. immo = yes, no
  6534.  
  6535. Possibilities:
  6536. draw (curtains, open or close) (money, withdraw, accumulate interest)
  6537. eke
  6538.  
  6539. ==> english/element.p <==
  6540. The name of what element ends in "h"?
  6541.  
  6542. ==> english/element.s <==
  6543. Bismuth.
  6544.  
  6545. "The Elements" by Tom Lehrer
  6546. Sung to the tune of "The Major-General's Song" from Gilbert & Sullivan's
  6547. "The Pirates of Penzance":
  6548.  
  6549. There's antimony, arsenic, aluminum, selenium
  6550. And hydrogen and oxygen and nitrogen and rhenium
  6551. And nickel, neodymium, neptunium, germanium
  6552. And iron, americium, ruthenium, uranium,
  6553. Europium, zirconium, lutetium, vanadium
  6554. And lanthanum and osmium and astatine and radium
  6555. And gold and protactinium and indium and gallium
  6556. And iodine and thorium and thulium and thallium.
  6557.  
  6558. There's yttrium, ytterbium, actinium, rubidium
  6559. And boron, gadolinium, niobium, iridium
  6560. And strontium and silicon and silver and samarium
  6561. And BISMUTH, bromine, lithium, beryllium and barium.
  6562.  
  6563. There's holmium and helium and hafnium and erbium
  6564. And phosphorous and francium and fluorine and terbium
  6565. And manganese and mercury, molybdenum, magnesium,
  6566. Dysprosium and scandium and cerium and cesium
  6567. And lead, praseodymium and platinum, plutonium,
  6568. Palladium, promethium, potassium, polonium
  6569. And tantalum, technetium, titanium, tellurium
  6570. And cadmium and calcium and chromium and curium.
  6571.  
  6572. There's sulfur, californium and fermium, berkelium
  6573. And also mendelevium, einsteinium, nobelium
  6574. And argon, krypton, neon, radon, xenon, zinc and rhodium
  6575. And chlorine, carbon, cobalt, copper, tungsten, tin and sodium.
  6576.  
  6577. These are the only ones of which the news has come to Ha'vard
  6578. And there may be many others but they haven't been discavard.
  6579.  
  6580. ==> english/equations.p <==
  6581. Each equation below contains the initials of words that will make the phrase
  6582. correct.  Figure out the missing words.  Lower case is used only to help the
  6583. initials stand out better.
  6584.  
  6585. Example:  26 = L. of the A.
  6586.     would be  26 = Letters of the Alphabet
  6587.  
  6588. 1 = G. L. for M. K.
  6589. 1 = S. C. in D. P.
  6590. 1 = S. S. for a M.
  6591. 1 = W. on a U.
  6592. 2 = H. in a W.
  6593. 2 = P. in a P.
  6594. 3 = B. M., S. H. T. R.]
  6595. 3 = D. of the C.
  6596. 3 = W. M.
  6597. 4 = Q. in a F. G.
  6598. 4 = S. in a Y.
  6599. 5 = D. in a Z. C.
  6600. 5 = D. of the C.
  6601. 5 = S. in the S. C.
  6602. 5 = T. on a F.
  6603. 6 = P. in a P.
  6604. 6 = T. Z. in the U. S.
  6605. 6 = of O. and a H. D. of the O.
  6606. 7 = C. in a R.
  6607. 7 = K. of F. in H. P.
  6608. 7 = W. of the W.
  6609. 8 = L. on a S.
  6610. 8 = L. on an O.
  6611. 8 = S. on a S. S.
  6612. 9 = D. in a Z. C., with the S. C.
  6613. 9 = L. of a C.
  6614. 9 = P. in the S. S.
  6615. 10 = L. I. B.
  6616. 11 = P. on a C. T.
  6617. 11 = P. on a F. T.
  6618. 12 = D. of C.
  6619. 12 = D. of J.
  6620. 12 = S. of the Z.
  6621. 12 = T. of I.
  6622. 13 = B. D.
  6623. 13 = S. on the A. F.
  6624. 14 = D. in a F.
  6625. 15 = M. on a D. M. C.
  6626. 16 = O. in the P.
  6627. 18 = H. on the G. C.
  6628. 20 = C. in a P.
  6629. 24 = B. B. B. in a P.
  6630. 24 = B. B. to a C.
  6631. 24 = H. in a D.
  6632. 25 = Y. of M. for a S. A.
  6633. 26 = L. of the A.
  6634. 29 = D. in F. in a L. Y.
  6635. 32 = D. F. at which W. F.
  6636. 36 = I. on a Y. S.
  6637. 40 = D. and N. of the G. F.
  6638. 43 = B. in E. C. of N.
  6639. 46 = C. in the H. B.
  6640. 50 = W. to L. Y. L.
  6641. 52 = W. in a Y.
  6642. 54 = C. in a D.
  6643. 57 = H. V.
  6644. 64 = S. on a C.
  6645. 76 = T. L. the B. P.
  6646. 88 = C. in the S.
  6647. 88 = P. K.
  6648. 90 = D. in a R. A.
  6649. 96 = T., by ?
  6650. 100 = B. of B. on a W.
  6651. 101 = D.
  6652. 101 = a S. M. L.
  6653. 200 = D. for P. G. in M.
  6654. 206 = B. in the H. B.
  6655. 365 = D. in a Y.
  6656. 432 = P. in a H.
  6657. 500 = M. in the I. F. H.
  6658. 500 = S. in a R.
  6659. 1000 = I. in N. Y.
  6660. 1000 = W. that a P. is W.
  6661. 1001 = A. N.
  6662. 20000 = L. U. the S.
  6663.  
  6664. ==> english/equations.s <==
  6665. This puzzle originally was printed in "Games" magazine in 1981, by Will Shortz.
  6666. Many people have added to it since then.
  6667.  
  6668. 1 = G. L. for M. K. (1 giant leap for man kind)
  6669. 1 = S. C. in D. P. (1 single calorie in diet pepsi)
  6670. 1 = S. S. for a M. (1 small step for a man)
  6671. 1 = W. on a U. (1 wheel on a unicycle)
  6672. 2 = H. in a W. (2 halves in a whole)
  6673. 2 = P. in a P. (2 peas in a pod)
  6674. 3 = B. M., S. H. T. R.] (3 blind mice, see how they run])
  6675. 3 = D. of the C. (Days of the Condor -- movie)
  6676. 3 = W. M. (3 wise men)
  6677. 4 = Q. in a F. G. (4 quarters in a football game)
  6678. 4 = S. in a Y. (4 seasons in a year)
  6679. 5 = D. in a Z. C. (5 digits in a zip code)
  6680. 5 = D. of the C. (Days of the Condor -- book)
  6681. 5 = S. in the S. C. (stars in the Southern Cross)
  6682. 5 = T. on a F. (5 toes on a foot)
  6683. 6 = P. in a P. (6 pigs in a poke)
  6684. 6 = T. Z. in the U. S. (time zones in the United States)
  6685. 6 = of O. and a H. D. of the O. (6 of one and a half dozen of the other)
  6686. 7 = C. in a R. (colors in a rainbow : ROYGBIV)
  6687. 7 = K. of F. in H. P. (7 kinds of fruit in hawaiian punch)
  6688. 7 = W. of the W. (7 wonders of the world)
  6689. 8 = L. on a S. (legs on a spider)
  6690. 8 = L. on an O. (8 legs on an octopus)
  6691. 8 = S. on a S. S. (8 sides on a stop sign)
  6692. 9 = D. in a Z. C., with the S. C. (digits in a zip code, with the street code)
  6693. 9 = L. of a C. (9 lives of a cat)
  6694. 9 = P. in the S. S. (9 planets in the solar system)
  6695. 10 = L. I. B. (10 little indian boys)
  6696. 11 = P. on a C. T. (11 players on a cricket team)
  6697. 11 = P. on a F. T. (11 players on a football team)
  6698. 12 = D. of C. (12 days of Christmas)
  6699. 12 = D. of J. (disciples of Jesus)
  6700. 12 = S. of the Z. (12 signs of the zodiac)
  6701. 12 = T. of I. (12 tribes of Israel)
  6702. 13 = B. D. (13 = baker's dozen)
  6703. 13 = S. on the A. F. (13 stripes on the American flag)
  6704. 14 = D. in a F. (14 days in a fortnight)
  6705. 15 = M. on a D. M. C. (15 men on a dead man's chest)
  6706. 16 = O. in the P. (ounces in the pound)
  6707. 18 = H. on the G. C. (18 holes on the golf course)
  6708. 20 = C. in a P. (20 cigarettes in a pack)
  6709. 24 = B. B. B. in a P. (24 black birds baked in a pie)
  6710. 24 = B. B. to a C. (24 beer bottles to a case)
  6711. 24 = H. in a D. (24 hours in a day)
  6712. 25 = Y. of M. for a S. A. (25 years of marriage for a silver anniversary)
  6713. 26 = L. of the A. (letters of the alphabet)
  6714. 29 = D. in F. in a L. Y. (29 days in Febuary in a leap year.)
  6715. 32 = D. F. at which W. F. (32 degrees Fahrenheit at which water freezes)
  6716. 36 = I. on a Y. S. (36 inches on a yard stick)
  6717. 40 = D. and N. of the G. F. (40 days and nights of the great flood)
  6718. 43 = B. in E. C. of N. (beans in each cup of Nescafe)
  6719. 46 = C. in the H. B. (chromosomes in the human body)
  6720. 50 = W. to L. Y. L. (50 ways to leave your lover)
  6721. 52 = W. in a Y. (52 weeks in a year)
  6722. 54 = C. in a D. (with the J.) (54 cards in a deck with the jokers)
  6723. 57 = H. V. (57 heinz varieties)
  6724. 64 = S. on a C. (64 squares on a checkerboard)
  6725. 76 = T. L. the B. P. (76 trombones led the big parade)
  6726. 88 = C. in the S. (constellations in the sky)
  6727. 88 = P. K. (88 piano keys)
  6728. 90 = D. in a R. A. (90 degrees in a right angle)
  6729. 96 = T., by ? (96 Tears, by ?)
  6730. 100 = B. of B. on a W. (100 bottles of beer on a wall)
  6731. 101 = D. (101 dalmations)
  6732. 101 = a S. M. L. (101, a silly millimeter longer)
  6733. 200 = D. for P. G. in M. (200 dollars for passing go in monopoly)
  6734. 206 = B. in the H. B. (206 bones in the human body)
  6735. 365 = D. in a Y. (365 days in a year)
  6736. 432 = P. in a H. (pints in a hogshead)
  6737. 500 = M. in the I. F. H. (500 miles in the Indianapolis Five Hundred)
  6738. 500 = S. in a R. (sheets in a ream)
  6739. 1000 = I. in N. Y. (1000 islands in new york)
  6740. 1000 = W. that a P. is W. (1000 words that a picture is worth)
  6741. 1001 = A. N.  (1001 arabian nights, as in tales of)
  6742. 20000 = L. U. the S. (20000 leagues under the sea)
  6743.  
  6744. ==> english/fossil.p <==
  6745. What are some examples of idioms that include obsolete words?
  6746.  
  6747. ==> english/fossil.s <==
  6748. These are called fossil expresions -- words that have dropped out of
  6749. common use but hang around in idioms.  Not all of them are separate
  6750. words, some are part of other words or have prefixes or suffixes
  6751. attached.  There are also words which have current meaning, but the
  6752. meaning in the idiom is unrelated to it.
  6753.  
  6754. idiom                   fossil          meaning of fossil
  6755. --------------------------------------------------
  6756. swashbuckler            buckler         small shield
  6757. newfangled              fangled         siezed
  6758. rank and file           file            column
  6759. to and fro              fro             from
  6760. gormless                gorm            attention
  6761. hem and haw             haw             make the sound "haw"
  6762. hem and haw             hem             make the sound "hem"
  6763. hue and cry             hue             outcry
  6764. kit and kaboodle        kaboodle        collection
  6765. out of kilter           kilter          order
  6766. kith and kin            kith            friends
  6767. let or hinderance       let             hinderance
  6768. footpad                 pad             highwayman
  6769. pratfall                prat            buttocks
  6770. rank and file           rank            row
  6771. raring to go            raring          enthusiastic
  6772. ruthless                ruth            compassion
  6773. short shrift            shrift          confession
  6774. spick-and-span          span            chunk of wood
  6775. spick-and-span          spick           nail (spike)
  6776. swashbuckler            swash           bluster or stagger
  6777. bank teller             tell            to count
  6778.  
  6779. ==> english/frequency.p <==
  6780. In the English language, what are the most frequently appearing:
  6781.         1) letters overall?
  6782.         2) letters BEGINNING words?
  6783.         3) final letters?
  6784.         4) digrams (ordered pairs of letters)?
  6785.  
  6786. ==> english/frequency.s <==
  6787. web2 = word list from Webster's Second Unabridged
  6788. web2a = hyphenated words and phrases from Webster's Second Unabridged
  6789. both = web2 + web2a
  6790. net = several gigabytes of Usenet traffic
  6791.  
  6792. 1) Most frequently appearing letters overall:
  6793. web2:   eiaorn tslcup mdhygb fvkwzx qj
  6794. both:   eairon tslcud pmhgyb fwvkzx qj
  6795. net:    etaoin srhldc umpfgy wbvkxj qz
  6796.  
  6797. 2) Most frequently appearing letters BEGINNING words:
  6798. web:    spcaut mbdrhi eofgnl wvkjqz yx
  6799. both:   spcatb umdrhf eigowl nvkqjz yx
  6800. net:    taisow cmbphd frnelu gyjvkx qz
  6801.  
  6802. 3) Most frequent final letters:
  6803. web:    eysndr ltacmg hkopif xwubzv jq
  6804. both:   eydsnr tlagcm hkpoiw fxbuzv jq
  6805. net:    estndr yolafg mhipuk cwxbvz jq
  6806.  
  6807. 4) Most frequent digrams (ordered pairs of letters)
  6808. web:    er in ti on te al an at ic en is re ra le ri ro st ne ar ...
  6809. both:   er in te ti on an re al at le en ra ic ar st ri ro ed ne ...
  6810. net:    th he in er re an on at te es or en ar ha is ou it to st nd ...
  6811.  
  6812. Program to compute this from word list in standard input:
  6813. #include <stdio.h>
  6814. #include <ctype.h>
  6815. typedef struct {
  6816.         int count;
  6817.         char name▌3¿;
  6818. } FREQ;
  6819.  
  6820. FREQ all▌256¿,initial▌256¿,terminal▌256¿,digram▌65536¿;
  6821.  
  6822. int compare(p,q)
  6823. FREQ *p,*q;
  6824. {       return q->count - p->count;
  6825. }
  6826.  
  6827. void sort_and_print(freq,count,description)
  6828. FREQ *freq;
  6829. int count;
  6830. char *description;
  6831. {   register FREQ *p;
  6832.  
  6833.     (void)qsort(freq,count,sizeof(*freq),compare);
  6834.     puts(description);
  6835.     for (p=freq;p<freq+count;p++)
  6836.         if (p->count) printf("%s %d\n",p->name,p->count);
  6837. }
  6838.  
  6839. main()
  6840. {   char s▌BUFSIZ¿;
  6841.     register char *p;
  6842.     register int i;
  6843.  
  6844.     while (gets(s)]=NULL) {
  6845.         if (islower(*s)) {
  6846.             initial▌*s¿.count++;
  6847.             sprintf(initial▌*s¿.name,"%c",*s);
  6848.             for (p=s;*p;p++) {
  6849.                 if (isalpha(*p)) {
  6850.                     all▌*p¿.count++;
  6851.                     sprintf(all▌*p¿.name,"%c",*p);
  6852.                     if (isalpha(p▌1¿)) {
  6853.                         i = p▌0¿*256 + p▌1¿;
  6854.                         digram▌i¿.count++;
  6855.                         sprintf(digram▌i¿.name,"%c%c",p▌0¿,p▌1¿);
  6856.                     }
  6857.                 }
  6858.             }
  6859.             terminal▌*--p¿.count++;
  6860.             sprintf(terminal▌*p¿.name,"%c",*p);
  6861.         }
  6862.     }
  6863.     sort_and_print(all,256,"overall character distribution: ");
  6864.     sort_and_print(initial,256,"initial character distribution: ");
  6865.     sort_and_print(terminal,256,"terminal character distribution: ");
  6866.     sort_and_print(digram,65536,"digram distribution: ");
  6867. }
  6868.  
  6869. ==> english/gry.p <==
  6870. Find three completely different words ending in "gry."
  6871.  
  6872. ==> english/gry.s <==
  6873. Aside from "angry" and "hungry" and words derived therefrom, there is
  6874. only one word ending with "-gry" in Webster's Third Unabridged: "aggry."
  6875. However, this word is defective in that it is part of a phrase "aggry beads."
  6876. The OED's usage examples all talk about "aggry beads."
  6877.  
  6878. Moving to older dictionaries, we find that "gry" itself is a word in Webster's
  6879. Second Unabridged (and the OED):
  6880.  
  6881. gry, n. ▌L. gry, a trifle; Gr. gry, a grunt¿
  6882.    1. a measure equal to one-tenth of a line. ▌Obs.¿ (Obs. = obsolete)
  6883.    2. anything very small. ▌Rare.¿
  6884.  
  6885. This is a list of 94 words, phrases and names ending in "gry":
  6886. ▌Explanation of references is given at the end of the list.¿
  6887.  
  6888. aggry ▌OED:1:182; W2; W3¿
  6889. Agry Dagh (Mount Agry) ▌EB11¿
  6890. ahungry ▌OED:1:194; FW; W2¿
  6891. angry ▌OED; FW; W2; W3¿
  6892. anhungry ▌OED:1:332; W2¿
  6893. Badagry ▌Johnston; EB11¿
  6894. Ballingry ▌Bartholomew:40; CLG:151; RD:164, pl.49¿
  6895. begry ▌OED:1:770,767¿
  6896. bewgry ▌OED:1:1160¿
  6897. bowgry ▌OED:1:1160¿
  6898. braggry ▌OED:1:1047¿
  6899. Bugry ▌TIG¿
  6900. Chockpugry ▌Worcester¿
  6901. Cogry ▌BBC¿
  6902. cony-gry ▌OED:2:956¿
  6903. conyngry ▌OED:2:956¿
  6904. Croftangry ▌DFC, as "Chrystal Croftangry"¿
  6905. dog-hungry ▌W2¿
  6906. Dshagry ▌Stieler¿
  6907. Dzagry ▌Andree¿
  6908. eard-hungry ▌CED (see "yird"); CSD¿
  6909. Echanuggry ▌Century:103-104, on inset map, Key 104 M 2¿
  6910. Egry ▌France; TIG¿
  6911. ever-angry ▌W2¿
  6912. fire-angry ▌W2¿
  6913. Gagry ▌EB11¿
  6914. gry (from Latin _gry_) ▌OED:4/2:475; W2¿
  6915. gry (from Romany _grai_) ▌W2¿
  6916. haegry ▌EDD (see "hagery")¿
  6917. half-angry ▌W2¿
  6918. hangry ▌OED:1:329¿
  6919. heart-angry ▌W2¿
  6920. heart-hungry ▌W2¿
  6921. higry pigry ▌OED:5/1:285¿
  6922. hogry ▌EDD (see "huggerie"); CSD¿
  6923. hogrymogry ▌EDD (see "huggerie"); CSD (as "hogry-mogry")¿
  6924. hongry ▌OED:5/1:459; EDD:3:282¿
  6925. huggrymuggry ▌EDD (see "huggerie"); CSD (as "huggry-muggry")¿
  6926. hungry ▌OED; FW; W2; W3¿
  6927. Hungry Bungry ▌Daily Illini, in ad for The Giraffe, Spring 1976¿
  6928. Jagry ▌EB11¿
  6929. kaingry ▌EDD (see "caingy")¿
  6930. land-hungry ▌W2¿
  6931. Langry ▌TIG; Times¿
  6932. Lisnagry ▌Bartholomew:489¿
  6933. MacLoingry ▌Phillips (as "Flaithbhertach MacLoingry")¿
  6934. mad-angry ▌OED:6/2:14¿
  6935. mad-hungry ▌OED:6/2:14¿
  6936. magry ▌OED:6/2:36, 6/2:247-48¿
  6937. malgry ▌OED:6/2:247¿
  6938. Margry ▌Indians (see "Pierre Margry" in bibliog., v.2, p.1204)¿
  6939. maugry ▌OED:6/2:247-48¿
  6940. mawgry ▌OED:6/2:247¿
  6941. meagry ▌OED:6/2:267¿
  6942. meat-hungry ▌W2¿
  6943. menagry ▌OED (see "managery")¿
  6944. messagry ▌OED¿
  6945. overangry ▌RH1; RH2¿
  6946. Pelegry ▌CE (in main index as "Raymond de Pelegry")¿
  6947. Pingry ▌Bio-Base; HPS:293-94, 120-21¿
  6948. podagry ▌OED; W2 (below the line)¿
  6949. Pongry ▌Andree (Supplement, p.572)¿
  6950. pottingry ▌OED:7/2:1195; Jamieson:3:532¿
  6951. puggry ▌OED:8/1:1573; FW; W2; W3¿
  6952. pugry ▌OED:8/1:1574¿
  6953. rungry ▌EDD:5:188¿
  6954. scavengry ▌OED (in 1715 quote under "scavengery")¿
  6955. Schtschigry ▌LG/1:2045; OSN:97¿
  6956. Seagry ▌TIG; EB11¿
  6957. Segry ▌Johnston; Andree¿
  6958. self-angry ▌W2¿
  6959. self-hungry ?
  6960. Shchigry ▌CLG:1747; Johnson:594; OSN:97,206; Times:185,pl.45¿
  6961. shiggry ▌EDD¿
  6962. Shtchigry ▌LG/1:2045; LG/2:1701¿
  6963. Shtshigry ▌Lipp¿
  6964. skugry ▌OED:9/2:156, 9/1:297; Jamieson:4:266¿
  6965. Sygry ▌Andree¿
  6966. Tangry ▌France¿
  6967. Tchangry ▌Johnson:594; LG/1:435,1117¿
  6968. Tchigry ▌Johnson:594¿
  6969. tear-angry ▌W2¿
  6970. tike-hungry ▌CSD¿
  6971. Tingry ▌France; EB11 (under "Princesse de Tingry")¿
  6972. toggry ▌Simmonds (as "Toggry", but all entries are capitalized)¿
  6973. ulgry ▌Partridge; Smith:24-25¿
  6974. unangry ▌W2¿
  6975. vergry ▌OED:12/1:123¿
  6976. Virgy ▌CLG:2090¿
  6977. Wirgy ▌CLG:2090; NAP:xxxix; Times:220, pl.62; WA:948¿
  6978. wind-angry.
  6979. wind-hungry ▌W2¿
  6980. yeard-hungry ▌CED (see "yird")¿
  6981. yerd-hungry ▌CED (see "yird"); OED¿
  6982. yird-hungry ▌CED (see "yird")¿
  6983. Ymagry ▌OED:1:1009 (col. 3, 1st "boss" verb), (variant of "imagery")¿
  6984.  
  6985. This list was gathered from the following articles:
  6986.  
  6987. George H. Scheetz. In Goodly Gree: With Goodwill. Word Ways 22:195 (Nov. 1989)
  6988. Murray R. Pearce. Who's Flaithbhertach MacLoingry? Word Ways 23:6 (Feb. 1990)
  6989. Harry B. Partridge. Gypsy Hobby Gry. Word Ways 23:9 (Feb. 1990)
  6990.  
  6991. References:
  6992. (Many references are of the form ▌Source:volume:page¿ or ▌Source:page¿.)
  6993.  
  6994. Andree, Richard. Andrees Handatlas (index volume). 1925.
  6995. Bartholomew, John. Gazetteer of the British Isles: Statistical and
  6996.         Topographical. 1887.
  6997. BBC = BBC Pronouncing Dictionary of English Names.
  6998. Bio-Base. (Microfiche) Detroit: Gale Research Company. 1980.
  6999. CE = Catholic Encyclopedia. 1907.
  7000. CED = Chambers English Dictionary. 1988.
  7001. Century = "India, Northern Part." The Century Atlas of the World. 1897, 1898.
  7002. CLG = The Colombia Lippincott Gazetteer of the World. L.E.Seltzer, ed. 1952.
  7003. CSD = Chambers Scots Dictionary. 1971 reprint of 1911 edition.
  7004. Daily Illini (University of Illinois at Urbana-Champaign).
  7005. DFC = Dictionary of Fictional Characters. 1963.
  7006. EB11 = Encyclopedia Britannica, 11th ed.
  7007. EDD = The English Dialect Dictionary. Joseph Wright, ed. 1898.
  7008. France = Map Index of France. G.H.Q. American Expeditionary Forces. 1918.
  7009. FW = Funk & Wagnalls New Standard Dictionary of the English Language. 1943.
  7010. HPS = The Handbook of Private Schools: An Annual Descriptive Survey of
  7011.         Independent Education, 66th ed. 1985.
  7012. Indians = Handbook of American Indians North of Mexico. F. W. Hodge. 1912.
  7013. Jamieson, John. An Etymological Dictionary of the Scottish Language. 1879-87.
  7014. Johnston, Keith. Index Geographicus... 1864.
  7015. LG/1 = Lippincott's Gazetteer of the World: A Complete Pronouncing Gazetteer
  7016.         or Geographical Dictionary of the World. 1888.
  7017. LG/2 = Lippincott's New Gazetteer: ... 1906.
  7018. Lipp = Lippincott's Pronouncing Gazetteer of the World. 1861, undated
  7019.         edition from late 1800's; 1902.
  7020. NAP = Narodowy Atlas Polski. 1973-1978 ▌Polish language¿
  7021. OED = The Oxford English Dictionary. 1933. ▌Form: OED:volume/part number if
  7022.         applicable:page¿
  7023. OSN: U.S.S.R. Volume 6, S-T. Official Standard Names Approved by the United
  7024.         States Board on Geographic Names. Gazetteer #42, 2nd ed. June 1970.
  7025. Partridge, Harry B. "Ad Memoriam Demetrii." Word Ways, 19 (Aug. 1986): 131.
  7026. Phillips, Lawrence. Dictionary of Biographical Reference. 1889.
  7027. RD = The Reader's Digest Complete Atlas of the British Isles, 1st ed. 1965.
  7028. RH1 = Random House Dictionary of the English Language, Unabridged. 1966.
  7029. RH2 = Random House Dictionary of the English Language, Second Edition
  7030.         Unabridged. 1987.
  7031. Simmonds, P.L. Commercial Dictionary of Trade Products. 1883.
  7032. Smith, John. The True Travels, Adventvres and Observations: London 1630.
  7033. Stieler, Adolph. Stieler's Handatlas (index volume). 1925.
  7034. TIG = The Times Index-Gazetteer of the World. 1965.
  7035. Times = The Times Atlas of the World, 7th ed. 1985.
  7036. W2 = Webster's New International Dictionary of the English Language,
  7037.         Second Edition, Unabridged. 1934.
  7038. W3 = Webster's Third New International Dictionary of the English Language,
  7039.         Unabridged. 1961.
  7040. WA = The World Atlas: Index-Gazetteer. Council of Ministires of the USSR, 1968.
  7041. Worcester, J.E. Universal Gazetteer, Second Edition. 1823.
  7042.  
  7043. Some words containing "gry" that do not end with "gry": agrypnia,
  7044. agrypnotic, Gryllidae, gryllid, gryllus, Gryllus, grylloblattid,
  7045. Gryllotalpa, gryllos, grypanian, Gryphaea, Gryll, Gryphaea, gryposis,
  7046. grysbok, gryphon, Gryphosaurus, Grypotherium, grysbuck.  Most of these
  7047. are in Webster's Second also with one from Webster's Third Edition and
  7048. one from the Random House Dictionary, Second Edition Unabridged.
  7049.  
  7050. ==> english/homographs.p <==
  7051. List all homographs (words that are spelled the same but pronounced differently)
  7052.  
  7053. ==> english/homographs.s <==
  7054. This list composed by Mark Brader <msb@sq.com>
  7055.  
  7056. Classes:
  7057.  
  7058. A - All of the following "defects" absent
  7059. B - Basic meanings are related
  7060. C - Capitalization differs ("capitonyms")
  7061. D - Different spellings also exist (US vs UK, hyphenation, etc.)
  7062. E - Equal pronunciations also exist (US vs UK, regional, etc.)
  7063. F - Foreign word, or may be distinguished with accent marks
  7064. G - Gcontrived :-), coined, jargon, or other uncommon word
  7065.  
  7066. N - Alleged, but I could not find support for this one in my dictionary
  7067.     and it is not familiar to me
  7068.  
  7069. 3 - 3-way homograph
  7070. 4 - 4-way homograph
  7071.  
  7072. B       abstract {corresponding noun and verb; henceforth abbreviated NV}
  7073. B       abuse {NV}
  7074. B       addict {NV}
  7075. B       advocate {NV}
  7076. BG      affect {alter; emotion}
  7077. B       affiliate {NV}
  7078. B       affix {NV}
  7079. G       agape {wide open; form of love}
  7080. B       aggregate {NV}
  7081. G       ai {sloth; ouch]}
  7082. BE      ally {NV}
  7083. B       alternate {NV}
  7084. BD      analyses {plural noun; singular verb (UK)}
  7085. B       animate {verb; adjective}
  7086. A       appropriate {take posession of; suitable}
  7087. B       approximate {verb; adjective}
  7088. E       are {form of to be; unit of area}
  7089. B       arithmetic {noun; adjective}
  7090. B       articulate {verb; adjective}
  7091. 4DFG    as {like; Roman coin; Persian card game; pl. of a}
  7092. B       aspirate {NV}
  7093. B       associate {NV}
  7094. B       attribute {NV}
  7095. C       august
  7096. A       axes {plural of ax; plural of axis}
  7097. A       bases {plural of base; plural of basis}
  7098. A       bass {~ fiddle; fishing for ~}
  7099. N       blessed
  7100. A       bow(ed) {~ and arrow; ~ to the king}
  7101. E       buffet {jostle; ~ lunch}
  7102. B       bustier {undergarment; more busty}
  7103. B       close {~ call; ~ the door}
  7104. B       closer {door ~; more close}
  7105. B       coagulate {NV}
  7106. G       coax {urge; coaxial cable}
  7107. 3FG     colon {":"; colonial farmer; Costa Rican monetary unit}
  7108. B       combat {NV}
  7109. B       combine {NV}
  7110. A       commune {take Communion; administrative district}
  7111. A       compact {closely arranged; treaty}
  7112. B       compound {NV}
  7113. B       compress {NV}
  7114. B       conduct {NV}
  7115. B       confect {NV}
  7116. B       confines {NV}
  7117. B       conflict {NV}
  7118. B       conglomerate {NV}
  7119. B       conjugate {NV}
  7120. BE      conserve {preserve; jam}
  7121. A       console {soothe; keyboard desk}
  7122. B       consort {NV}
  7123. B       construct {NV}
  7124. B       consummate {verb; adjective}
  7125. N       contact
  7126. E       content {what is contained; satisfied}
  7127. B       contest {NV}
  7128. B       contract {NV}
  7129. B       contrast {NV}
  7130. N       convent
  7131. A       converse {logic term; to talk}
  7132. B       convert {NV}
  7133. B       convict {NV}
  7134. BE      coordinate {NV}
  7135. FG      dame {woman; term in the game of Go}
  7136. DE      decameter {poetic line with 10 feet; 10 meters (US)}
  7137. B       defect {flaw; turn traitor}
  7138. E       defense {sports term; fortification}
  7139. BE      delegate {NV}
  7140. B       deliberate {adjective; verb}
  7141. A       desert {leave alone; Sahara ~}
  7142. B       desolate {adjective; verb}
  7143. D       dingy {dull; small boat}
  7144. BE      discharge {NV}
  7145. N       divers {plural diver; various}
  7146. F       do {perform; tonic note of scale}
  7147. A       does {~ the buck see the ~?}
  7148. A       dove {dived; pigeon}
  7149. F       dozen {12; stun (Scottish)}
  7150. B       drawer {one who draws; chest of ~s}
  7151. B       duplicate {NV}
  7152. B       elaborate {verb; adjective}
  7153. A       entrance {door; delight}
  7154. BDE     envelop {NV}
  7155. N       envelope
  7156. N       ergotism {logical reasoning; ergot poisoning}
  7157. B       escort {NV}
  7158. N       escrow
  7159. B       essay {piece of writing; try}
  7160. B       estimate {NV}
  7161. CFG     ewe {female sheep; African language}
  7162. B       excuse {NV}
  7163. B       exploit {NV}
  7164. BF      expose {NV}
  7165. B       ferment {NV}
  7166. N       fiasco {failure; bottle}
  7167. BDE     fillet {cut of meat/fish; band of ribbon/wood}
  7168. G       formal {ceremonious; methylal}
  7169. DEG     genet {civetlike animal; horselike animal}
  7170. A       gill {volume unit; organ in fish}
  7171. A       glower {sullen look; one that glows}
  7172. B       graduate {NV}
  7173. F       he {pronoun; Hebrew letter}
  7174. CE      herb {name; plant}
  7175. A       hinder {hamper; posterior}
  7176. B       house {NV}
  7177. B       import {NV}
  7178. A       incense {infuriate; perfume for burning}
  7179. B       increase {NV}
  7180. B       initiate {NV}
  7181. B       insert {NV}
  7182. B       insult {NV}
  7183. B       intern {NV}
  7184. A       intimate {~ relations; to suggest}
  7185. A       invalid {cripple; erroneous}
  7186. B       invite {NV}
  7187. G       is {form of to be; plural of i}
  7188. B       jagged {slashed or cut; having a zigzag edge}
  7189. C       Job
  7190. BCF     jubilate {rejoice; joyous song}
  7191. CF      junker/Junker
  7192. 3A      lather {suds; lath worker; lathe worker}
  7193. A       lead {~ pipe; ~ astray}
  7194. B               {past tense verb; adjective}
  7195. BE      legged {past tense verb; adjective}
  7196. CF      Lima
  7197. B       live {~ in peace; ~ audience}
  7198. B       lives {~ in peace; for all of our ~}
  7199. D       lower {to let down; frown}
  7200. F       manes {plural of mane; Roman gods}
  7201. F       mate {friend; type of tea}
  7202. N       mead
  7203. A       minute {60 seconds; tiny}
  7204. B       misconduct {NV}
  7205. BE      mobile {movable; wind-blown sculpture}
  7206. B       moderate {NV}
  7207. EG      molar {back tooth; chemical term}
  7208. A       moped {brooded; fun vehicle}
  7209. B       mouse {rodent; to hunt them}
  7210. B       mouth {NV}
  7211. A       mow {pile of hay; to cut down}
  7212. B       multiply {verb; adverb}
  7213. A       number {decimal ~; more numb}
  7214. B       object {thing; complain}
  7215. E       offense {sports term; attack}
  7216. 3DG     os {bone; esker; pl. of o}
  7217. A       overage {too old; surplus}
  7218. BD      paralyses {plural noun; singular verb (UK)}
  7219. A       pasty {pastelike; British meat pie}
  7220. 3FG     pate {head; food paste; porcelain paste for ceramics}
  7221. A       peaked {sharply pointed; unhealthy looking}
  7222. A       peer {equal; one who pees}
  7223. B       perfect {verb; adjective}
  7224. G       periodic {regularly occurring; ~ acids, HIO4 and related substances}
  7225. B       permit {NV}
  7226. C       Placer
  7227. C       polish
  7228. A       poll {head; group of students}
  7229. B       predicate {NV}
  7230. N       premise
  7231. A       present {current; Christmas ~}
  7232. E       primer {intro book/material (US); device for priming}
  7233. B       proceeds {goes; income}
  7234. B       produce {give rise to; fruits and vegetables}
  7235. B       progress {to move forward; work in ~}
  7236. A       project {planned undertaking; to throw forward}
  7237. N       prospect
  7238. B       protest {NV}
  7239. A       pussy {cat; infected}
  7240. B       putter/putting {golf club; one that puts}
  7241. DG      rabat {clerical garment; pottery piece used for polishing}
  7242. DG      rabbi {clerical garment; Jewish religious official}
  7243. B       ragged {teased; tattered}
  7244. F       re {pertaining to; 2nd note of scale}
  7245. B       read {present tense; past tense}
  7246. C       Reading
  7247. F       real {actual; former Spanish coin}
  7248. B       rebel {NV}
  7249. B       recess {NV}
  7250. B       recoil {NV}
  7251. B       record {NV}
  7252. D       recreate {relax; create again}
  7253. 3BD     redress {compensate; compensation; dress again}
  7254. B       refill {NV}
  7255. B       refund {NV}
  7256. B       refuse {NV}
  7257. B       regress {NV}
  7258. B       reject {NV}
  7259. N       repent {regret; creeping}
  7260. B       replay {NV}
  7261. D       represent {stand for; present again}
  7262. B       rerun {NV}
  7263. D       research {investigate; search again}
  7264. A       resent {be indignant; sent again}
  7265. D       reserve {hold back; serve again}
  7266. D       resign {quit; sign again}
  7267. D       resolve {settle dispute; solve again}
  7268. D       resort {vacation spot; sort again}
  7269. F       resume {work summary; restart}
  7270. A       river {watercourse; one who rives}
  7271. F       rose {flower; wine}
  7272. DE      routing {making a route for (US spelling); woodworking term}
  7273. A       row {a fight; ~,~,~ your boat}
  7274. DF      sake {purpose; Japanese drink}
  7275. 3AF     salve {ointment; salvage; hail]}
  7276. N       second
  7277. B       segment {NV}
  7278. B       separate {NV}
  7279. A       severer {cutter; more severe}
  7280. 3AG     sewer {one who sews; storm ~; head servant at table}
  7281. A       shower {one who shows; ~ stall}
  7282. B       syndicate {NV}
  7283. A       singer {one who singes; one who sings}
  7284. A       skied {past tense of ski; past tense of sky}
  7285. A       slaver {slave taker; drool}
  7286. A       slough {swamp; cast-off}
  7287. A       sow {~ seeds; female pig}
  7288. A       stingy {meager; able to sting}
  7289. B       subject {NV}
  7290. A       supply {in a supple way; ~ and demand}
  7291. B       survey {NV}
  7292. B       suspect {NV}
  7293. N       swinger {whopper; one that swings}
  7294. CF      tang {flavor; Chinese dynasty}
  7295. A       tarry {covered in tar; dawdle}
  7296. A       tear {~ down; shed a ~}
  7297. A       thou {you; slang for thousand}
  7298. A       thymic {of thyme; of thymus}
  7299. A       tier {one who ties; row or rank}
  7300. B       torment {NV}
  7301. A       tower {one who tows; leaning ~}
  7302. B       transfer {NV}
  7303. B       transplant {NV}
  7304. B       transport {NV}
  7305. DG      unionized {~ labor; ~ hydrogen}
  7306. B       upset {NV}
  7307. G       us {we; plural of u}
  7308. B       use {NV}
  7309. A       violist {viol player; viola player}
  7310. A       wind {~ the clock; north ~}
  7311. CF      worms
  7312. A       wound {injury; wrapped around}
  7313. N       yak {ox; laugh}
  7314.  
  7315.  
  7316. ==> english/homophones.p <==
  7317. What words have four or more spellings that sound alike?
  7318.  
  7319. ==> english/homophones.s <==
  7320. air, aire, are, ayr, ayer, e'er, ere, err, heir
  7321. cense, cents, scents, sense
  7322. eau, eaux, O, oh, owe
  7323.  
  7324. ==> english/j.ending.p <==
  7325. What words and names end in j?
  7326.  
  7327. ==> english/j.ending.s <==
  7328. Following is a compilation of words ending in j from various
  7329. dictionaries.  Capitalized words and words marked as foreign
  7330. are included, but to keep the list to a managable size,
  7331. personal and place names are excluded.
  7332.  
  7333.  
  7334. aflaj    plural of falaj (Cham)
  7335. benj     variant of bhang - hemp plant (NI2)
  7336. bhimraj  the rachet-tailed drongo (F&W)
  7337. Bhumij   branch of Munda tribes in India (NI3)
  7338. Chuj     a people of Northwestern Guatemala (NI3)
  7339. esraj    an Indian musical instrument with 3 or 4 strings (OED2)
  7340. falaj    a water channel as part of the ancient irrigation
  7341.          system of Oman (Cham)
  7342. Funj     variant of Fung - a people dominant in Sennar (NI3)
  7343. gaj      Omanese coin (NI2)
  7344. genj     a common type of cotton cloth in Sudan (F&W)
  7345. gunj     a grannery in India (NI2)
  7346. hadj     variant of hajj (NI3)
  7347. haj      variant of hajj (NI3)
  7348. hajilij  the bito - a small scrubby tree that grows in dry
  7349.          parts of Africa and Asia (NI2)
  7350. hajj     pilgimage to Mecca (NI3)
  7351. hij      obsolete form of hie or high (OED2)
  7352. Jubaraj  variant of Yuvaraja - the male heir to an Indian
  7353.          pricipality (OED2)
  7354. kaleej   variant of kalij (NI3)
  7355. kalij    any of several crested Indian pheasants (NI3)
  7356. kankrej  guzerat - a breed of Indian cattle (NI3)
  7357. kharaj   a tax on unbelievers (NI2)
  7358. Khawarij plural of Kharijite - a member of the oldest
  7359.          religious sect of Islam (NI3)
  7360. khiraj   variant of kharaj (NI2)
  7361. kilij    a Turkish saber with a crescent shaped blade (RHD)
  7362. kurunj   variant of kurung - the Indian beech (NI2)
  7363. Maharaj  variant of Maharaja - East Indian prince (OED2)
  7364. munj     a tough Asiatic grass (NI3)
  7365. naranj   Maldive Island name for mancala - an Arabian board
  7366.          game (CD)
  7367. pakhawaj a doubleheaded drum used in Indian music (OED2)
  7368. raj      rule (NI3)
  7369. saj      the Indian laurel (NI2)
  7370. samaj    Hindu religious society (NI3)
  7371. sohmaj   variant of samaj (NI2)
  7372. somaj    variant of samaj (NI2)
  7373. svaraj   variant of swaraj (F&W)
  7374. swaraj   local self-government in India (NI3)
  7375. taj      a tall conical cap worn by Moslems (NI3)
  7376. tedj     variant of tej (OED2)
  7377. tej      Ethiopian mead (OED2)
  7378. Viraj    in Hindu Mythology, the mysterious primeval being
  7379.          when differentiating itself into male and female (F&W)
  7380. Yuvaraj  same as Jubaraj (OED2)
  7381. Yuveraj  same as Jubaraj (OED2)
  7382. Yuvraj   same as Jubaraj (OED2)
  7383. zij      Persian astronomical tables (F&W)
  7384.  
  7385. This list is almost certainly not complete.  For example, on
  7386. page 187 of Beyond Language, Dmitri Borgmann has "Udruj" in a
  7387. word list.  What reference he dug this word out of is unknown;
  7388. the combined efforts of the NPL electronic mailing list could
  7389. not produce the source of this word.  So additions to this list
  7390. will be welcomed by the author.
  7391.  
  7392.  
  7393. REFERENCES
  7394.  
  7395. CD - The Century Dictionary and Cyclopedia, 1911
  7396. Cham - Chambers English Dictionary, 1988
  7397. F&W - Funk & Wagnall's New Standard Dictionary of the English
  7398.       Language, 1941
  7399. NI2 - Webster's New International Dictionary, Second Edition,
  7400.       1942
  7401. NI3 - Webster's Third New International Dictionary, 1981
  7402. OED2 - Oxford English Dictionary, Second Edition, 1989
  7403. RHD - Random House Dictionary of the English Language, 1966
  7404.  
  7405. ---
  7406. Dan Tilque      --      dant@logos.WR.TEK.COM
  7407.  
  7408. ==> english/ladder.p <==
  7409. Find the shortest word ladders stretching between the following pairs:
  7410. hit - ace
  7411. pig - sty
  7412. four - five
  7413. play - game
  7414. green - grass
  7415. wheat - bread
  7416. order - chaos
  7417. order - impel
  7418. sixth - hubby
  7419. speedy - comedy
  7420. chasing - robbers
  7421. effaces - cabaret
  7422. griming - goblets
  7423. vainest - injects
  7424. vainest - infulae
  7425.  
  7426. ==> english/ladder.s <==
  7427. Using every unabridged dictionary available, the best yet found are:
  7428. hit ait act ace
  7429. pig peg seg sey sty
  7430. four foud fond find fine five
  7431. play blay bray bras baas bams gams game
  7432. green grees greys grays grass
  7433. wheat theat treat tread bread
  7434. order older elder eider cider cides codes coles colls coals chals chaos
  7435. order ormer armer ammer amper imper impel
  7436. sixth sixty silty silly sally sably sabby nabby nubby hubby
  7437. speedy speeds steeds steers sheers shyers sayers payers papers papery popery
  7438. popely pomely comely comedy
  7439. griming priming prising poising toising toiling coiling colling collins collies
  7440. dollies doilies dailies bailies bailees bailers failers fablers gablers gabbers
  7441. gibbers gibbets gobbets goblets
  7442. chasing ceasing cessing messing massing masting marting martins martens martels
  7443. cartels carpels carpers campers cambers combers cobbers combers robbers
  7444. vainest fainest fairest sairest saidest saddest maddest middest mildest wildest
  7445. wiliest winiest waniest caniest cantest contest confest confess confers conners
  7446. canners fanners fawners pawners pawnees pawnces paunces jaunces jaunced jaunted
  7447. saunted stunted stented stenned steined stained spained splined splines salines
  7448. savines savings pavings parings earings enrings endings ondings ondines undines
  7449. unlines unlives unwives unwires unwares unbares unbared unpared unpaged uncaged
  7450. incaged incased incised incises incites indites indices indicts inducts indults
  7451. insults insulas insulae infulae
  7452.  
  7453. This is not another travelling salesman - it is merely finding the diameter of
  7454. connected components of that graph.  The simple algorithm for this is to do
  7455. one depth first search from each word, resulting in an O(n*m) worst case
  7456. algorithm (where n is the number of words, and m is the number of arcs).  In
  7457. practice, it is actually somewhat better, since the graph breaks down into
  7458. many connected components.  However, the diameters (and solutions) depend on
  7459. what dictionary is used.  Here are the results from various dictionaries:
  7460.  
  7461. From /usr/dict/words (restricted to words all lower case alphabetical) (19,694
  7462. words): sixth - hubby (46 steps)
  7463.  
  7464. From the official scrabble players dictionary (94,276 words): effaces -
  7465. cabaret (57 steps)
  7466.  
  7467. From the british official scrabble words (134,051 words): vainest - infulae
  7468. (73 steps)
  7469.  
  7470. From webster's ninth new collegiate dictionary (abridged) (78, 167 words):
  7471. griming - goblets (56 steps)
  7472.  
  7473. From all of the above, merged  (180,676 words): vainest - injects (58 steps)
  7474.  
  7475. To see the effect the dictionary has on paths, here are the lengths of the
  7476. shortest paths these pairs, and for the ones mentioned in previous posts, for
  7477. each dictionary (a - means that there is no path using only words from that
  7478. dictionary):
  7479.  
  7480.                   UDW OSPD OSW  W9 ALL
  7481.     hit - ace      5    3   3   5   3
  7482.     pig - sty      -    5   4   5   4
  7483.    four - five     6    6   5   7   5
  7484.    play - game     8    7   7   8   7
  7485.   green - grass   13    4   4   7   4
  7486.   wheat - bread    6    6   6   6   6
  7487.   sixth - hubby   46    9   9   -   9
  7488. effaces - cabaret  -   57   -   -  33
  7489. vainest - infulae  -    -  73   -  52
  7490. griming - goblets  -   22  19  56  15
  7491. vainest - injects  -    -  72   -  58
  7492.  
  7493. ==> english/less.ness.p <==
  7494. Find a word that forms two other words, unrelated in meaning, when "less"
  7495. and "ness" are added.
  7496.  
  7497. ==> english/less.ness.s <==
  7498. base -> baseless, baseness
  7499. light -> lightless, lightness
  7500. sound -> soundless, soundness
  7501. wit -> witless, witness
  7502.  
  7503. ==> english/letter.rebus.p <==
  7504. Define the letters of the alphabet using self-referential common phrases (e.g.,
  7505. "first of all" defines "a").
  7506.  
  7507. ==> english/letter.rebus.s <==
  7508. A       first of all, midday
  7509. B       fifth of bourbon, starting block
  7510. C       fifth of scotch
  7511. D       end of the world, back of my hand
  7512. E       end of the line, beginning of the end
  7513. F       starting friction, front
  7514. G       middle of the night, starting gate
  7515. H       end of the earth, top of the heap, middle of nowhere
  7516. I       next of kin
  7517. J       center of project
  7518. K       bottom of the deck, two of a kind
  7519. L       bottom of the barrel, starting line
  7520. M       top of my head
  7521. N       center of attention, final countdown, end run
  7522. O       second in command
  7523. P       bottom of the heap, the first of painters, starting point
  7524. Q       at the front of the queue, top quality
  7525. R       middle of the road, center of inertia
  7526. S       _Last of the Mohicans_, start of something big
  7527. T       top o' the morning, one's wit's end, bottom of my heart, last, central
  7528. U       second guess
  7529. V       center of gravity
  7530. W       end of the rainbow, top of the world
  7531. X       wax finish, climax
  7532. Y       top of your head, center of the cyclone, early years, final extremity
  7533. Z       led zeppelin
  7534.  
  7535. ==> english/lipograms.p <==
  7536. What books have been written without specific letters, vowels, etc.?
  7537.  
  7538. ==> english/lipograms.s <==
  7539. Such a book is called a lipogram.
  7540.  
  7541. A novel-length example in English (omitting e) exists, titled _Gadsby_.
  7542.  
  7543. Georges Perec wrote a French novel titled _La Disparition_ which does
  7544. not contain the letter 'e',  except in a few bits of text that the
  7545. publisher had to include in or on the book somewhere -- such as the
  7546. author's name :-).  But these were all printed in red, making them
  7547. somehow ``not count''.
  7548.  
  7549. Perec also wrote another novel in which `e' was the only vowel.
  7550.  
  7551. In _La Disparition_, unlike _Gadsby_, the lipogrammatic
  7552. technique is reflected in the story.  Objects disappear or become
  7553. invisible.  We know, however, more or less why the characters can't
  7554. find things like eggs or even remember their names -- because the
  7555. words for them can't be used.
  7556.  
  7557. Amazingly, it's been ``translated'' into English (by Harry Mathews, I
  7558. think).
  7559.  
  7560. Another work which manages to ▌almost¿ adhere to restrictive
  7561. alphabetic rules while also remaining readable as well as providing
  7562. amusement and literary satisfaction (though you have to like
  7563. disjointed fiction) is _Alphabetical Africa_ by Walter Abish.  The
  7564. rules (which of course he doesn't explain, you can't help noticing
  7565. most of them) have to do with initial letters of words.  There are 52
  7566. chapters.  In the first, all words begin with `a'; in the second, all
  7567. words begin with either `a' or `b'; etc, until all words are allowed
  7568. in chapter 26.  Then in the second half, the letters are taken away
  7569. one by one.  It's remarkable when, for instance, you finally get `the'
  7570. and realize how much or little you missed it; earlier, when `I' comes
  7571. in, you feel something like the difference between third- and
  7572. first-person narration.  As one of the blurbs more or less says (I
  7573. don't have it here to quote), reading this is like slowly taking a
  7574. deep breath and letting it out again.
  7575.  
  7576. ----
  7577. Mitch Marks  mitchell@cs.uchicago.edu
  7578.  
  7579. ==> english/multi.lingual.p <==
  7580. What words in multiple languages are related in interesting ways?
  7581.  
  7582. ==> english/multi.lingual.s <==
  7583. Synonymous reversals:
  7584.     Dutch: nier (kidney), French: rein
  7585.     French: etats, English: state
  7586.  
  7587. ==> english/near.palindrome.p <==
  7588. What are some long near palindromes, i.e., words that except for one
  7589. letter would be palindromes?
  7590.  
  7591. ==> english/near.palindrome.s <==
  7592. Here are the longest near palindromes in Webster's Ninth Collegiate:
  7593. catalatic               footstool               red pepper
  7594. detonated               locofocos               red spider
  7595. dew-clawed              nabataean               retreater
  7596. eisegesis               possessor               stargrass
  7597. foolproof               ratemeter               webmember
  7598.  
  7599. ==> english/palindromes.p <==
  7600. What are some long palindromes?
  7601.  
  7602. ==> english/palindromes.s <==
  7603. The first words spoken were a palindrome:
  7604.     Madam, I'm Adam.
  7605. or perhaps:
  7606.     Madam in Eden, I'm Adam.
  7607. The response, of course, must have been:
  7608.     Eve
  7609.  
  7610. Napolean's lament:
  7611.     Able was I ere I saw Elba.
  7612. Has been improved with:
  7613.     Unremarkable was I ere I saw Elba, Kramer, nu?
  7614.  
  7615. A fish is a:
  7616.     laminar animal
  7617.  
  7618. Other palindromes in ascending length (drum roll please):
  7619.     Dennis sinned.
  7620.     Sir, I'm Iris.
  7621.     Sup not on pus.
  7622.     Name no one man.
  7623.     Naomi, did I moan?
  7624.     Enid and Edna dine.
  7625.     Revenge Meg? Never]
  7626.     No lemons, no melon.
  7627.     A Toyota's a Toyota.
  7628.     Ma is a nun, as I am.
  7629.     He harasses Sarah, eh?
  7630.     Niagara, O roar again]
  7631.     He lived as a devil, eh?
  7632.     Nurse, I spy gupsies, run]
  7633.     Sit on a potato pan, Otis]
  7634.     Slap a ham on Omaha, pals]
  7635.     A slut nixes sex in Tulsa.
  7636.     Rats live on no evil star.
  7637.     Ten animals I slam in a net.
  7638.     Go deliver a dare, vile dog.
  7639.     Was it a car or a cat I saw?
  7640.     Was it Eliot's toilet I saw?
  7641.     Al lets Della call Ed Stella.
  7642.     Draw, O Caeser, erase a coward.
  7643.     Did Eve salt an atlas?  Eve did.
  7644.     No pinot noir on Orion to nip on.
  7645.     Naomi, sex at noon taxes] I moan.
  7646.     Evil I did dwell; lewd did I live.
  7647.     Yo, bad anaconda had no Canada boy .
  7648.     Egad] A base tone denotes a bad age.
  7649.     Satan, oscillate my metallic sonatas.
  7650.     Red dude kill lion. No ill-liked udder.
  7651.     I roamed under it as a tired, nude Maori.
  7652.     To Peru, named llama mall 'De Manure Pot'.
  7653.     Straw?  No, too stupid a fad.  I put soot on warts.
  7654.     Now, Ned, I am a maiden nun; Ned, I am a maiden won.
  7655.     Here we no got conical ill lilac in octogon ewer, eh?
  7656.     Salamander a ton now. Raw war won not, a Red Nam, alas.
  7657.     Fool] A dog lives sad a boxer, Rex. O bad ass evil god aloof]
  7658.     'Tenor Octopus Night' netted a cadet tenth ginsu pot, coronet.
  7659.     Won total, I am a pro. Bali radar I labor. Pa, mail a tot now]
  7660.     Yo, boy] Trap gnus, nude. 'Kangaroo Rag' naked unsung party, O boy]
  7661.     Did I strap red nude, red rump, also slap murdered underparts? I did]
  7662.     Doc, note: I dissent.  A fast never prevents a fatness.  I diet on cod.
  7663.     So regards Rat's Lib: regrets no more hero monster gerbil stars' drag Eros.
  7664.     Degas, are we not drawn onward, we freer few, drawn onward to new eras aged?
  7665.     Garret, I ogle. Enemy democrats party; trap star comedy men, eel goiter rag.
  7666.     Sagas emit taxes, rat snot, or pastrami. I'm Arts, a proton star - sex at
  7667.         times a gas.
  7668.     Dr. Ana, Cataracts. Uranium enema smarts if fist rams, Amen] Emu in a
  7669.         rust car at a canard.
  7670.     T. Eliot, top bard, notes putrid tang emanating, is sad; I'd assign it a
  7671.         name: gnat dirt upset on drab pot toilet.
  7672.  
  7673.  
  7674.  
  7675. Those wonderful proper names:
  7676.     Dennis, Nell, Edna, Leon, Nedra, Anita, Rolf, Nora, Alice, Carol, Leo,
  7677.     Jane, Reed, Dena, Dale, Basil, Rae, Penny, Lana, Dave, Denny, Lena,
  7678.     Ida, Bernadette, Ben, Ray, Lila, Nina, Jo, Ira, Mara, Sara, Mario, Jan,
  7679.     Ina, Lily, Arne, Bette, Dan, Reba, Diane, Lynn, Ed, Eva, Dana, Lynne,
  7680.     Pearl, Isabel, Ada, Ned, Dee, Rena, Joel, Lora, Cecil, Aaron, Flora,
  7681.     Tina, Arden, Noel, and Ellen sinned.
  7682.  
  7683. A poem:
  7684. Mood's mode]
  7685. Pallas, I won]
  7686. (Diaper pane, sold entire.)
  7687. Melt till ever sere, hide it.
  7688. Drown a more vile note;
  7689. (Tar of rennet.)
  7690. Ah, trowel, baton, eras ago.
  7691. The reward? A "nisi." Two nag.
  7692.  
  7693. Otary tastes putrid, yam was green.
  7694. Odes up and on; stare we.
  7695. Rats nod. Nap used one-erg saw.
  7696. (May dirt upset satyr?)
  7697.  
  7698. A toga now; 'tis in a drawer, eh?
  7699. Togas are notable.
  7700. (Worth a tenner for Ate`.)
  7701. Tone liver. O Man, word-tied I.
  7702.  
  7703. Here's revel]
  7704. Little merit, Ned? Lose, Nap?
  7705. Repaid now is all apedom's doom.
  7706.     -- Hubert Phillips:
  7707.  
  7708.      Headmaster's Palindromic List on his Memo Pad:
  7709.  
  7710. Test on Erasmus                       Dr of Law
  7711. Deliver soap                          Stop dynamo (OTC)
  7712. Royal: phone no.?                     Tel: Law re Kate Race
  7713. Ref. Football.                        Caps on for prep
  7714. Is sofa sitable on?                   Pots- no tops
  7715. XI--Staff over                        Knit up ties ('U')
  7716. Sub-edit Nurse's order                Ned (re paper)
  7717. Canning is on test (snub slip-up)     Eve's simple hot dish (crib)
  7718. Birch (Sid) to help Miss Eve          Pupil's buns
  7719. Reaper den                            T-set: no sign in a/c
  7720. Use it                                Red roses
  7721. Put inkspot on stopper                Run Tide Bus?
  7722. Prof.--no space                       Rev off at six
  7723. Caretaker (wall, etc.)                Noel Bat is a fossil
  7724. Too mand d*** pots                    Lab to offer one 'Noh' play--
  7725. Wal for duo?  (I'd name Dr. O)           or 'Pals Reviled'?
  7726. See few owe fees (or demand IOU?)     Sums are not set.
  7727.     -- Joyce Johnson
  7728.         (_New_Statesman_ competition in 1967.  126 words, 467 letters)
  7729.  
  7730.  
  7731. Some word (not letter) palindromes:
  7732. So patient a doctor to doctor a patient so.
  7733. Girl, bathing on Bikini, eyeing boy, finds boy eyeing bikini on bathing girl.
  7734.  
  7735. In German:
  7736.     Ein Neger mit Gazelle zagt im Regen nie.
  7737.  
  7738. In Serbo-Croat:
  7739.     Ana voli Milovana.
  7740.     Ana nabra par banana.
  7741.     Imena Amen nema, a me mi.
  7742.     U pero soli i los o repu.
  7743.     Ako jad moli silom daj oka.
  7744.     Odano mati pita: a ti pitam, o nado?
  7745.     Evo sam iza padam mada pazim asove.
  7746.               v       v         v       v
  7747.     A krt u razu mi laze no one zalim u zaru trka.
  7748.  
  7749. Palindromes in other languages that are palindromes in English:
  7750. Hebrew: aba or abba, English: dad
  7751. German: tat, English: deed
  7752.  
  7753. The timeless classic:
  7754.     A man, a plan, a canal; Panama?
  7755. Has been improved by:
  7756.     A dog, a plan, a canal: pagoda]
  7757.         -- anonymous
  7758.  
  7759.     A man, a plan, a cat, a canal; Panama?
  7760.         -- Jim Saxe, plan file @ CMU, 9 October 1983
  7761.  
  7762.     A man, a plan, a cat, a ham, a yak, a yam, a hat, a canal--Panama]
  7763.         -- Guy Jacobson, plan file @ CMU late 1983
  7764.  
  7765.     A man, a plan, a caret, a ban, a myriad, a sum, a lac, a liar, a hoop, a
  7766. pint, a catalpa, a gas, an oil, a bird, a yell, a vat, a caw, a pax, a wag,
  7767. a tax, a nay, a ram, a cap, a yam, a gay, a tsar, a wall, a car, a luger, a
  7768. ward, a bin, a woman, a vassal, a wolf, a tuna, a nit, a pall, a fret, a
  7769. watt, a bay, a daub, a tan, a cab, a datum, a gall, a hat, a fag, a zap, a
  7770. say, a jaw, a lay, a wet, a gallop, a tug, a trot, a trap, a tram, a torr, a
  7771. caper, a top, a tonk, a toll, a ball, a fair, a sax, a minim, a tenor, a
  7772. bass, a passer, a capital, a rut, an amen, a ted, a cabal, a tang, a sun, an
  7773. ass, a maw, a sag, a jam, a dam, a sub, a salt, an axon, a sail, an ad, a
  7774. wadi, a radian, a room, a rood, a rip, a tad, a pariah, a revel, a reel, a
  7775. reed, a pool, a plug, a pin, a peek, a parabola, a dog, a pat, a cud, a nu,
  7776. a fan, a pal, a rum, a nod, an eta, a lag, an eel, a batik, a mug, a mot, a
  7777. nap, a maxim, a mood, a leek, a grub, a gob, a gel, a drab, a citadel, a
  7778. total, a cedar, a tap, a gag, a rat, a manor, a bar, a gal, a cola, a pap, a
  7779. yaw, a tab, a raj, a gab, a nag, a pagan, a bag, a jar, a bat, a way, a
  7780. papa, a local, a gar, a baron, a mat, a rag, a gap, a tar, a decal, a tot, a
  7781. led, a tic, a bard, a leg, a bog, a burg, a keel, a doom, a mix, a map, an
  7782. atom, a gum, a kit, a baleen, a gala, a ten, a don, a mural, a pan, a faun,
  7783. a ducat, a pagoda, a lob, a rap, a keep, a nip, a gulp, a loop, a deer, a
  7784. leer, a lever, a hair, a pad, a tapir, a door, a moor, an aid, a raid, a
  7785. wad, an alias, an ox, an atlas, a bus, a madam, a jag, a saw, a mass, an
  7786. anus, a gnat, a lab, a cadet, an em, a natural, a tip, a caress, a pass, a
  7787. baronet, a minimax, a sari, a fall, a ballot, a knot, a pot, a rep, a
  7788. carrot, a mart, a part, a tort, a gut, a poll, a gateway, a law, a jay, a
  7789. sap, a zag, a fat, a hall, a gamut, a dab, a can, a tabu, a day, a batt, a
  7790. waterfall, a patina, a nut, a flow, a lass, a van, a mow, a nib, a draw, a
  7791. regular, a call, a war, a stay, a gam, a yap, a cam, a ray, an ax, a tag, a
  7792. wax, a paw, a cat, a valley, a drib, a lion, a saga, a plat, a catnip, a
  7793. pooh, a rail, a calamus, a dairyman, a bater, a canal--Panama.
  7794.                 --Dan Hoey, 'discovered' in 1984.
  7795.  
  7796. Dan goes on to say "...a little work on the search algorithm could make
  7797. it several times as long."
  7798.  
  7799. The entire book _Satire: Veritas_ is a palindrome, it starts
  7800. "Sir, I stra..." and ends "...  Art, sir, is Satire: Veritas."
  7801.  
  7802. References:
  7803.     Palindromes and Anagrams
  7804.     Howard W. Bergerson
  7805.     Dover Publications
  7806.     New York, 1973
  7807.     ISBN 0-486-20664-5.
  7808.  
  7809.     The Oxford Guide to Word Games, chapter 11, titled "Palindromes"
  7810.     Tony Augarde
  7811.  
  7812. ==> english/pangram.p <==
  7813. A "pangram" is a sentence containing all 26 letters.
  7814. What is the shortest pangram (measured by number of letters or words)?
  7815. What is the shortest word list using all 26 letters in alphabetical order?
  7816. In reverse alphabetical order?
  7817.  
  7818. ==> english/pangram.s <==
  7819. The single-letter words that have meanings unrelated to their letter shapes
  7820. or sounds, position in the alphabet, etc. are:
  7821. a - indefinite article; on; in; at; to; he; him; she; her; they; them; it; I;
  7822.     have; of; all
  7823. c - 100; cocaine; programming language
  7824. d - 500
  7825. e - base of natural logs; eccentricity; enlarging
  7826. g - acceleration of gravity; general ability; $1000; general audience
  7827. i - one; unit vector in x direction; personal pronoun; in; aye
  7828. j - one; unit vector in y direction
  7829. k - 1000; 1024; strikeout; unit vector in z direction
  7830. l - 50; ell; elevated railroad
  7831. m - 1000; em; pica; an antigen of human blood
  7832. n - an indefinite number; en; an antigen of human blood
  7833. o - oh
  7834. q - quality of oscillatory circuit
  7835. R - one of the three Rs; restricted audience
  7836. t - t-shirt
  7837. u - upper class
  7838. v - five
  7839. w - w particle
  7840. x - unknown quantity; atmospherics; adults only
  7841. y - unknown quantity; YMCA
  7842. z - unknown quantity; buzzing sound; z particle
  7843. It is therefore advisable to exclude single-letter words, with the
  7844. possible exception of 'a'.
  7845.  
  7846. As always, word acceptability varies with the dictionaries used.  We use these:
  7847. 9C  - Merriam-Webster's Ninth New Collegiate Dictionary, 1986
  7848. NI3 - Merriam-Webster's Third New International Dictionary, 1961
  7849. NI2 - Merriam-Webster's New International Dictionary, Second Edition, 1935
  7850. OED - Oxford English Dictionary with Supplements, 1933 - 85
  7851. '+' indicates obsolete, dialectical, slang, or otherwise substandard word.
  7852.  
  7853. Some exceptional pangrams:
  7854. Using only words in 9C:
  7855. Sympathizing would fix Quaker objectives. (5 words, 36 letters)
  7856. Quick brown fox, jump over the lazy dogs. (8 words, 32 letters)
  7857. Pack my box with five dozen liquor jugs. (8 words, 32 letters)
  7858. Jackdaws love my big sphinx of quartz. (7 words, 31 letters)
  7859. The five boxing wizards jump quickly. (6 words, 31 letters)
  7860. How quickly daft jumping zebras vex. (6 words, 30 letters)
  7861. Quartz glyph job vex'd cwm finks. (6 words?, 26 letters)
  7862. Cwm, fjord-bank glyphs vext quiz. (6 words, 26 letters, Dmitri Borgmann)
  7863. Using words in 9C and NI3:
  7864. Veldt jynx grimps waqf zho buck. (6 words, 26 letters, Michael Jones)
  7865. Using words in 9C, NI3 and NI3+:
  7866. Squdgy fez, blank jimp crwth vox. (6 words, 26 letters, Claude Shannon)
  7867. Using words in 9C, NI3, NI2 and NI2+:
  7868. Phlegms fyrd wuz qvint jackbox. (5 words, 26 letters, Dmitri Borgmann)
  7869.  
  7870. Some exceptional panalphabetic word lists:
  7871. jackbox viewfinder phlegmy quartz (4 words, 31 letters, Mary Hazard)
  7872. benzoxycamphors quick-flowing juventude (3 words, 36 letters, Darryl Francis)
  7873.  
  7874. Some exceptional nearly panalphabetic isogrammatic word lists:
  7875. blacksmith gunpowdery (2 words, 20 letters)
  7876. humpbacks frowzy tingled (3 words, 22 letters)
  7877.  
  7878. Some exceptional panalphabetic word lists with letters in alphabetical order:
  7879. Using only words in 9C:
  7880. a BCD ef ghi jack limn op querist ulva wax oyez (11 words, 37 letters)
  7881. ABC defog hijack limn op querist ulva wax oyez (9 words, 38 letters)
  7882. Using words in 9C and NI3:
  7883. a BCD ef ghi jak limn op qres to uva wax oyez (12 words, 34 letters)
  7884. ABC defy ghi jak limn op qres to uva wax oyez (11 words, 35 letters)
  7885. ABC defy ghi jak limn opaquers turves wax oyez (9 words, 38 letters)
  7886. scabicide afghani jokul manrope querist purview oxygenize (7 words, 51 letters)
  7887. Using words in 9C, NI3 and NI3+:
  7888. a BCD ef ghi jak limn op QRS to uva wax yez (12 words, 32 letters)
  7889. ABC defy ghi jak limn op QRS to uva wax yez (11 words, 33 letters)
  7890. ab cad ef ghi jak limn op qre stun vow ox yez (12 words, 34 letters)
  7891. ABC defy ghi jak limn op querist uva wax yez (10 words, 35 letters)
  7892. Using words in 9C, NI3, NI3+, NI2 and NI2+:
  7893. ABC def ghi jak limn op qre struv wax yez (10 words, 32 letters)
  7894. ABC def ghi jak limn opaquer struv wax yez (9 words, 34 letters)
  7895. Using words in 9C, NI3, NI3+, NI2, NI2+ and the OED:
  7896. ABC defog hij klam nop QRS tu vow XYZ (9 words, 29 letters, Jeff Grant)
  7897. ABC def ghi jak limn op qres tu vow XYZ (10 words, 30 letters)
  7898. ABC defog hij klam nop querist uvrow XYZ (8 words, 33 letters, Jeff Grant)
  7899. ABC defyghe bij sklim nop querist uvrow XYZ (8 words, 36 letters)
  7900. ABC defog hijack limnophil querist uvrow XYZ (7 words, 38 letters, Jeff Grant)
  7901.  
  7902. Some exceptional panalphabetic word lists with letters in reverse alpha order:
  7903. Using words from 9C:
  7904. lazy ox ow vug tsar quip on milk jib hag fed cab a (13 words, 38 letters)
  7905. lazy ox wave uts reequip on milk jihad gifted cabal (10 words, 42 letters)
  7906. Using words from 9C and NI3:
  7907. lazy ox ow vug tsar quip on milk jib hag fed caba (12 words, 38 letters)
  7908. lazy ox wave uts roque pon milk jihad gifted caba (10 words, 40 letters)
  7909. Using words from 9C, NI3 and NI2:
  7910. zo yex wu vug tsar quip on milk jib hag fed caba (12 words, 37 letters)
  7911. zo yex wave uts roque pon milk jihad gifted caba (10 words, 39 letters)
  7912.  
  7913. All words are main entries in 9C except the following:
  7914. 9C: ghi (at 'ghee')
  7915. NI3: caba, fyrd, jak, opaquers, pon, qre(s), squdgy, uva
  7916. NI3+: jimp, QRS (at 'QRS complex'), sklim, vox (at 'vox populi'), yez
  7917. NI2: benzoxycamphors, jackbox, limnophil, quick-flowing, yex, zo
  7918. NI2+: def, juventude, klam, quar, qvint, struv, tu, wuz
  7919. OED: defyghe (at 'defy'), bij (at 'buy'), hij, nop, uvrow (at 'yuffrouw'),
  7920.         XYZ (at 'X')
  7921.  
  7922. The first time I saw this pangram was in Gyles Brandeth's _The Joy of Lex_.
  7923. It appeared there as:
  7924.  
  7925. Waltz, nymph, for quick jigs vex Bud.  (7 words, 28 letters, proper noun.)
  7926.  
  7927. I always wondered why they didn't try modifying it as:
  7928.  
  7929. Waltz, nymph, for quick jigs vex buds.  (7 words, 29 letters, no proper noun.)
  7930.  
  7931. However, why fast dances would irritate incipient flowers is beyond me,
  7932. so I tried again:
  7933.  
  7934. Waltz, dumb nymph, for quick jigs vex.  (7 words, 29 letters, no proper noun,
  7935.                                          makes more sense.)
  7936.  
  7937. However, sounds kind of sexist, and we can maybe chop off a letter and
  7938. eliminate the sexism, although suffering some loss of sense:
  7939.  
  7940. Waltz, bud nymph, for quick jigs vex.  (7 words, 28 letters, no proper noun,
  7941.                                         makes less sense.)
  7942.  
  7943. There are river nymphs and tree nymphs and mountain nymphs, so there can
  7944. be nymphs of the aforementioned incipent flowers, right?  Sense is a matter
  7945. of opinion, so you can move the bud around or turn it into another imperative
  7946. verb rather than a noun-as-adjective:
  7947.  
  7948. Waltz, nymph, bud, for quick jigs vex.  (7 words, 28 letters, no proper noun,
  7949.                                          sense is dubious.)
  7950. ▌We've all heard of budding youth, right?¿
  7951.  
  7952. Waltz, nymph, for quick bud jigs vex.  (7 words, 28 letters, no proper noun,
  7953.                                         sense is dubious.)
  7954. ▌Yeah, we've all learned to dance a merry jig that looks like one of those
  7955. infamous incipient flowers.¿
  7956.  
  7957. Dub waltz, nymph, for quick jigs vex.  (7 words, 28 letters, no proper noun,
  7958.                                         came up with this on the spot and
  7959.                                         actually it looks pretty good])
  7960. ▌The idea being that a nymph, being in control of the soundtrack for a TV
  7961. sitcom, has to change the music to which a grandmother is listening, from
  7962. something from Ireland to something from Strauss.¿
  7963.  
  7964.     -- Stephen Joseph Smith <sjsmith@cs.umd.edu>
  7965.  
  7966. It is fairly straightforward, if time-consuming, to search for minimal
  7967. pangrams given a suitable lexicon, and the enclosed program does this.
  7968. The run time is of the order of 20 MIPS-days if fed `Official Scrabble
  7969. Words', a document nominally listing all sufficiently short words
  7970. playable in tournament Scrabble in Britain.
  7971.  
  7972. I also enclose a lexicon which will reproduce the OSW results much more
  7973. quickly.
  7974.  
  7975. The results are dominated by onomatopoeic interjections (`pst', `sh',
  7976. etc.), and words borrowed from Welsh (`cwm', `crwth') and Arabic (`qat',
  7977. `suq').  Other lexicons will contain a very different leavening of such
  7978. words, and yield a very different set of pangrams.
  7979.  
  7980. Readers are invited to form sentences (or, less challenging, newspaper
  7981. headlines) from these pangrams.  Few are amenable to this sort of thing.
  7982.  
  7983.     -- Steve Thomas
  7984.  
  7985. -----cut-----here-----
  7986. #include <stdio.h>
  7987. #include <ctype.h>
  7988.  
  7989. extern void *malloc ();
  7990. extern void *realloc ();
  7991.  
  7992. long getword ();
  7993.  
  7994. #define MAXWORD 26
  7995. int list▌MAXWORD¿;
  7996. int lp;
  7997.  
  7998. struct list {
  7999.         struct list *next;
  8000.         char *word;
  8001. };
  8002.  
  8003. struct word {
  8004.         long mask;
  8005.         struct list *list;
  8006. } *w;
  8007. int wp;
  8008. int wsize;
  8009.  
  8010. char wordbuf▌BUFSIZ¿;
  8011.  
  8012. char *letters = "qxjzvwfkbyhpmgcdultnoriase";
  8013.  
  8014. int cmp (ap, bp)
  8015. struct word *ap, *bp;
  8016. {
  8017.         char *p;
  8018.         long a = ap->mask, b = bp->mask;
  8019.  
  8020.         for (p = letters; *p; p++)
  8021.         {
  8022.                 long m = 1L << (*p - 'a');
  8023.  
  8024.                 if ((a & m) ]= (b & m))
  8025.                         if ((a & m) ]= 0)
  8026.                                 return -1;
  8027.                         else
  8028.                                 return 1;
  8029.         }
  8030.         return 0;
  8031. }
  8032.  
  8033. void *
  8034. newmem (p, size)
  8035. void *p;
  8036. int size;
  8037. {
  8038.         if (p)
  8039.                 p = realloc (p, size);
  8040.         else
  8041.                 p = malloc (size);
  8042.         if (p == NULL) {
  8043.                 fprintf (stderr, "Out of memory\n");
  8044.                 exit (1);
  8045.         }
  8046.         return p;
  8047. }
  8048.  
  8049. char *
  8050. dupstr (s)
  8051. char *s;
  8052. {
  8053.         char *p = newmem ((void *)NULL, strlen (s) + 1);
  8054.  
  8055.         strcpy (p, s);
  8056.         return p;
  8057. }
  8058.  
  8059. main (argc, argv)
  8060. int argc;
  8061. char **argv;
  8062. {
  8063.         long m;
  8064.         int i, j;
  8065.  
  8066.         while ((m = getword (stdin)) ]= 0)
  8067.         {
  8068.                 if (wp >= wsize)
  8069.                 {
  8070.                         wsize += 1000;
  8071.                         w = newmem (w, wsize * sizeof (struct word));
  8072.                 }
  8073.                 w▌wp¿.mask = m;
  8074.                 w▌wp¿.list = newmem ((void *)NULL, sizeof (struct list));
  8075.                 w▌wp¿.list->word = dupstr (wordbuf);
  8076.                 w▌wp¿.list->next = (struct list *)NULL;
  8077.                 wp++;
  8078.         }
  8079.         qsort (w, wp, sizeof (struct word), cmp);
  8080.         for (i = 1, j = 1; j < wp; j++)
  8081.         {
  8082.                 if (w▌j¿.mask == w▌i - 1¿.mask) {
  8083.                         w▌j¿.list->next = w▌i - 1¿.list;
  8084.                         w▌i - 1¿.list = w▌j¿.list;
  8085.                 } else
  8086.                         w▌i++¿ = w▌j¿;
  8087.         }
  8088.         wp = i;
  8089.         pangram (0L, 0, letters);
  8090.         exit (0);
  8091. }
  8092.  
  8093. pangram (sofar, min, lets)
  8094. long sofar;
  8095. int min;
  8096. char *lets;
  8097. {
  8098.         register int i;
  8099.         register long must;
  8100.  
  8101.         if (sofar == 0x3ffffff) {
  8102.                 print ();
  8103.                 return;
  8104.         }
  8105.         for (; *lets; lets++)
  8106.                 if ((sofar & (1 << (*lets - 'a'))) == 0)
  8107.                         break;
  8108.         must = 1 << (*lets - 'a');
  8109.         for (i = min; i < wp; i++)
  8110.         {
  8111.                 if (w▌i¿.mask & sofar)
  8112.                         continue;
  8113.                 if ((w▌i¿.mask & must) == 0)
  8114.                         continue;
  8115.                 list▌lp++¿ = i;
  8116.                 pangram (w▌i¿.mask ! sofar, i + 1, lets);
  8117.                 --lp;
  8118.         }
  8119. }
  8120.  
  8121. long
  8122. getword (fp)
  8123. FILE *fp;
  8124. {
  8125.         long mask, m;
  8126.         char *p;
  8127.         char c;
  8128.  
  8129.         while (fgets (wordbuf, sizeof (wordbuf), fp) ]= NULL) {
  8130.                 p = wordbuf;
  8131.                 mask = 0L;
  8132.                 while ((c = *p++) ]= '\0') {
  8133.                         if (]islower (c))
  8134.                                 break;
  8135.                         m = 1L << (c - 'a');
  8136.                         if ((mask & m) ]= 0)
  8137.                                 break;
  8138.                         mask != m;
  8139.                 }
  8140.                 if (c == '\n')
  8141.                         p▌-1¿ = c = '\0';
  8142.                 if (c == '\0' && mask)
  8143.                         return mask;
  8144.         }
  8145.         return 0;
  8146. }
  8147.  
  8148. print ()
  8149. {
  8150.         int i;
  8151.  
  8152.         for (i = 0; i < lp; i++)
  8153.         {
  8154.                 struct word *p = &w▌list▌i¿¿;
  8155.                 struct list *l;
  8156.  
  8157.                 if (p->list->next == NULL)
  8158.                         printf ("%s", p->list->word);
  8159.                 else {
  8160.                         printf ("(");
  8161.                         for (l = p->list; l; l = l->next) {
  8162.                                 printf ("%s", l->word);
  8163.                                 if (l->next)
  8164.                                         printf (" ");
  8165.                         }
  8166.                         printf (")");
  8167.                 }
  8168.                 if (i ]= lp - 1)
  8169.                         printf (" ");
  8170.         }
  8171.         printf ("\n");
  8172.         fflush (stdout);
  8173. }
  8174. -----and-----here-----
  8175. ankh
  8176. bad
  8177. bag
  8178. bald
  8179. balk
  8180. balks
  8181. band
  8182. bandh
  8183. bang
  8184. bank
  8185. bap
  8186. bard
  8187. barf
  8188. bark
  8189. bed
  8190. beds
  8191. beg
  8192. bend
  8193. benj
  8194. berk
  8195. berks
  8196. bez
  8197. bhang
  8198. bid
  8199. big
  8200. bight
  8201. bilk
  8202. bink
  8203. bird
  8204. birds
  8205. birk
  8206. bisk
  8207. biz
  8208. blad
  8209. blag
  8210. bland
  8211. blank
  8212. bled
  8213. blend
  8214. blight
  8215. blimp
  8216. blin
  8217. blind
  8218. blink
  8219. blintz
  8220. blip
  8221. blitz
  8222. block
  8223. blond
  8224. blunk
  8225. blunks
  8226. bod
  8227. bods
  8228. bog
  8229. bok
  8230. boks
  8231. bold
  8232. bond
  8233. bong
  8234. bonk
  8235. bonks
  8236. bop
  8237. bord
  8238. bords
  8239. bosk
  8240. box
  8241. brad
  8242. brank
  8243. bred
  8244. brink
  8245. brinks
  8246. brod
  8247. brods
  8248. brog
  8249. brogh
  8250. broghs
  8251. bud
  8252. bug
  8253. bugs
  8254. bulk
  8255. bulks
  8256. bump
  8257. bumps
  8258. bund
  8259. bunds
  8260. bung
  8261. bungs
  8262. bunk
  8263. bunks
  8264. burd
  8265. burds
  8266. burg
  8267. burgh
  8268. burghs
  8269. burk
  8270. burks
  8271. burp
  8272. busk
  8273. by
  8274. ch
  8275. crwth
  8276. cwm
  8277. cwms
  8278. dab
  8279. dag
  8280. dak
  8281. damp
  8282. dap
  8283. deb
  8284. debs
  8285. debt
  8286. deft
  8287. delf
  8288. delfs
  8289. delft
  8290. delph
  8291. delphs
  8292. depth
  8293. derv
  8294. dervs
  8295. dhak
  8296. dib
  8297. dig
  8298. dight
  8299. dink
  8300. dinks
  8301. dirk
  8302. disk
  8303. div
  8304. divs
  8305. dob
  8306. dobs
  8307. dog
  8308. dop
  8309. dorp
  8310. dowf
  8311. drab
  8312. draft
  8313. drib
  8314. dribs
  8315. drip
  8316. drop
  8317. drub
  8318. drubs
  8319. drunk
  8320. drunks
  8321. dub
  8322. dug
  8323. dugs
  8324. dung
  8325. dunk
  8326. dunks
  8327. dup
  8328. dusk
  8329. dwarf
  8330. dzo
  8331. dzos
  8332. fad
  8333. fag
  8334. falx
  8335. fank
  8336. fard
  8337. fax
  8338. fed
  8339. fend
  8340. fends
  8341. fenks
  8342. fez
  8343. fib
  8344. fid
  8345. fig
  8346. fight
  8347. fink
  8348. firk
  8349. fisk
  8350. fix
  8351. fiz
  8352. fjord
  8353. fjords
  8354. flab
  8355. flag
  8356. flak
  8357. flank
  8358. flap
  8359. flax
  8360. fled
  8361. fleg
  8362. flex
  8363. flight
  8364. flimp
  8365. flip
  8366. flisk
  8367. flix
  8368. flog
  8369. flogs
  8370. flong
  8371. flongs
  8372. flop
  8373. flops
  8374. flub
  8375. flump
  8376. flumps
  8377. flung
  8378. flunk
  8379. flux
  8380. fob
  8381. fobs
  8382. fog
  8383. fogs
  8384. fold
  8385. folds
  8386. folk
  8387. folks
  8388. fond
  8389. fonds
  8390. fop
  8391. fops
  8392. ford
  8393. fords
  8394. fork
  8395. forks
  8396. fox
  8397. frab
  8398. frank
  8399. frap
  8400. fremd
  8401. fright
  8402. friz
  8403. frog
  8404. frond
  8405. frump
  8406. frumps
  8407. fub
  8408. fud
  8409. fug
  8410. fugs
  8411. fund
  8412. funds
  8413. funk
  8414. funks
  8415. fy
  8416. fyrd
  8417. fyrds
  8418. gab
  8419. gad
  8420. gamb
  8421. gamp
  8422. gap
  8423. gawk
  8424. gawp
  8425. ged
  8426. geld
  8427. gib
  8428. gid
  8429. gif
  8430. gild
  8431. gink
  8432. gip
  8433. gju
  8434. gjus
  8435. gled
  8436. glib
  8437. glid
  8438. glift
  8439. glitz
  8440. glob
  8441. globs
  8442. glyph
  8443. glyphs
  8444. gob
  8445. god
  8446. gold
  8447. golf
  8448. golfs
  8449. golp
  8450. golps
  8451. gonk
  8452. gov
  8453. govs
  8454. gowd
  8455. gowf
  8456. gowfs
  8457. gowk
  8458. gowks
  8459. graft
  8460. graph
  8461. grub
  8462. grypt
  8463. gub
  8464. gubs
  8465. gulf
  8466. gulfs
  8467. gulp
  8468. gulph
  8469. gulphs
  8470. gunk
  8471. gup
  8472. gym
  8473. gymp
  8474. gyp
  8475. gyps
  8476. hadj
  8477. hank
  8478. hyp
  8479. hyps
  8480. jab
  8481. jag
  8482. jak
  8483. jamb
  8484. jap
  8485. jark
  8486. jerk
  8487. jerks
  8488. jib
  8489. jibs
  8490. jig
  8491. jimp
  8492. jink
  8493. jinks
  8494. jinx
  8495. jird
  8496. jirds
  8497. jiz
  8498. job
  8499. jobs
  8500. jog
  8501. jogs
  8502. jud
  8503. juds
  8504. jug
  8505. jugs
  8506. junk
  8507. junks
  8508. jynx
  8509. kang
  8510. kant
  8511. kaw
  8512. keb
  8513. kebs
  8514. ked
  8515. kef
  8516. kefs
  8517. keg
  8518. kelp
  8519. kemb
  8520. kemp
  8521. kep
  8522. kerb
  8523. kerbs
  8524. kerf
  8525. kerfs
  8526. kex
  8527. khan
  8528. khud
  8529. khuds
  8530. kid
  8531. kids
  8532. kif
  8533. kifs
  8534. kight
  8535. kild
  8536. kiln
  8537. kilp
  8538. kind
  8539. kinds
  8540. king
  8541. kip
  8542. klepht
  8543. knag
  8544. knight
  8545. knob
  8546. knobs
  8547. knub
  8548. knubs
  8549. kob
  8550. kobs
  8551. kond
  8552. kop
  8553. kops
  8554. kraft
  8555. krantz
  8556. kranz
  8557. kvetch
  8558. ky
  8559. kynd
  8560. kynds
  8561. lav
  8562. lev
  8563. lez
  8564. link
  8565. luz
  8566. lynx
  8567. mawk
  8568. nabk
  8569. nth
  8570. pad
  8571. park
  8572. pawk
  8573. pax
  8574. ped
  8575. peg
  8576. pegh
  8577. peghs
  8578. pelf
  8579. pelfs
  8580. penk
  8581. perk
  8582. perv
  8583. pervs
  8584. phang
  8585. phiz
  8586. phlox
  8587. pig
  8588. pight
  8589. pix
  8590. pleb
  8591. plebs
  8592. pled
  8593. plight
  8594. plink
  8595. plod
  8596. plongd
  8597. plonk
  8598. pluck
  8599. plug
  8600. plumb
  8601. plumbs
  8602. plunk
  8603. ply
  8604. pod
  8605. polk
  8606. polks
  8607. pong
  8608. pork
  8609. pox
  8610. poz
  8611. prex
  8612. prod
  8613. prof
  8614. prog
  8615. pst
  8616. pub
  8617. pud
  8618. pug
  8619. pugh
  8620. pulk
  8621. pulks
  8622. punk
  8623. pyx
  8624. qat
  8625. qats
  8626. qibla
  8627. qiblas
  8628. quark
  8629. quiz
  8630. sh
  8631. skelf
  8632. skid
  8633. skrump
  8634. skug
  8635. sphinx
  8636. spiv
  8637. squawk
  8638. st
  8639. sunk
  8640. suq
  8641. swiz
  8642. sylph
  8643. tank
  8644. thilk
  8645. tyg
  8646. vamp
  8647. van
  8648. vang
  8649. vant
  8650. veg
  8651. veld
  8652. velds
  8653. veldt
  8654. vend
  8655. vends
  8656. verb
  8657. verbs
  8658. vet
  8659. vex
  8660. vibs
  8661. vild
  8662. vint
  8663. vly
  8664. vox
  8665. vug
  8666. vugs
  8667. vuln
  8668. waltz
  8669. wank
  8670. welkt
  8671. whack
  8672. zag
  8673. zap
  8674. zarf
  8675. zax
  8676. zed
  8677. zek
  8678. zeks
  8679. zel
  8680. zig
  8681. zigs
  8682. zimb
  8683. zimbs
  8684. zing
  8685. zings
  8686. zip
  8687. zips
  8688. zit
  8689. zurf
  8690. zurfs
  8691.  
  8692. ==> english/phonetic.letters.p <==
  8693. What does "FUNEX" mean?
  8694.  
  8695. ==> english/phonetic.letters.s <==
  8696. FUNEX? (Have you any eggs?)
  8697. SVFX. (Yes, we have eggs.)
  8698. FUNEM? (Have you any ham?)
  8699. SVFM. (Yes, we have ham.)
  8700. FUMNX? (Have you ham and eggs?)
  8701. S,S:VFM,VFX,VFMNX] (Yes, yes: we have ham, we have eggs, we have ham and eggs])
  8702.  
  8703. CD ED BD DUCKS? (See the itty bitty ducks?)
  8704. MR NOT DUCKS] (Them are not ducks])
  8705. OSAR, CDEDBD WINGS? (Oh yes they are, see the itty bitty wings?)
  8706. LILB MR DUCKS] (Well I'll be, them are ducks])
  8707.  
  8708. In Spanish:
  8709. SOCKS. (Eso si que es.)
  8710.  
  8711. ==> english/piglatin.p <==
  8712. What words in pig latin also are words?
  8713.  
  8714. ==> english/piglatin.s <==
  8715. cess    ->      essay
  8716. coke    ->      okay
  8717. lawn    ->      onlay
  8718. lout    ->      outlay
  8719. lover   ->      overlay
  8720. plover  ->      overplay
  8721. plunder ->      underplay
  8722. sass    ->      assay
  8723. stout   ->      outstay
  8724. trash   ->      ashtray
  8725. wear    ->      airway
  8726. wonder  ->      underway
  8727.  
  8728.  
  8729. ==> english/pleonasm.p <==
  8730. What are some redundant terms that occur frequently (like "ABM missile")?
  8731.  
  8732. ==> english/pleonasm.s <==
  8733. 11.5% APR
  8734. ABM missile
  8735. ABS system
  8736. AC current
  8737. ACT tests
  8738. AMOCO Oil Co.
  8739. APL programming language
  8740. ATM macine
  8741. BASIC Code
  8742. BBS System
  8743. CAD design
  8744. CNN news network
  8745. DC current
  8746. DMZ zone
  8747. DOS operating system
  8748. GMT time
  8749. Geirangerfjorden (Fjord Fjord Fjord)
  8750. HIV virus
  8751. ISBN number
  8752. ISDN network
  8753. LCD display
  8754. LED diode
  8755. La Brea Tar Pits
  8756. Los Altos Hills (The Hills Hills)
  8757. MIDI Interface
  8758. Mount Fujiyama (Mount Mountain)
  8759. NATO organization
  8760. NFS File System
  8761. PCV valve
  8762. PIN number
  8763. RAM (or ROM) memory
  8764. Ruidoso River (Noisy River River)
  8765. SALT talks
  8766. SAT test
  8767. SCSI Interface
  8768. SEATO organization
  8769. VIN number
  8770. floccinoccinihlipilification (from 4 latin words meaning "nothing")
  8771. hoi polloi (a genuine bilingual redundancy)
  8772. hot water heater
  8773.  
  8774. ==> english/plurals/collision.p <==
  8775. Two words, spelled and pronounced differently, have plurals spelled
  8776. the same but pronounced differently.
  8777.  
  8778. ==> english/plurals/collision.s <==
  8779. axe and axis -> axes
  8780. base and basis -> bases
  8781. ellipse and ellipsis -> ellipses
  8782.  
  8783. ==> english/plurals/doubtful.number.p <==
  8784. A little word of doubtful number,
  8785. a foe to rest and peaceful slumber.
  8786. If you add an "s" to this,
  8787. great is the metamorphosis.
  8788. Plural is plural now no more,
  8789. and sweet what bitter was before.
  8790. What am I?
  8791.  
  8792. ==> english/plurals/doubtful.number.s <==
  8793. cares -> caress
  8794.  
  8795. ==> english/plurals/drop.s.p <==
  8796. What plural is formed by DROPPING the terminal "s" in a word?
  8797.  
  8798. ==> english/plurals/drop.s.s <==
  8799. necropolis -> necropoli
  8800.  
  8801. ==> english/plurals/endings.p <==
  8802. List a plural ending with each letter of the alphabet.
  8803.  
  8804. ==> english/plurals/endings.s <==
  8805. Legend
  8806. 0 = plural formed (basically) by adding letter
  8807. 1 = plural spelled differently from singular
  8808. 2 = ditto, plural contains punctuation
  8809. 3 = plural spelled the same as singular
  8810.  
  8811. All entries are from Merriam-Webster's Ninth Collegiate Dictionary,
  8812. except those marked "(NI3)", which are from the Third International.
  8813. Entries in brackets are probable dictionary artifacts.
  8814.  
  8815. A 0 VAS VASA
  8816. B 1 SLUBBI SLEYB (NI3)
  8817. C 0 CALPULLI CALPULLEC (NI3)
  8818. D 2 GRANT-IN-AID GRANTS-IN-AID
  8819. E 0 ALA ALAE
  8820. F 1 SHARIF ASHRAF (NI3)
  8821. G 0 AIRE AIRIG (NI3)
  8822. H 0 LIRA LIROTH
  8823. I 0 BAN BANI
  8824. J 1 KHARIJITE KHAWARIJ (NI3)
  8825. K 0 PULI PULIK
  8826. L 1 ARMFUL ARMSFUL
  8827. M 0 GOY GOYIM
  8828. N 0 KRONE KRONEN
  8829. O 2 DERRING-DO DERRINGS-DO (NI3) ▌1 MEO MIAO/MIXTECA MIXTECO/PAPIOPIO PAPIO/SUMU SUMO (NI3)¿
  8830. P 2 AIDE-DE-CAMP AIDES-DE-CAMP
  8831. Q 3 QARAQALPAQ QARAQALPAQ (NI3)
  8832. R 0 KRONE KRONER
  8833. S 0 A AS
  8834. T 0 MATZO MATZOT
  8835. U 0 HALER HALERU
  8836. V 3 TIV TIV (NI3)
  8837. W 2 SON-IN-LAW SONS-IN-LAW ▌1 KWAPA QUAPAW (NI3)¿
  8838. X 0 EAU EAUX
  8839. Y 0 GROSZ GROSZY
  8840. Z 3 HERTZ HERTZ
  8841.  
  8842. ==> english/plurals/french.p <==
  8843. What English word, when spelled backwards, is its French plural?
  8844.  
  8845. ==> english/plurals/french.s <==
  8846. state/etats
  8847.  
  8848. ==> english/plurals/man.p <==
  8849. Words ending with "man" make their plurals by adding "s".
  8850.  
  8851. ==> english/plurals/man.s <==
  8852. caiman
  8853. doberman
  8854. German
  8855. human
  8856. leman
  8857. ottoman
  8858. pitman
  8859. Pullman
  8860. Roman
  8861. shaman
  8862. talisman
  8863.  
  8864. ==> english/plurals/switch.first.p <==
  8865. What plural is formed by switching the first two letters?
  8866.  
  8867. ==> english/plurals/switch.first.s <==
  8868. falaj -> aflaj (Chambers English Dictionary)
  8869.  
  8870. ==> english/portmanteau.p <==
  8871. What are some words formed by combining together parts of other words?
  8872.  
  8873. ==> english/portmanteau.s <==
  8874. Such words are called "Portmanteau" words.  Here is a very incomplete list:
  8875. beefalo         beef, buffalo
  8876. brunch          breakfast, lunch
  8877. chortle         chuckle, snort
  8878. fantabulous     fantastic, fabulous
  8879. flare           flame, glare
  8880. flounder        flounce, founder
  8881. glimmer         gleam, shimmer
  8882. glitz           glamour, ritz
  8883. liger           lion, tiger
  8884. motel           motor, hotel
  8885. smash           smack, mash
  8886. smog            smoke, fog
  8887. squiggle        squirm, wiggle
  8888. tangelo         tangerine, pomelo
  8889. tigon           tiger, lion
  8890. ****
  8891. Unless noted otherwise, all words occur in Webster's Third New International
  8892. Dictionary, Merriam-Webster, Springfield, MA, 1961.
  8893.  
  8894. ==> english/potable.color.p <==
  8895. Find words that are both beverages and colors.
  8896.  
  8897. ==> english/potable.color.s <==
  8898. burgundy
  8899. champagne
  8900. chartreuse
  8901. chocolate
  8902. claret
  8903. cocoa
  8904. coffee
  8905. cream
  8906. midori (Japanese for green. Does Japanese count?)
  8907. rose
  8908. wine
  8909.  
  8910. ==> english/rare.trigraphs.p <==
  8911. What trigraphs (three-letter combinations) occur in only one word?
  8912.  
  8913. ==> english/rare.trigraphs.s <==
  8914. Here is a list of all the trigraphs which occur exactly once in the union of
  8915. _Official Scrabble Words_ (First Edition), the _Official Scrabble Players
  8916. Dictionary_ and _Webster's Unabridged Dictionary (Second Edition)_,
  8917. together with the words in which they occur.
  8918.  
  8919. The definition of "word" is a problematic.  For example, lots of words
  8920. starting deoxy- contain the trigraph `eox', but no others do.  Should
  8921. `eox' be on the list?
  8922.  
  8923. Common words are marked with a *.
  8924.  
  8925. aae baaed
  8926. adq*headquarter headquarters
  8927. ajs svarajs
  8928. aqs talaqs
  8929. bks nabks
  8930. bze subzero
  8931. cda ducdame
  8932. dph*headphone headphones
  8933. dsf*handsful
  8934. dts veldts
  8935. dzu kudzu kudzus
  8936. ekd*weekday weekdays
  8937. evh evhoe
  8938. evz evzone evzones
  8939. exv sexvalent
  8940. ezv*rendezvous
  8941. fhu cliffhung
  8942. fjo fjord fjords
  8943. fsp*offspring offsprings
  8944. gds smaragds
  8945. ggp*eggplant eggplants
  8946. gnb signboard signboards
  8947. gnp*signpost signposted signposting signposts
  8948. gnt sovereignty
  8949. gty hogtying
  8950. gza*zigzag zigzagged zigzagging zigzaggy zigzags
  8951. hds camanachds
  8952. hky droshky
  8953. hlr kohlrabi kohlrabies kohlrabis
  8954. hrj lehrjahre
  8955. hyx asphyxia asphyxias asphyxiate asphyxies asphyxy
  8956. itv mitvoth
  8957. iwy skiwy
  8958. ixg sixgun
  8959. jds slojds
  8960. jje hajjes
  8961. jki pirojki pirojki
  8962. jym jymold
  8963. kky yukky
  8964. ksg*thanksgiving
  8965. kuz yakuza
  8966. kvo mikvoth
  8967. kyj*skyjack skyjacked skyjacker skyjackers skyjacking skyjackings skyjacks
  8968. llj killjoy killjoys
  8969. lmd filmdom filmdoms
  8970. ltd*meltdown meltdowns
  8971. lxe calxes
  8972. lzy schmalzy
  8973. mds fremds
  8974. mfy comfy
  8975. mhs ollamhs
  8976. mky dumky
  8977. mmm dwammming
  8978. mpg*campground
  8979. mss bremsstrahlung
  8980. muo muon muonic muonium muoniums muons
  8981. nhs sinhs
  8982. njy benjy
  8983. nuu continuum
  8984. obg hobgoblin hobgoblins
  8985. ojk pirojki
  8986. okc*bookcase bookcases
  8987. ovk sovkhoz sovkhozes sovkhozy
  8988. pev*grapevine grapevines
  8989. pfs dummkopfs
  8990. php ephphatha
  8991. pss topssmelt
  8992. pyj pyjama pyjamaed pyjamas
  8993. siq physique physiques
  8994. slt juslted
  8995. smk besmkes
  8996. spb*raspberries raspberry
  8997. spt claspt
  8998. swy swythe
  8999. syg*easygoing
  9000. szy groszy
  9001. tux*tux tuxedo tuxedoes tuxedos tuxes
  9002. tvy outvying
  9003. tzu tzuris
  9004. ucd ducdame
  9005. vho evhoe
  9006. vkh sovkhoz sovkhozes sovkhozy sovkhos
  9007. vly vly
  9008. vns eevns
  9009. voh evohe
  9010. vun avuncular
  9011. wcy gawcy
  9012. wdu*sawdust sawdusted sawdusting sawdusts sawdusty
  9013. wfr bowfront
  9014. wft ewftes
  9015. xeu exeunt
  9016. xgl foxglove foxgloves
  9017. xiw taxiway taxiways
  9018. xls cacomixls
  9019. xtd nextdoor
  9020. xva sexvalent
  9021. yks bashlyks
  9022. yrf gyrfalcon gyrfalcons
  9023. ysd paysd
  9024. yxy asphyxy
  9025. zhk pirozhki
  9026. zow zowie
  9027. zwo*buzzword buzzwords
  9028. zzs*buzzsaw
  9029.  
  9030. ==> english/records/pronunciation/silent.p <==
  9031. What words have an exceptional number of silent letters?
  9032.  
  9033. ==> english/records/pronunciation/silent.s <==
  9034.   longest sequence  BROUGHAM (4, UGHA)
  9035.   for each letter  AISLE, COMB, INDICT,
  9036.    HANDSOME, TWITCHED, HALFPENNY, GNOME, MYRRH, BUSINESS, MARIJUANA, KNOCK,
  9037.    TALK, MNEMONIC, AUTUMN, PEOPLE, PSYCHE, CINQCENTS, FORECASTLE, VISCOUNT,
  9038.    HAUTBOY, PLAQUE, FIVEPENCE, WRITE, TABLEAUX, PRAYER, RENDEZVOUS
  9039.   homophones, for each letter  O(A)R, LAM(B), S(C)ENT,
  9040.    LE(D)GER, DO(E), WAF(F), REI(G)N, (H)OUR, WA(I)VE, HAJ(J)I, (K)NOT, HA(L)VE,
  9041.    PRIM(M)ER, DAM(N), J(O)UST, (P)SALTER, ?, CAR(R)IES, (S)CENT, TARO(T),
  9042.    B(U)Y, ?, T(W)O, ?, RE(Y), BIZ(Z)
  9043. ****
  9044. Unless noted otherwise, all words occur in Webster's Third New International
  9045. Dictionary, Merriam-Webster, Springfield, MA, 1961.
  9046.  
  9047. ==> english/records/pronunciation/spelling.p <==
  9048. What words have exceptional ways to spell sounds?
  9049.  
  9050. ==> english/records/pronunciation/spelling.s <==
  9051.   same spelling, different sound -OUGH (7)
  9052.    BOUGH, COUGH, DOUGH, HICCOUGH, LOUGH, ROUGH, THROUGH
  9053.   different spelling, same sound AIR (9)
  9054.    AIR, AIRE, ARE, AYR, AYER, E'ER, ERE, ERR, HEIR
  9055. ****
  9056. Unless noted otherwise, all words occur in Webster's Third New International
  9057. Dictionary, Merriam-Webster, Springfield, MA, 1961.
  9058.  
  9059. ==> english/records/pronunciation/syllable.p <==
  9060. What words have an exceptional number of letters per syllable?
  9061.  
  9062. ==> english/records/pronunciation/syllable.s <==
  9063.   longest for each number of syllables
  9064.    one SCRAUNCHED  ▌SQUIRRELLED (11)¿ two SCRATCHBRUSHED (14)
  9065.    one, for each letter  ARCHED, BROUGHAMS, CRAUNCHED, DRAUGHTS,
  9066.     EARTHED, FLINCHED, GROUCHED, HAUNCHED, ITCHED, JOUNCED, KNIGHTS, LAUNCHED,
  9067.     MOOCHED, NAUGHTS, OINKED, PREACHED, QUETCHED, REACHED, SCRAUNCHED,
  9068.     THOUGHTS, UMPHS, VOUCHED, WREATHED, XYSTS, YEARNED, ZOUAVES
  9069.    two, for each letter  ARCHFIENDS, BREAKTHROUGHS, CLOTHESHORSE,
  9070.     DRAUGHTBOARDS, EARTHTONGUES, FLAMEPROOFED, GREATHEART, HAIRSBREADTHS,
  9071.     INTHRALLED, JUNETEENTHS, KNICKKNACKS, LIGHTWEIGHTS, MOOSETONGUES,
  9072.     NIGHTCLOTHES, OUTSTRETCHED, PLOUGHWRIGHTS, QUICKTHORNS, ROUGHSTRINGS,
  9073.     SCRATCHBRUSHED, THROATSTRAPS, UNSTRETCHED, VERSESMITHS, WHERETHROUGH,
  9074.     XANTHINES, YOURSELVES, ZEITGEISTS
  9075.   shortest for each number of syllables
  9076.    two AA  three AREA (4) ▌O'IO (3)¿ four IEIE (4) five OXYOPIA (7)
  9077.    six ONIOMANIA  ▌AMIOIDEI (8)¿ seven EPIDEMIOLOGY (12) ▌OMOHYOIDEI (10)¿
  9078.    eight EPIZOOTIOLOGY  nine EPIZOOTIOLOGICAL (16) ten EPIZOOTIOLOGICALLY
  9079.     twelve HUMUHUMUNUKUNUKUAPUAA (21)
  9080. ****
  9081. Unless noted otherwise, all words occur in Webster's Third New International
  9082. Dictionary, Merriam-Webster, Springfield, MA, 1961.
  9083.  
  9084. ==> english/records/spelling/longest.p <==
  9085. What is the longest word in the English language?
  9086.  
  9087. ==> english/records/spelling/longest.s <==
  9088. The longest word to occur in both English and American "authoritative"
  9089. unabridged dictionaries is "pneumonoultramicroscopicsilicovolcanoconiosis."
  9090.  
  9091. The following is a brief citation history of this "word."
  9092.  
  9093. New York Herald Tribune, February 23, 1935, p. 3
  9094. "Pneumonoultramicroscopicsilicovolcanokoniosis succeeded
  9095. electrophotomicrographically as the longest word in the English
  9096. language recognized by the National Puzzlers' League at the opening
  9097. session of the organization's 103d semi-annual meeting held yesterday
  9098. at the Hotel New Yorker.
  9099.  
  9100. The puzzlers explained that the forty-five-letter word is the name of a
  9101. special form of silicosis caused by ultra-microscopic particles of
  9102. siliceous volcanic dust."
  9103.  
  9104. Everett M. Smith (b. 1/1/1894), President of NPL and Radio News Editor
  9105. of the Christian Science Monitor, cited the word at the convention.
  9106. Smith was also President of the Yankee Puzzlers of Boston.
  9107. It is not known whether Smith coined the word.
  9108.  
  9109. "Bedside Manna. The Third Fun in Bed Book.", edited by Frank Scully,
  9110. Simon and Schuster, New York, 1936, p. 87
  9111. "There's been a revival in interest in spelling, but Greg Hartswick,
  9112. the cross word king and world's champion speller, is still in control
  9113. of the situation.  He'd never get any competition from us, that's
  9114. sure, though pronouncing, let alone spelling, a 44 letter word like:
  9115.         Pneumonoultramicrosopicsilicovolkanakoniosis,
  9116. a disease caused by ultra-microscopic particles of sandy volcanic dust
  9117. might give even him laryngitis."
  9118.  
  9119. It is likely that Scully, who resided in New York in February 1935,
  9120. read the Herald Tribune article and slightly misremembered the word.
  9121.  
  9122. Supplement to the Oxford English Dictionary, 1936
  9123. Both "-coniosis" and "-koniosis" are cited.
  9124. "a factitious word alleged to mean 'a lung disease caused by the inhalation
  9125. of very fine silica dust' but occurring chiefly as an instance of a very long
  9126. word."
  9127.  
  9128. Webster's first cite is "-koniosis" in the addendum to the Second Edition.
  9129. The Third Edition changes the "-koniosis" to "-coniosis."
  9130.  
  9131. I conjecture that this "word" was coined by word puzzlers, who then
  9132. worked assiduously to get it into the major unabridged dictionaries
  9133. (perhaps with a wink from the editors?)  to put an end to the endless
  9134. squabbling about what is the longest word.
  9135.  
  9136. ==> english/records/spelling/most.p <==
  9137. What word has the most variant spellings?
  9138.  
  9139. ==> english/records/spelling/most.s <==
  9140. catercorner
  9141.  
  9142. There's eight spellings in Webster's Third.
  9143.  
  9144. catercorner
  9145. cater-cornered
  9146. catacorner
  9147. cata-cornered
  9148. catty-corner
  9149. catty-cornered
  9150. kitty-corner
  9151. kitty-cornered
  9152.  
  9153. If you look in Random House, you will find one more which doesn't appear
  9154. in Web3, but it only differs by a hyphen:
  9155.  
  9156. cater-corner
  9157.  
  9158. ---
  9159. Dan Tilque    --     dant@techbook.com
  9160.  
  9161. ==> english/records/spelling/operations.on.words/deletion.p <==
  9162. What exceptional words turn into other words by deletion of letters?
  9163.  
  9164. ==> english/records/spelling/operations.on.words/deletion.s <==
  9165.    longest beheadable word  P(REDETERMINATION) (16/15)
  9166.     longest for each letter (6-88,181,198,213,13-159,14-219,15-155,16-96,220,
  9167.      17-85) APATHETICALLY, BLITHESOME, CHASTENING, DEMULSIFICATION,
  9168.      EMOTIONLESSNESS, FUTILITARIANISM, GASTRONOMICALLY, HEDRIOPHTHALMA,
  9169.      IDENTIFICATION, JUNCTIONAL, KINAESTHETIC, LIMITABLENESS, METHYLACETYLENE,
  9170.      NEOPALEOZOIC, OENANTHALDEHYDE, PREDETERMINATION, QUINTA, REVOLUTIONARILY,
  9171.      SELECTIVENESS, TREASONABLENESS, UPRAISER, VINDICATION, WHENCEFORWARD,
  9172.      XANTHOPHYLLITE, YOURSELVES, ZOOSPORIFEROUS
  9173.     longest beheadable down to a single letter  PRESTATE (8)
  9174.    longest curtailable word (not a plural)  (BULLETIN)G (9)
  9175.     longest curtailable down to a single letter  LAMBASTES
  9176.    longest alternately beheadable and curtailable word  ASHAMED (7)
  9177.    longest arbitrarily beheadable and curtailable (all subsequences words)
  9178.      SHADES (6)
  9179.    longest terminal ellision word  D(EPILATION)S (11)
  9180.    longest letter subtraction down to a single letter  STRANGLING,
  9181.     STRANGING, STANGING, STAGING, SAGING, AGING, GING, GIN, IN, I
  9182.    longest charitable word (subtract letter anywhere)
  9183.     PLEATS: LEATS,PEATS,PLATS,PLEAS,PLEAT
  9184.    shortest stingy word (no deletion possible)  PRY (3)
  9185. ****
  9186. Unless noted otherwise, all words occur in Webster's Third New International
  9187. Dictionary, Merriam-Webster, Springfield, MA, 1961.
  9188.  
  9189. ==> english/records/spelling/operations.on.words/insertion.and.deletion.p <==
  9190. What exceptional words turn into other words by both insertion and
  9191. deletion of letters?
  9192.  
  9193. ==> english/records/spelling/operations.on.words/insertion.and.deletion.s <==
  9194.    longest word both charitable and hospitable
  9195.     AMY: AM,AY,MY;GAMY,ARMY,AMOY,AMYL
  9196.    shortest word both stingy and hostile  IMPETUOUS (9)
  9197. ****
  9198. Unless noted otherwise, all words occur in Webster's Third New International
  9199. Dictionary, Merriam-Webster, Springfield, MA, 1961.
  9200.  
  9201. ==> english/records/spelling/operations.on.words/insertion.p <==
  9202. What exceptional words turn into other words by insertion of letters?
  9203.  
  9204. ==> english/records/spelling/operations.on.words/insertion.s <==
  9205.    longest hydration (double reheadment)  (D,R)EVOLUTIONIST (12/13)
  9206.    longest hospitable word (insert letter anywhere)
  9207.     CARES: SCARES, CHARES, CADRES, CARIES, CARETS, CARESS
  9208.    shortest hostile word (no deletion possible)  SYZYGY (6)
  9209. ****
  9210. Unless noted otherwise, all words occur in Webster's Third New International
  9211. Dictionary, Merriam-Webster, Springfield, MA, 1961.
  9212.  
  9213. ==> english/records/spelling/operations.on.words/movement.p <==
  9214. What exceptional words turn into other words by movement of letters?
  9215.  
  9216. ==> english/records/spelling/operations.on.words/movement.s <==
  9217.    longest word allowing exchange of letters (metallege)
  9218.     CONSERVATIONAL, CONVERSATIONAL
  9219.    longest head-to-tail shift
  9220.     SPECULATION, PECULATIONS
  9221.    longest double head-to-tail shift
  9222.     STABLE-TABLES-ABLEST
  9223.    longest complete cyclic transposal  ATE-TEA-EAT (3)
  9224. ****
  9225. Unless noted otherwise, all words occur in Webster's Third New International
  9226. Dictionary, Merriam-Webster, Springfield, MA, 1961.
  9227.  
  9228. ==> english/records/spelling/operations.on.words/substitution.p <==
  9229. What exceptional words turn into other words by substitution of letters?
  9230.  
  9231. ==> english/records/spelling/operations.on.words/substitution.s <==
  9232. longest onalosi (substitution in every position possible)
  9233.  PASTERS: MASTERS,POSTERS,PALTERS,PASSERS,PASTORS,PASTELS,PASTERN
  9234. shortest isolano (no substitution possible)
  9235.  ECRU
  9236. longest word, all letters changed to other letters in minimum number of
  9237. steps, yielding another word  THUMBING-THUMPING-TRUMPING-TRAMPING-
  9238.  TRAPPING-CRAPPING-CRAPPIES-CRAPPOES
  9239. longest word girders  BADGER/SUNLIT, BUDLET/SANGIR (6)
  9240. longest word with full vowel substitution
  9241.  CL(A,E,I,O,U)CKING (8) also Y D(A,E,I,O,U,Y)NE (4)
  9242. longest words with vowel substitutions
  9243.  DESTRUCTIBILITIES, DISTRACTIBILITIES (17)
  9244. longest word constant-letter-shifted to another PRIMERO-SULPHUR (7)
  9245.  arithmetical-letter-shifted DREAM-ETHER (5)
  9246.  constant-shift-with-transposal (shiftgrams) AEROPHANE-SILVERITE (9)
  9247. longest word pair shifted one position on typewriter keyboard WAXIER-ESCORT (6)
  9248. longest word pair confusable on a telephone keypad AMOUNTS-CONTOUR (7)
  9249. ****
  9250. Unless noted otherwise, all words occur in Webster's Third New International
  9251. Dictionary, Merriam-Webster, Springfield, MA, 1961.
  9252.  
  9253. ==> english/records/spelling/operations.on.words/transposition.p <==
  9254. What exceptional words turn into other words by transposition of letters?
  9255.  
  9256. ==> english/records/spelling/operations.on.words/transposition.s <==
  9257.    longest reversal  DESSERTS,STRESSED (8)
  9258.    longest well-mixed transposal
  9259.     CINEMATOGRAPHER, MEGACHIROPTERAN  (15)
  9260.    longest transposition list
  9261.     APERS, APRES, ASPER, PARES, PARSE, PEARS, PRASE, PRESA, RAPES, REAPS, SPARE,
  9262.     SPEAR (12)
  9263.     ANGRIEST, ANGRITES, ASTRINGE, GAIRTENS, GANISTER, GANTRIES, GRANITES,
  9264.     INGRATES, RANGIEST, TEARINGS  (10) ▌SATING(ER), SIGNATE(R), TANGIER(S) (3)¿
  9265.     ANORETICS, ATROSCINE, CANOTIERS, CERTOSINA, CONARITES, CREATIONS, REACTIONS,
  9266.     TRICOSANE (8)
  9267.  
  9268.   transposition with deletion, insertion, or substitution
  9269.    longest well-mixed transdeletion
  9270.     SONOLUMINESCENCES, UNECONOMICALNESSES (17/18)
  9271.    longest word transdeletable to a single letter
  9272.     CONCENTRATIONS-CONSTERNATION-CONTORNIATES-TRANSECTION-
  9273.     STENTORIAN-TRANSIENT-ENTRAINS-NASTIER-ASTERN-TEARS-SATE-TEA-AT-A (14)
  9274.    longest Baltimore transdeletion (word transdeletable on every letter)
  9275.      IDOLATERS: DELATORS, SOTERIAL, DILATERS, ASTEROID,
  9276.     STOLIDER, SOREDIAL, DILATORS, DIASTOLE, TAILORED  (9)
  9277.    shortest word that cannot be transadded to another word  SYZYGY (6)
  9278.    longest well-mixed transubstitution
  9279.     MICROELECTROPHORESIS, SPECTROCOLORIMETRIES (20)
  9280. ****
  9281. Unless noted otherwise, all words occur in Webster's Third New International
  9282. Dictionary, Merriam-Webster, Springfield, MA, 1961.
  9283.  
  9284. ==> english/records/spelling/operations.on.words/words.within.words.p <==
  9285. What exceptional words contain other words?
  9286.  
  9287. ==> english/records/spelling/operations.on.words/words.within.words.s <==
  9288.    longest non-trivial charade  IN-DISC-RIM-IN-A-TI-ON (16)
  9289.     longest forward and reverse charade
  9290.      MAT-HE-MA-TI-CAL, LAC-IT-AM-EH-TAM
  9291.    longest snowball or rhopalic  T-EM-PER-AMEN-TALLY (15)
  9292.    longest reverse rhopalic  HETERO-TRANS-PLAN-TAT-IO-N (21)
  9293.    highest ratio of subwords/length (logogram)
  9294.     FIRESTONE: RE, TO, ON, NO, IF, FIR, IRE, RES, TON, ONE, NOT, RIF, FIRE,
  9295.     IRES, REST, TONE, FIRES, STONE, SERIF (20/9)
  9296.    longest charlinkade  FORESTALL: FOREST, ALL; FORE, REST, TALL (9)
  9297.    longest alternade  TRIENNIALLY: TINILY, RENAL (11)
  9298.    shortest three-letter-minimum word deletions
  9299.     PILGRIMAGE: RIM, GAG, PILE; GRIM, LAG, PIE
  9300. ****
  9301. Unless noted otherwise, all words occur in Webster's Third New International
  9302. Dictionary, Merriam-Webster, Springfield, MA, 1961.
  9303.  
  9304. ==> english/records/spelling/sets.of.words/nots.and.crosses.p <==
  9305. What is the most number of letters that can be fit into a three by three grid
  9306. of words, such that no letter is repeated in any row, column or diagonal?
  9307.  
  9308. ==> english/records/spelling/sets.of.words/nots.and.crosses.s <==
  9309. Games magazine ran a contest on this.  The winner had 62:
  9310.  
  9311.                 proxying!   buckwash  !   veldt
  9312.                 ------------------------------------
  9313.                 stumbled!      j      !  zincography
  9314.                 ------------------------------------
  9315.                 whack   ! providently !   bumfs
  9316.  
  9317. Here are some good tries:
  9318.  
  9319.                 backsword !thumpingly !   fez
  9320.                 ----------------------------------
  9321.                 vexingly  !      q    ! throwbacks = 61
  9322.                 ----------------------------------
  9323.                 thump     ! beadworks !   jingly
  9324.  
  9325.                 backsword ! thumpingly!  vex
  9326.                 ----------------------------------
  9327.                 vexingly  !      q    ! throwbacks = 60
  9328.                 ----------------------------------
  9329.                 thump     !  bedrocks !   flying
  9330.  
  9331.                 subjack   !downrightly!    fez
  9332.                 ----------------------------------
  9333.                 novelwright!     q    !  backups   = 59
  9334.                 ----------------------------------
  9335.                 pyx       !  subface  ! downright
  9336.  
  9337.                 krafts    ! exhuming  ! blowzy
  9338.                 ----------+-----------+-----------
  9339.                 phylum    !     j     ! transfixed = 56
  9340.                 ----------+-----------+-----------
  9341.                 vexing    ! folkways  ! chump
  9342.  
  9343.                 klutz     ! cymograph !   fend
  9344.                 ----------+-----------+-----------
  9345.                 exscind   !     j     ! kymograph = 54
  9346.                 ----------+-----------+-----------
  9347.                 myograph  !flunked    !   vibs
  9348. ****
  9349. Unless noted otherwise, all words occur in Webster's Third New International
  9350. Dictionary, Merriam-Webster, Springfield, MA, 1961.
  9351.  
  9352. ==> english/records/spelling/sets.of.words/squares.p <==
  9353. What are some exceptional word squares (square crosswords with no blanks)?
  9354.  
  9355. ==> english/records/spelling/sets.of.words/squares.s <==
  9356. Word squares are a particular example of a type of crossword known
  9357. as "forms".  They were more popular early in the 20th century than
  9358. they are now, but people still like to compose and solve them.  Forms
  9359. appear every month in the _Enigma_, which is the monthly publication
  9360. of the National Puzzlers' League.  The membership fee is $13 for
  9361. the first year, and information may be obtained from:
  9362.  
  9363.   David A. Rosen
  9364.   207 East 27th St. #3K
  9365.   New York, NY 10016
  9366.  
  9367. All members have the option of choosing a nom de plume; for example,
  9368. I go by the nom "Cubist".  Another good place to find information on
  9369. forms is in _Word Ways_, which is a quarterly journal of recreational
  9370. linguistics:
  9371.  
  9372.   _Word Ways_
  9373.   Spring Valley Road
  9374.   Morristown, NJ  07960
  9375.  
  9376. I'll have a paper appearing here at some point on the "support" of a
  9377. form (which I'll discuss below).
  9378.  
  9379. Word squares come in two flavors, regular and double.  In regular word
  9380. squares the words are the same across and down; in double word squares
  9381. all words are different.  The largest legitimate word square has order
  9382. 9 (although Jeff Grant has come close to the 10), and what is considered
  9383. to be the finest example was discovered by Eric Albert via computer search:
  9384.  
  9385.   necessism
  9386.   existence
  9387.   circumfer
  9388.   escarping
  9389.   sturnidae
  9390.   sempitern
  9391.   infidelic
  9392.   scenarize
  9393.   mergences
  9394.  
  9395. All words appear in from Webster's New International Dictionary, Second
  9396. Edition.  It's the *only* single-source 9-square known, and its only
  9397. flaw is that "Sturnidae" is a proper (capitalized) word.  All words
  9398. are also solid-form (no phrases, spaces, punctuation marks, etc.).
  9399.  
  9400. Eric was using about 63,000 words when he discovered his square.  Using
  9401. about 78,500 9-letter words, I found an additional square:
  9402.  
  9403.   bortsches
  9404.   overtrust
  9405.   reparence
  9406.   trabeatae
  9407.   strestell
  9408.   creatural
  9409.   hunterite
  9410.   escalates
  9411.   steelless
  9412.  
  9413. All are in the OED, except for "trabeatae", which is in NI2.  This
  9414. makes this square arguably the second-best ever discovered.  All
  9415. words are uncapitalized and solid-form, but it has the flaw of using
  9416. more than one source.  It is, however, the *only* known 9-square that
  9417. uses only uncapitalized, solid-form dictionary words.
  9418.  
  9419. There are about 2000 9-squares known, all of which were constructed
  9420. by hand except for the two noted above.  Almost all of these use
  9421. very obscure sources of words.  As a general rule of thumb, if you
  9422. discover a new form via computer search, it is probably going to be
  9423. of high quality, since it is hard to obtain computer-readable word
  9424. lists that contain *really* obscure words.
  9425.  
  9426. The largest known double word-squares are of order-8.  They are
  9427. considered to be about as hard to construct as a regular word
  9428. square of order-9, and this is substantiated by the work I've
  9429. done on the mathematics of form construction.   The following
  9430. fine example was constructed by Jeff Grant (see his article in
  9431. _Word Ways_, Vol. 25 Num. 1, pp. 9-12):
  9432.  
  9433.   trattled
  9434.   hemerine
  9435.   apotomes
  9436.   metapore
  9437.   nailings
  9438.   aloisias
  9439.   tentmate
  9440.   assessed
  9441.  
  9442. All are dictionary terms, but there are some weak entries, e.g.
  9443.  
  9444.   Aloisias: individuals named Aloisia, a feminine form of Aloysius
  9445.   occurring in the 16th and 17th century in parish registers of
  9446.   Hinton Charterhouse, England (The Oxford Dictionary of English
  9447.   Christian Names, 3rd Edition, E.G. Withycombe, 1977)
  9448.  
  9449. Such words are, however, dear to the heart of logologists]  For
  9450. other examples of double squares see the article mentioned above.
  9451.  
  9452. There are also many other types of forms.  Some of the most common
  9453. are pyramids, stars, and diamonds, and some come in regular and double
  9454. varieties, and some are inherently double (e.g. rectangles).
  9455.  
  9456. How hard is it to discover a square, anyway, and how many are there?
  9457. As a data point, my program using the main (Air Force) entries in
  9458. NI2 (26,332 words), found only seven 8x8 squares.  This took an hour
  9459. to run.  They are:
  9460.  
  9461. outtease  appetite  unabated  acetated  interact  repeated  repeated
  9462. unweaned  prenaris  nopinene  cadinene  neomenia  evenmete  evenmete
  9463. twigsome  perscent  apostate  edentate  toxicant  pectosic  pectosic
  9464. teguexin  ensconce  bistered  tindered  emittent  entresol  entresol
  9465. easement  taconite  antehall  antehall  rectoral  amoebula  amoebula
  9466. anoxemia  irenicum  tearable  tearable  anaerobe  tessular  tessular
  9467. seminist  tincture  entellus  entellus  cinnabar  etiolate  etiolate
  9468. edentate  esteemer  deedless  deedless  tattlery  declarer  declared
  9469.  
  9470. If the heuristic mathematics are worked out, the number of different
  9471. words in your word-list before you'd expect to find a regular word
  9472. square of order-n (the "support") is about e^{(n-1)/2}, where e ~ 15.7.
  9473. For a double word square of order-n the support is about e^{n/2}.
  9474. There is a simple algorithm which is more precise, and this gives a
  9475. support of 75,641 for a regular 9-square, and a support of 272,976
  9476. for a double 9-square (using my 9-letter word list), which agrees
  9477. well with reality.
  9478. --
  9479. Chris Long, 265 Old York Rd., Bridgewater, NJ  08807-2618
  9480. clong@remus.rutgers.edu
  9481.  
  9482. ==> english/records/spelling/single.words.p <==
  9483. What words have exceptional lengths, patterns
  9484.  
  9485. ==> english/records/spelling/single.words.s <==
  9486. Word Records from Webster's Third
  9487.  
  9488. Spelling
  9489.  
  9490. Letter Patterns
  9491.  
  9492. Entire Word
  9493. longest word trinitrophenylmethylnitramine (29,1)
  9494. longest palindrome kinnikinnik (11,1)
  9495. longest beginning with a palindrome adinida (7,1)
  9496. longest beginning with b palindrome boob (4,1)
  9497. longest beginning with c palindrome carac civic (5,2)
  9498. longest beginning with d palindrome deified devoved (7,2)
  9499. longest beginning with e palindrome ecce esse (4,2)
  9500. longest beginning with f palindrome f (1,1)
  9501. longest beginning with g palindrome goog (4,1)
  9502. longest beginning with h palindrome hagigah halalah (7,2)
  9503. longest beginning with i palindrome igigi imami (5,2)
  9504. longest beginning with j palindrome j (1,1)
  9505. longest beginning with k palindrome kinnikinnik (11,1)
  9506. longest beginning with l palindrome lemel level lysyl (5,3)
  9507. longest beginning with m palindrome malayalam (9,1)
  9508. longest beginning with n palindrome nauruan (7,1)
  9509. longest beginning with o palindrome oppo otto (4,2)
  9510. longest beginning with p palindrome peeweep (7,1)
  9511. longest beginning with q palindrome qazaq (5,1)
  9512. longest beginning with r palindrome reviver rotator (7,2)
  9513. longest beginning with s palindrome sawbwas seesees seities sememes (7,4)
  9514. longest beginning with t palindrome terret tibbit tippit (6,3)
  9515. longest beginning with u palindrome uku ulu utu (3,3)
  9516. longest beginning with v palindrome vav (3,1)
  9517. longest beginning with w palindrome waw wow (3,2)
  9518. longest beginning with x palindrome x (1,1)
  9519. longest beginning with y palindrome yaray (5,1)
  9520. longest beginning with z palindrome z (1,1)
  9521. longest with middle a palindrome halalah rotator (7,2)
  9522. longest with middle b palindrome sawbwas (7,1)
  9523. longest with middle c palindrome soccos succus (6,2)
  9524. longest with middle d palindrome murdrum (7,1)
  9525. longest with middle e palindrome sememes (7,1)
  9526. longest with middle f palindrome deified (7,1)
  9527. longest with middle g palindrome degged (6,1)
  9528. longest with middle h palindrome aha ihi oho (3,3)
  9529. longest with middle i palindrome hagigah reviver (7,2)
  9530. longest with middle j palindrome kajak (5,1)
  9531. longest with middle k palindrome kinnikinnik (11,1)
  9532. longest with middle l palindrome hallah selles (6,2)
  9533. longest with middle m palindrome sammas (6,1)
  9534. longest with middle n palindrome adinida (7,1)
  9535. longest with middle o palindrome devoved (7,1)
  9536. longest with middle p palindrome tippit (6,1)
  9537. longest with middle q palindrome q (1,1)
  9538. longest with middle r palindrome nauruan (7,1)
  9539. longest with middle s palindrome seesees (7,1)
  9540. longest with middle t palindrome seities (7,1)
  9541. longest with middle u palindrome alula arura (5,2)
  9542. longest with middle v palindrome civic level rever tevet (5,4)
  9543. longest with middle w palindrome peeweep (7,1)
  9544. longest with middle x palindrome sexes (5,1)
  9545. longest with middle y palindrome malayalam (9,1)
  9546. longest with middle z palindrome kazak qazaq (5,2)
  9547. longest tautonym tangantangan (12,1)
  9548. longest beginning with a tautonym akeake atlatl (6,2)
  9549. longest beginning with b tautonym bellabella (10,1)
  9550. longest beginning with c tautonym caracara chowchow couscous (8,3)
  9551. longest beginning with d tautonym dugdug dumdum (6,2)
  9552. longest beginning with e tautonym ee (2,1)
  9553. longest beginning with f tautonym froufrou (8,1)
  9554. longest beginning with g tautonym ganggang greegree guitguit (8,3)
  9555. longest beginning with h tautonym hotshots (8,0) ?
  9556. longest beginning with i tautonym ipilipil (8,1)
  9557. longest beginning with j tautonym juju (4,1)
  9558. longest beginning with k tautonym kavakava kawakawa khuskhus kohekohe kouskous kukukuku (8,6)
  9559. longest beginning with l tautonym lapulapu lavalava lomilomi (8,3)
  9560. longest beginning with m tautonym mahimahi makomako matamata murumuru (8,4)
  9561. longest beginning with n tautonym nagnag (6,1)
  9562. longest beginning with o tautonym oo (2,1)
  9563. longest beginning with p tautonym palapala pioupiou piripiri poroporo (8,4)
  9564. longest beginning with q tautonym quiaquia (8,1)
  9565. longest beginning with r tautonym riroriro (8,1)
  9566. longest beginning with s tautonym sweeswee (8,1)
  9567. longest beginning with t tautonym tangantangan (12,1)
  9568. longest beginning with u tautonym ulaula (6,1)
  9569. longest beginning with v tautonym valval verver (6,2)
  9570. longest beginning with w tautonym wallawalla (10,1)
  9571. longest beginning with x tautonym ? (0,0) ?
  9572. longest beginning with y tautonym yariyari (8,1)
  9573. longest beginning with z tautonym zoozoo (6,1)
  9574. longest head 'n' tail einsteins muckamuck okeydokey overcover pungapung tarantara trinitrin (9,7)
  9575. longest with middle a head 'n' tail muckamuck pungapung (9,2)
  9576. longest with middle b head 'n' tail aba (3,1)
  9577. longest with middle c head 'n' tail overcover (9,1)
  9578. longest with middle d head 'n' tail okeydokey (9,1)
  9579. longest with middle e head 'n' tail arear caeca (5,2)
  9580. longest with middle f head 'n' tail efe ofo (3,2)
  9581. longest with middle g head 'n' tail aggag algal edged magma (5,4)
  9582. longest with middle h head 'n' tail outshouts (9,0) ?
  9583. longest with middle i head 'n' tail trinitrin (9,1)
  9584. longest with middle j head 'n' tail anjan (5,1)
  9585. longest with middle k head 'n' tail arkar kokko (5,2)
  9586. longest with middle l head 'n' tail ingling khalkha (7,2)
  9587. longest with middle m head 'n' tail bamba bombo mamma pampa (5,4)
  9588. longest with middle n head 'n' tail tarantara (9,1)
  9589. longest with middle o head 'n' tail ingoing mesomes (7,2)
  9590. longest with middle p head 'n' tail apa (3,1)
  9591. longest with middle q head 'n' tail q (1,1)
  9592. longest with middle r head 'n' tail adrad kurku ugrug verve (5,4)
  9593. longest with middle s head 'n' tail hotshot (7,1)
  9594. longest with middle t head 'n' tail einsteins (9,1)
  9595. longest with middle u head 'n' tail mauma shush siusi veuve (5,4)
  9596. longest with middle v head 'n' tail ava eve (3,2)
  9597. longest with middle w head 'n' tail abwab (5,1)
  9598. longest with middle x head 'n' tail manxman (7,1)
  9599. longest with middle y head 'n' tail calycal (7,1)
  9600. longest with middle z head 'n' tail z (1,1)
  9601.  
  9602. Subset of Word
  9603. longest internal palindrome kinnikinniks sensuousness sensuousnesses (11,3)
  9604. longest internal tautonym anhydrohydroxyprogesterone anhydrohydroxyprogesterones kinnikinnick kinnikinnicks kinnikinnics kinnikinniks magnetophotophoresis methylethylpyridine micromicrofarad neuroneuronal trimethylethylene (10,11)
  9605. longest repeated prefix kinnikinnick kinnikinnicks kinnikinnics kinnikinniks micromicrofarad neuroneuronal (10,6)
  9606. most consecutive doubled letters bookkeeper bookkeeping (3,2)
  9607. most doubled letters possessionlessness possessionlessnesses successlessness successlessnesses (4,4)
  9608. longest two cadence humuhumunukunukuapuaa humuhumunukunukuapuaas (8,2)
  9609. longest three cadence effervescence effervescences extendednesses neglectednesses pervertednesses redheadednesses reflectednesses unexpectednesses vallabhacharya vallabhacharyas (5,10)
  9610. longest four cadence alveolopalatal coproporphyrinuria coproporphyrinurias distributivities gastroschisises humuhumunukunukuapuaa humuhumunukunukuapuaas inevitabilities roentgenometries somesthesises stresslessness stresslessnesses (4,12)
  9611. longest five cadence indecipherablenesses recollectivenesses (4,2)
  9612.  
  9613. Letter Counts
  9614.  
  9615. Lipograms
  9616. longest letters from first half hamamelidaceae (14,1)
  9617. longest letters from second half nonsupports (11,0) ?
  9618. longest without ab hydroxydesoxycorticosterone (27,1)
  9619. longest without abcd philoprogenitivenesses (22,1)
  9620. longest without a to h supposititiously (16,1)
  9621. longest without a to k monotonously synonymously tumultuously voluptuously (12,4)
  9622. longest without a to n prototropy zoosporous (10,2)
  9623. longest without a to q susurrus (8,1)
  9624. longest without a to s tutty (5,1)
  9625. longest without e humuhumunukunukuapuaas macracanthorhynchiasis phonocardiographically prorhipidoglossomorpha supradiaphragmatically (22,5)
  9626. longest without et humuhumunukunukuapuaas phonocardiographically prorhipidoglossomorpha (22,3)
  9627. longest without eta coccidioidomycosis (18,1)
  9628. longest without etai phyllospondylous (16,1)
  9629. longest without etain chlorophyllous chromosomology chrysochlorous phyllomorphous polymorphously scolopophorous (14,6)
  9630. longest without etains promorphology (13,1)
  9631.  
  9632. Letter Choices
  9633.  
  9634. Vowels
  9635. longest all vowels aiee ieie (4,2)
  9636. longest each vowel once entwicklungsroman (17,1)
  9637. longest each vowel & y once cylindrocellular phosphuranylites ventriculography (16,3)
  9638. shortest each vowel once eulogia eutocia eutopia isourea sequoia (7,5)
  9639. shortest each vowel & y once oxyuridae (9,1)
  9640. shortest vowels in order caesious (8,1)
  9641. shortest vowels & y in order facetiously (11,1)
  9642. longest vowels in order abstentious (11,1)
  9643. longest vowels & y in order abstemiously (12,1)
  9644. shortest vowels in reverse order muroidea (8,1)
  9645. shortest vowels & y in reverse order ? (80,0) ?
  9646. longest vowels in reverse order subcontinental (14,1)
  9647. longest vowels & y in reverse order ? (0,0) ?
  9648. longest one vowel strengths (9,1)
  9649. longest two vowels schwartzbrots (13,1)
  9650. longest containing a univocalic tathagatagarbhas (16,1)
  9651. longest containing e univocalic strengthlessnesses (18,1)
  9652. longest containing i univocalic instinctivistic (15,1)
  9653. longest containing o univocalic loxolophodonts (14,1)
  9654. longest containing u univocalic struldbrugs (11,1)
  9655. longest containing y univocalic glycyls gypsyfy khlysts khlysty phytyls pyrryls qyrghyz rhythms styryls thymyls tyddyns (7,11)
  9656. longest alternating vowel-consonant hypovitaminosises (17,1)
  9657. longest alternating vowel-consonant excluding y aluminosilicates diketopiperazine epicoracohumeral (16,3)
  9658.  
  9659. Consonants
  9660. longest consonant string bergschrund bergschrunds catchphrase eschscholtzia eschscholtzias festschrift festschriften festschrifts goldschmidtine goldschmidtines goldschmidtite goldschmidtites lachsschinken lachsschinkens latchstring mischsprache mischsprachen nachschlag nachschlage nachschlags promptscript veldtschoen weltschmerz weltschmerzes (6,24)
  9661. longest one consonant assessees coccaceae (9,2)
  9662. longest two consonant nauseousnesses sensuousnesses (14,2)
  9663.  
  9664. Isograms
  9665. longest isogram dermatoglyphics (15,1)
  9666. longest pair isogram scintillescent (14,1)
  9667. longest trio isogram deeded (6,1)
  9668. longest tetrad isogram kukukuku (8,1)
  9669. longest polygram unprosperousnesses (18,0) ?
  9670. longest pyramid chachalaca deadheaded disseisees evennesses keennesses kinnikinic rememberer sassanians sereneness sleeveless susurruses (10,11)
  9671. most repeated letters dihydroxycholecalciferol hydroxydesoxycorticosterone hysterosalpingographies methyldihydromorphinone microspectrophotometrically octamethylpyrophosphoramide phosphatidylethanolamine pseudohermaphroditism tetrabromophenolphthalein tetraiodophenolphthalein trinitrophenylmethylnitramine (9,11)
  9672. highest containing a repeated palaeacanthocephala tathagatagarbha tathagatagarbhas (6,3)
  9673. highest containing b repeated bubbybush flibbertigibbet flibbertigibbets flibbertigibbety (4,4)
  9674. highest containing c repeated chroococcaceae chroococcaceous circumcrescence circumcrescences echinococcic micrococcaceae (5,6)
  9675. highest containing d repeated condiddled dadded deadheaded dendrodendritic diddered diddled diddledees didodecahedron disbudded dodded doddered doddled driddled dunderheaded dunderheadedness dunderheadednesses dyakisdodecahedral dyakisdodecahedron dyakisdodecahedrons fiddledeedee fiddleheaded granddaddy lepidodendrid lepidodendrids lepidodendroid muddleheaded muddleheadedness muddleheadednesses muddyheaded puddingheaded skedaddled woodshedded (4,32)
  9676. highest containing e repeated ethylenediaminetetraacetate (7,1)
  9677. highest containing f repeated chiffchaff chiffchaffs giffgaff giffgaffed giffgaffing giffgaffs riffraff (4,7)
  9678. highest containing g repeated aggregating aggreging chugalugging gagging gaggling ganggang ganggangs gigging giggling gigglingly glugging goggling grigging grogging guggling lallygagging lollygagging zigzagging (4,18)
  9679. highest containing h repeated ichthyophthiriasis ichthyophthirius ichthyophthiriuses rhamphorhynchid rhamphorhynchids rhamphorhynchoid rhamphorhynchus (4,7)
  9680. highest containing i repeated dirigibilities discriminabilities distinguishabilities divisibilities ignitibilities indiscernibilities indiscerptibilities indistinguishability indivisibility infinitesimalities intelligibilities invincibilities (6,12)
  9681. highest containing j repeated ajonjoli ajonjolis avijja avijjas djokjakarta gastrojejunal gastrojejunostomy hajj hajjes hajji hajjis jajman jajmani jajmans jajoba jejuna jejunal jejune jejunely jejuneness jejunenesses jejunities jejunity jejunostomies jejunostomy jejunum jeremejevite jeremejevites jimberjawed jimjams jinglejangle jinglejangles jinjili jinjilis jipijapa jipijapas jirajara jirajaras jiujitsu jiujitsus jiujutsu jiujutsus jogjakarta jojoba jujitsu jujitsus juju jujube jujubes jujus jujut
  9682.  
  9683.  
  9684. highest containing k repeated kakkak kakkaks knickknack knickknackatories knickknackatory knickknackeries knickknackery knickknacky kukukuku kukukukus (4,10)
  9685. highest containing l repeated allochlorophyll allochlorophylls alloplastically intellectualistically lillypillies lillypilly polysyllabically (5,7)
  9686. highest containing m repeated dynamometamorphism hamamelidanthemum immunocompromised mammatocumulus mammectomies mammectomy mammiform mammilliform mammogram mammonism mammonisms mesembryanthemum mesembryanthemums meshummadim mohammedanism mohammedanisms muhammadanism muhammadanisms mummiform tetramethylammonium thermometamorphism zamzummim zamzummims (4,23)
  9687. highest containing n repeated inconvenientness inconvenientnesses nannoplankton nannoplanktonic nondenominational nondenominationalism nonentanglement nonintervention noninterventionist syngenesiotransplantation unconvincingness unconvincingnesses (5,12)
  9688. highest containing o repeated monogonoporous pseudomonocotyledonous (6,2)
  9689. highest containing p repeated aplopappus haplopappus hyperleptoprosopic hyperleptoprosopy snippersnapper whippersnapper (4,6)
  9690. highest containing q repeated qaraqalpaq qaraqalpaqs (3,2)
  9691. highest containing r repeated ferriprotoporphyrin ferroprotoporphyrin (5,2)
  9692. highest containing s repeated possessionlessnesses (9,1)
  9693. highest containing t repeated ethylenediaminetetraacetate tetrasubstituted throttlebottom totipotentiality yttrotantalite (5,5)
  9694. highest containing u repeated humuhumunukunukuapuaa humuhumunukunukuapuaas (9,2)
  9695. highest containing v repeated overconservative ovoviviparity ovoviviparous ovoviviparously ovoviviparousness vulvovaginitis (3,6)
  9696. highest containing w repeated bowwow bowwows powwow powwowed powwowing powwows swallowwort whillywhaw whillywhaws whitlowwort williwaw williwaws willowware willowweed willowworm willywaw willywaws (3,17)
  9697. highest containing x repeated dextropropoxyphene executrix executrixes exlex exlexes exonarthex exotoxic exotoxin hexachlorocyclohexane hexahydroxy hexaxon hexoxide hydroxydeoxycorticosterone hydroxydesoxycorticosterone maxixe maxixes myxoxanthin oxyhexactine oxyhexaster paxwax paxwaxes paxywaxies paxywaxy saxifrax saxifraxes saxitoxin sextuplex xanthotoxin xanthoxenite xanthoxenites xanthoxylaceae xanthoxyletin xanthoxyletins xanthoxylin xanthoxylins xanthoxylum xanthoxylums (2,37)
  9698. highest containing y repeated acetylphenylhydrazine acetylphenylhydrazines anhydrohydroxyprogesterone anhydrohydroxyprogesterones brachydactyly chylophylly cryptozygy cystopyelography cytophysiologically cytophysiology dacryocystorhinostomy dactylosymphysis dihydroxyphenylalanine dyssynergy glycolytically gypsyfy gypsyfying hydrodynamically hydronymy hydroxydeoxycorticosterone hydroxydesoxycorticosterone hydroxyethyl hydroxyethylation hydroxyethylations hydroxylysine hydroxymethyl hydroxymethylation
  9699.  
  9700.  
  9701. highest containing z repeated pizzazz pizzazzes razzmatazz razzmatazzes (4,4)
  9702. most different letters blepharoconjunctivitis pseudolamellibranchiata pseudolamellibranchiate psychogalvanometric (16,4)
  9703. highest ratio length/letters kukukuku (400,1)
  9704. highest ratio length/letters (no tautonyms) senselessnesses (375,1)
  9705. lowest length 16 ratio length/letters ventriculography (106,1)
  9706. lowest length 17 ratio length/letters entwicklungsroman hydrobasaluminite pterygomandibular (113,3)
  9707. lowest length 18 ratio length/letters carboxyhemoglobins entwicklungsromane hyperglobulinemias psychogalvanometer ventriculographies (120,5)
  9708. lowest length 19 ratio length/letters psychogalvanometric (118,1)
  9709. lowest length 20 ratio length/letters brachycephalizations dimethyltubocurarine encephalomyocarditis magnetofluiddynamics moschellandsbergites (133,5)
  9710. lowest length 21 ratio length/letters diphenylthiocarbazone pseudolamellibranchia sphygmomanometrically (140,3)
  9711. lowest length 22 ratio length/letters blepharoconjunctivitis (137,1)
  9712. lowest length 23 ratio length/letters pseudolamellibranchiata pseudolamellibranchiate (143,2)
  9713. lowest length 24 ratio length/letters diphenylaminechlorarsine laryngotracheobronchitis meningoencephalomyelitis (171,3)
  9714. lowest length 25 ratio length/letters spectroheliokinematograph (166,1)
  9715.  
  9716. Letter Appearance
  9717. longest narrow letters (ACEMNORSUVWXZ) erroneousnesses verrucosenesses (15,2)
  9718. longest tall letters (BDFGHIJKLPQTY) lighttight lillypilly (10,2)
  9719. longest vertical-symmetry letters (AHIMOTUVWXY) homotaxia thymomata (9,2)
  9720. longest horizontal-symmetry letters (BCDEHIKOX) checkbook checkhook chookchie (9,3)
  9721. highest ratio of dotted letters (IJ) jinjili (71,1)
  9722.  
  9723. Typewriter
  9724. longest top row proprietory proterotype rupturewort (11,3)
  9725. longest middle row shakalshas (10,1)
  9726. longest in order wettish (7,1)
  9727. longest in reverse order bourree chapote chappie chappow gouttee (7,5)
  9728. longest left hand tesseradecades (14,0) ?
  9729. longest right hand hypolimnion kinnikinnik (11,2)
  9730. longest alternating hands leucocytozoans (14,1)
  9731. longest one finger deeded humhum hummum muhuhu muumuu (6,5)
  9732. longest adjacent keys assessees redresser redresses seeresses sweeswees (9,5)
  9733.  
  9734. Puzzle
  9735. longest formed with chemical symbols nonrepresentationalism (22,1)
  9736. longest formed with US postal codes convallarias (12,1)
  9737. longest formed with compass points newnesses sweeswees (9,2)
  9738. longest formed with piano notes cabbaged fabaceae fagaceae (8,3)
  9739.  
  9740. Letter Order
  9741.  
  9742. Alphabetical
  9743. longest letters in order aegilops (8,1)
  9744. longest letters in order with repeats aegilops (8,1)
  9745. longest letters in reverse order sponged wronged (7,2)
  9746. longest letters in reverse order with repeats trollied (8,1)
  9747. longest roller-coaster decriminalizations provincializations (18,2)
  9748. longest no letters in place trinitrophenylmethylnitramine (29,1)
  9749. most letters in place abudefduf agammaglobulinemias archencephalon archetypical archetypically syngenesiotransplantation (5,6)
  9750. most letters in place shifted cooperatively daughterlinesses definitivenesses gymnoplast gymnoplasts inoperative inoperativeness inopportunely intraoperatively neighborlinesses operatively postoperatively preoperatively undefendablenesses unoperative unspiritually (6,16)
  9751. most consecutive letters in order consecutively bierstube bierstuben bierstubes gymnopaedia gymnopaedias gymnopaedic gymnopedia gymnopedias gymnophiona gymnoplast gymnoplasts klavierstuck limnopithecus limnoplankton limnoplanktonic overstudy overstuff overstuffed semnopithecus semnopitheque semnopitheques thamnophile thamnophiles thamnophiline thamnophilus thamnophis understudy (4,27)
  9752. most consecutive letters in order aborticide aborticides abscinded absconded abscondence abscondences alimentotherapy aluminographies aluminography aluminotype aluminotypes ambuscade ambuscaded ambuscades helminthosporia helminthosporin helminthosporins helminthosporium helminthosporiums helminthosporoid laminograph laminographic laminographies laminography laminosioptes limnograph limnopithecus limnoplankton limnoplanktonic luminophor luminophors luminoscope opaquers reconstructive reconstructively
  9753.  
  9754.  
  9755. most consecutive letters appropinquates appropinquations appropinquities equiponderates equiponderations perquisition perquisitions preconquest propinquities quadruplications sesquiterpenoid sesquiterpenoids (8,12)
  9756. highest ratio of consecutive letters to length klompen (85,1)
  9757.  
  9758. ==> english/repeat.p <==
  9759. What is a sentence containing the most repeated words, without:
  9760.         using quotation marks,
  9761.         using proper names,
  9762.         using a language other than English,
  9763.         anything else distasteful.
  9764.  
  9765. ==> english/repeat.s <==
  9766. Five "had"s in a row:
  9767.  
  9768. The parents were unable to conceive, so they hired someone else to
  9769. be a surrogate.
  9770.  
  9771. The parents had had a surrogate have their child.
  9772.  
  9773. The parents had had had their child.
  9774.  
  9775. The child had had no breakfast.
  9776.  
  9777. The child whose parents had had had had had no breakfast.
  9778.  
  9779. ==> english/repeated.words.p <==
  9780. What is a sentence with the same word several times repeated?
  9781.  
  9782. ==> english/repeated.words.s <==
  9783. It is true for all that, that that that that that that signifies, is not
  9784. the one to which I refer.
  9785.  
  9786. Here are some steps to understanding the entire sentence:
  9787.     That is not the one to which I refer.
  9788.     That (that that that signifies) is not the one to which I refer.
  9789.     That that that that that that signifies, is not the one to which I refer.
  9790.  
  9791. In Annamite:
  9792.     Ba ba ba ba.
  9793.         (Three ladies gave a box on the ear to the favorite of the Prince.)
  9794.  
  9795. ==> english/rhyme.p <==
  9796. What English words are hard to rhyme?
  9797.  
  9798. "Rhyme is the identity in sound of an accented vowel in a word...and
  9799. of all consonantal and vowel sounds following it; with a difference in
  9800. the sound of the consonant immediately preceding the accented vowel."
  9801. (From The Complete Rhyming Dictionary by Clement Wood).  Appropriately
  9802. Wood says a couple of pages later, "If a poet commences, 'October is
  9803. the wildest month' he has estopped himself from any rhyme; since
  9804. "month" has no rhyme in English."
  9805.  
  9806. ==> english/rhyme.s <==
  9807. NI3 = Merriam-Webster's Third New International Dictionary
  9808. NI2 = Merriam-Webster's New International Dictionary, Second Edition
  9809. RHD = Random House Unabridged Dictionary
  9810. + means slang, foreign, obsolete, dialectical, etc.
  9811.  
  9812. Word            Rhyme                                   Assonance
  9813. --------------- --------------------------------------- --------------------
  9814. aitch           brache (NI2+), taich (NI2+)             naish
  9815. angry           unangry (NI2+)                          aggry
  9816. angst                                                   lanx
  9817. beards          weirds
  9818. breadth                                                 death
  9819. bulb                                                    pulp
  9820. carpet          charpit
  9821. chimney         timne, polymny (NI2+)
  9822. cusp            wusp (NI2)                              bust
  9823. depth                                                   stepped
  9824. eighth                                                  faith
  9825. else                                                    belts
  9826. exit            direxit (RHD+)                          sexist
  9827. fiends          teinds, piends
  9828. filched         hilched (NI3+), milched (NI2)           zilch
  9829. filth           spilth, tilth
  9830. fifth                                                   drift
  9831. film            pilm (NI3+)                             kiln
  9832. fluxed          luxed (NI3+), muxed (NI3+)              ducked
  9833. glimpsed                                                rinsed
  9834. gospel                                                  hostile
  9835. gulf                                                    pulse
  9836. jinxed          outminxed (?)                           blinked
  9837. leashed         niched, tweesht (NI2+)
  9838. liquid                                                  wicked
  9839. mollusk                                                 smallest
  9840. mouthed         southed
  9841. month                                                   grumph
  9842. mulcts                                                  bulks
  9843. mulched         gulched (NI3+)                          bulged
  9844. ninth                                                   pint
  9845. oblige                                                  bides
  9846. oomph           sumph (NI3+)
  9847. orange          sporange
  9848. pint            jint (NI2+)                             bind
  9849. poem            phloem, proem
  9850. pregnant        regnant
  9851. purple          curple (NI3+), hirple (NI3+)
  9852. puss            schuss
  9853. rhythm          smitham
  9854. scalds          balds, caulds (NI3+), faulds (NI3+)
  9855. scarce          clairce (NI2), hairse (NI2+)            cares
  9856. sculpts                                                 gulps
  9857. silver          chilver (NI3+)
  9858. sixth                                                   kicks
  9859. spirit          squiret (NI2+)
  9860. tenth           nth                                     bent
  9861. tsetse          baronetcy, intermezzi, theetsee
  9862. tuft            yuft
  9863. twelfth                                                 health
  9864. widow           kiddo
  9865. width                                                   bridge
  9866. window          indo, lindo
  9867. wolf                                                    bulls
  9868.  
  9869. ==> english/self.ref.letters.p <==
  9870. Construct a true sentence of the form: "This sentence contains _ a's, _ b's,
  9871. _ c's, ...," where the numbers filling in the blanks are spelled out.
  9872.  
  9873. ==> english/self.ref.letters.s <==
  9874. A little history of the problem, culled from the pages of _Metamagical
  9875. Themas_, Hofstadter's collection of his _Scientific American_ columns.
  9876. First mention of it is in the Jan. '82 column, a followup to one on self-
  9877. referential sentences.  Lee Sallows opened the field with a sentence that
  9878. began "Only the fool would take trouble to verify that his sentence was
  9879. composed of ten a's ...." etc.
  9880.  
  9881. Then in the addendum to the Jan.'83 column on viral sentences, Hofstadter
  9882. quotes Sallows describing his Pangram Machine, "a clock-driven cascade of
  9883. sixteen Johnson-counters," to tackle the problem. An early success was:
  9884.         "This pangram tallies five a's, one b, one c, two d's, twenty-
  9885.         eight e's, eight f's, six g's, eight h's, thirteen i's, one j,
  9886.         one k, three l's, two m's, eighteen n's, fifteen o's, two p's,
  9887.         one q, seven r's, twenty-five s's, twenty-two t's, four u's, four
  9888.         v's, nine w's, two x's, four y's, and one z."
  9889.  
  9890. Sallows wagered ten guilders that no-one could create a perfect self-
  9891. documenting sentence beginning, "This computer-generated pangram contains
  9892. ...." within ten years.
  9893.  
  9894. It was solved very quickly, after Sallows' challenge appeared in Dewdny's
  9895. Oct. '84 SA column.  Larry Tesler solved it by a method Hofstadter calls
  9896. "Robinsonizing," which involves starting with an arbitrary set of values
  9897. for each letter, getting the true values when the sentence is made, and
  9898. plugging the new values back in, making a feedback loop. Eventually, you
  9899. can zero in on a set of values that work.  Tesler's sentence:
  9900.         This computer-generated pangram contains six a's, one b, three
  9901.         c's, three d's, thirty-seven e's, six f's, three g's, nine h's,
  9902.         twelve i's, one j, one k, two l's, three m's, twenty-two n's,
  9903.         thirteen o's, three p's, one q, fourteen r's, twenty-nine s's,
  9904.         twenty-four t's, five u's, six v's, seven w's, four x's, five
  9905.         y's, and one z.
  9906.  
  9907. The method of solution (called "Robinsonizing," after the logician Raphael
  9908. Robinson) is as follows:
  9909. 1)      Fix the count of a's.
  9910. 2)      Fix the count of b's.
  9911. 3)      Fix the count of c's.
  9912. ...
  9913. 26)   Fix the count of z's.
  9914. Then, if the sentence is still wrong, go back to step 1.
  9915.  
  9916. Most attempts will fall into long loops (what Hofstadter calls attractive
  9917. orbits), but with a good computer program, it's not too hard to find a
  9918. Robinsonizing sequence that zeros in on a fixed set of values.
  9919.  
  9920. The February and May 1992 _Word Ways_ have articles on this subject,
  9921. titled "In Quest of a Pangram, (Part 1)" by Lee Sallows.  It tells of his
  9922. search for a self-referential pangram of the form, "This pangram
  9923. contains _ a's, ..., and one z."  (He built special hardware to search
  9924. for them.)  Two such pangrams given in the article are:
  9925.  
  9926.     This pangram lists four a's, one b, one c, two d's,
  9927.     twenty-nine e's, eight f's, three g's, five h's, eleven i's,
  9928.     one j, one k, three l's, two m's twenty-two n's, fifteen o's,
  9929.     two p's, one q, seven r's, twenty-six s's, nineteen t's, four
  9930.     u's, five v's, nine w's, two x's, four y's, and one z.
  9931.  
  9932.     This pangram contains four a's, one b, two c's, one d, thirty
  9933.     e's, six f's, five g's, seven h's, eleven i's, one j, one k,
  9934.     two l's, two m's eighteen n's, fifteen o's, two p's, one q,
  9935.     five r's, twenty-seven s's, eighteen t's, two u's, seven v's,
  9936.     eight w's, two x's, three y's, & one z.
  9937.  
  9938. It also contains one in Dutch by Rudy Kousbroek:
  9939.  
  9940.     Dit pangram bevat vijf a's, twee b's, twee c's, drie d's,
  9941.     zesenveertig e's, vijf f's, vier g's, twee h's, vijftien i's,
  9942.     vier j's, een k, twee l's, twee m's, zeventien n's, een o,
  9943.     twee p's, een q, zeven r's, vierentwintig s's, zestien t's,
  9944.     een u, elf v's, acht w's, een x, een y, and zes z's.
  9945.  
  9946. References:
  9947. Dewdney, A.K. Scientific American, Oct. 1984, pp 18-22.
  9948. Sallows, L.C.F. Abacus, Vol.2, No.3, Spring 1985, pp 22-40.
  9949. Sallows, L.C.F. Word Ways, Feb. & May 1992
  9950. Hofstadter, D. Scientific American, Jan. 1982, pp 12-17.
  9951.  
  9952. ==> english/self.ref.numbers.p <==
  9953. What true sentence has the form: "There are _ 0's, _ 1's, _ 2's, ...,
  9954. in this sentence"?
  9955.  
  9956. ==> english/self.ref.numbers.s <==
  9957. There are 1 0's, 7 1's, 3 2's, 2 3's, 1 4's, 1 5's, 1 6's, 2 7's, 1 8's,
  9958. and 1 9's in this sentence.
  9959.  
  9960. There are 1 0's, 11 1's, 2 2's, 1 3's, 1 4's, 1 5's, 1 6's, 1 7's, 1 8's
  9961. and 1 9's in this sentence.
  9962.  
  9963. ==> english/self.ref.words.p <==
  9964. What sentence describes its own word, syllable and letter count?
  9965.  
  9966. ==> english/self.ref.words.s <==
  9967. This sentence contains ten words, eighteen syllables, and sixty-four letters.
  9968.  
  9969. ==> english/sentence.p <==
  9970. Find a sentence with words beginning with the letters of the alphabet, in order.
  9971.  
  9972. ==> english/sentence.s <==
  9973. After boxes containing dynamite exploded furiously
  9974. generating hellish inferno jet killing laboring miners,
  9975. novice operator, paralyzed, quickly refuses surgical treatment
  9976. until veteran workers x-ray youth zealously.
  9977.  
  9978. A big cuddly dog emitted fierce growls happily ignoring joyful kids licking
  9979. minute nuts on pretty queer rotten smelly toadstalls underneath vampires
  9980. who x-rayed young zombies.
  9981.  
  9982. ==> english/snowball.p <==
  9983. Construct the longest coherent sentence you can such that the nth
  9984. word is n letters long.
  9985.  
  9986. ==> english/snowball.s <==
  9987. I
  9988. do
  9989. not
  9990. know
  9991. where
  9992. family
  9993. doctors
  9994. acquired
  9995. illegibly
  9996. perplexing
  9997. handwriting;
  9998. nevertheless,
  9999. extraordinary
  10000. pharmaceutical
  10001. intellectuality,
  10002. counterbalancing
  10003. indecipherability,
  10004. transcendentalizes
  10005. intercommunications'
  10006. incomprehensibleness.
  10007.  
  10008. ==> english/spoonerisms.p <==
  10009. List some exceptional spoonerisms.
  10010.  
  10011. ==> english/spoonerisms.s <==
  10012. Original by Spooner himself:
  10013.  
  10014. I am afraid you have tasted the whole worm, and must
  10015. therefore take the next town drain.
  10016.  
  10017. Some years ago in the Parliament, a certain member known for his quick and
  10018. rapier wit, cut across a certain other member who was trying to make some
  10019. bad joke.  He called him a "Shining Wit" then apologized for making a
  10020. Spoonerism.
  10021.  
  10022. Another famous broadcast fluff was on the Canadian Broadcasting
  10023. Corporation, which an announcer identified as the "Canadian
  10024. Broadcorping Castration."
  10025.  
  10026. Oh yes, another radio announcer one that has sort of crept into
  10027. common English usage is "one swell foop".
  10028.  
  10029. A friend of mine had just eaten dinner in the school
  10030. cafeteria, and he didn't look very happy. Another of
  10031. my friends said, "John, what's wrong?" Knowing exactly
  10032. what he was saying, he said, "It's the bound grief I
  10033. had for dinner]"
  10034.  
  10035. A radio announcer, talking about a royal visit (or some such) said the
  10036. visitor would be greeted with a "twenty one sun galoot".
  10037.  
  10038. There are several fractured fables based on spoonerisms, such as:
  10039.  
  10040. A king on a desert island was so beloved by his people, they decided to
  10041. give him a very special gift for the anniversary of his coronation.  So
  10042. after much thought, they decided to make him a throne out of seashells,
  10043. which were plentiful on the island.  And when it was finished, they
  10044. presented it to the king, who loved it.  But he soon discovered it was
  10045. very uncomfortable to sit on.  So he told his subjects it was too
  10046. special to use everyday (so as not to hurt their feelings) and put it in
  10047. the attic of his palace (which was, of course, a hut like all the other
  10048. dwellings on the island), planning to use it just for special occasions.
  10049. But that night, it fell through the ceiling of his bedroom and landed
  10050. on top of him, killing him instantly.  And the moral of the story is:
  10051. Those who live in grass houses shouldn't stow thrones]
  10052.  
  10053. ==> english/states.p <==
  10054. What long words have all bigrams either a postal state code or its reverse?
  10055.  
  10056. ==> english/states.s <==
  10057. 10 paramarine
  10058. 10 indentment
  10059. 10 cacocnemia
  10060. 9 amendment
  10061. 9 paramimia
  10062. 9 paramenia
  10063. 9 paralinin
  10064. 9 paralalia
  10065. 9 palilalia
  10066. 9 palapalai
  10067. 8 scalawag
  10068. 8 memorial
  10069.  
  10070. Disallowing reversals of state codes the longest common ones are:
  10071.  
  10072. 8 malarial
  10073. 7 malaria
  10074. 6 scalar
  10075. 6 marine
  10076. 5 flaky
  10077.  
  10078. Terry Donahue
  10079.  
  10080. ==> english/telegrams.p <==
  10081. Since telegrams cost by the word, phonetically similar messages can be cheaper.
  10082. See if you can decipher these extreme cases:
  10083.  
  10084. UTICA CHANSON MIGRATE INVENTION ANNUAL KNOBBY SORRY IN FACTUAL BEEN CLOVER.
  10085.  
  10086. WEED LICHEN ICE CHEST FOREARM OTHER DISGUISE DELIMIT.
  10087.  
  10088. CANCEL MYOCARDIA ITS INFORMAL FUNCTION.
  10089.  
  10090. YEARN AFFIX, LOST UKASE, UGANDA JAIL, CONSERVE TENURES YACHT APPEAL.
  10091.  
  10092. EYELET SHEILA INDIA HOUSE SHEILAS TURKEY.
  10093.  
  10094. BOB STILT SEA, CANTANKEROUS BOAT, HUMUS GOAD IMMORTAL DECOS GUARD.
  10095.  
  10096. MARY SINBAD SHEER TOURNEY AUGUSTA WIND NOCTURNE TOOTHBRUSH.
  10097.  
  10098. WHINE YOSEMITE NAMES SOY CAN PHILATELIST.
  10099.  
  10100. ALBEIT DETRACT UNIVERSE EDIFY MUSTAFA TICKET TICKET IN.
  10101.  
  10102. ==> english/telegrams.s <==
  10103. These are from an old "Games" magazine:
  10104.  
  10105. UTICA CHANSON MIGRATE INVENTION ANNUAL KNOBBY SORRY IN FACTUAL BEEN CLOVER.
  10106.         You take a chance on my great invention and you'll not be sorry.
  10107.          In fact, you'll be in clover.
  10108.  
  10109. WEED LICHEN ICE CHEST FOREARM OTHER DISGUISE DELIMIT.
  10110.         We'd like a nice chest for our mother; the sky's the limit.
  10111.  
  10112. CANCEL MYOCARDIA ITS INFORMAL FUNCTION.
  10113.         Can't sell my ol' car dear; it's in for malfunction.
  10114.  
  10115. YEARN AFFIX, LOST UKASE, UGANDA JAIL, CONSERVE TENURES YACHT APPEAL.
  10116.         You're in a  fix. Lost your case. You goin' to jail.
  10117.         Can serve ten years. You ought to appeal.
  10118.  
  10119. EYELET SHEILA INDIA HOUSE SHEILAS TURKEY.
  10120.         I let Sheila in their house; she lost her key.
  10121.  
  10122. BOB STILT SEA, CANTANKEROUS BOAT, HUMUS GOAD IMMORTAL DECOS GUARD.
  10123.         Bob's still at sea; can't anchor his boat. You must go to him
  10124.         or tell the coast guard.
  10125.  
  10126. MARY SINBAD SHEER TOURNEY AUGUSTA WIND NOCTURNE TOOTHBRUSH.
  10127.         Mary's in bed; she hurt her knee.  A gust of wind
  10128.         knocked her into the brush.
  10129.  
  10130. WHINE YOSEMITE NAMES SOY CAN PHILATELIST.
  10131.         Why don't you (why'n'ya) send me the names, so I can
  10132.         fill out a list.
  10133.  
  10134. ALBEIT DETRACT UNIVERSE EDIFY MUSTAFA TICKET TICKET IN.
  10135.         I'll be at the track and I have a receipt if I must have a ticket to
  10136.          get in.
  10137.  
  10138. ==> english/trivial.p <==
  10139. Consider the free non-abelian group on the twenty-six letters of the
  10140. alphabet with all relations of the form <word1> = <word2>, where <word1>
  10141. and <word2> are homophones (i.e. they sound alike but are spelled
  10142. differently).  Show that every letter is trivial.
  10143.  
  10144. For example, be = bee, so e is trivial.
  10145.  
  10146. ==> english/trivial.s <==
  10147. be = bee ==> e is trivial;
  10148. ail = ale ==> i is trivial;
  10149. week = weak ==> a is trivial;
  10150. lie = lye ==> y is trivial;
  10151. to = too ==> o is trivial;
  10152. two = to ==> w is trivial;
  10153. hour = our ==> h is trivial;
  10154. faggot = fagot ==> g is trivial;
  10155. bowl = boll ==> l is trivial;
  10156. gell = jel ==> j is trivial;
  10157. you = ewe ==> u is trivial;
  10158. damn = dam ==> n is trivial;
  10159. limb = limn ==> b is trivial;
  10160. bass = base ==> s is trivial;
  10161. cede = seed ==> c is trivial;
  10162. knead = need ==> k is trivial;
  10163. add = ad ==> d is trivial;
  10164. awful = offal ==> f is trivial;
  10165. gram = gramme ==> m is trivial;
  10166. grip = grippe ==> p is trivial;
  10167. cue = queue ==> q is trivial;
  10168. carrel = carol ==> r is trivial;
  10169. butt = but ==> t is trivial;
  10170. lox = locks ==> x is trivial;
  10171. tsar = czar ==> z is trivial;
  10172. vlei = flay ==> v is trivial.
  10173.  
  10174. For a related problem, see _The Jimmy's Book_ (_The American Mathematical
  10175. Monthly_, Vol. 93, Num. 8 (Oct. 1986), p.  637):
  10176.  
  10177. Consider the free group on twenty-six letters A, ..., Z.  Mod out by
  10178. the relation that defines two words to be equivalent if (a) one is a
  10179. permutation of the other and (b) each appears as a legitimate English
  10180. word in the dictionary.  Identify the center of this group.
  10181.  
  10182.     -- clong@remus.rutgers.edu (Chris Long)
  10183.  
  10184. ==> english/weird.p <==
  10185. Make a sentence containing only words that violate the "i before e" rule.
  10186.  
  10187. ==> english/weird.s <==
  10188. From the May, 1990 _Word Ways_:
  10189.  
  10190. That is IE - Or, Is That EI?
  10191.  
  10192. by Paul Leopold
  10193. Stockholm, Sweden
  10194.  
  10195.     "Seeing wherein neither weirdly-veiled sovereign deigned
  10196.     agreeing, their feisty heirs, leisurely eyeing eight heinous
  10197.     deity-freightened reindeer sleighs, counterfeited spontaneity,
  10198.     freeing rein (reveille, neighing]); forfeited obeisance,
  10199.     fleeing neighborhood.  Kaleidoscopically-veined foreign
  10200.     heights being seized, either reigned, sleight surfeited,
  10201.     therein; reinvented skein-dyeing; reiteratedly inveighed,
  10202.     feigning weighty seismological reinforcement."
  10203.  
  10204. The above passage appears in a book on the ecological conservation
  10205. measures of the enlightened plutocracies of antiquity, Ancient
  10206. Financier Aristocracies' Conscientious Scientific Species Policies,
  10207. by Creighton Leigh Peirce and Keith Leiceister Reid. . . .
  10208.  
  10209. Any beings decreeing such ogreish, albeit nonpareil,
  10210. homogeneity must be nucleic protein-deficient from sauteing
  10211. pharmacopoeial caffeine and codeine]
  10212.  
  10213. From an 'fgrep cie /usr/dict/words', with similiar words removed.
  10214. ancient coefficient concierge conscience conscientious deficient efficient
  10215. financier glacier hacienda Muncie omniscient proficient science
  10216. Societe(?) society species sufficient
  10217.  
  10218. A search through Webster's on-line dictionary produced the following exceptions:
  10219.  
  10220. Word: *cie*
  10221. Possible matches are:
  10222.   1. -facient               2. abortifacient          3. ancien regime
  10223.   4. ancient                5. ancientry              6. boccie
  10224.   7. cenospecies            8. christian science      9. coefficient
  10225.  10. concierge             11. conscience            12. conscience money
  10226.  13. conscientious         14. conscientious objector15. deficiency
  10227.  16. deficiency disease    17. deficient             18. domestic science
  10228.  19. earth science         20. ecospecies            21. efficiency
  10229.  22. efficiency engineer   23. efficient             24. facies
  10230.  25. fancier               26. financier             27. genospecies
  10231.  28. geoscience            29. glacier               30. glacier theory
  10232.  31. habeas corpus ad subjiciendum32. hacienda       33. inconscient
  10233.  34. inefficiency          35. inefficient           36. insufficience
  10234.  37. insufficiency         38. insufficient          39. international scientific vocabulary
  10235.  40. library science       41. liquefacient          42. mental deficiency
  10236.  43. mutafacient           44. natural science       45. nescience
  10237.  46. omniscience           47. omniscient            48. physical science
  10238.  49. political science     50. precieux              51. prescience
  10239.  52. prescientific         53. prima facie           54. proficiency
  10240.  55. proficient            56. pseudoscience         57. rubefacient
  10241.  58. science               59. science fiction       60. scient
  10242.  61. sciential             62. scientific            63. scientific method
  10243.  64. scientism             65. scientist             66. scientistic
  10244.  67. secret society        68. self-sufficiency      69. self-sufficient
  10245.  70. social science        71. social scientist      72. societal
  10246.  73. society               74. society verse         75. somnifacient
  10247.  76. specie                77. species               78. stupefacient
  10248.  79. sub specie aeternitatis80. subspecies            81. sufficiency
  10249.  82. sufficient            83. sufficient condition  84. superficies
  10250.  85. type species          86. unscientific          87. valenciennes
  10251.  88. vers de societe
  10252.  
  10253. ==> english/word.boundaries.p <==
  10254. List some sentences that can be radically altered by changing word boundaries
  10255. and punctuation.
  10256.  
  10257. ==> english/word.boundaries.s <==
  10258. Issues topping our mail: manslaughter.
  10259. Is Sue stopping our mailman's laughter?
  10260.  
  10261. The real ways I saw it.
  10262. There always is a wit.
  10263.  
  10264. You read evil tomes, Tim, at Ed's issue.
  10265. "You're a devil, Tom]" estimated sis Sue.
  10266.  
  10267. ==> english/word.torture.p <==
  10268. What is the longest word all of whose contiguous subsequences are words?
  10269.  
  10270. ==> english/word.torture.s <==
  10271. This problem was discussed in _Word Ways_ in 1974-5.  In August 1974,
  10272. Ralph Beaman, in an article titled "Word Torture", offered the word
  10273. SHADES, from which one obtains HADES, SHADE; ADES, HADE, SHAD; DES, ADE,
  10274. HAD, SHA; ES, DE, AD, HA, SH; S, E, D, A, H.  All of these are words
  10275. given in Webster's Third.
  10276.  
  10277. Since that time, a serious search has been launched for a seven-letter
  10278. word.  The near misses so far are:
  10279. Date    Person           Word     Missing
  10280. Aug 74  Ralph Beaman     GAMINES  INES, GAMI, NES, INE
  10281. Nov 74  Dmitri Borgmann  ABASHED  INE, NES, ABASHE, BASHE, ASHE (all in OED)
  10282. May 75  David Robinson   GUNITES  GU, GUNIT (using Webster's Second)
  10283. May 75  David Robinson   ETAMINE  ETAMI, TAMI (using Webster's Second)
  10284. May 75  Ralph Beaman     MORALES  RAL (using Webster's Second)
  10285. Aug 75  Tom Pulliam      SHEAVES  EAV (using Webster's Second)
  10286.  
  10287. Webster's Second has been used for most of the attempts since it
  10288. contains so many more words than Webster's Third.  The seven-letter
  10289. plateau remains to be achieved.
  10290.  
  10291. ==> games/chess/knight.control.p <==
  10292. How many knights does it take to attack or control the board?
  10293.  
  10294. ==> games/chess/knight.control.s <==
  10295. Fourteen knights are required to attack every square:
  10296.  
  10297.     1   2   3   4   5   6   7   8
  10298.    ___ ___ ___ ___ ___ ___ ___ ___
  10299. h !   !   !   !   !   !   !   !   !
  10300.    --- --- --- --- --- --- --- ---
  10301. g !   !   ! N ! N ! N ! N !   !   !
  10302.    --- --- --- --- --- --- --- ---
  10303. f !   !   !   !   !   !   !   !   !
  10304.    --- --- --- --- --- --- --- ---
  10305. e !   ! N ! N !   !   ! N ! N !   !
  10306.    --- --- --- --- --- --- --- ---
  10307. d !   !   !   !   !   !   !   !   !
  10308.    --- --- --- --- --- --- --- ---
  10309. c !   ! N ! N ! N ! N ! N ! N !   !
  10310.    --- --- --- --- --- --- --- ---
  10311. b !   !   !   !   !   !   !   !   !
  10312.    --- --- --- --- --- --- --- ---
  10313. a !   !   !   !   !   !   !   !   !
  10314.    --- --- --- --- --- --- --- ---
  10315.  
  10316. Three knights are needed to attack h1, g2, and a8; two more for b1, a2,
  10317. and b3, and another two for h7, g8, and f7.
  10318.  
  10319. The only alternative pattern is:
  10320.  
  10321.     1   2   3   4   5   6   7   8
  10322.    ___ ___ ___ ___ ___ ___ ___ ___
  10323. h !   !   !   !   !   !   !   !   !
  10324.    --- --- --- --- --- --- --- ---
  10325. g !   !   ! N !   !   ! N !   !   !
  10326.    --- --- --- --- --- --- --- ---
  10327. f !   !   ! N ! N ! N ! N !   !   !
  10328.    --- --- --- --- --- --- --- ---
  10329. e !   !   !   !   !   !   !   !   !
  10330.    --- --- --- --- --- --- --- ---
  10331. d !   !   ! N ! N ! N ! N !   !   !
  10332.    --- --- --- --- --- --- --- ---
  10333. c !   ! N ! N !   !   ! N ! N !   !
  10334.    --- --- --- --- --- --- --- ---
  10335. b !   !   !   !   !   !   !   !   !
  10336.    --- --- --- --- --- --- --- ---
  10337. a !   !   !   !   !   !   !   !   !
  10338.    --- --- --- --- --- --- --- ---
  10339.  
  10340. Twelve knights are needed to control (attack or occupy) the board:
  10341.  
  10342.     1   2   3   4   5   6   7   8
  10343.    ___ ___ ___ ___ ___ ___ ___ ___
  10344. a !   !   !   !   !   !   !   !   !
  10345.    --- --- --- --- --- --- --- ---
  10346. b !   !   ! N !   !   !   !   !   !
  10347.    --- --- --- --- --- --- --- ---
  10348. c !   !   ! N ! N !   ! N ! N !   !
  10349.    --- --- --- --- --- --- --- ---
  10350. d !   !   !   !   !   ! N !   !   !
  10351.    --- --- --- --- --- --- --- ---
  10352. e !   !   ! N !   !   !   !   !   !
  10353.    --- --- --- --- --- --- --- ---
  10354. f !   ! N ! N !   ! N ! N !   !   !
  10355.    --- --- --- --- --- --- --- ---
  10356. g !   !   !   !   !   ! N !   !   !
  10357.    --- --- --- --- --- --- --- ---
  10358. h !   !   !   !   !   !   !   !   !
  10359.    --- --- --- --- --- --- --- ---
  10360.  
  10361. Each knight can control at most one of the twelve squares a1, b1, b2,
  10362. h1, g1, g2, a8, b8, b7, h8, g8, g7.  This position is unique up to
  10363. reflection.
  10364.  
  10365. References
  10366.     Martin Gardner, _Mathematical Magic Show_.
  10367.  
  10368. ==> games/chess/mutual.check.p <==
  10369. What position is a stalemate for both sides and is reachable in a legal game
  10370. (including the requirement to prevent check)?
  10371.  
  10372. ==> games/chess/mutual.check.s <==
  10373. Put the following configuration in one corner:
  10374.  
  10375. !
  10376. !  x
  10377. !  P   x
  10378. !B P   P
  10379. !K R B
  10380. +---------
  10381.  
  10382. ("x" is a Black pawn), and the same with colors reversed in the h8
  10383. corner.
  10384.  
  10385. --Noam D. Elkies (elkies@zariski.harvard.edu)
  10386.   Dept. of Mathematics, Harvard University
  10387.  
  10388. ==> games/chess/mutual.stalemate.p <==
  10389. What's the minimal number of pieces in a legal mutual stalemate?
  10390.  
  10391. ==> games/chess/mutual.stalemate.s <==
  10392. 6.
  10393.  
  10394. W Kh8 e6 f7 h7  B Kf8 e7
  10395. W Kb1  B Ka3 b2 b3 b4 a4
  10396. W Kf1  B Kh1 Bg1 f2 f3 h2
  10397.  
  10398. ==> games/chess/queens.p <==
  10399. How many ways can eight queens be placed so that they control the board?
  10400.  
  10401. ==> games/chess/queens.s <==
  10402. 92.  The following program uses a backtracking algorithm to count positions:
  10403.  
  10404. #include <stdio.h>
  10405.  
  10406. static int count = 0;
  10407.  
  10408. void try(int row, int left, int right) {
  10409.    int poss, place;
  10410.    if (row == 0xFF) ++count;
  10411.    else {
  10412.       poss = ~(row!left!right) & 0xFF;
  10413.       while (poss ]= 0) {
  10414.          place = poss & -poss;
  10415.          try(row!place, (left!place)<<1, (right!place)>>1);
  10416.          poss &= ~place;
  10417.          }
  10418.       }
  10419.    }
  10420.  
  10421. void main() {
  10422.    try(0,0,0);
  10423.    printf("There are %d solutions.\n", count);
  10424.    }
  10425. --
  10426. Tony Lezard IS tony@mantis.co.uk OR tony%mantis.co.uk@uknet.ac.uk
  10427. OR EVEN arl10@phx.cam.ac.uk if all else fails.
  10428.  
  10429. ==> games/chess/size.of.game.tree.p <==
  10430. How many different positions are there in the game tree of chess?
  10431.  
  10432. ==> games/chess/size.of.game.tree.s <==
  10433. Consider the following assignment of bit strings to square states:
  10434.  
  10435.         Square State            Bit String
  10436.         ------ -----            --- ------
  10437.  
  10438.         Empty                   0
  10439.         White Pawn              100
  10440.         Black Pawn              101
  10441.         White Rook              11111
  10442.         Black Rook              11110
  10443.         White Knight            11101
  10444.         Black Knight            11100
  10445.         White Bishop            11011
  10446.         Black Bishop            11010
  10447.         White Queen             110011
  10448.         Black Queen             110010
  10449.         White King              110001
  10450.         Black King              110000
  10451.  
  10452. Record a position by listing the bit string for each of the 64 squares.
  10453. For a position with all the pieces still on the board, this will take
  10454. 164 bits.  As pieces are captured, the number of bits needed goes down.
  10455. As pawns promote, the number of bits go up.  For positions where a King
  10456. and Rook are in position to castle if castling is legal, we will need
  10457. a bit to indicate if in fact castling is legal.  Same for positions
  10458. where an en-passant capture may be possible.  I'm going to ignore these
  10459. on the grounds that a more clever encoding of a position than the one
  10460. that I am proposing could probably save as many bits as I need for these
  10461. considerations, and thus conjecture that 164 bits is enough to encode a
  10462. chess position.
  10463.  
  10464. This gives an upper bound of 2^164 positions, or 2.3x10^49 positions.
  10465.  
  10466. Jurg Nievergelt, of ETH Zurich, quoted the number 2^70 (or about 10^21) in
  10467. e-mail, and referred to his paper "Information content of chess positions",
  10468. ACM SIGART Newsletter 62, 13-14, April 1977, to be reprinted in "Machine
  10469. Intelligence" (ed Michie), to appear 1990.
  10470.  
  10471. Note that this latest estimate, 10^21, is not too intractable:
  10472. 10^7 computers running at 10^7 positions per second could scan those
  10473. in 10^7 seconds, which is less than 6 months.
  10474.  
  10475. In fact, suppose there is a winning strategy in chess for white.  Suppose
  10476. further that the strategy starts from a strong book opening, proceeds through
  10477. middle game with only moves that DT would pick using the singular
  10478. extension technique, and finally ends in an endgame that DT can analyze
  10479. completely.  The book opening might take you ten moves into the game and
  10480. DT has demonstarted its ability to analyze mates-in-20, so how many nodes
  10481. would DT really have to visit?  I suggest that by using external storage
  10482. such a optical WORM memory, you could easily build up a transposition
  10483. table for such a midgame.  If DT did not find a mate, you could progressively
  10484. expand the width of the search window and add to the table until it did.
  10485. Of course there would be no guarantee of success, but the table built
  10486. would be useful regardless.  Also, you could change the book opening and
  10487. add to the table.  This project could continue indefinitely until finally
  10488. it must solve the game (possibly using denser and denser storage media as
  10489. technology advances).
  10490.  
  10491. What do you think?
  10492.  
  10493. -------
  10494.  
  10495. I think you are a little bit too optimistic about the feasibility.  Solving
  10496. mate-in-19 when the moves are forcing is one thing, but solving mate-in-19
  10497. when the moves are not forcing is another.  Of course, human beings are no
  10498. better at the latter task.  But to solve the game in the way you described
  10499. would seem to require the ability to handle the latter task.  Anyway, we
  10500. cannot really think about doing the sort of thing you described; DT is just a
  10501. poor man's chess machine project (relatively speaking).
  10502.                                                 --Hsu
  10503.  
  10504. i dont think that you understand the numbers involved.
  10505. the size of the tree is still VERY large compared to all
  10506. the advances that you cite. (speed of DT, size of worms,
  10507. endgame projects, etc) even starting a project will probably
  10508. be a waste of time since the next advance will overtake it
  10509. rather than augment it. (if you start on a journey to the
  10510. stars today, you will be met there by humans)
  10511. ken
  10512.  
  10513. ==> games/cigarettes.p <==
  10514. The game of cigarettes is played as follows:
  10515. Two players take turns placing a cigarette on a circular table.  The cigarettes
  10516. can be placed upright (on end) or lying flat, but not so that it touches any
  10517. other cigarette on the table.  This continues until one person looses by not
  10518. having a valid position on the table to place a cigarette.
  10519.  
  10520. Is there a way for either of the players to guarantee a win?
  10521.  
  10522. ==> games/cigarettes.s <==
  10523. The first person wins by placing a cigarette at the center of the table,
  10524. and then placing each of his cigarettes in a position symmetric (with
  10525. respect to the center) to the place the second player just moved.  If the
  10526. second player could move, then symmetrically, so can the first player.
  10527.  
  10528. ==> games/connect.four.p <==
  10529. Is there a winning strategy for Connect Four?
  10530.  
  10531. ==> games/connect.four.s <==
  10532. An AI program has solved Connect Four for the standard 7 x 6 board.
  10533. The conclusion: White wins, was confirmed by the brute force check made by
  10534. James D. Allen, which has been published in rec.games.programmer.
  10535.  
  10536. The program called VICTOR consists of a pure knowledge-based evaluation
  10537. function which can give three values to a position:
  10538.  1 won by white,
  10539.  0 still unclear.
  10540. -1 at least a draw for Black,
  10541.  
  10542. This evaluation function is based on 9 strategic rules concerning the game,
  10543. which all nine have been (mathematically) proven to be correct.
  10544. This means that a claim made about the game-theoretical value of a position
  10545. by VICTOR, is correct, although no search tree is built.
  10546. If the result 1 or -1 is given, the program outputs a set of rules applied,
  10547. indicating the way the result can be achieved.
  10548. This way one evaluation can be used to play the game to the end without any
  10549. extra calculation (unless the position was still unclear, of course).
  10550.  
  10551. Using the evaluation function alone, it has been shown that Black can at least
  10552. draw the game on any 6 x (2n) board. VICTOR found an easy strategy for
  10553. these boardsizes, which can be taught to anyone within 5 minutes. Nevertheless,
  10554. this strategy had not been encountered before by any humans, as far as I know.
  10555.  
  10556. For 7 x (2n) boards a similar strategy was found, in case White does not
  10557. start the game in the middle column. In these cases Black can therefore at
  10558. least draw the game.
  10559.  
  10560. Furthermore, VICTOR needed only to check a few dozen positions to show
  10561. that Black can at least draw the game on the 7 x 4 board.
  10562.  
  10563. Evaluation of a position on a 7 x 4 or 7 x 6 board costs between 0.01 and 10
  10564. CPU seconds on a Sun4.
  10565.  
  10566. For the 7 x 6 board too many positions were unclear. For that reason a
  10567. combination of Conspiracy-Number Search and Depth First Search was used
  10568. to determine the game-theoretical value. This took several hundreds of hours
  10569. on a Sun4.
  10570.  
  10571. The main reason for the large amount of search needed, was the fact that in
  10572. many variations, the win for White was very difficult to achieve.
  10573. This caused many positions to be unclear for the evaluation function.
  10574.  
  10575. Using the results of the search, a database will be constructed
  10576. of roughly 500.000 positions with their game-theoretical value.
  10577. Using this datebase, VICTOR can play against humans or other programs,
  10578. winning all the time (playing White).  The average move takes less
  10579. than a second of calculation (search in the database or evaluation
  10580. of the position by the evaluation function).
  10581.  
  10582. Some variations are given below (columns and rows are numbered as is customary
  10583. in chess):
  10584.  
  10585. 1. d1, ..  The only winning move.
  10586.  
  10587. After 1. .., a1 wins 2. e1. Other second moves for White has not been
  10588. checked yet.
  10589. After 1. .., b1 wins 2. f1. Other second moves for White has not been
  10590. checked yet.
  10591. After 1. .., c1 wins 2. f1. Only 2 g1 has not been checked yet. All other
  10592. second moves for White give Black at least a draw.
  10593. After 1. .., d2 wins 2. d3. All other second moves for White give black
  10594. at least a draw.
  10595.  
  10596. A nice example of the difficulty White has to win:
  10597.  
  10598. 1. d1, d2
  10599. 2. d3, d4
  10600. 3. d5, b1
  10601. 4. b2]
  10602.  
  10603. The first three moves for White are forced, while alternatives at the
  10604. fourth moves of White are not checked yet.
  10605.  
  10606. A variation which took much time to check and eventually turned out
  10607. to be at least a draw for Black, was:
  10608.  
  10609. 1. d1, c1
  10610. 2. c2?, .. f1 wins, while c2 does not.
  10611. 2. .., c3 Only move which gives Black the draw.
  10612. 3. c4, .. White's best chance.
  10613. 3. .., g1]] Only 3 .., d2 has not been checked completely, while all
  10614.             other third moves for Black have been shown to lose.
  10615.  
  10616. The project has been described in my 'doctoraalscriptie' (Master thesis)
  10617. which has been supervised by Prof.Dr H.J. van den Herik of the
  10618. Rijksuniversiteit Limburg (The Netherlands).
  10619.  
  10620. I will give more details if requested.
  10621.  
  10622. Victor Allis.
  10623. Vrije Universiteit van Amsterdam.
  10624. The Netherlands.
  10625. victor@cs.vu.nl
  10626.  
  10627. ==> games/craps.p <==
  10628. What are the odds in craps?
  10629.  
  10630. ==> games/craps.s <==
  10631. The game of craps:
  10632. There is a person who rolls the two dice, and then there is the house.
  10633. 1) On the first roll, if a 7 or 11 comes up, the roller wins.
  10634.    If a 2, 3, or 12 comes up the house wins.
  10635.    Anything else is a POINT, and more rolling is necessary, as per rule 2.
  10636. 2) If a POINT appears on the first roll, keep rolling the dice.
  10637.    At each roll, if the POINT appears again, the roller wins.
  10638.    At each roll, if a 7 comes up, the house wins.
  10639.    Keep rolling until the POINT or a 7 comes up.
  10640.  
  10641. Then there are the players, and they are allowed to place their bets with
  10642. either the roller or with the house.
  10643.  
  10644. -----
  10645. My computations:
  10646.  
  10647.  
  10648.  
  10649.  
  10650.  
  10651. On the first roll, P.roller.trial(1) = 2/9, and P.house.trial(1) = 1/9.
  10652. Let  P(x) stand for the probability of a 4,5,6,8,9,10 appearing.
  10653. Then on the second and onwards rolls, the probability is:
  10654.  
  10655. Roller:
  10656.                          ---                        (i - 2)
  10657. P.roller.trial(i) =      \   P(x)   *   ((5/6 - P(x))         *   P(x)
  10658. (i > 1)                  /
  10659.                          ---
  10660.                          x = 4,5,6,8,9,10
  10661.  
  10662. House:
  10663.                         ---                        (i - 2)
  10664. P.house.trial(i) =      \   P(x)   *   ((5/6 - P(x))         *   1/6
  10665. (i > 1)                 /
  10666.                         ---
  10667.                         x = 4,5,6,8,9,10
  10668.  
  10669. Reasoning (roller): For the roller to win on the ith trial, a POINT
  10670. should have appeared on the first trial (the first P(x) term), and the
  10671. same POINT should appear on the ith trial (the last P(x) term). All the in
  10672. between trials should come up with a number other than 7 or the POINT
  10673. (hence the (5/6 - P(x)) term).
  10674. Similar reasoning holds for the house.
  10675.  
  10676. The numbers are:
  10677. P.roller.trial(i) (i > 1) =
  10678.  
  10679.                 (i-1)                 (i-1)                     (i-1)
  10680.  1/72 * (27/36)      + 2/81 * (26/36)        + 25/648 * (25/36)
  10681.  
  10682.  
  10683. P.house.trial(i) (i > 1) =
  10684.  
  10685.                 (i-1)                 (i-1)                     (i-1)
  10686.  2/72 * (27/36)      + 3/81 * (26/36)        + 30/648 * (25/36)
  10687.  
  10688.  
  10689. -------------------------------------------------
  10690. The total probability comes to:
  10691. P.roller = 2/9   +   (1/18 + 4/45 + 25/198)  = 0.4929292929292929..
  10692. P.house  = 1/9   +   (1/9  + 2/15 + 15/99)  =  0.5070707070707070..
  10693.  
  10694. which is not even.
  10695. ===========================================================================
  10696.  
  10697. ==
  10698. Avinash Chopde                           (with standard disclaimer)
  10699. abc@unhcs.unh.edu, abc@unh.unh.edu            {.....}]uunet]unh]abc
  10700.  
  10701. ==> games/crosswords/cryptic/clues.p <==
  10702. What are some clues (indicators) used in cryptics?
  10703.  
  10704. ==> games/crosswords/cryptic/clues.s <==
  10705. The following list is derived from indicators used in a variety of
  10706. crosswords: the letters in the left column are the letters being
  10707. indicated; the right hand column is how these letters might be
  10708. indicated in a clue.
  10709.  
  10710. Caveat emptor: many of the entries in this list would be considered
  10711. unsound in some puzzles (some of these unsound indicators are marked
  10712. with a +). Entries marked * are used mostly in advanced cryptics.
  10713.  
  10714. I would welcome corrections and additions to the list.
  10715.  
  10716. ----------------------------------------------------------------------
  10717. a               Austria
  10718. a               I
  10719. a               academician
  10720. a               accepted
  10721. a               ace
  10722. a               acre
  10723. a               active
  10724. a               adult
  10725. a               advanced
  10726. a               afternoon
  10727. a               aleph
  10728. a               alpha
  10729. a               amateur
  10730. a               ampere
  10731. a               an/ane
  10732. a               angstrom
  10733. a               answer
  10734. a               ante
  10735. a               are (metric)
  10736. a               articles - English
  10737. a               associate
  10738. a               atomic
  10739. a               ay
  10740. a               aye
  10741. a               before
  10742. a               blood group
  10743. a               bomb
  10744. a               effect
  10745. a               examination
  10746. a               fifty
  10747. a               film
  10748. a               first character
  10749. a               first class
  10750. a               first letter
  10751. a               five hundred
  10752. a               five thousand
  10753. a               good
  10754. a               high class
  10755. a               it
  10756. a               key +
  10757. a               level
  10758. a               midday
  10759. a               note +
  10760. a               one *
  10761. a               paper
  10762. a               road
  10763. a               strings (violin)
  10764. a               un
  10765. a               unit
  10766. a               violin string
  10767. a               vitamin
  10768. a               year
  10769. aa              motoring organisation
  10770. ab              able seaman
  10771. ab              hand
  10772. ab              rating
  10773. ab              sailor/salt/seaman
  10774. ab              tar
  10775. abbe            priest (Fr.)
  10776. abe             Lincoln
  10777. abel            first victim
  10778. abel            murder victim
  10779. abel            second child
  10780. abel            third man
  10781. able            can
  10782. able            expert
  10783. abo             native
  10784. ac              account
  10785. ac              accountant
  10786. ac              aircraftsman
  10787. ac              alternating current
  10788. ac              before Christ
  10789. ac              bill
  10790. ac              current
  10791. aca             accountant
  10792. acas            peacemakers
  10793. acc             account
  10794. acc             bill
  10795. ace             card
  10796. ace             champion
  10797. ace             expert
  10798. ace             one
  10799. ace             pilot
  10800. ace             service
  10801. ace             winner
  10802. act             decree
  10803. act             performance
  10804. actor           tree
  10805. ad              Christian era
  10806. ad              advertisement
  10807. ad              after date
  10808. ad              before the day
  10809. ad              contemporary
  10810. ad              in the modern age
  10811. ad              in the year of our Lord
  10812. ad              modern times
  10813. ad              notice
  10814. ad              now
  10815. ad              nowadays
  10816. ad              our time/era
  10817. ad              period
  10818. ad              present day
  10819. ad              promotion
  10820. ad              puff
  10821. ad              this era
  10822. ad              today
  10823. adam            first character
  10824. adam            first person
  10825. adam            number one
  10826. add             sum
  10827. add             tot
  10828. aden            port
  10829. admin           management
  10830. admin           running
  10831. ado             business
  10832. ado             difficulty
  10833. ado             fuss
  10834. ado             row
  10835. ado             trouble
  10836. ae              aged
  10837. ae              poet
  10838. aet             aged
  10839. ag              silver
  10840. aga             Muslim leader
  10841. age             (long) time
  10842. age             mature
  10843. age             period
  10844. agent           spy
  10845. agm             annual meeting
  10846. agm             meeting
  10847. agm             yearly meeting
  10848. ai              capital
  10849. ai              first class
  10850. ai              good
  10851. ai              high class
  10852. ai              main road
  10853. ai              sloth
  10854. ail             trouble
  10855. ain             own (Scot.)
  10856. air             appearance
  10857. air             display
  10858. air             song
  10859. aire            river
  10860. ait             island
  10861. al              Alabama
  10862. al              Alan
  10863. al              Albania
  10864. al              Albert
  10865. al              Capone
  10866. al              aluminium
  10867. al              gangster
  10868. al              one pound
  10869. ala             Alabama
  10870. ala             after the style of
  10871. ala             in the style of
  10872. ala             to the (Fr.)
  10873. ala             wings
  10874. alas            Alaska
  10875. alb             one pound
  10876. ale             beer
  10877. aleph           Hebrew letter
  10878. all             completely
  10879. all             everybody
  10880. all             everything
  10881. alp             mountain
  10882. alp             peak
  10883. alph            river
  10884. alpha           Greek letter
  10885. alpha           beginning
  10886. alpha           first character
  10887. alpha           first letter
  10888. am              America
  10889. am              American
  10890. am              I am
  10891. am              admitting
  10892. am              boasting
  10893. am              half day
  10894. am              hymns
  10895. am              in the morning
  10896. am              morning
  10897. am              self-confessed
  10898. amen            final word
  10899. amen            last word
  10900. amer            American
  10901. ammo            missiles
  10902. amos            bookmaker
  10903. amp             one member
  10904. an              I
  10905. an              articles - English
  10906. an              before
  10907. an              if (old word)
  10908. an              one *
  10909. an              un
  10910. ana             tales
  10911. ane             I
  10912. ane             one
  10913. ane             one
  10914. ankh            life symbol
  10915. anne            princess
  10916. anon            now
  10917. ans             answer
  10918. ans             brief reply
  10919. ans             collection
  10920. ans             short answer
  10921. ant             if it (old word)
  10922. ant             six-footer
  10923. ant             social worker
  10924. ant             soldier
  10925. ant             worker
  10926. ape             copy
  10927. ape             primate
  10928. aq              water
  10929. ar              arrive/arrival
  10930. ar              year of reign
  10931. ara             academician
  10932. ara             artist
  10933. ara             painter
  10934. arab            horse
  10935. arc             curve
  10936. argo            old ship
  10937. aria            song
  10938. arm             gun
  10939. arm             limb
  10940. arm             member
  10941. arr             arrive/arrival
  10942. art             contrivance
  10943. art             craft
  10944. art             cunning
  10945. art             painting
  10946. art             skill
  10947. as              Anglo-Saxon
  10948. as              ayes
  10949. as              ays
  10950. as              like
  10951. as              one's
  10952. as              specifically
  10953. as              when
  10954. ash             remains
  10955. ash             tree
  10956. asia            continent
  10957. aside           one fifteen
  10958. asis            existing state
  10959. asp             snake
  10960. ass             donkey
  10961. asti            wine
  10962. ate             goddess
  10963. ate             mischief
  10964. athena          goddess
  10965. ation           at one on
  10966. atoll           bikini
  10967. au              gold
  10968. au              to the (Fr.)
  10969. aus             Australia
  10970. aux             to the (Fr.)
  10971. av              bible
  10972. av              lived so long
  10973. ave             average
  10974. ave             greeting
  10975. ave             hail
  10976. ave             road
  10977. ave             way
  10978. aver            average
  10979. avon            county
  10980. ay              I
  10981. ay              agreement
  10982. ay              always
  10983. ay              ever
  10984. ay              yes
  10985. aye             I
  10986. aye             I say
  10987. aye             agreement
  10988. aye             always
  10989. aye             ever
  10990. aye             yes
  10991. az              Azed
  10992. az              scope, plenty of
  10993. b               Bach
  10994. b               Beethoven
  10995. b               Belgium
  10996. b               Brahms
  10997. b               Britain
  10998. b               British
  10999. b               a follower
  11000. b               bachelor
  11001. b               baron
  11002. b               bedbug
  11003. b               bee
  11004. b               bel
  11005. b               beta
  11006. b               beth
  11007. b               bishop
  11008. b               black
  11009. b               blood group
  11010. b               bloody
  11011. b               book *
  11012. b               born
  11013. b               boron
  11014. b               bowled
  11015. b               boy
  11016. b               breadth
  11017. b               inferior
  11018. b               key +
  11019. b               magnetic flux
  11020. b               note +
  11021. b               paper
  11022. b               second
  11023. b               second class
  11024. b               second letter
  11025. b               three hundred
  11026. b               three thousand
  11027. b               vitamin
  11028. ba              Bachelor of Arts
  11029. ba              airline
  11030. ba              bachelor
  11031. ba              barium
  11032. ba              degree
  11033. ba              graduate
  11034. ba              scholar
  11035. bac             airline
  11036. bacon           philosopher
  11037. ban             curse
  11038. ban             outlaw
  11039. ban             prohibition
  11040. bar             Inn
  11041. bar             lawyers
  11042. bar             prevent
  11043. bar             save
  11044. barb            horse
  11045. bat             fly-by-night
  11046. bb              bees
  11047. bb              books
  11048. bb              very black
  11049. bc              ancient times
  11050. bc              before Christ
  11051. bc              period
  11052. bd              beady
  11053. bd              bound
  11054. bd              cleric
  11055. bd              theologian
  11056. be              exist
  11057. be              live
  11058. bea             airline
  11059. bear            speculator
  11060. bed             in bed
  11061. bee             buzzer
  11062. bee             group of workers
  11063. bee             six-footer
  11064. bee             social worker
  11065. bee             worker
  11066. bef             Gort's men
  11067. bess            queen
  11068. beta            Greek letter
  11069. beth            Hebrew letter
  11070. bi              double
  11071. bi              two (double)
  11072. bird            prison
  11073. bis             two (twice)
  11074. bit             chewed
  11075. bit             piece
  11076. biz             business
  11077. bk              book
  11078. bl              British company
  11079. bl              lawman
  11080. bl              lawyer
  11081. blue            Conservative
  11082. bm              British Museum
  11083. bm              doctor
  11084. bo              American man
  11085. boa             snake
  11086. board           directors
  11087. bob             old shilling
  11088. bp              bishop
  11089. br              Britain
  11090. br              British
  11091. br              British Rail
  11092. br              bank rate
  11093. br              branch
  11094. br              bridge
  11095. br              brig
  11096. br              brother
  11097. br              brown *
  11098. br              lines/landline
  11099. br              railway(s)
  11100. br              trains
  11101. br              transport
  11102. bra             female support(er)
  11103. bra             support
  11104. bra             undergarment
  11105. brass           money
  11106. brat            child
  11107. bren            gun
  11108. brer            rabbit
  11109. bridal          old wedding
  11110. bro             brother
  11111. bs              bees
  11112. bst             summer time
  11113. bull            American policeman
  11114. bull            gold
  11115. bus             transport
  11116. c               Celsius/centigrade
  11117. c               Charles
  11118. c               Conservative
  11119. c               Cuba
  11120. c               about (approx.)
  11121. c               approx(imately)
  11122. c               around
  11123. c               cape
  11124. c               caput
  11125. c               carbon
  11126. c               caught
  11127. c               cedi
  11128. c               cent/centime
  11129. c               centi-
  11130. c               century
  11131. c               chapter
  11132. c               circa
  11133. c               club
  11134. c               cold
  11135. c               complex number
  11136. c               copyright
  11137. c               coulomb
  11138. c               electrical capacity
  11139. c               hundred
  11140. c               hundred thousand
  11141. c               key +
  11142. c               lot
  11143. c               many
  11144. c               note +
  11145. c               roughly
  11146. c               sea
  11147. c               see
  11148. c               speed of light
  11149. c               spring
  11150. c               tap
  11151. c               vitamin
  11152. ca              about (approx.)
  11153. ca              accountant
  11154. ca              approx(imately)
  11155. ca              calcium
  11156. ca              roughly
  11157. cab             transport
  11158. cade            conspirator
  11159. cain            first murderer
  11160. cain            killer
  11161. cain            murderer
  11162. cal             California
  11163. cam             river
  11164. can             able to +
  11165. can             is able to
  11166. can             prison
  11167. can             vessel
  11168. cantuar         archbishop
  11169. cap             chapter
  11170. cap             international
  11171. car             carat
  11172. car             transport
  11173. carnation       motor race
  11174. cart            transport
  11175. cat             jazz fan
  11176. cato            censor
  11177. cattle          neat
  11178. cave            warning
  11179. cb              Seabee (Amer.)
  11180. cc              county council
  11181. cc              seas
  11182. cc              small measure
  11183. cc              small quantity
  11184. cc              two hundred
  11185. cd              diplomat
  11186. cd              seedy
  11187. ce              Church of England
  11188. ce              church
  11189. ce              engineers
  11190. ce              this (Fr.)
  11191. cent            money
  11192. cet             this (Fr.)
  11193. ch              China
  11194. ch              Companion of Honour
  11195. ch              Switzerland
  11196. ch              award
  11197. ch              central heating
  11198. ch              champion
  11199. ch              chapter
  11200. ch              chief
  11201. ch              child
  11202. ch              church
  11203. ch              companion
  11204. ch              honour
  11205. ch              order
  11206. cha             tea
  11207. chai            gypsy woman
  11208. chair           president
  11209. chal            gypsy
  11210. char            daily
  11211. che             guerrilla
  11212. che             revolutionary
  11213. cher            dear (Fr.)
  11214. chere           dear (Fr.)
  11215. chi             Greek letter
  11216. ci              Channel Islands
  11217. ci              hundred and one
  11218. cia             secret service
  11219. cia             spies
  11220. cid             captain
  11221. cid             chief
  11222. cid             detectives
  11223. cid             police
  11224. cid             spanish hero
  11225. cinc            commander
  11226. cl              chlorine
  11227. cl              class
  11228. cl              clause
  11229. cl              gas - chlorine
  11230. cl              hundred and fifty
  11231. co              Colombia
  11232. co              business
  11233. co              care of
  11234. co              cobalt
  11235. co              commander
  11236. co              commanding officer
  11237. co              company
  11238. co              county
  11239. co              firm
  11240. co              gas - carbon monoxide
  11241. co              house
  11242. co              objector
  11243. co              officer
  11244. cod             fish
  11245. cod             swimmer
  11246. col             neck
  11247. col             pass
  11248. cole            old king
  11249. colon           stop
  11250. com             commander
  11251. comb            hairdresser
  11252. composer        scorer
  11253. con             Conservative
  11254. con             against
  11255. con             party
  11256. con             politician
  11257. con             study
  11258. con             swindle
  11259. con             trick
  11260. cooler          prison
  11261. core            decentralise
  11262. corn            naval commander
  11263. cot             bed
  11264. cot             house
  11265. cow             lower
  11266. cow             neat
  11267. cr              credit
  11268. cr              crown
  11269. cr              king
  11270. cs              Civil Service
  11271. cs              Czechoslovakia
  11272. cs              hundreds
  11273. cs              seas
  11274. ct              Connecticut
  11275. ct              carat
  11276. ct              caught
  11277. ct              cent/centime
  11278. ct              court
  11279. ct              small weight
  11280. ct              weight
  11281. cu              copper
  11282. cu              see you
  11283. cue             queue
  11284. cure            priest (Fr.)
  11285. cutie           pretty girl
  11286. cv              autobiography
  11287. cy              see why
  11288. d               (old) penny
  11289. d               Dee
  11290. d               Democrat
  11291. d               Deutsch
  11292. d               Germany
  11293. d               Schubert's works
  11294. d               copper
  11295. d               damn
  11296. d               date
  11297. d               daughter
  11298. d               day
  11299. d               dead/died
  11300. d               deci
  11301. d               degree
  11302. d               delete
  11303. d               delta
  11304. d               deserted
  11305. d               deuterium
  11306. d               diameter
  11307. d               diamond
  11308. d               differential operator
  11309. d               electrical flux
  11310. d               five hundred
  11311. d               four
  11312. d               four thousand
  11313. d               hundreds
  11314. d               key +
  11315. d               lot
  11316. d               many +
  11317. d               mark
  11318. d               note +
  11319. d               notice
  11320. d               number
  11321. d               strings (violin)
  11322. d               violin string
  11323. d               vitamin
  11324. da              American lawyer
  11325. da              District Attorney
  11326. da              agreement - foreign (Russ.)
  11327. da              dagger *
  11328. da              lawman
  11329. da              lawyer
  11330. da              yes (Russ.)
  11331. dab             expert
  11332. dad             father
  11333. dad             old man
  11334. dail            Irish house
  11335. dam             barrier
  11336. dam             mother
  11337. dam             restrain
  11338. dame            lady
  11339. dan             tribe
  11340. das             articles - German
  11341. das             the (Ger.)
  11342. dc              Washington
  11343. dc              Washington
  11344. dc              current
  11345. dd              cleric
  11346. dd              days
  11347. dd              divine
  11348. dd              doctor
  11349. dd              doctor of divinity
  11350. dd              theologian
  11351. de              from (Fr.)
  11352. de              of (Fr.)
  11353. dean            good man
  11354. dec             Christmas period
  11355. dec             last month
  11356. decanter        Tantalus' prisoner
  11357. dee             river
  11358. deed            indeed
  11359. deed            legal document
  11360. deep            in the main
  11361. deep            main
  11362. deep            sea
  11363. deg             degree
  11364. del             of the (Ital.)
  11365. dela            from the (Fr.)
  11366. dela            of the (Fr.)
  11367. demi            half
  11368. den             retreat
  11369. den             study
  11370. der             articles - German
  11371. der             the (Ger.)
  11372. derby           horse race
  11373. des             of the (Fr.)
  11374. det             detective
  11375. di              double
  11376. di              five hundred and one
  11377. di              princess
  11378. di              two (double)
  11379. die             articles - German
  11380. die             the (Ger.)
  11381. dime            12.5 cents
  11382. dior            designer
  11383. dis             Hell
  11384. dis             Pluto
  11385. dis             underworld
  11386. disc            circle
  11387. disc            record
  11388. disc            ring
  11389. dish            pretty girl
  11390. dit             named
  11391. dit             reported
  11392. dit             said (Fr.)
  11393. dit             say (Fr.)
  11394. diy             amateur's department
  11395. dk              Denmark
  11396. dm              mark
  11397. do              (the) same
  11398. do              act
  11399. do              cheat
  11400. do              cook
  11401. do              ditto
  11402. do              note
  11403. do              party
  11404. do              work
  11405. dodo            double act
  11406. doh             note
  11407. don             fellow
  11408. don             nobleman
  11409. don             put on
  11410. don             university teacher
  11411. down            county
  11412. dr              dead reckoning
  11413. dr              doctor
  11414. dr              dram
  11415. dr              drawer
  11416. dr              healer
  11417. drake           bowler
  11418. dt              alcoholic state
  11419. dt              psychotic state
  11420. du              from the (Fr.)
  11421. du              of the (Fr.)
  11422. dutch           wife
  11423. e               Asian
  11424. e               Edward
  11425. e               Elizabeth
  11426. e               England/English
  11427. e               Spain
  11428. e               boat
  11429. e               bridge players
  11430. e               direction
  11431. e               east/eastern
  11432. e               eight
  11433. e               eight thousand
  11434. e               energy +
  11435. e               epsilon
  11436. e               eta
  11437. e               five
  11438. e               five thousand
  11439. e               key +
  11440. e               layer
  11441. e               logarithm base
  11442. e               low grade
  11443. e               note +
  11444. e               orient
  11445. e               oriental
  11446. e               point
  11447. e               quarter
  11448. e               strings (violin)
  11449. e               two hundred and fifty
  11450. e               two hundred and fifty thousand
  11451. e               universal set
  11452. e               violin string
  11453. e               vitamin
  11454. ea              East Africa
  11455. ea              each
  11456. ea              river *
  11457. ea              running water
  11458. ea              water
  11459. ear             listener
  11460. ear             organ
  11461. ear             spike *
  11462. earp            lawman
  11463. eat             Tanzania
  11464. ebor            archbishop
  11465. ec              London district
  11466. ec              city
  11467. eccles          Cakesville
  11468. ed              Edward
  11469. ed              editor
  11470. ed              journalist
  11471. eden            garden
  11472. eden            old Prime Minister
  11473. eden            paradise
  11474. edison          inventor
  11475. edit            censor
  11476. ee              ease
  11477. eel             fish
  11478. eel             swimmer
  11479. eer             always
  11480. eer             ever
  11481. eer             invariably
  11482. eg              for example
  11483. eg              for instance
  11484. egg             bomb
  11485. egg             cocktail
  11486. egg             encourage
  11487. eight           rowing boat
  11488. ein             number one (Ger.)
  11489. el              American railway
  11490. el              American railway
  11491. el              articles - Spanish
  11492. el              measure
  11493. el              printer's measure
  11494. el              small measure
  11495. el              the (Span.)
  11496. eld             old age
  11497. eli             priest
  11498. eli             prophet
  11499. elia            writer
  11500. ell             four feet
  11501. ell             length
  11502. ell             measure
  11503. ely             city
  11504. ely             city
  11505. ely             see
  11506. em              measure
  11507. em              printer's measure
  11508. em              small measure
  11509. em              small square
  11510. em              them
  11511. en              measure
  11512. en              printer's measure
  11513. en              small measure
  11514. eng             England/English
  11515. ent             otorhinolaryngology
  11516. entry           record
  11517. eon             age
  11518. eon             time
  11519. ep              record
  11520. er              Cockney girl
  11521. er              QE
  11522. er              difficulty
  11523. er              ever
  11524. er              hesitation
  11525. er              king
  11526. er              monarch
  11527. er              queen
  11528. er              royal badge
  11529. era             generation
  11530. erasmus         old scholar
  11531. ere             always
  11532. ere             before
  11533. ergo            so
  11534. eric            gradually
  11535. erie            lake
  11536. err             blunder
  11537. err             sin
  11538. err             wander
  11539. erse            Gaelic
  11540. es              French art
  11541. es              ease
  11542. esp             sixth sense
  11543. esp             telepathy
  11544. est             is (Fr.)
  11545. et              Egypt
  11546. et              alien
  11547. et              and (Fr.)
  11548. et              exotic
  11549. et              extraterrestrial
  11550. et              film
  11551. eta             Greek letter
  11552. eta             estimated time of arrival
  11553. eta             illegal army
  11554. eta             terrorists
  11555. eton            college
  11556. eton            educational establishment
  11557. eton            school
  11558. etty            artist
  11559. eur             continent
  11560. eve             first lady
  11561. eve             first mate
  11562. eve             lady
  11563. eve             woman
  11564. ew              bridge partners
  11565. ew              partnership
  11566. ex              former
  11567. ex              from
  11568. ex              late
  11569. ex              one time
  11570. exe             river
  11571. eye             I
  11572. eye             I say
  11573. eye             seer
  11574. eyot            island
  11575. ezra            pound
  11576. f               Fahrenheit
  11577. f               France
  11578. f               Friday
  11579. f               clef
  11580. f               farad
  11581. f               farthing
  11582. f               fathom
  11583. f               fellow
  11584. f               female
  11585. f               feminine
  11586. f               filly
  11587. f               fine
  11588. f               fluorine
  11589. f               folio
  11590. f               following
  11591. f               foot
  11592. f               force
  11593. f               forte
  11594. f               forty
  11595. f               frequency
  11596. f               gas - fluorine
  11597. f               hole
  11598. f               key +
  11599. f               loud
  11600. f               noisy
  11601. f               note +
  11602. f               strong
  11603. f               vitamin
  11604. f               woman
  11605. fa              football
  11606. fa              note
  11607. fah             note
  11608. fal             river
  11609. fare            China area (Far East)
  11610. fast            firm
  11611. fe              iron
  11612. fed             American detective
  11613. ff              folios
  11614. ff              followings
  11615. ff              fortissimo
  11616. ff              very loud
  11617. ff              very loud
  11618. ff              very strong
  11619. ff              very strong
  11620. fig             small illustration
  11621. fir             tree
  11622. firm            business
  11623. firm            company
  11624. fist            duke
  11625. fl              flourished
  11626. fla             Florida
  11627. flower          bloomer
  11628. flu             illness
  11629. fo              Foreign Office
  11630. fo              folio
  11631. fob             free on board
  11632. foc             free of charge
  11633. fol             folio
  11634. fool            dessert
  11635. foot            infantry
  11636. for             free on rail
  11637. force           police
  11638. fore            warning
  11639. four            rowing boat
  11640. fr              French
  11641. fr              father
  11642. fr              fragment
  11643. fr              franc
  11644. fr              frequently
  11645. ft              feet
  11646. ft              foot
  11647. ft              measure
  11648. fz              forced
  11649. g               Gauss
  11650. g               George
  11651. g               Germany
  11652. g               agent
  11653. g               clef
  11654. g               four hundred
  11655. g               gamma
  11656. g               gamut
  11657. g               gee
  11658. g               girl
  11659. g               gram(me)
  11660. g               grand (Amer.)
  11661. g               gravity
  11662. g               guinea
  11663. g               gulf
  11664. g               key +
  11665. g               man
  11666. g               midnight
  11667. g               note +
  11668. g               strings (violin)
  11669. g               suit
  11670. g               thousand
  11671. g               thousand
  11672. g               violin string
  11673. g               weight
  11674. g-man           American detective
  11675. ga              Georgia
  11676. gab             gift
  11677. gad             tribe
  11678. gael            Gaelic
  11679. gal             girl
  11680. gar             fish
  11681. gat             gun
  11682. gate            old goat
  11683. gb              Great Britain
  11684. gb              our islands
  11685. gee             horse
  11686. gee             little horse
  11687. geiger          counter
  11688. gel             jelly
  11689. gen             Genesis
  11690. gen             general
  11691. gen             information
  11692. gen             low down
  11693. gent            fellow
  11694. george          pilot
  11695. gg              Gee-gee
  11696. gg              horse
  11697. gg              little horse
  11698. gi              American soldier
  11699. gi              doughboy
  11700. gi              fighter
  11701. gi              government issue
  11702. gi              private
  11703. gi              serving man
  11704. gi              soldier
  11705. gladiator       old fighter
  11706. glc             capital authority
  11707. gm              counter
  11708. go              bargain
  11709. go              energy
  11710. go              in good condition
  11711. go              ready
  11712. go              success
  11713. go              traffic signal
  11714. go              work
  11715. gotham          New York
  11716. gp              doctor
  11717. gr              Greece
  11718. gr              Greek
  11719. gr              King George
  11720. gr              grain
  11721. gr              grammar
  11722. gr              gramme
  11723. gr              grouse
  11724. gr              king
  11725. gr              small weight
  11726. gr              weight
  11727. grant           general
  11728. grass           informer
  11729. grist           miller's corn
  11730. grs             gunmen
  11731. gs              general service
  11732. gs              general staff
  11733. gt              fast car
  11734. gt              sports car
  11735. gu              guinea
  11736. gu              old fiddle
  11737. gue             old fiddle
  11738. h               Dirac's constant
  11739. h               Hungary
  11740. h               Planck's constant
  11741. h               bomb
  11742. h               gas - hydrogen
  11743. h               hand
  11744. h               hard
  11745. h               heart
  11746. h               height
  11747. h               henry
  11748. h               horse
  11749. h               hospital
  11750. h               hot
  11751. h               hour
  11752. h               house
  11753. h               husband
  11754. h               hydrant
  11755. h               hydrogen
  11756. h               tap
  11757. h               two hundred
  11758. h               two hundred thousand
  11759. h               vitamin
  11760. ha              half ditch
  11761. ha              laugh
  11762. ha              this year
  11763. haha            ditch
  11764. haha            laugh
  11765. hair net        lock keeper
  11766. ham             (poor) actor
  11767. han             Chinese dynasty
  11768. hand            worker
  11769. has             bears
  11770. haw             hedge
  11771. he              (high) explosive
  11772. he              His Excellency
  11773. he              ambassador
  11774. he              excellency
  11775. he              gas - helium
  11776. he              governor
  11777. he              helium
  11778. he              legate
  11779. he              male
  11780. he              our man
  11781. he              the man
  11782. head            point
  11783. hearth          hard ground
  11784. hebe            goddess
  11785. hehe            laugh
  11786. hen             female
  11787. hen             layer
  11788. her             female
  11789. her             the woman's
  11790. her             woman
  11791. hf              half
  11792. hg              Dad's army
  11793. hg              mercury
  11794. hh              very hard
  11795. hi              greeting
  11796. hi              hello
  11797. hic             here in Rome
  11798. hic             this (Lat.)
  11799. him             male
  11800. hm              Her Majesty
  11801. hm              His Majesty
  11802. hm              king
  11803. hm              queen
  11804. ho              house
  11805. hobo            tramp
  11806. hock            prison
  11807. hol             short break
  11808. hp              hire purchase
  11809. hp              never-never
  11810. hr              hour
  11811. hs              here is
  11812. ht              high tension
  11813. hun             German
  11814. hun             barbarian
  11815. i               Italy
  11816. i               a
  11817. i               an/ane
  11818. i               ay
  11819. i               aye
  11820. i               che
  11821. i               electrical current
  11822. i               eye
  11823. i               first person
  11824. i               imaginary number
  11825. i               iodine
  11826. i               iota
  11827. i               island
  11828. i               line
  11829. i               lunchtime
  11830. i               number one
  11831. i               one
  11832. i               one
  11833. i               single
  11834. i               square root of -1
  11835. i               straight line
  11836. i               un
  11837. i               unit
  11838. i               upright
  11839. i               yours truly
  11840. ia              Iowa
  11841. iam             I am
  11842. iam             admitting
  11843. iam             afternoon
  11844. iam             boasting
  11845. iam             early morning
  11846. iam             self-confessed
  11847. ian             Scot/Scotsman
  11848. ians            one answer
  11849. ib              in the same place
  11850. ib              same place
  11851. ibid            in the same place
  11852. ic              I see
  11853. ic              hundred
  11854. ic              in charge
  11855. ic              ninety nine
  11856. ice             diamond(s)
  11857. ice             hard water
  11858. iceni           old people
  11859. id              I had
  11860. id              I would
  11861. id              fish
  11862. id              genius
  11863. id              identification
  11864. id              instinct
  11865. id              same
  11866. id              that (Lat.)
  11867. ida             princess
  11868. ide             fish
  11869. idem            said
  11870. ie              that is (that's)
  11871. if              condition
  11872. if              provided/providing
  11873. ign             gin cocktail/sling
  11874. ii              eleven
  11875. ii              eyes
  11876. ii              two
  11877. il              Israel
  11878. il              articles - Italian
  11879. il              one pound
  11880. il              the (Ital.)
  11881. ilb             one pound
  11882. ill             I shall/will
  11883. ill             Illinois
  11884. ill             badly
  11885. ill             unwell
  11886. im              I am
  11887. im              admitting
  11888. im              boasting
  11889. im              self-confessed
  11890. imp             little devil
  11891. imp             mischevious child
  11892. imp             one member
  11893. impi            soldiers
  11894. in              at home
  11895. in              batting
  11896. in              elected
  11897. in              fashionable
  11898. in              favoured
  11899. in              in fashion
  11900. in              not out
  11901. in              playing
  11902. in              trendy
  11903. in              wearing
  11904. ina             princess
  11905. inch            island
  11906. ind             India
  11907. ine             oriental
  11908. ing             gin cocktail/sling
  11909. inn             local
  11910. insect          six-footer
  11911. intens          decimally
  11912. io              cry of triumph
  11913. io              joyful cry
  11914. io              maiden
  11915. io              ten
  11916. io              triumphant cry
  11917. ioc             dime (Amer.)
  11918. iom             Isle of Man
  11919. iom             island
  11920. iom             man
  11921. ion             number one returning
  11922. iota            Greek letter
  11923. ious            credit notes
  11924. ious            promises to pay
  11925. ip              trivial sum
  11926. ir              Iran
  11927. ira             illegal army
  11928. ira             terrorists
  11929. ire             anger
  11930. ire             rage
  11931. irl             Ireland
  11932. is              Iceland
  11933. is              ayes
  11934. is              ays
  11935. is              eyes
  11936. is              island
  11937. is              one's
  11938. isis            goddess
  11939. isis            river
  11940. isle            man
  11941. ism             doctrine
  11942. ism             theory
  11943. iss             exists
  11944. ist             first
  11945. it              Italian
  11946. it              sex appeal
  11947. it              the thing
  11948. iv              four
  11949. iv              ivy
  11950. iv              tea-time
  11951. ive             I have
  11952. ix              nine
  11953. j               Jack
  11954. j               Japan
  11955. j               heat
  11956. j               jay
  11957. j               joule
  11958. j               judge
  11959. j               justice
  11960. j               knave
  11961. j               one
  11962. j               pen
  11963. j               square root of -1
  11964. ja              Jamaica
  11965. ja              agreement - foreign (Ger.)
  11966. ja              yes (Ger.)
  11967. jack            sailor/salt/seaman
  11968. je              I, being French
  11969. je              In Paris, I
  11970. jo              little woman
  11971. jo              sweetheart
  11972. job             bookmaker
  11973. jock            Scot/Scotsman
  11974. joe             American soldier
  11975. jolly           Marine
  11976. jug             prison
  11977. k               Kay
  11978. k               Khmer Republic
  11979. k               Kirkpatrick
  11980. k               Koechel
  11981. k               Mozart's works
  11982. k               Scarlatti's works
  11983. k               constant
  11984. k               kappa
  11985. k               kelvin
  11986. k               kilo
  11987. k               king
  11988. k               knight
  11989. k               monarch
  11990. k               potassium
  11991. k               thousand
  11992. k               twenty
  11993. k               twenty thousand
  11994. k               two hundred and fifty
  11995. k               vitamin
  11996. ka              double *
  11997. ka              genius *
  11998. ka              individuality
  11999. key             opener
  12000. kg              cagey
  12001. kine            cattle
  12002. kine            neat
  12003. kine            ox
  12004. kish            graphite
  12005. kl              kale
  12006. km              kilometre
  12007. kn              cayenne
  12008. knee            bender
  12009. ko              decisive blow
  12010. ko              kick off
  12011. ko              knock out
  12012. ko              stunner
  12013. kr              krypton
  12014. kt              knight
  12015. kv              cave (beware)
  12016. kv              cavy
  12017. ky              Kentucky
  12018. l               Labour
  12019. l               Liberal
  12020. l               Luxembourg
  12021. l               angle
  12022. l               apprentice
  12023. l               corner
  12024. l               el
  12025. l               elevated railway
  12026. l               ell
  12027. l               fifty
  12028. l               fifty thousand
  12029. l               half century
  12030. l               hand
  12031. l               inductance
  12032. l               inexperienced driver
  12033. l               la(m)bda
  12034. l               lake
  12035. l               lambert
  12036. l               latin
  12037. l               latitude
  12038. l               league
  12039. l               learner
  12040. l               learning
  12041. l               left
  12042. l               length
  12043. l               licentiate
  12044. l               line
  12045. l               lira/lire
  12046. l               litre
  12047. l               long
  12048. l               lumen
  12049. l               luminance
  12050. l               many +
  12051. l               money
  12052. l               new driver
  12053. l               novice
  12054. l               number plate
  12055. l               one pound
  12056. l               overhead railway
  12057. l               port
  12058. l               pound
  12059. l               pupil
  12060. l               railway
  12061. l               side
  12062. l               sovereign
  12063. l               student
  12064. l               trainee
  12065. l               tyro
  12066. l               vitamin
  12067. la              Los Angeles
  12068. la              Louisiana
  12069. la              articles - French
  12070. la              articles - Italian
  12071. la              articles - Spanish
  12072. la              look *
  12073. la              note
  12074. la              the (Fr.)
  12075. la              the (Ital.)
  12076. la              the (Span.)
  12077. lab             Labour
  12078. lab             laboratory
  12079. lab             party
  12080. lab             politician
  12081. lab             science centre
  12082. lac             aircraftsman
  12083. lac             hundred thousand
  12084. lah             note
  12085. lakh            hundred thousand
  12086. lam             beat
  12087. lam             pound
  12088. lambda          Greek letter
  12089. lamed           Hebrew letter
  12090. lar             Libya
  12091. laud            archbishop
  12092. lay             song
  12093. lb              one pound
  12094. lb              pound
  12095. le              articles - French
  12096. le              the (Fr.)
  12097. lear            king
  12098. lee             general
  12099. leg             limb
  12100. leg             member
  12101. leg             on
  12102. leg             support
  12103. lei             flowers
  12104. lei             wreath *
  12105. lely            artist
  12106. les             articles - French
  12107. les             the French
  12108. let             allow(ed)
  12109. let             hindrance
  12110. let             permit(ted)
  12111. let             service
  12112. let             with a tenant
  12113. lewis           gun
  12114. lh              left hand
  12115. li              fifty one
  12116. lib             Liberal
  12117. lib             book
  12118. lib             party
  12119. lib             politician
  12120. limn            old paint
  12121. line            shipping company
  12122. ling            fish
  12123. ling            swimmer
  12124. lips            mouthpiece
  12125. lips            speakers
  12126. lit             drunk
  12127. lit             loaded
  12128. lit             settled
  12129. ll              ells
  12130. ll              els
  12131. ll              fifty pounds
  12132. ll              fifty-fifty
  12133. lner            old railway
  12134. lo              look
  12135. lo              see
  12136. loch            lake
  12137. log             maths function
  12138. log             record
  12139. los             articles - Spanish
  12140. loser           fabulous hare
  12141. lot             large amount
  12142. loti            first item in sale
  12143. lower           cow
  12144. lowing          neat sound
  12145. lp              long playing
  12146. lp              record
  12147. ls              ells
  12148. ls              els
  12149. lso             orchestra
  12150. lt              Lieutenant
  12151. lt              officer
  12152. lud             old king
  12153. lum             chimney (Scot.)
  12154. lv              meal ticket
  12155. m               Bond's boss
  12156. m               French man
  12157. m               Malta
  12158. m               Monday
  12159. m               em
  12160. m               lot
  12161. m               maiden
  12162. m               maiden over
  12163. m               male
  12164. m               man
  12165. m               many +
  12166. m               mare
  12167. m               mark
  12168. m               married
  12169. m               masculine
  12170. m               mass
  12171. m               master
  12172. m               measure
  12173. m               member
  12174. m               meso-
  12175. m               meta-
  12176. m               meter (metre)
  12177. m               midday
  12178. m               mile
  12179. m               modulus
  12180. m               monsieur
  12181. m               month
  12182. m               moon
  12183. m               motorway
  12184. m               mu
  12185. m               noon
  12186. m               roof
  12187. m               small square
  12188. m               spymaster
  12189. m               thousand
  12190. m               vitamin
  12191. ma              Master of Arts
  12192. ma              academic
  12193. ma              degree
  12194. ma              educated man
  12195. ma              graduate
  12196. ma              master
  12197. ma              mother
  12198. ma              old woman
  12199. ma              scholar
  12200. mab             fairy queen
  12201. mab             queen
  12202. mac             Scot/Scotsman
  12203. main            deep
  12204. main            sea
  12205. mal             bad French
  12206. mam             mother
  12207. man             Friday
  12208. man             fellow
  12209. man             fighter
  12210. man             hand
  12211. man             husband
  12212. man             island
  12213. man             piece (chess)
  12214. man             soldier
  12215. man             worker
  12216. manifesto       show-ring
  12217. mass            Massachusetts
  12218. mass            service
  12219. maxim           gun
  12220. may             can
  12221. mayo            tree ring
  12222. mb              doctor
  12223. mc              master of ceremonies
  12224. mc              medal
  12225. md              doctor
  12226. md              one thousand five hundred
  12227. me              first person
  12228. me              note
  12229. me              number one
  12230. men             people
  12231. men             soldiers
  12232. ment            intended
  12233. ment            meant
  12234. ment            on purpose
  12235. ment            understood
  12236. mer             sea (Fr.)
  12237. met             New York opera
  12238. met             police
  12239. mg              magnesium
  12240. mi              main road
  12241. mi              motorway
  12242. mi              note
  12243. mig             aeroplane
  12244. mill            economist
  12245. min             half minute
  12246. min             thirty seconds
  12247. ming            Chinese dynasty
  12248. ming            china
  12249. mini            car
  12250. miss            Mississippi
  12251. mm              French men
  12252. mm              ems
  12253. mm              medal
  12254. mn              Merchant Navy
  12255. mo              Missouri
  12256. mo              doctor
  12257. mo              half minute
  12258. mo              month
  12259. mo              second
  12260. mo              short time
  12261. mo              time
  12262. mo              way of working
  12263. mog             cat
  12264. mon             Monday
  12265. mon             Scot/Scotsman
  12266. moo             low
  12267. moo             neat sound
  12268. mos             months
  12269. moses           lawgiver
  12270. mot             car test
  12271. mot             test
  12272. moth            fly-by-night
  12273. mp              fairly quiet
  12274. mp              member
  12275. mp              member of parliament
  12276. mp              military police
  12277. mp              mounted police
  12278. mp              mountie(s)
  12279. mp              policeman
  12280. mp              politician
  12281. mp              representative
  12282. mph             rate
  12283. mph             speed
  12284. mps             chemist
  12285. mr              mister
  12286. ms              ems
  12287. ms              handwriting
  12288. ms              manuscript
  12289. ms              paper
  12290. ms              text
  12291. ms              writing
  12292. mss             manuscripts
  12293. mss             papers
  12294. mt              hill
  12295. mt              mountain
  12296. mtb             torpedo boat
  12297. mu              Greek character
  12298. mu              Greek letter
  12299. mum             mother
  12300. mum             quiet
  12301. mum             silence
  12302. mur             wall (Fr.)
  12303. mutt            dog
  12304. my              gracious me
  12305. n               Avogadro's number
  12306. n               Norway
  12307. n               born
  12308. n               bridge players
  12309. n               direction
  12310. n               en
  12311. n               gas - nitrogen
  12312. n               half an em
  12313. n               indefinite number
  12314. n               knight
  12315. n               midday
  12316. n               name
  12317. n               neper
  12318. n               neuter
  12319. n               new
  12320. n               newton
  12321. n               ninety
  12322. n               ninety thousand
  12323. n               nitrogen
  12324. n               noon
  12325. n               north(ern)
  12326. n               note
  12327. n               noun
  12328. n               nu
  12329. n               number
  12330. n               point
  12331. n               pole +
  12332. n               quarter
  12333. n               unfavourable aspect
  12334. n               unknown number
  12335. n               unlimited number
  12336. na              North America
  12337. na              no (Scot.)
  12338. na              not (Scot.)
  12339. na              sodium
  12340. nae             no (Scot.)
  12341. nae             not (Scot.)
  12342. nag             horse
  12343. nag             little horse
  12344. nat             born
  12345. nation          people
  12346. nation          race
  12347. national        horse race
  12348. nb              nota bene
  12349. nb              note
  12350. nco             non-commissioned officer
  12351. nco             sergeant
  12352. nd              North Dakota
  12353. nd              no date
  12354. ne              Durham area
  12355. ne              Humberside
  12356. ne              Tyneside
  12357. ne              born
  12358. ne              bridge opponents
  12359. ne              gas - neon
  12360. ne              neon
  12361. ne              north-east
  12362. ne              not (old word)
  12363. ne              quarter
  12364. neat            cattle
  12365. neat            ox
  12366. ned             donkey
  12367. ned             little horse
  12368. nee             born
  12369. nemo            submariner
  12370. ness            head
  12371. ness            loch
  12372. ness            point
  12373. net             capture
  12374. net             fabric
  12375. ney             Marshal
  12376. ng              no good
  12377. ni              Northern Ireland
  12378. ni              Ulster
  12379. ni              nickel
  12380. nib             writer
  12381. nick            prison
  12382. nie             close (old word)
  12383. nie             near (old word)
  12384. nil             love
  12385. nil             nothing
  12386. nitre           chemical
  12387. nitre           fertiliser
  12388. nj              New Jersey
  12389. nl              not clear
  12390. nl              not far
  12391. nl              not permitted
  12392. nn              ens
  12393. no              New Orleans
  12394. no              indefinite number
  12395. no              not out
  12396. no              number
  12397. no              play (Jap.)
  12398. no              refusal
  12399. noh             play (Jap.)
  12400. noi             leading
  12401. noi             no one
  12402. noi             number one
  12403. non             no (Fr.)
  12404. nose            informer
  12405. np              North Pole
  12406. np              pole
  12407. ns              bridge partners
  12408. ns              ens
  12409. ns              not specific
  12410. ns              partnership
  12411. nt              Holy Writ
  12412. nt              New Testament
  12413. nt              Northern Territories
  12414. nt              book(s)
  12415. nt              books
  12416. nt              good book
  12417. nt              part of Bible
  12418. nt              preservationists
  12419. nt              scriptures
  12420. nu              Greek character
  12421. nu              Greek letter
  12422. nu              name unknown
  12423. nu              unidentified
  12424. num             miners
  12425. number          anaesthetic
  12426. nun             bluetit
  12427. nurse           shark
  12428. nus             dawn
  12429. nus             students' (union)
  12430. nut             teachers
  12431. nv              envy
  12432. nw              Merseyside
  12433. nw              bridge opponents
  12434. nw              north-west
  12435. nw              quarter
  12436. ny              Gotham
  12437. ny              New York
  12438. o               Ohio
  12439. o               around
  12440. o               aught
  12441. o               bald patch
  12442. o               ball
  12443. o               blob
  12444. o               blood group
  12445. o               cavity
  12446. o               cipher
  12447. o               circle
  12448. o               circuit
  12449. o               circular letter
  12450. o               dial
  12451. o               disc
  12452. o               duck
  12453. o               egg
  12454. o               eleven
  12455. o               eleven thousand
  12456. o               empty
  12457. o               examination
  12458. o               full moon
  12459. o               gas - oxygen
  12460. o               globe
  12461. o               gulf
  12462. o               hole
  12463. o               hollow
  12464. o               hoop
  12465. o               loop
  12466. o               love
  12467. o               naught
  12468. o               nil
  12469. o               no
  12470. o               nothing
  12471. o               nought
  12472. o               oh
  12473. o               omicron
  12474. o               opening
  12475. o               orb
  12476. o               ortho-
  12477. o               ought
  12478. o               owe
  12479. o               oxygen
  12480. o               pellet
  12481. o               ring
  12482. o               round
  12483. o               spangle
  12484. oaks            horse race
  12485. oates           conspirator
  12486. ob              old boy
  12487. obe             award
  12488. obe             honour
  12489. obe             order
  12490. obit            dead/died
  12491. obit            final message
  12492. obit            last word
  12493. oc              commander
  12494. oc              officer commanding
  12495. odo             bishop
  12496. oe              old English
  12497. oe              old Etonian
  12498. offa            old king
  12499. og              ogee
  12500. og              own goal
  12501. og              soccer blunder
  12502. oic             officer in charge
  12503. ok              acceptable
  12504. ok              all right
  12505. ok              approval
  12506. ok              authorisation
  12507. ok              correct
  12508. ok              fine
  12509. ok              okay
  12510. old man         captain
  12511. ole             cry of delight
  12512. om              award
  12513. om              honour
  12514. om              order
  12515. omega           Greek letter
  12516. omega           final letter
  12517. omega           final word
  12518. omega           last letter
  12519. omitting        skipping
  12520. on              about
  12521. on              acting
  12522. on              being broadcast
  12523. on              leg
  12524. on              on the menu
  12525. on              operating
  12526. on              performing
  12527. on              playing
  12528. one             lunchtime
  12529. one             undivided
  12530. oo              duck's eggs
  12531. oo              ohs
  12532. oo              owes
  12533. oo              spectacles
  12534. oom             Dutch uncle
  12535. op              operation
  12536. op              opposite
  12537. op              opus
  12538. op              out of print
  12539. op              work
  12540. optic           seer
  12541. opus            work
  12542. or              alternative
  12543. or              alternatively
  12544. or              before *
  12545. or              gold
  12546. or              yellow
  12547. oral            examination
  12548. oral            test
  12549. ord             no way
  12550. ord             ring road
  12551. orion           hunter
  12552. orpheus         classical musician
  12553. os              Ordinary Seaman
  12554. os              big
  12555. os              large letters
  12556. os              ohs
  12557. os              old style
  12558. os              outsize
  12559. os              owes
  12560. os              sailor
  12561. os              very large
  12562. ost             east (Ger.)
  12563. ot              Holy Writ
  12564. ot              Old Testament
  12565. ot              book(s)
  12566. ot              good book
  12567. ot              occupational therapy
  12568. ot              part of Bible
  12569. ot              scriptures
  12570. ou              Open University
  12571. oui             agreement - foreign (Fr.)
  12572. oui             yes (Fr.)
  12573. ouija           Franco-German agreement
  12574. ous             nothing to America
  12575. ouse            river
  12576. out             away
  12577. out             not at home
  12578. out             unfashionable
  12579. over            maiden
  12580. ox              bull
  12581. ox              neat
  12582. oxonian         dark blue
  12583. oy              oh, why
  12584. oz              ounce
  12585. oz              small weight
  12586. oz              weight
  12587. oz              wizard place
  12588. p               Celt
  12589. p               Kelt
  12590. p               Portugal
  12591. p               copper
  12592. p               four hundred
  12593. p               four hundred thousand
  12594. p               page
  12595. p               park(ing)
  12596. p               participle
  12597. p               pawn
  12598. p               pea
  12599. p               pedal
  12600. p               pee
  12601. p               peg
  12602. p               penny
  12603. p               phosphorous
  12604. p               pi
  12605. p               piano
  12606. p               pint
  12607. p               poise
  12608. p               power
  12609. p               president
  12610. p               prince
  12611. p               quiet
  12612. p               small change
  12613. p               soft
  12614. p               softly
  12615. p               vitamin
  12616. pa              Panama
  12617. pa              father
  12618. pa              old man
  12619. pan             god
  12620. par             standard
  12621. para            Brazilian
  12622. para            airborne soldier
  12623. parent          source
  12624. pas             dance
  12625. pas             step
  12626. pate            head
  12627. pawn            piece (chess)
  12628. pawnbroker      uncle
  12629. pb              lead
  12630. pc              copper
  12631. pc              policeman
  12632. pe              Peru
  12633. pe              gym
  12634. pe              physical education
  12635. pe              training
  12636. peg             tee
  12637. pen             author
  12638. pen             enclosure
  12639. pen             prison
  12640. pen             writer
  12641. per             by
  12642. per             for each
  12643. per             through
  12644. pet             cherished
  12645. pet             favourite
  12646. pg              paying guest
  12647. ph              local
  12648. phi             Greek letter
  12649. pi              Greek character
  12650. pi              Greek letter
  12651. pi              confusion *
  12652. pi              good
  12653. pi              religious
  12654. pi              upright
  12655. pier            mole
  12656. pit             Hell
  12657. pitt            old Prime Minister
  12658. pl              Poland
  12659. pla             mountain retreat
  12660. pla             port authority
  12661. plato           philosopher
  12662. plo             illegal army
  12663. plo             terrorists
  12664. plot            garden
  12665. pm              Prime Minister
  12666. pm              afternoon
  12667. pm              half day
  12668. pm              in the afternoon
  12669. po              Italian flower
  12670. po              Post Office
  12671. po              airman
  12672. po              palladium
  12673. po              pole
  12674. po              river
  12675. polo            Merchant of Venice
  12676. poly            college
  12677. pony            twenty five pounds
  12678. pony            twenty-five pounds
  12679. pop             father
  12680. pop             old man
  12681. port            left
  12682. port            wine
  12683. pp              pages
  12684. pp              peas
  12685. pp              pees
  12686. pp              pianissimo
  12687. pp              very quiet/soft(ly)
  12688. pr              Puerto Rico
  12689. pr              Romans/Roman people
  12690. pr              electoral system
  12691. pr              image building
  12692. pr              president
  12693. pr              price
  12694. pr              prince
  12695. pr              public relations
  12696. pr              two
  12697. pra             academician
  12698. pra             artist
  12699. pra             painter
  12700. pres            president
  12701. prison          bird
  12702. pro             expert
  12703. pro             for
  12704. pro             public relations officer
  12705. prof            academic
  12706. provos          terrorists
  12707. ps              footnote
  12708. ps              peas
  12709. ps              pees
  12710. ps              postscript
  12711. ps              second thoughts
  12712. pt              part
  12713. pt              physical training
  12714. pt              pint
  12715. pt              platinum
  12716. pt              point
  12717. pt              post town
  12718. pt              stop
  12719. pt              training
  12720. pub             local
  12721. pv              peavy
  12722. q               Celt
  12723. q               Kelt
  12724. q               Quebec
  12725. q               Queensland
  12726. q               boat
  12727. q               cue
  12728. q               electrical charge
  12729. q               farthing
  12730. q               five hundred
  12731. q               five hundred thousand
  12732. q               koppa
  12733. q               ninety
  12734. q               ninety thousand
  12735. q               quality
  12736. q               queen
  12737. q               query
  12738. q               question
  12739. q               queue
  12740. q               quintal
  12741. q               rational numbers
  12742. qp              kewpie (doll)
  12743. qt              cutie
  12744. qt              quart
  12745. qt              quiet
  12746. qu              quart
  12747. qu              queen
  12748. quad            prison
  12749. que             what (Fr.)
  12750. qui             who (Fr.)
  12751. r               Reaumur
  12752. r               Regina
  12753. r               Republican
  12754. r               Rex
  12755. r               Romania
  12756. r               are
  12757. r               arithmetic
  12758. r               castle
  12759. r               eighty
  12760. r               eighty thousand
  12761. r               hand
  12762. r               king
  12763. r               monarch
  12764. r               month
  12765. r               queen
  12766. r               radius
  12767. r               rain
  12768. r               rand
  12769. r               reading
  12770. r               real numbers
  12771. r               recipe
  12772. r               resistance
  12773. r               rho
  12774. r               right
  12775. r               river
  12776. r               road
  12777. r               rontgen unit
  12778. r               rook
  12779. r               royal
  12780. r               run
  12781. r               side
  12782. r               take *
  12783. r               writing
  12784. ra              Royal Academician
  12785. ra              Royal Academy
  12786. ra              Royal Artillery
  12787. ra              academician
  12788. ra              academy
  12789. ra              artillery
  12790. ra              artist
  12791. ra              big guns
  12792. ra              gunmen
  12793. ra              gunner(s)
  12794. ra              painter
  12795. ra              soldiers
  12796. ra              sun (god)
  12797. rab             Butler
  12798. rac             motoring organisation
  12799. race            people
  12800. rada            academy
  12801. raf             fliers
  12802. raf             service
  12803. rage            fashion
  12804. rain            waterfall
  12805. ram             butter
  12806. ram             music school
  12807. ram             sheep
  12808. ran             managed
  12809. ran             smuggled
  12810. ras             head
  12811. ras             prince
  12812. rat             art nouveau
  12813. rat             desert fighter
  12814. rat             desert(er)
  12815. rat             scab
  12816. rat             strikebreaker
  12817. rate            speed
  12818. rc              Roman Catholic
  12819. rc              church
  12820. rd              little way
  12821. rd              road
  12822. rd              way
  12823. re              Royal Engineer(s)
  12824. re              about
  12825. re              again
  12826. re              concerning
  12827. re              engineer(s)
  12828. re              note
  12829. re              over
  12830. re              religious education
  12831. re              sapper(s)
  12832. re              soldiers
  12833. re              touching
  12834. rebecca         Welsh riots
  12835. rec             recipe
  12836. rec             take
  12837. red             anarchist
  12838. red             bloody
  12839. red             cent *
  12840. red             communist
  12841. red             leftist
  12842. red             revolutionary
  12843. red             socialist
  12844. regan           princess
  12845. regan           wicked sister
  12846. reine           French queen
  12847. reme            engineers
  12848. rene            French man
  12849. rep             agent
  12850. rep             salesman
  12851. rep             traveller
  12852. res             old thing
  12853. resh            Hebrew letter
  12854. ret             soak
  12855. rev             priest
  12856. rev             vicar
  12857. rex             cat
  12858. rg              argy(-bargy)
  12859. rh              right hand
  12860. ri              Rhode Island
  12861. ri              king emperor
  12862. rib             wife
  12863. rid             clear
  12864. rid             free
  12865. ring            circle
  12866. rio             port
  12867. rip             final message
  12868. rip             last word
  12869. river           banker
  12870. river           flower
  12871. river           runner
  12872. rly             lines/landline
  12873. rly             railway
  12874. rly             transport
  12875. rly             way
  12876. rm              Marine
  12877. rm              Royal Marine(s)
  12878. rm              jolly
  12879. rm              old Irish magistrate
  12880. rm              resident magistrate
  12881. rma             Sandhurst
  12882. rms             mailboat
  12883. rn              Navy
  12884. rn              Royal Navy
  12885. rn              fleet
  12886. rn              sailors
  12887. rn              service
  12888. ro              right hand
  12889. roc             fabulous bird
  12890. rock            diamond
  12891. rod             fast car
  12892. rod             pole
  12893. rod             sports car
  12894. roi             French king
  12895. roi             king (Fr.)
  12896. rom             gypsy
  12897. rose            flower
  12898. rosinante       poor horse
  12899. rot             corruption
  12900. rot             decay
  12901. rot             rubbish
  12902. rr              Right Reverend
  12903. rr              Rolls Royce
  12904. rr              ars
  12905. rr              bishop
  12906. rr              car
  12907. rsm             non-commissioned officer
  12908. rt              arty
  12909. rt              right
  12910. ru              football
  12911. ru              rugby
  12912. ruc             Irish police
  12913. rue             street (Fr.)
  12914. run             manage
  12915. run             smuggle
  12916. rur             Capek's play
  12917. rv              bible
  12918. ry              landline
  12919. ry              line(s)
  12920. ry              little way
  12921. ry              rail
  12922. ry              railway
  12923. ry              transport
  12924. ry              way
  12925. s               Bach's works
  12926. s               God's
  12927. s               Sabbath
  12928. s               Saturday
  12929. s               Schmieder
  12930. s               Sweden
  12931. s               as
  12932. s               bend
  12933. s               bob
  12934. s               bridge players
  12935. s               direction
  12936. s               dollar
  12937. s               es
  12938. s               ess
  12939. s               has
  12940. s               his
  12941. s               is
  12942. s               largesse
  12943. s               old Bob
  12944. s               old shilling
  12945. s               paragon
  12946. s               part of collar
  12947. s               point
  12948. s               pole +
  12949. s               quarter
  12950. s               saint
  12951. s               second
  12952. s               seven
  12953. s               seven thousand
  12954. s               shilling
  12955. s               side
  12956. s               siemens
  12957. s               sister
  12958. s               snow
  12959. s               society
  12960. s               son
  12961. s               south(ern)
  12962. s               space
  12963. s               spade
  12964. s               square
  12965. s               stokes
  12966. s               sulphur
  12967. s               sun
  12968. s               us
  12969. sa              South Africa
  12970. sa              South America
  12971. sa              essay
  12972. sa              it
  12973. sa              sex appeal
  12974. sa              without date
  12975. sad             blue
  12976. sailor          able seaman
  12977. saint           paragon
  12978. salt            able seaman
  12979. sam             uncle
  12980. sas             soldiers
  12981. sat             Saturday
  12982. satin           dressmaker
  12983. sc              namely
  12984. sc              self-contained
  12985. sc              small capitals
  12986. sc              specifically
  12987. sc              that is
  12988. sc              viz
  12989. scot            fine
  12990. scot            tax
  12991. scr             scruple
  12992. sculptor        blockbuster
  12993. sd              South Dakota
  12994. sd              without a day fixed
  12995. sdp             nationalists
  12996. se              Home Counties
  12997. se              London Area
  12998. se              bridge opponents
  12999. se              quarter
  13000. se              south-east
  13001. sea             deep
  13002. sea             main
  13003. sec             dry
  13004. sec             second
  13005. sec             short time
  13006. sec             time
  13007. see             look
  13008. seed            children
  13009. semi            half
  13010. semi            house
  13011. sent            ecstatic
  13012. set             group
  13013. set             put
  13014. seth            fourth man
  13015. sgt             non-commissioned officer
  13016. sgt             sergeant
  13017. sh              hush
  13018. sh              quiet
  13019. sh              silence
  13020. she             (the) woman
  13021. she             female
  13022. she             lady
  13023. she             novel
  13024. sherpa          mountaineer
  13025. si              South Island
  13026. si              agreement - foreign (Span.)
  13027. si              note
  13028. si              silicon
  13029. si              yes (Ital.)
  13030. si              yes (Span.)
  13031. sib             relation
  13032. sic             thus
  13033. side            team
  13034. silk            dressmaker
  13035. sin             err
  13036. sin             evil
  13037. sin             without
  13038. sin             wrong
  13039. sine            maths function
  13040. sine            without
  13041. sir             knight
  13042. sis             sister
  13043. sly             tinker
  13044. sm              French king
  13045. sm              French queen
  13046. sm              king (Fr.)
  13047. sm              non-commissioned officer
  13048. smith           economist
  13049. sn              Essen
  13050. sn              partnership
  13051. sn              tin
  13052. so              ergo
  13053. so              note
  13054. so              therefore
  13055. so              thus
  13056. so              well
  13057. soh             note
  13058. sol             note
  13059. sol             sun-god
  13060. som             county
  13061. some            approx(imately)
  13062. son             issue
  13063. sop             soprano
  13064. sos             appeal
  13065. sp              South Pole
  13066. sp              childless
  13067. sp              odds
  13068. sp              pole
  13069. sp              species
  13070. sp              starting price
  13071. sp              without children
  13072. spa             spring
  13073. spire           Oxford dreamer
  13074. spy             agent
  13075. spy             mole
  13076. square          unfashionable
  13077. sr              old railway
  13078. sr              senior
  13079. srn             nurse
  13080. ss              German soldier
  13081. ss              Sunday School
  13082. ss              liner
  13083. ss              saints
  13084. ss              ship
  13085. ss              steamship
  13086. st              good man
  13087. st              hush
  13088. st              little way
  13089. st              paragon
  13090. st              road
  13091. st              saint
  13092. st              silence
  13093. st              stone
  13094. st              street
  13095. st              stumped
  13096. st              thoroughfare
  13097. st              way
  13098. st              weight
  13099. stag            speculator
  13100. sten            gun
  13101. stet            don't change it
  13102. stir            prison
  13103. stop            traffic signal
  13104. sts             saints
  13105. sty             filthy place
  13106. stye            eyesore
  13107. su              Soviet Union
  13108. sub             U-boat
  13109. sub             stand-in
  13110. sub             substitute
  13111. sub             warship
  13112. supra           over
  13113. sure            certain
  13114. sw              Cornwall
  13115. sw              Devon
  13116. sw              bridge opponents
  13117. sw              quarter
  13118. sw              south-west
  13119. swift           screecher
  13120. swiss roll      jammed cylinder
  13121. sx              Essex
  13122. t               Thailand
  13123. t               Tuesday
  13124. t               bandage
  13125. t               bar
  13126. t               bone
  13127. t               cart
  13128. t               cloth
  13129. t               cross
  13130. t               crossed
  13131. t               half dry
  13132. t               hundred and sixty
  13133. t               hundred and sixty thousand
  13134. t               junction
  13135. t               model +
  13136. t               peg
  13137. t               perfect letter
  13138. t               plate
  13139. t               rail
  13140. t               shirt
  13141. t               short time
  13142. t               square
  13143. t               tau
  13144. t               te
  13145. t               tea
  13146. t               tee
  13147. t               tesla
  13148. t               the
  13149. t               time
  13150. t               ton(ne)
  13151. t               tritium
  13152. ta              Territorial Army
  13153. ta              army
  13154. ta              cheers
  13155. ta              reserves
  13156. ta              soldiers
  13157. ta              terriers
  13158. ta              territorials
  13159. ta              thank you
  13160. ta              thanks
  13161. ta              volunteers
  13162. tab             label
  13163. tace            silence
  13164. tag             label
  13165. tan             beat
  13166. tan             brown
  13167. tan             maths function
  13168. tar             able seaman
  13169. tar             art nouveau
  13170. tar             sailor/salt/seaman
  13171. tata            Tosti's song
  13172. tata            goodbye
  13173. tate            gallery
  13174. tau             cross
  13175. tay             river
  13176. tb              torpedo boat
  13177. td              medal
  13178. te              Lawrence
  13179. te              note
  13180. tea             leaves
  13181. tec             detective
  13182. ted             Edward
  13183. ted             Heath
  13184. tee             peg
  13185. teen            old injury
  13186. tees            river
  13187. tell            archer
  13188. temp            secretary
  13189. ten             PM's address
  13190. tene            old injury
  13191. tent            wine
  13192. ter             three (triple)
  13193. ter             thrice
  13194. test            educational journal
  13195. test            examination
  13196. test            match
  13197. teth            Hebrew letter
  13198. the             article
  13199. the             articles - English
  13200. ti              note
  13201. tic             note
  13202. tic             spasm
  13203. tic             twitching
  13204. tier            row
  13205. time            father
  13206. times           daily
  13207. timon           misanthrope
  13208. tin             can
  13209. tin             cash
  13210. tin             money
  13211. tin             vessel
  13212. tiny            small
  13213. tion            empty container
  13214. tit             bird
  13215. tit             inferior horse
  13216. tit             poor horse
  13217. tnt             big banger
  13218. tnt             explosive
  13219. tod             fox
  13220. todo            commotion
  13221. toe             extremity
  13222. toe             member
  13223. tom             big bell
  13224. tom             cat
  13225. tome            book
  13226. ton             fashion
  13227. ton             hundred
  13228. ton             large amount
  13229. ton             weight
  13230. tonne           weight
  13231. tor             hell
  13232. tor             hill
  13233. tor             mountain
  13234. tor             point
  13235. tor             prominence
  13236. tory            Conservative
  13237. tory            party
  13238. tory            politician
  13239. tp              teepee
  13240. tr              Turkey
  13241. tr              transaction
  13242. tr              translation
  13243. tram            transport
  13244. tree            actor
  13245. tres            very (Fr.)
  13246. tri             three (triple)
  13247. tri             thrice
  13248. troy            ancient city
  13249. troy            old city
  13250. try             attempt
  13251. try             essay
  13252. ts              teas
  13253. ts              tees
  13254. tt              abstaining
  13255. tt              dry
  13256. tt              on the wagon
  13257. tt              race
  13258. tt              teas
  13259. tt              tees
  13260. tt              teetotal
  13261. tt              teetotaller
  13262. tt              thank you
  13263. tu              tradesmen
  13264. tuck            friar
  13265. twelve          eec
  13266. two             company
  13267. u               Conservative
  13268. u               Uruguay
  13269. u               Utah
  13270. u               about turn
  13271. u               acceptable
  13272. u               bend
  13273. u               boat
  13274. u               educational establishment
  13275. u               ewe
  13276. u               film
  13277. u               for all to see
  13278. u               high class
  13279. u               on view to all
  13280. u               posh
  13281. u               socially acceptable
  13282. u               suitable for children
  13283. u               superior
  13284. u               trap
  13285. u               tube
  13286. u               turn
  13287. u               union/Unionist
  13288. u               universal
  13289. u               university
  13290. u               upper class
  13291. u               uppish
  13292. u               upsilon
  13293. u               uranium
  13294. u               yew
  13295. u               you
  13296. uc              you see
  13297. uk              United Kingdom
  13298. uk              this country
  13299. uk              this island
  13300. ule             rubber
  13301. ult             last month
  13302. um              doubt
  13303. um              hesitation
  13304. un              United Nations
  13305. un              international
  13306. un              number one (Fr.)
  13307. un              one
  13308. un              one (dialect)
  13309. un              peacekeepers
  13310. una             number one (Ital.)
  13311. unco            very (Scot.)
  13312. une             number one (Fr.)
  13313. uno             international organisation
  13314. uno             number one (Ital.)
  13315. up              at university
  13316. up              excited
  13317. up              in court
  13318. up              mounted
  13319. up              riding
  13320. up              superior
  13321. uq              you queue
  13322. ur              ancient city
  13323. ur              hesitation
  13324. ur              old city
  13325. ur              primitive
  13326. ur              you are
  13327. ure             river
  13328. uru             Uruguay
  13329. us              America
  13330. us              American
  13331. us              as above
  13332. us              ewes
  13333. us              no good
  13334. us              transatlantic
  13335. us              undersecretary
  13336. us              use
  13337. us              useless
  13338. us              yews
  13339. us              you and me
  13340. usa             America
  13341. use             application
  13342. use             custom
  13343. use             employ(ment)
  13344. use             practice
  13345. use             practise
  13346. ussr            Soviet Union
  13347. ut              note
  13348. ute             half minute
  13349. uu              ewes
  13350. uu              use
  13351. uu              yews
  13352. ux              wife
  13353. v               Vatican
  13354. v               against
  13355. v               agent
  13356. v               bomb
  13357. v               day
  13358. v               five
  13359. v               look
  13360. v               neck
  13361. v               neckline
  13362. v               notch
  13363. v               opposing
  13364. v               see
  13365. v               sign
  13366. v               vanadium
  13367. v               vee
  13368. v               velocity
  13369. v               verb
  13370. v               verse
  13371. v               versus
  13372. v               very
  13373. v               victory
  13374. v               vide
  13375. v               volt
  13376. v               volume
  13377. v               win
  13378. va              Virginia
  13379. vad             nurse
  13380. vale            farewell
  13381. vale            goodbye
  13382. vat             tax
  13383. vau             Hebrew letter
  13384. vb              verb
  13385. ve              victory
  13386. ver             rev up
  13387. very            light
  13388. vet             surgeon
  13389. vg              for example
  13390. vi              half dozen
  13391. vi              six
  13392. via             old way
  13393. vid             see
  13394. vid             tanner/sixpence
  13395. vide            look
  13396. vide            see
  13397. vin             French wine
  13398. vip             big noise
  13399. vip             tanner/sixpence
  13400. vir             man/Roman
  13401. vis             viscount
  13402. vj              victory
  13403. vo              left hand
  13404. vol             book
  13405. vol             volume
  13406. vy              various years
  13407. w               Wednesday
  13408. w               Welsh
  13409. w               William
  13410. w               bridge players
  13411. w               direction
  13412. w               point
  13413. w               quarter
  13414. w               tungsten
  13415. w               watt
  13416. w               weak
  13417. w               west(ern)
  13418. w               whole numbers
  13419. w               wicket
  13420. w               width
  13421. w               wife
  13422. w               woman
  13423. ward            disadvantage (drawback)
  13424. washington      young feller
  13425. we              partnership
  13426. we              you and I
  13427. wee             little
  13428. wee             minor
  13429. wee             small
  13430. who             doctor
  13431. wi              Mayfair
  13432. wi              West Indies
  13433. wi              Westminster
  13434. winner          fabulous tortoise
  13435. wise            youth leaders
  13436. wist            knew (old word)
  13437. women           monstrous regiment
  13438. woof            bark
  13439. wt              small weight
  13440. wt              weight
  13441. x               Christ
  13442. x               PM's address
  13443. x               Xmas
  13444. x               across
  13445. x               body
  13446. x               chi
  13447. x               chromosome
  13448. x               cross
  13449. x               draw
  13450. x               ex,Exe
  13451. x               film
  13452. x               illiterate's signature
  13453. x               kiss
  13454. x               particle
  13455. x               ray
  13456. x               sign of love
  13457. x               sign of the times
  13458. x               spot marked
  13459. x               ten
  13460. x               ten thousand
  13461. x               thousand
  13462. x               times
  13463. x               unknown
  13464. x               vitamin
  13465. x               vote
  13466. x               wrong sign
  13467. x               xi
  13468. xc              ninety
  13469. xi              eleven
  13470. xi              side
  13471. xi              team
  13472. xl              excel
  13473. xv              side
  13474. xv              team
  13475. y               alloy
  13476. y               chromosome
  13477. y               level
  13478. y               measure
  13479. y               moth
  13480. y               one hundred and fifty
  13481. y               one hundred and fifty thousand
  13482. y               track
  13483. y               unknown
  13484. y               why
  13485. y               yard
  13486. y               year
  13487. y               yen
  13488. y               young
  13489. y               yttrium
  13490. yard            detectives
  13491. yd              measure
  13492. ye              the (old word)
  13493. ye              you (old word)
  13494. yea             agreement
  13495. yew             tree
  13496. yr              year
  13497. yr              your
  13498. ys              wise
  13499. ys              youth leaders
  13500. yt              that (old word)
  13501. yu              jade
  13502. yule            you will, say
  13503. yy              wise
  13504. z               Zambia
  13505. z               bar
  13506. z               bend
  13507. z               cedilla
  13508. z               final letter
  13509. z               integers
  13510. z               izzard
  13511. z               last character
  13512. z               last letter
  13513. z               omega
  13514. z               seven
  13515. z               seven thousand
  13516. z               sound of sleep
  13517. z               zed
  13518. z               zee
  13519. z               zero
  13520. z               zeta
  13521. zo              cross *
  13522. zr              Zaire
  13523. zz              (sound of) snoring
  13524. ----------------------------------------------------------------------
  13525.  
  13526. --
  13527. Ross Beresford,   ! Email (trusted): rberesfo@cix.compulink.co.uk
  13528. 10 Wagtail Close, !          (work): ross@siesoft.co.uk
  13529. Twyford, Reading, !    (under test): ross@dickens.demon.co.uk
  13530. RG10 9ED, UK      !
  13531.  
  13532. ==> games/crosswords/cryptic/double.p <==
  13533. Each clue has two solutions, one for each diagram; one of the answers
  13534. to 1ac. determines which solutions are for which diagram.
  13535.  
  13536. All solutions are in Chamber's and Webster's Third except for one solution
  13537. of each of 1dn, 3dn and 4dn, which can be found in Webster's 2nd. edition.
  13538.  
  13539. #######################################################################
  13540. #1   !2   !    !    !3   !4   !5   #1   !2   !    !    !3   !4   !5   #
  13541. #    !    !    !    !    !    !    #    !    !    !    !    !    !    #
  13542. #----+----###########----#----#----#----+----###########----#----#----#
  13543. #6   !    !7   !    !    #    #    #6   !    !7   !    !    #    #    #
  13544. #    !    !    !    !    #    #    #    !    !    !    !    #    #    #
  13545. #----#----#----######----#----#----#----#----#----######----#----#----#
  13546. #    #    #    #8   !    !    !    #    #    #    #8   !    !    !    #
  13547. #    #    #    #    !    !    !    #    #    #    #    !    !    !    #
  13548. #----#----#----######----#----#----#----#----#----######----#----#----#
  13549. #9   !    !    !    #    #    #    #9   !    !    !    #    #    #    #
  13550. #    !    !    !    #    #    #    #    !    !    !    #    #    #    #
  13551. #----#----#----######----#----#----#----#----#----######----#----#----#
  13552. #    #    #10  !    !    !    !    #    #    #10  !    !    !    !    #
  13553. #    #    #    !    !    !    !    #    #    #    !    !    !    !    #
  13554. #----#----#----###########----+----#----#----#----###########----+----#
  13555. #11  !    !    !    !    !    !    #11  !    !    !    !    !    !    #
  13556. #    !    !    !    !    !    !    #    !    !    !    !    !    !    #
  13557. #######################################################################
  13558.  
  13559. Ac.
  13560. 1.  What can have distinctive looking heads spaced about more prominently
  13561.         right. (7)
  13562. 6.  Vermin that can overrun fish and t'English tor perhaps. (5)
  13563. 8.  Old testament reversal - Adam's conclusion, start of sin.
  13564.         Felines initially with everything there. (4)
  13565. 9.  Black initiated cut, oozed out naturally. (4)
  13566. 10. For instance, 11 with spleen dropping I count? (5)
  13567. 11. Provoked explosion of grenade. (7)
  13568.  
  13569. Dn.
  13570. 1.  Some of club taking part in theatrical function, for the equivalent
  13571.         of a fraction of a pound. (6)
  13572. 2.  Close-in light meter in one formation originally treated as limestone. (6)
  13573. 3.  Xingu River hombres having symmetrical shape. (5)
  13574. 4.  About sex-appeal measure - what waitresses should be? (6)
  13575. 5.  Old penny, least damaged, was preserved. (6)
  13576. 7.  IRA to harm ruling Englishman; extremes could be belonging to group. (5)
  13577.  
  13578. ==> games/crosswords/cryptic/double.s <==
  13579. +-+-+-+-+-+-+-+-+-+-+-+-+-+-+
  13580. !r e d c a p s!d e x t r a l!
  13581. + + +-+-+ + + + + +-+-+ + + +
  13582. !o t t e r!o!a!r o a c h!s!a!
  13583. + + + +-+ + + + + + +-+ + + +
  13584. !u!a!h!f a l l!a!z!m!t o m s!
  13585. + + + +-+ + + + + + +-+ + + +
  13586. !b l e d!r!i!t!c o o n!m!i!t!
  13587. + + + +-+ + + + + + +-+ + + +
  13588. !l!o!i r a t e!m!o!n o b l e!
  13589. + + + +-+-+ + + + + +-+-+ + +
  13590. !e n r a g e d!a n g e r e d!
  13591. +-+-+-+-+-+-+-+-+-+-+-+-+-+-+
  13592.  
  13593. Notes.
  13594. Left grid: Ac. 1. R + spaced (anag). 6. T'E tor (anag). 8. F-all. 9. B-led.
  13595. 10. I-rate. Dn. 1. Ro-ub-le. 2. T.A.L. in one (anag). 4. it in pole.
  13596. 5. anag of D+least. 7. anag of initial letters.
  13597. Right grid: Ac. 1. D-extra-L. 6. 3 mngs. 8. OT (rev) + m-s.
  13598. 9. initial letters. 10. No.-b(i)le. Dn. Dra-c-ma. 2. Zoo(m) in one (anag).
  13599. 3. hidden. 4. SA (rev) + mile. 5. anag of D+least. 7. anag of final letters.
  13600.  
  13601. --------------------------------------------------------------------
  13602.  
  13603. How I built it: it was hard]
  13604. Basically, I started with a couple of word pairs which were easy to clue
  13605. (e.g. enraged/angered - same meaning and anagrams of each other)
  13606. and built a grid around them, trying to ensure corresponding words
  13607. had something in common, either in meaning (their, among) or structure,
  13608. (EtalON, EOzooN) and making sure that there was at least one word
  13609. which could be used to distinguish the two grids (dextral).
  13610. The clues were built in one of two ways:
  13611. either the words had a common definition, and so a subsidiary indication
  13612. which could refer to either was needed; or it was necessary to define each
  13613. word in such a way that it was a subsidiary definition for all or part
  13614. of the corresponding word, and deal with any remaining parts as before.
  13615. I think the single hardest part was finding a definition of "interferometer"
  13616. which could also be interpreted as "zoo" or "ozo".
  13617.  
  13618.                                         Roy
  13619.                                         rt@ukc.ac.uk
  13620.  
  13621. ==> games/crosswords/cryptic/intro.p <==
  13622. What are the rules for cluing cryptic crosswords?
  13623. ==> games/crosswords/cryptic/intro.s <==
  13624. This is a brief set of instructions for solving cryptic crossword puzzles
  13625. for those of you who are intrigued by these puzzles, but haven't known how
  13626. to begin solving them.  For a more complete introduction, send a
  13627. self-addressed, stamped envelope to The Atlantic Puzzler, 745 Boylston
  13628. Street, Boston, Mass.  02116.
  13629.  
  13630. The characteristic common to all cryptic crossword puzzles is the format of
  13631. the clues.  Each clue is a miniature word puzzle consisting of a straight
  13632. definition of the answer and a cryptic definition of the answer.  For
  13633. example,
  13634.  
  13635.      Axle is poorly splined (7)
  13636.  
  13637. yields SPINDLE.  Axle is the straight definition.  The cryptic definition
  13638. (poorly splined) indicates an anagram of "splined".  The number in
  13639. parentheses is the number of letters in the answer.  Punctuation and
  13640. capitalization may be ignored in interpreting the clues.
  13641.  
  13642. There are only eight categories of clues, as follows:
  13643.  
  13644. 1. Anagram
  13645.  
  13646.    An anagram is a word formed by mixing up the letters of another word.  An
  13647.    anagram clue is indicated by some word that means "mixed up", for
  13648.    example, out, crazy, bizarre, insane, etc.  One or more words may
  13649.    contribute to the anagram.  For example:
  13650.  
  13651.       Tim goes insane from selfishness (7)
  13652.  
  13653.       for EGOTISM (anagram of "Tim goes")
  13654.  
  13655. 2. Double Definition
  13656.  
  13657.    A double definition is simply two definitions of the word.  Most two-word
  13658.    clues are double definitions.  For example:
  13659.  
  13660.       Release without charge (4)
  13661.  
  13662.       for FREE
  13663.  
  13664. 3. Container
  13665.  
  13666.    A container clue indicates that something is to be put in (or wrapped
  13667.    around) something else.  A container is indicated by phrases such as
  13668.    eaten by, contains, in, gobbles, etc.  For example:
  13669.  
  13670.       In Missouri, consumed by fear (7)
  13671.  
  13672.       for AMONGST (MO = Missouri in ANGST = fear)
  13673.  
  13674. 4. Hidden Word
  13675.  
  13676.    A hidden word is a word embedded in another word or words.  It is
  13677.    indicated by phrases such as spot in, hides, at the heart of, covers,
  13678.    etc.  For example:
  13679.  
  13680.       Worn spot in paper at typo (5)
  13681.  
  13682.       for RATTY (find ratty in "paper at typo")
  13683.  
  13684. 5. Reversal
  13685.  
  13686.    A reversal is a definition of a word with the letters reversed.  It is
  13687.    indicated by words such as back, reversed, up (for down clues), leftward
  13688.    (for across clues), etc.  For example:
  13689.  
  13690.    Egad] Ray entirely reversed the lot of cloth (7)
  13691.  
  13692.    for YARDAGE ("Egad] Ray" reversed)
  13693.  
  13694. 6. Homophone
  13695.  
  13696.    A homophone definition is a definition of a word that sounds the same as
  13697.    the answer, but is spelled differently.  A homophone is indicated by
  13698.    words such as in audience, I hear, mouthed, verbally, etc.  For example:
  13699.  
  13700.    Regrets prank, I hear (4)
  13701.  
  13702.    for RUES (the homophone is RUSE = prank)
  13703.  
  13704. 7. Charade
  13705.  
  13706.    In a charade, the pieces of the word are "spelled" out in order.  There
  13707.    are no auxiliary words that indicate a charade.  For example:
  13708.  
  13709.    Excite a jerk extremist (7)
  13710.  
  13711.    for FANATIC (FAN = excite, A, TIC = jerk)
  13712.  
  13713. 8. Deletion
  13714.  
  13715.    A deletion is a clue where you are instructed to remove a part of some
  13716.    word to make another word.  For example,
  13717.  
  13718.    Times with poor wages (4)
  13719.  
  13720.    for AGES (with-poor WAGES, where with is abbreviated by W)
  13721.  
  13722. Often the clue types are combined.  Some common examples are 1) hidden word
  13723. reversals where the answer is found backwards embedded in other words, and
  13724. 2) containers or charades where the parts are anagrams.  For example:
  13725.  
  13726.    Car shops have broken gear immersed in gasoline. (7)
  13727.  
  13728.    for GARAGES (RAGE = gear anagram in GAS = gasoline)
  13729.  
  13730. All manner of common abbreviations, acronyms, and other symbology such as
  13731. roman numerals are allowed.  For example:
  13732.  
  13733.    c    one hundred, cup, or centigrade
  13734.    vi   six
  13735.    h    hot
  13736.    s    small
  13737.    ca   california
  13738.  
  13739. Two punctuation marks at the end of the clue have been reserved for special
  13740. meaning.  A question mark (?) indicates that the straight clue is not
  13741. entirely straight (usually a pun).  For example:
  13742.  
  13743.    I tie down mascara holder soundly? (7)
  13744.  
  13745.    for EYELASH (homophone of "I lash", mascara holder is a punning
  13746.       definition of EYELASH)
  13747.  
  13748. An exclamation point (]) indicates that some part (usually all) of the clue
  13749. overlaps.  For example, the straight definition may also be the anagram
  13750. indicator.  Here is an example that entirely overlaps:
  13751.  
  13752.    A moped also has these] (6)
  13753.  
  13754.    for PEDALS (hidden word)
  13755.  
  13756. Here, the entire clue indicates the hidden word, but the entire clue is
  13757. also a straight definition of the answer.
  13758.  
  13759. Give it a try]  Cryptic crossword puzzles are a lot of fun.
  13760.  
  13761. -- Steve Koehler
  13762.    ucsd.edu]telesoft]koehler
  13763.    telesoft]koehler@ucsd.edu
  13764.    koehler@telesoft.com
  13765.  
  13766. ==> games/go-moku.p <==
  13767. For a game of k in a row on an n x n board,  for what values of k and n is
  13768. there a win?  Is (the largest such) k eventually constant or does it increase
  13769. with n?
  13770.  
  13771. ==> games/go-moku.s <==
  13772. Berlekamp, Conway, and Guy's _Winning_Ways_ reports proof that the
  13773. maximum k is between 4 and 7 inclusive, and it appears to be 5 or 6.
  13774. They report:
  13775.  
  13776. . 4-in-a-row is a draw on a 5x5 board (C. Y. Lee), but not on a 4x30
  13777.     board (C. Lustenberger).
  13778.  
  13779. . N-in-a-row is shown to be a draw on a NxN board for N>4, using a
  13780.     general pairing technique devised by A. W. Hales and R. I. Jewett.
  13781.  
  13782. . 9-in-a-row is a draw even on an infinite board, a 1954 result of H. O.
  13783.     Pollak and C. E. Shannon.
  13784.  
  13785. . More recently, the pseudonymous group T. G. L. Zetters showed that
  13786.     8-in-a-row is a draw on an infinite board, and have made some
  13787.     progress on showing infinite 7-in-a-row to be a draw.
  13788.  
  13789. Go-moku is 5-in-a-row played on a 19x19 go board.  It is apparently a
  13790. win for the first player, and so the Japanese have introduced several
  13791. 'handicaps' for the first player (e.g., he must win with _exactly_
  13792. five: 6-in-a-row doesn't count), but apparently the game is still a win
  13793. for the first player.  None of these apparent results have been
  13794. proven.
  13795.  
  13796. ==> games/hi-q.p <==
  13797. What is the quickest solution of the game Hi-Q (also called Solitair)?
  13798.  
  13799. For those of you who aren't sure what the game looks like:
  13800.  
  13801. 32 movable pegs ("+") are arranged on the following board such that
  13802. only the middle position is empty ("-"). Just to be complete: the board
  13803. consists of only these 33 positions.
  13804.  
  13805.           1 2 3 4 5 6 7
  13806.         1     + + +
  13807.         2     + + +
  13808.         3 + + + + + + +
  13809.         4 + + + - + + +
  13810.         5 + + + + + + +
  13811.         6     + + +
  13812.         7     + + +
  13813.  
  13814. A piece moves on this board by jumping over one of its immediate
  13815. neighboor (horizontally or vertically) into an empty space opposite.
  13816. The peg that was jumped over, is hit and removed from the board.  A
  13817. move can contain multiple hits if you use the same peg to make the
  13818. hits.
  13819.  
  13820. You have to end with one peg exactly in the middle position (44).
  13821.  
  13822. ==> games/hi-q.s <==
  13823. 1:      46*44
  13824. 2:      65*45
  13825. 3:      57*55
  13826. 4:      54*56
  13827. 5:      52*54
  13828. 6:      73*53
  13829. 7:      43*63
  13830. 8:      75*73*53
  13831. 9:      35*55
  13832. 10:     15*35
  13833. 11:     23*43*63*65*45*25
  13834. 12:     37*57*55*53
  13835. 13:     31*33
  13836. 14:     34*32
  13837. 15:     51*31*33
  13838. 16:     13*15*35
  13839. 17:     36*34*32*52*54*34
  13840. 18:     24*44
  13841.  
  13842. Found by Ernest Bergholt in 1912 and was proved to be minimal by John Beasley
  13843. in 1964.
  13844.  
  13845. References
  13846.         The Ins and Outs of Peg Solitaire
  13847.         John D Beasley
  13848.         Oxford U press, 1985
  13849.         ISBN 0-19-853203-2
  13850.  
  13851.         Winning Ways, Vol. 2, Ch. 23
  13852.         Berlekamp, E.R.
  13853.         Academic Press, 1982
  13854.         ISBN 01-12-091102-7
  13855.  
  13856. ==> games/jeopardy.p <==
  13857. What are the highest, lowest, and most different scores contestants
  13858. can achieve during a single game of Jeopardy?
  13859.  
  13860. ==> games/jeopardy.s <==
  13861. highest: $283,200.00, lowest: -$29,000.00, biggest difference: $309,700.00
  13862.  
  13863. (1) Our theoretical contestant has an itchy trigger finger, and rings in with
  13864.     an answer before either of his/her opponents.
  13865.  
  13866. (2) The daily doubles (1 in the Jeopardy] round, 2 in the Double Jeopardy]
  13867.     round) all appear under an answer in the $100 or $200 rows.
  13868.  
  13869. (3) All answers given by our contestant are (will be?) correct.
  13870.  
  13871. Therefore:
  13872.  
  13873. Round 1 (Jeopardy]): Max. score per category: $1500.
  13874.                      For 6 categories - $100 for the DD, that's $8900.
  13875.                      Our hero bets the farm and wins - score: $17,800.
  13876.  
  13877. Round 2 (Double Jeopardy]):
  13878.                      Max. score per category: $3000.
  13879.                      Assume that the DDs are found last, in order.
  13880.                      For 6 categories - $400 for both DDs, that's $17,600.
  13881.                      Added to his/her winnings in Round 1, that's $35,400.
  13882.                      After the 1st DD, where the whole thing is wagered,
  13883.                      the contestant's score is $70,800.  Then the whole
  13884.                      amount is wagered again, yielding a total of $141,600.
  13885.  
  13886. Round 3 (Final Jeopardy]):
  13887.                      Our (very greedy] :) hero now bets the whole thing, to
  13888.                      see just how much s/he can actually win.  Assuming that
  13889.                      his/her answer is right, the final amount would be
  13890.                      $283,200.
  13891.  
  13892. But the contestant can only take home $100,000; the rest is donated to
  13893. charity.
  13894.  
  13895. To calculate the lowest possible socre:
  13896.  
  13897. -1500 x 6 = -9000 + 100 = -8900.
  13898.  
  13899. On the Daily Double that appears in the 100 slot, you bet the maximum
  13900. allowed, 500, and lose. So after the first round, you are at -9400.
  13901.  
  13902. -3000 x 6 = -18000 + 400 = -17600
  13903.  
  13904. On the two Daily Doubles in the 200 slots, bet the maximum allowed, 1000. So
  13905. after the second round you are at -9400 + -19600 = -29000. This is the
  13906. lowest score you can achieve in Jeopardy before the Final Jeopardy round.
  13907.  
  13908. The caveat here is that you *must* be the person sitting in the left-most
  13909. seat (either a returning champion or the luckier of the three people who
  13910. come in after a five-time champion "retires") at the beginning of the game,
  13911. because otherwise you will not have control of the board when the first
  13912. Daily Double comes along.
  13913.  
  13914. ==> games/knight.tour.p <==
  13915. For what board sizes is a knight's tour possible?
  13916.  
  13917. ==> games/knight.tour.s <==
  13918. A tour exists for boards of size 1x1, 3x4, 3xN with N >= 7, 4xN with N >= 5,
  13919. and MxN with N >= M >= 5.  In other words, for all rectangles except 1xN
  13920. (excluding the trivial 1x1), 2xN, 3x3, 3x5, 3x6, 4x4.
  13921.  
  13922. With the exception of 3x8 and 4xN, any even-sized board which allows a tour
  13923. will also allow a closed (reentrant) tour.
  13924.  
  13925. On an odd-sided board, there is one more square of one color than
  13926. of the other.  Every time a knight moves, it moves to a square of
  13927. the other color than the one it is on.  Therefore, on an odd-sided
  13928. board, it must end the last move but one of the complete, reentrant
  13929. tour on a square of the same color as that on which it started.
  13930. It is then impossible to make the last move, for that move would end
  13931. on a square of the same color as it begins on.
  13932.  
  13933. Here is a solution for the 7x7 board (which is not reentrant).
  13934.      ------------------------------------
  13935.      ! 17 !  6 ! 33 ! 42 ! 15 !  4 ! 25 !
  13936.      ------------------------------------
  13937.      ! 32 ! 47 ! 16 !  5 ! 26 ! 35 ! 14 !
  13938.      ------------------------------------
  13939.      !  7 ! 18 ! 43 ! 34 ! 41 ! 24 !  3 !
  13940.      ------------------------------------
  13941.      ! 46 ! 31 ! 48 ! 27 ! 44 ! 13 ! 36 !
  13942.      ------------------------------------
  13943.      ! 19 !  8 ! 45 ! 40 ! 49 !  2 ! 23 !
  13944.      ------------------------------------
  13945.      ! 30 ! 39 ! 10 ! 21 ! 28 ! 37 ! 12 !
  13946.      ------------------------------------
  13947.      !  9 ! 20 ! 29 ! 38 ! 11 ! 22 !  1 !
  13948.      ------------------------------------
  13949.  
  13950. Here is a solution for the 5x5 board (which is not reentrant).
  13951.      --------------------------
  13952.      !  5 ! 10 ! 15 ! 20 !  3 !
  13953.      --------------------------
  13954.      ! 16 ! 21 !  4 !  9 ! 14 !
  13955.      --------------------------
  13956.      ! 11 !  6 ! 25 !  2 ! 19 !
  13957.      --------------------------
  13958.      ! 22 ! 17 !  8 ! 13 ! 24 !
  13959.      --------------------------
  13960.      !  7 ! 12 ! 23 ! 18 !  1 !
  13961.      --------------------------
  13962.  
  13963. Here is a reentrant 2x4x4 tour:
  13964.          0 11 16  3    15  4  1 22
  13965.         19 26  9 24     8 23 14 27
  13966.         10  5 30 17    31 12 21  2
  13967.         29 18 25  6    20  7 28 13
  13968. A reentrant 4x4x4 tour can be constructed by splicing two copies.
  13969.  
  13970. It shouldn't be much more work now to completely solve the problem of which 3D
  13971. rectangular boards allow tours.
  13972.  
  13973. ==> games/nim.p <==
  13974. Place 10 piles of 10 $1 bills in a row.  A valid move is to reduce
  13975. the last i>0 piles by the same amount j>0 for some i and j; a pile
  13976. reduced to nothing is considered to have been removed.  The loser
  13977. is the player who picks up the last dollar, and they must forfeit
  13978. half of what they picked up to the winner.
  13979.  
  13980. 1)  Who is the winner in Waldo Nim, the first or the second player?
  13981.  
  13982. 2)  How much more money than the loser can the winner obtain with best
  13983.     play on both parties?
  13984.  
  13985. ==> games/nim.s <==
  13986. For the particular game described we only need to consider positions for
  13987. which the following condition holds for each pile:
  13988.  
  13989.         (number of bills in pile k) + k >= (number of piles) + 1
  13990.  
  13991. A GOOD position is defined as one in which this condition holds,
  13992. with equality applying only to one pile P, and all piles following P
  13993. having the same number of bills as P.
  13994. ( So the initial position is GOOD, the special pile being the first. )
  13995. I now claim that if I leave you a GOOD position, and you make any move,
  13996. I can move back to a GOOD position.
  13997.  
  13998. Suppose there are n piles and the special pile is numbered (n-p+1)
  13999. (so that the last p piles each contain p bills).
  14000. (1) You take p bills from p or more piles;
  14001.   (a) If p = n, you have just taken the last bill and lost.
  14002.   (b) Otherwise I reduce pile (n-p) (which is now the last) to 1 bill.
  14003. (2) You take p bills from r(<p) piles;
  14004.     I take r bills from (p-r) piles.
  14005. (3) You take q(<p) bills from p or more piles;
  14006.     I take (p-q) bills from q piles.
  14007. (4) You take q(<p) bills from r(<p) piles;
  14008.   (a) q+r>p; I take (p-q) bills from (q+r-p) piles
  14009.   (b) q+r<=p; I take (p-q) bills from (q+r) piles
  14010.  
  14011. Verifying that each of the resulting positions is GOOD is tedious
  14012. but straightforward.  It is left as an exercise for the reader.
  14013.  
  14014.     -- RobH
  14015.  
  14016. ==> games/othello.p <==
  14017. How good are computers at Othello?
  14018.  
  14019. ==> games/othello.s <==
  14020. The interesting game in which computers are undoubted masters of all they
  14021. survey is Othello, where Kai-Fu Lee's (CMU) program "Bill" is so good it can
  14022. only play itself to learn to get better.  Bill has a fantastically
  14023. correct and efficient evaluation function, that recently has been further
  14024. improved by learning coefficients for additional terms made up of the
  14025. pair-wise combination of the four old terms.  This improved  the quality
  14026. of the play approximately as much as searching an extra two ply.
  14027.  
  14028. Bill is so good it can beat lots of players with no search at all.  Its
  14029. 6 or 7 ply search sweeps aside all opposition (though Kai-Fu says that some
  14030. very good players are now coming along in Japan, and he is not sure whether
  14031. Bill would beat them).  One interesting question remaining in Othello is
  14032. the game theoretic value of the starting position.  Bill's results seem
  14033. to indicate that the first player has an advantage.  It appears that,
  14034. since Kai-Fu has published all his evaluation material, someone could
  14035. build an Othello machine, and produce a constructive proof (as was done
  14036. for Cubic) that it is a win for the first player.
  14037.  
  14038. ==> games/risk.p <==
  14039. What are the odds when tossing dice in Risk?
  14040.  
  14041. ==> games/risk.s <==
  14042. Attacker using 3 dice, Defender using 2:
  14043.  
  14044.     Probability that Attacker wins 2 = 2323 / 7776
  14045.     Probability that Attacker wins 1 = 3724 / 7776
  14046.     Probability that Attacker wins 0 = 1729 / 7776
  14047.  
  14048. Attacker using 3 dice, Defender using 1:
  14049.  
  14050.     Probability that Attacker wins 1 = 855 / 1296
  14051.     Probability that Attacker wins 0 = 441 / 1296
  14052.  
  14053. Attacker using 2 dice, Defender using 2:
  14054.  
  14055.     Probability that Attacker wins 2 = 225 / 1296
  14056.     Probability that Attacker wins 1 = 630 / 1296
  14057.     Probability that Attacker wins 0 = 441 / 1296
  14058.  
  14059. Attacker using 2 dice, Defender using 1:
  14060.  
  14061.     Probability that Attacker wins 1 = 125 / 216
  14062.     Probability that Attacker wins 0 = 91 / 216
  14063.  
  14064. Attacker using 1 dice, Defender using 2:
  14065.  
  14066.     Probability that Attacker wins 1 = 90 / 216
  14067.     Probability that Attacker wins 0 = 126 / 216
  14068.  
  14069. Attacker using 1 dice, Defender using 1:
  14070.  
  14071.     Probability that Attacker wins 1 = 15 / 36
  14072.     Probability that Attacker wins 0 = 21 / 36
  14073.  
  14074. ==> games/rubiks.clock.p <==
  14075. How do you quickly solve Rubik's clock?
  14076.  
  14077. ==> games/rubiks.clock.s <==
  14078.                           Solution to Rubik's Clock
  14079.  
  14080. The solution to Rubik's Clock is very simple and the clock can be
  14081. "worked" in 10-20 seconds once the solution is known.
  14082.  
  14083. In this description of how to solve the clock I will describe
  14084. the different clocks as if they were on a map (e.g. N,NE,E,SE,S,SW,W,NW);
  14085. this leaves the middle clock which I will just call M.
  14086. To work the Rubik's clock choose one side to start from; it does
  14087. not matter from which side you start.  Your initial goal
  14088. will be to align the N,S,E,W and M clocks.  Use the following algorithm
  14089. to do this:
  14090.  
  14091.         ▌1¿  Start with all buttons in the OUT position.
  14092.  
  14093.         ▌2¿  Choose a N,S,E,W clock that does not already have the
  14094.              same time as M (i.e. not aligned with M).
  14095.  
  14096.         ▌3¿  Push in the closest two buttons to the clock you chose in ▌2¿.
  14097.  
  14098.         ▌4¿  Using the knobs that are farest away from the clock you chose in
  14099.              ▌2¿ rotate the knob until M and the clock you chose are aligned.
  14100.              The time on the clocks at this point does not matter.
  14101.  
  14102.         ▌5¿  Go back to ▌1¿ until N,S,E,W and M are in alignment.
  14103.  
  14104.         ▌6¿  At this point N,S,E,W and M should all have the same time.
  14105.              Make sure all buttons are out and rotate any knob
  14106.              until N,S,E,W and M are pointing to 12 oclock.
  14107.  
  14108. Now turn the puzzle over and repeat steps ▌1¿-▌6¿ for this side.  DO NOT
  14109. turn any knobs other than the ones described in ▌1¿-▌6¿.  If you have
  14110. done this correctly then on both sides of the puzzle N,S,E,W and M will
  14111. all be pointing to 12.
  14112.  
  14113. Now to align NE,SE,SW,NW.  To finish the puzzle you only need to work from
  14114. one side.  Choose a side and use the following algorithm  to align the
  14115. corners:
  14116.  
  14117.         ▌1¿  Start with all buttons OUT on the side you're working from.
  14118.  
  14119.         ▌2¿  Choose a corner that is not aligned.
  14120.  
  14121.         ▌3¿  Press the button closest to that corner in.
  14122.  
  14123.         ▌4¿  Using any knob except for that corner's knob rotate all the
  14124.              clocks until they are in line with the corner clock.
  14125.              (Here "all the clocks" means N,S,E,W,M and any other clock
  14126.              that you have already aligned)
  14127.              There is no need at this point to return the clocks to 12
  14128.              although if it is less confusing you can.  Remember to
  14129.              return all buttons to their up position before you do so.
  14130.  
  14131.         ▌5¿  Return to ▌1¿ until all clocks are aligned.
  14132.  
  14133.         ▌6¿  With all buttons up rotate all the clocks to 12.
  14134.  
  14135. ==> games/rubiks.cube.p <==
  14136. What is known about bounds on solving Rubik's cube?
  14137.  
  14138. ==> games/rubiks.cube.s <==
  14139. The "official" world record was set by Minh Thai at the 1982 World
  14140. Championships in Budapest Hungary, with a time of 22.95 seconds.
  14141.  
  14142. Keep in mind mathematicians provided standardized dislocation patterns
  14143. for the cubes to be randomized as much as possible.
  14144.  
  14145. The fastest cube solvers from 19 different countries had 3 attempts each
  14146. to solve the cube as quickly as possible.   Minh and several others have
  14147. unofficially solved the cube in times between 16 and 19 seconds.
  14148. However, Minh averages around 25 to 26 seconds after 10 trials, and by
  14149. best average of ten trials is about 27 seconds (although it is usually
  14150. higher).
  14151.  
  14152. Consider that in the World Championships 19 of the world's fastest cube
  14153. solvers each solved the cube 3 times and no one solved the cube in less
  14154. than 20 seconds...
  14155.  
  14156. God's algorithm is the name given to an as yet (as far as I know)
  14157. undiscovered method to solve the rubik's cube in the least number of moves;
  14158. as apposed to using 'canned' moves.
  14159.  
  14160. The known lower bound is 18 moves. This is established by looking at things
  14161. backwards: suppose we can solve a position in N moves. Then by running the
  14162. solution backwards, we can also go from the solved position to the position
  14163. we started with in N moves. Now we count how many sequences of N moves there
  14164. are from the starting position, making certain that we don't turn the same
  14165. face twice in a row:
  14166.  
  14167.   N=0: 1 (empty) sequence;
  14168.   N=1: 18 sequences (6 faces can be turned, each in 3 different ways)
  14169.   N=2: 18*15 sequences (take any sequence of length 1, then turn any of the
  14170.        five faces which is not the last face turned, in any of 3 different
  14171.        ways);
  14172.   N=3: 18*15*15 sequences (take any sequence of length 2, then turn any of
  14173.        the five faces which is not the last face turned, in any of 3
  14174.        different ways);
  14175.   :
  14176.   :
  14177.   N=i: 18*15^(i-1) sequences.
  14178.  
  14179. So there are only 1 + 18 + 18*15 + 18*15^2 + ... + 18*15^(n-1) sequences of
  14180. moves of length n or less. This sequence sums to (18/14)*(15^n - 1) + 1.
  14181. Trying particular values of n, we find that there are about 8.4 * 10^18
  14182. sequences of length 16 or less, and about 1.3 times 10^20 sequences of
  14183. length 17 or less.
  14184.  
  14185. Since there are 2^10 * 3^7 * 8] * 12], or about 4.3 * 10^19, possible
  14186. positions of the cube, we see that there simply aren't enough sequences of
  14187. length 16 or less to reach every position from the starting position. So not
  14188. every position can be solved in 16 or less moves - i.e. some positions
  14189. require at least 17 moves.
  14190.  
  14191. This can be improved to 18 moves by being a bit more careful about counting
  14192. sequences which produce the same position. To do this, note that if you turn
  14193. one face and then turn the opposite face, you get exactly the same result as
  14194. if you'd done the two moves in the opposite order. When counting the number
  14195. of essentially different sequences of N moves, therefore, we can split into
  14196. two cases:
  14197.  
  14198. (a) Last two moves were not of opposite faces. All such sequences can be
  14199.     obtained by taking a sequence of length N-1, choosing one of the 4 faces
  14200.     which is neither the face which was last turned nor the face opposite
  14201.     it, and choosing one of 3 possible ways to turn it. (If N=1, so that the
  14202.     sequence of length N-1 is empty and doesn't have a last move, we can
  14203.     choose any of the 6 faces.)
  14204.  
  14205. (b) Last two moves were of opposite faces. All such sequences can be
  14206.     obtained by taking a sequence of length N-2, choosing one of the 2
  14207.     opposite face pairs that doesn't include the last face turned, and
  14208.     turning each of the two faces in this pair (3*3 possibilities for how it
  14209.     was turned). (If N=2, so that the sequence of length N-2 is empty and
  14210.     doesn't have a last move, we can choose any of the 3 opposite face
  14211.     pairs.)
  14212.  
  14213. This gives us a recurrence relation for the number X_N of sequences of
  14214. length N:
  14215.  
  14216.   N=0: X_0                               = 1 (the empty sequence)
  14217.   N=1: X_1 = 18 * X_0                    = 18
  14218.   N=2: X_2 = 12 * X_1     + 27 * X_0     = 243
  14219.   N=3: X_3 = 12 * X_2     + 18 * X_1     = 3240
  14220.   :
  14221.   :
  14222.   N=i: X_i = 12 * X_(i-1) + 18 * X_(i-2)
  14223.  
  14224. If you do the calculations, you find that X_0 + X_1 + X_2 + ... + X_17 is
  14225. about 2.0 * 10^19. So there are fewer essentially different sequences of
  14226. moves of length 17 or less than there are positions of the cube, and so some
  14227. positions require at least 18 moves.
  14228.  
  14229. The upper bound of 50 moves is I believe due to Morwen Thistlethwaite, who
  14230. developed a technique to solve the cube in a maximum of 50 moves. It
  14231. involved a descent through a chain of subgroups of the full cube group,
  14232. starting with the full cube group and ending with the trivial subgroup (i.e.
  14233. the one containing the solved position only). Each stage involves a careful
  14234. examination of the cube, essentially to work out which coset of the target
  14235. subgroup it is in, followed by a table look-up to find a sequence to put it
  14236. into that subgroup. Needless to say, it was not a fast technique]
  14237.  
  14238. But it was fascinating to watch, because for the first three quarters or so
  14239. of the solution, you couldn't really see anything happening - i.e. the
  14240. position continued to appear random] If I remember correctly, one of the
  14241. final subgroups in the chain was the subgroup generated by all the double
  14242. twists of the faces - so near the end of the solution, you would suddenly
  14243. notice that each face only had two colours on it. A few moves more and the
  14244. solution was complete. Completely different from most cube solutions, in
  14245. which you gradually see order return to chaos: with Morwen's solution, the
  14246. order only re-appeared in the last 10-15 moves.
  14247.  
  14248. With God's algorithm, of course, I would expect this effect to be even more
  14249. pronounced: someone solving the cube with God's algorithm would probably
  14250. look very much like a film of someone scrambling the cube, run in reverse]
  14251.  
  14252. Finally, something I'd be curious to know in this context: consider the
  14253. position in which every cubelet is in the right position, all the corner
  14254. cubelets are in the correct orientation, and all the edge cubelets are
  14255. "flipped" (i.e. the only change from the solved position is that every edge
  14256. is flipped). What is the shortest sequence of moves known to get the cube
  14257. into this position, or equivalently to solve it from this position? (I know
  14258. of several sequences of 24 moves that do the trick.)
  14259.  
  14260. The reason I'm interested in this particular position: it is the unique
  14261. element of the centre of the cube group. As a consequence, I vaguely suspect
  14262. (I'd hardly like to call it a conjecture :-) it may lie "opposite" the
  14263. solved position in the cube graph - i.e. the graph with a vertex for each
  14264. position of the cube and edges connecting positions that can be transformed
  14265. into each other with a single move. If this is the case, then it is a good
  14266. candidate to require the maximum possible number of moves in God's
  14267. algorithm.
  14268.  
  14269.     -- David Seal dseal@armltd.co.uk
  14270.  
  14271. To my knowledge, no one has ever demonstrated a specific cube position
  14272. that takes 15 moves to solve.  Furthermore, the lower bound is known to
  14273. be greater than 15, due to a simple proof.
  14274.  
  14275. The way we know the lower bound is by working backwards counting how
  14276. many positions we can reach in a small number of moves from the solved
  14277. position.  If this is less than 43,252,003,274,489,856,000 (the total
  14278. number of positions of Rubik's cube) then you need more than that
  14279. number of moves to reach the other positions of the cube.  Therefore,
  14280. those positions will require more moves to solve.
  14281.  
  14282. The answer depends on what we consider a move.  There are three common
  14283. definitions.  The most restrictive is the QF metric, in which only a
  14284. quarter-turn of a face is allowed as a single move.  More common is
  14285. the HF metric, in which a half-turn of a face is also counted as a
  14286. single move.  The most generous is the HS metric, in which a quarter-
  14287. turn or half-turn of a central slice is also counted as a single move.
  14288. These metrics are sometimes called the 12-generator, 18-generator, and
  14289. 27-generator metrics, respectively, for the number of primitive moves.
  14290. The definition does not affect which positions you can get to, or even
  14291. how you get there, only how many moves we count for it.
  14292.  
  14293. The answer is that even in the HS metric, the lower bound is 16,
  14294. because at most 17,508,850,688,971,332,784 positions can be reached
  14295. within 15 HS moves.  In the HF metric, the lower bound is 18, because
  14296. at most 19,973,266,111,335,481,264 positions can be reached within 17
  14297. HF moves.  And in the QT metric, the lower bound is 21, because at
  14298. most 39,812,499,178,877,773,072 positions can be reached within 20 QT
  14299. moves.
  14300.  
  14301.     -- jjfink@skcla.monsanto.com writes:
  14302.  
  14303.  
  14304. Lately in this conference I've noted several messages related to Rubik's
  14305. Cube and Square 1. I've been an avid cube fanatic since 1981 and I've
  14306. been gathering cube information since.
  14307.  
  14308. Around Feb. 1990 I started to produce the Domain of the Cube Newsletter,
  14309. which focuses on Rubik's Cube and all the cube variants produced to
  14310. date. I include notes on unproduced prototype cubes which don't even
  14311. exist, patent information, cube history (and prehistory), computer
  14312. simulations of puzzles, etc. I'm up to the 4th issue.
  14313.  
  14314. Anyways, if you're interested in other puzzles of the scramble by
  14315. rotation type you may be interested in DOTC. It's available free to
  14316. anyone interested. I am especially interested in contributing articles
  14317. for the newsletter, e.g. ideas for new variants, God's Algorithm.
  14318.  
  14319. Anyone ever write a Magic Dodecahedron simulation for a computer? Anyone
  14320. understand Morwen Thistlethwaite's 50 move solution to Rubik's Cube? I'd
  14321. love to hear from you.
  14322.  
  14323. Drop me a SASE (say empire size) if you're interested in DOTC or if you
  14324. would like to exchange notes on Rubik's Cube, Square 1 etc.
  14325.  
  14326. I'm also interested in exchanging puzzle simulations, e.g. Rubik's Cube,
  14327. Twisty Torus, NxNxN Simulations, etc, for Amiga and IBM computers. I've
  14328. written several Rubik's Cube solving programs, and I'm trying to make
  14329. the definitive puzzle solving engine. I'm also interested in AI programs
  14330. for Rubik's Cube and the like.
  14331.  
  14332. Ideal Toy put out the Rubik's Cube Newsletter, starting with
  14333. issue #1 on May 1982. There were 4 issues in all, and I'm missing
  14334. #3.
  14335.  
  14336. I have:    #1, May 1982
  14337.            #2, Aug 1982
  14338.            #3, Aug 1983
  14339.  
  14340. I am willing to trade photocopies with anyone to obtain #3.
  14341.  
  14342. There was another sort of magazine, published in several languages
  14343. called Rubik's Logic and Fantasy in space. I believe there were 8
  14344. issues in all. Unfortunately I don't have any of these] I'm willing
  14345. to buy these off anyone interesting in selling. I would like to get the
  14346. originals if at all possible...
  14347.  
  14348. I'm also interested in buying any books on the cube or related puzzles.
  14349. In particular I am _very_ interested in obtaining the following:
  14350.  
  14351. Cube Games                               Don Taylor, Leanne Rylands
  14352. Official Solution to Alexander's Star    Adam Alexander
  14353. The Amazing Pyraminx                     Dr. Ronald Turner-Smith
  14354. The Winning Solution                     Minh Thai
  14355. The Winning Solution to Rubik's Revenge  Minh Thai
  14356. Simple Solutions to Cubic Puzzles        James G. Nourse
  14357.  
  14358. I'm also interested in buying puzzles of the mechanical type.
  14359. I'm still missing the Pyraminx Star (basically a Pyraminx with more tips
  14360. on it), the Puck, and Hungarian Rings.
  14361.  
  14362. If anyone out here is a fellow collector I'd like to hear from you.
  14363. If you have a cube variant which you think is rare, or an idea for a
  14364. cube variant we could swap notes.
  14365.  
  14366. I'm in the middle of compiling an exhaustive library for computer
  14367. simulations of puzzles. This includes simulations of all Uwe Meffert's
  14368. puzzles which he prototyped but _never_ produced. In fact, I'm in the
  14369. middle of working on a Pyraminx Hexagon solver. What? Never heard of it?
  14370. Meffert did a lot of other puzzles which never were made.
  14371.  
  14372. I invented some new "scramble by rotation" puzzles myself. My favourite
  14373. creation is the Twisty Torus. It is a torus puzzle with segments (which
  14374. slide around 360 degrees) with multiple rings around the circumference.
  14375.  
  14376. The computer puzzle simulation library I'm forming will be described
  14377. in depth in DOTC #4 (The Domain of the Cube Newsletter). So if you
  14378. have any interesting computer puzzle programs please email me and
  14379. tell me all about them]
  14380.  
  14381. Also to the people interested in obtaining a subscription to DOTC,
  14382. who are outside of Canada (which it seems is just about all of you])
  14383. please don't send U.S. or non-Canadian stamps (yeah, I know I said
  14384. Self-Addressed Stamped Envelope before). Instead send me an
  14385. international money order in Canadian funds for $6. I'll send you
  14386. the first 4 issues (issue #4 is almost finished).
  14387.  
  14388. Mark Longridge
  14389. Address: 259 Thornton Rd N, Oshawa Ontario Canada, L1J 6T2
  14390. Email:   mark.longridge@canrem.com
  14391.  
  14392. One other thing, the six bucks is not for me to make any money. This
  14393. is only to cover the cost of producing it and mailing it. I'm
  14394. just trying to spread the word about DOTC and to encourage other
  14395. mechanical puzzle lovers to share their ideas, books, programs and
  14396. puzzles. Most of the programs I've written and/or collected are
  14397. shareware for C64, Amiga and IBM. I have source for all my programs
  14398. (all in C or Basic) and I am thinking of providing a disk with the
  14399. 4th issue of DOTC. If the response is favourable I will continue
  14400. to provide disks with DOTC.
  14401.  
  14402.     -- Mark Longridge <mark.longridge@canrem.com> writes:
  14403.  
  14404. It may interest people to know that in the latest issue of "Cubism For Fun" %
  14405. (# 28 that I just received yesterday) there is an article by Herbert Kociemba
  14406. from Darmstadt.  He describes a program that solves the cube.  He states that
  14407. until now he has found no configuration that required more than 21 turns to
  14408. solve.
  14409.  
  14410. He gives a 20 move manoeuvre to get at the "all edges flipped/
  14411. all corners twisted" position:
  14412.         DF^2U'B^2R^2B^2R^2LB'D'FD^2FB^2UF'RLU^2F'
  14413. or in Varga's parlance:
  14414.         dofitabiribirilobadafodifobitofarolotifa
  14415.  
  14416. Other things #28 contains are an analysis of Square 1, an article about
  14417. triangular tilings by Martin Gardner, and a number of articles about other
  14418. puzzles.
  14419. --
  14420. %  CFF is a newsletter published by the Dutch Cubusts Club NKC.
  14421. Secretary:
  14422.         Anneke Treep
  14423.         Postbus 8295
  14424.         6710 AG  Ede
  14425.         The Netherlands
  14426. Membership fee for 1992 is DFL 20 (about$ 11).
  14427. --
  14428.     -- dik t. winter <dik@cwi.nl>
  14429.  
  14430. References:
  14431.  
  14432. E. C. Turner & K. F. Gold, "Rubik's Groups", American Mathematical Monthly,
  14433.    vol. 92 (1985), pp. 617-629.
  14434.  
  14435. Cubelike Puzzles - What Are They and How Do You Solve Them?
  14436. J.A. Eidswick   A.M.M. March, 1986
  14437.  
  14438. Rubik's Revenge: The Group Theoretical Solution
  14439. Mogens Esrom Larsen   A.M.M. June-July, 1985
  14440.  
  14441. The Group of the Hungarian Magic Cube
  14442. Chris Rowley   Proceedings of the First Western Austrialian
  14443.                 Conference on Algebra, 1982
  14444.  
  14445. Rubik's Cubic Compendium
  14446. Erno Rubik, Tamas Varga, et al
  14447. (Ed by David Singmaster)
  14448. Oxford University Press, 1987
  14449. (Some chapters on mathematics of the cube.)
  14450.  
  14451. David Singmaster, _Notes on Rubik's `Magic Cube'_
  14452.  
  14453. "Winning Ways"
  14454. by
  14455. Berlekamp, Elwyn R.
  14456. Conway, John H.
  14457. Guy, Richard K.
  14458. Volume two, pages 760-768, 808, 809
  14459.  
  14460. ==> games/rubiks.magic.p <==
  14461. How do you solve Rubik's Magic?
  14462.  
  14463. ==> games/rubiks.magic.s <==
  14464. The solution is in a 3x3 grid with a corner missing.
  14465.  
  14466. +---+---+---+          +---+---+---+---+
  14467. ! 3 ! 5 ! 7 !          ! 1 ! 3 ! 5 ! 7 !
  14468. +---+---+---+          +---+---+---+---+
  14469. ! 1 ! 6 ! 8 !          ! 2 ! 4 ! 6 ! 8 !
  14470. +---+---+---+          +---+---+---+---+
  14471. ! 2 ! 4 !              Original Shape
  14472. +---+---+
  14473.  
  14474. To get the 2x4 "standard" shape into this shape, follow this:
  14475. 1.  Lie it flat in front of you (4 going across).
  14476. 2.  Flip the pair (1,2) up and over on top of (3,4).
  14477. 3.  Flip the ONE square (2) up and over (1).
  14478. ▌Note:  if step 3 won't go, start over, but flip the entire original shape
  14479.         over (exposing the back).¿
  14480. 4.  Flip the pair (2,4) up and over on top of (5,6).
  14481. 5.  Flip the pair (1,2) up and toward you on top of (blank,4).
  14482. 6.  Flip the ONE square (2) up and left on top of (1).
  14483. 7.  Flip the pair (2,4) up and toward you.
  14484.  
  14485. Your puzzle won't be completely solved, but this is how to get the shape.
  14486. Notice that 3,5,6,7,8 don't move.
  14487.  
  14488. ==> games/scrabble.p <==
  14489. What are some exceptional scrabble games?
  14490.  
  14491. ==> games/scrabble.s <==
  14492. The shortest scrabble game:
  14493.  
  14494. The Scrabble Players News, Vol. XI No. 49, June 1983, contributed by
  14495. Kyle Corbin of Raleigh, NC:
  14496.  
  14497.                  ▌J¿
  14498.                 J U S
  14499.                   S O X
  14500.                    ▌X¿U
  14501.  
  14502. which can be done in 4 moves, JUS, SOX, ▌J¿US, and ▌X¿U.
  14503.  
  14504. In SPN Vol. XI, No. 52, December 1983, Alan Frank presented what
  14505. he claimed is the shortest game where no blanks are used, also
  14506. four moves:
  14507.  
  14508.                   C
  14509.                  WUD
  14510.                 CUKES
  14511.                  DEY
  14512.                   S
  14513.  
  14514. This was followed in SPN, Vol. XII No. 54, April 1984, by Terry Davis
  14515. of Glasgow, KY:
  14516.  
  14517.                   V
  14518.                 V O▌X¿
  14519.                  ▌X¿U,
  14520.  
  14521. which is three moves.  He noted that the use of two blanks prevents
  14522. such plays as VOLVOX.  Unfortunately, it doesn't prevent SONOVOX.
  14523.  
  14524. - - - - - - - - - - - - - - - - - - - - - - - - - - - - - - - - - - - -
  14525. Record for the highest scrabble score in a single turn (in a legal position):
  14526.  
  14527. According to the Scrabble Players Newspaper (since renamed to
  14528. Scrabble Players News) issue 44, p13, the highest score for one
  14529. turn yet discovered, using the Official Scrabble Players
  14530. Dictionary, 1st ed. (the 2nd edition is now in use in club and
  14531. tournament play) and the Websters 9th New Collegiate Dictionary,
  14532. was the following:
  14533.  
  14534. d i s e q u i l i b r a t e D
  14535. . . . . . . . e . . . . . . e
  14536. . . . . . . . e . . . . . o m
  14537. r a d i o a u t o g r a p(h)Y
  14538. . . . . . . . . . . . w a s T
  14539. . . . . . . . . . . b e . . h
  14540. . . . . . . . . . . a . . g o
  14541. . . . c o n j u n c t i v a L
  14542. . . . . . . . . . . . . . n o
  14543. . . . . . . . f i n i k i n G
  14544. . . . . . . . a . . . (l) e i
  14545. . . . . . . . d . s p e l t Z
  14546. . . . . . . w e . . . . . . e
  14547. . . . . . . r . . . . . . o r
  14548. m e t h o x y f l u r a n e S
  14549.  
  14550. for 1682 points.
  14551.  
  14552.  
  14553. According to the May 1986 issue of GAMES, the highest known score achievable
  14554. in one turn is 1,962 points.  The word is BENZOXYCAMPHORS formed across the
  14555. three triple-word scores on the bottom of the board.  Apparently it was
  14556. discovered by Darryl Francis, Ron Jerome, and Jeff Grant.
  14557.  
  14558. As for other Scrabble trivia, the highest-scoring first move based on the
  14559. Official Scrabble Players Dictionary is 120 points, with the words JUKEBOX,
  14560. QUIZZED, SQUEEZE, or ZYMURGY.  If Funk & Wagnall's New Standard Dictionary
  14561. is used then ZYXOMMA, worth 130 points, can be formed.
  14562.  
  14563. The highest-scoring game, based on Webster's Second and Third and on the
  14564. Oxford English Dictionary, was devised by Ron Jerome and Ralph Beaman and
  14565. totalled 4,142 points for the two players.  The highest-scoring words in
  14566. the game were BENZOXYCAMPHORS, VELVETEEN, and JACKPUDDINGHOOD.
  14567.  
  14568. The following example of a SCRABBLE game produced a score of 2448 for one
  14569. player and 1175 for the final word.  It is taken from _Beyond Language_ (1967)
  14570. by Dmitri Borgman (pp. 217-218).  He credits this solution to Mrs. Josefa H.
  14571. Byrne of San Francisco and implies that all words can be found in _Webster's
  14572. Second Edition_.  The two large words (multiplied by 27 as they span 3 triple
  14573. word scores) are ZOOPSYCHOLOGIST (a psychologist who treats animals rather
  14574. than humans) and PREJUDICATENESS (the condition or state of being decided
  14575. beforehand).  The asterisks (*) represent the blank tiles. (Please excuse
  14576. any typo's).
  14577.  
  14578.            Board                        Player1                 Player2
  14579.  
  14580. Z O O P S Y C H O L O G I S T    ABILITY             76   ERI, YE     9
  14581. O N         H A   U     R O W    MAN, MI             10   EN          2
  14582. *         R I B   R O V E   I    FEN, FUN            14   MANIA       7
  14583. L           T I K E         G    TABU                12   RIB         6
  14584. O             L                  NEXT                11   AM          4
  14585. G             I                  AX                   9   END         6
  14586. I             T                  IT, TIKE            10   LURE        6
  14587. *             Y E                LEND, LOGIC*AL      79   OO*LOGICAL  8
  14588. A               R                FUND, JUD           27   ATE, MA     7
  14589. L E N D       M I                ROVE                14   LO          2
  14590.     E         A             Q    DARE, DE            13   ES, ES, RE  6
  14591. W A X     F E N             U    RE, ROW             14   IRE, IS, SO 7
  14592. E   T A B U   I             A    DARED, QUAD         22   ON          4
  14593. E         N   A M   D A R E D    WAX, WEE            27   WIG         9
  14594. P R E J U D I C A T E N E S S    CHIT, HA            14   ON          2
  14595.                                  PREJUDICATENESS,
  14596.                                    AN, MANIAC,
  14597.                                    QUADS, WEEP      911   OOP         8
  14598.                                  ZOOPSYCHOLOGIST,
  14599.                                    HABILITY, TWIG,
  14600.                                    ZOOLOGICAL      1175
  14601.                                  --------------------------------------
  14602.                                  Total:            2438              93
  14603.  
  14604.                                  F, N, V, T in
  14605.                                  loser's hand:      +10             -10
  14606.                                  --------------------------------------
  14607.                                  Final Score:      2448              83
  14608.  
  14609.  
  14610. ---------------------------------------------------------------------------
  14611. It is possible to form the following 14 7-letter OSPD words from the tiles:
  14612. HUMANLY
  14613. FATUOUS
  14614. AMAZING
  14615. EERIEST
  14616. ROOFING
  14617. TOILERS
  14618. QUIXOTE
  14619. JEWELRY
  14620. CAPABLE
  14621. PREVIEW
  14622. BIDDERS
  14623. HACKING
  14624. OVATION
  14625. DONATED
  14626.  
  14627. ==> games/square-1.p <==
  14628. Does anyone have any hints on how to solve the Square-1 puzzle?
  14629.  
  14630. ==> games/square-1.s <==
  14631.                                 SHAPES
  14632.  
  14633. 1. There are 29 different shapes for a side, counting reflections:
  14634.     1 with 6 corners, 0 edges
  14635.     3 with 5 corners, 2 edges
  14636.    10 with 4 corners, 4 edges
  14637.    10 with 3 corners, 6 edges
  14638.     5 with 2 corners, 8 edges
  14639.  
  14640. 2. Naturally, a surplus of corners on one side must be compensated
  14641.    by a deficit of corners on the other side.  Thus there are 1*5 +
  14642.    3*10 + C(10,2) = 5 + 30 + 55 = 90 distinct combinations of shapes,
  14643.    not counting the middle layer.
  14644.  
  14645. 3. You can reach two squares from any other shape in at most 7 transforms,
  14646.    where a transform consists of (1) optionally twisting the top, (2)
  14647.    optionally twisting the bottom, and (3) flipping.
  14648.  
  14649. 4. Each transform toggles the middle layer between Square and Kite,
  14650.    so you may need 8 transforms to reach a perfect cube.
  14651.  
  14652. 5. The shapes with 4 corners and 4 edges on each side fall into four
  14653.    mutually separated classes.  Side shapes can be assigned values:
  14654.    0: Square, Mushroom, and Shield; 1: Left Fist and Left Paw; 2:
  14655.    Scallop, Kite, and Barrel; 3. Right Fist and Right Paw.  The top
  14656.    and bottom's sum or difference, depending on how you look at them,
  14657.    is a constant.  Notice that the side shapes with bilateral symmetry
  14658.    are those with even values.
  14659.  
  14660. 6. To change this constant, and in particular to make it zero, you must
  14661.    attain a position that does not have 4 corners and 4 edges on each
  14662.    side.  Almost any such position will do, but returning to 4 corners
  14663.    and 4 edges with the right constant is left to your ingenuity.
  14664.  
  14665. 7. If the top and bottom are Squares but the middle is a Kite, just flip
  14666.    with the top and bottom 30deg out of phase and you will get a cube.
  14667.  
  14668.                                 COLORS
  14669.  
  14670. 1. I do not know the most efficient way to restore the colors.  What
  14671.    follows is my own suboptimal method.  All flips keep the yellow
  14672.    stripe steady and flip the blue stripe.
  14673.  
  14674. 2. You can permute the corners without changing the edges, so first
  14675.    get the edges right, then the corners.
  14676.  
  14677. 3. This transformation sends the right top edge to the bottom
  14678.    and the left bottom edge to the top, leaving the other edges
  14679.    on the same side as they started:  Twist top 30deg cl, flip,
  14680.    twist top 30deg ccl, twist bottom 150deg cl, flip, twist bottom
  14681.    30deg cl, twist top 120deg cl, flip, twist top 30deg ccl, twist
  14682.    bottom 150deg cl, flip, twist bottom 30deg cl.  Cl and ccl are
  14683.    defined looking directly at the face.  With this transformation
  14684.    you can eventually get all the white edges on top.
  14685.  
  14686. 4. Check the parity of the edge sequence on each side.  If either is
  14687.    wrong, you need to fix it.  Sorry -- I don't know how]  (See any
  14688.    standard reference on combinatorics for an explanation of parity.)
  14689.  
  14690. 5. The following transformation cyclically permutes ccl all the top edges
  14691.    but the right one and cl all the bottom edges but the left one.  Apply
  14692.    the transformation in 3., and turn the whole cube 180deg.  Repeat.
  14693.    This is a useful transformation, though not a cure-all.
  14694.  
  14695. 6. Varying the transformation in 3. with other twists will produce other
  14696.    results.
  14697.  
  14698. 7. The following transformation changes a cube into a Comet and Star:
  14699.    Flip to get Kite and Kite.  Twist top and bottom cl 90deg and flip to get
  14700.    Barrel and Barrel.  Twist top cl 30 and bottom cl 60 and flip to get
  14701.    Scallop and Scallop.  Twist top cl 60 and bottom cl 120 and flip to
  14702.    get Comet and Star.  The virtue of the Star is that it contains only
  14703.    corners, so that you can permute the corners without altering the edges.
  14704.  
  14705. 8. To reach a Lemon and Star instead, replace the final bottom cl 120 with
  14706.    a bottom cl 60.  In both these transformation the Star is on the bottom.
  14707.  
  14708. 9. The following transformation cyclically permutes all but the bottom
  14709.    left rear.  It sends the top left front to the bottom, and the bottom
  14710.    left front to the top.  Go to Comet and Star.  Twist star cl 60.
  14711.    Go to Lemon and Star -- you need not return all the way to the cube, but
  14712.    do it if you're unsure of yourself by following 7 backwards.  Twist star
  14713.    cl 60.  Return to cube by following 8 backwards.  With this transformation
  14714.    you should be able to get all the white corners on top.
  14715.  
  14716. 10. Check the parity of the corner sequences on both sides.  If the bottom
  14717.    parity is wrong, here's how to fix it:  Go to Lemon and Star.  The
  14718.    colors on the Star will run WWGWWG.  Twist it 180 and return to cube.
  14719.  
  14720. 11. If the top parity is wrong, do the same thing, except that when you
  14721.    go from Scallop and Scallop to Lemon and Star, twist the top and bottom
  14722.    ccl instead of cl.  The colors on the Star should now run GGWGGW.
  14723.  
  14724. 12. Once the parity is right on both sides, the basic method is to
  14725.    go to Comet and Star, twist the star 120 cl (it will be WGWGWG),
  14726.    return to cube, twist one or both sides, go to Comet and Star,
  14727.    undo the star twist, return to cube, undo the side twists.
  14728.    With no side twists, this does nothing.  If you twist the top,
  14729.    you will permute the top corners.  If you twist the bottom,
  14730.    you will permute the bottom corners.  Eventually you will get
  14731.    both the top and the bottom right.  Don't forget to undo the
  14732.    side twists -- you need to have the edges in the right places.
  14733.  
  14734. Happy twisting....
  14735. --
  14736. Col. G. L. Sicherman
  14737. gls@windmill.att.COM
  14738.  
  14739. ==> games/think.and.jump.p <==
  14740. THINK & JUMP:  FIRST THINK, THEN JUMP UNTIL YOU
  14741.                ARE LEFT WITH ONE PEG]                      O - O   O - O
  14742.                                                           / \ / \ / \ / \
  14743.                                                          O---O---O---O---O
  14744. BOARD DESCRIPTION:  To the right is a model of            \ / \ / \ / \ /
  14745.                     the Think & Jump board.  The       O---O---O---O---O---O
  14746.                     O's represent holes which         / \ / \ / \ / \ / \ / \
  14747.                     contain pegs.                    O---O---O---O---O---O---O
  14748.                                                       \ / \ / \ / \ / \ / \ /
  14749.                                                        O---O---O---O---O---O
  14750. DIRECTIONS:  To play this brain teaser, you begin         / \ / \ / \ / \
  14751.              by removing the center peg.  Then,          O---O---O---O---O
  14752.              moving any direction in the grid,            \ / \ / \ / \  /
  14753.              jump over one peg at a time,                  O - O   O - O
  14754.              removing the jumped peg - until only
  14755.              one peg is left.  It's harder then it looks.
  14756.              But it's more fun than you can imagine.
  14757.  
  14758. SKILL CHART:
  14759.  
  14760.         10 pegs left - getting better
  14761.          5 pegs left - true talent
  14762.          1 peg  left - you're a genius
  14763.  
  14764. Manufactured by Pressman Toy Corporation, NY, NY.
  14765.  
  14766. ==> games/think.and.jump.s <==
  14767. Three-color the board in the obvious way.  The initial configuration has 12
  14768. of each color, and each jump changes the parity of all three colors.  Thus,
  14769. it is impossible to achieve any position where the colors do not have the
  14770. same parity; in particular, (1,0,0).
  14771.  
  14772. If you remove the requirement that the initially-empty cell must be at the
  14773. center, the game becomes solvable.  The demonstration is left as an exercise.
  14774.  
  14775. Karl Heuer   rutgers]harvard]ima]haddock]karl   karl@haddock.ima.isc.com
  14776.  
  14777.  
  14778.  
  14779.  
  14780. Here is one way of reducing Think & Jump to two pegs.
  14781.  
  14782.  
  14783. Long simplifies Balsley's scintillating snowflake solution:
  14784.  
  14785. 1  U-S           A - B   C - D
  14786. 2  H-U          / \ / \ / \ / \
  14787. 3  V-T         E---F---G---H---I
  14788. 4  S-H          \ / \ / \ / \ /
  14789. 5  D-M       J---K---L---M---N---O
  14790. 6  F-S      / \ / \ / \ / \ / \ / \
  14791. 7  Q-F     P---Q---R---S---T---U---V
  14792. 8  A-L      \ / \ / \ / \ / \ / \ /
  14793. 9  S-Q       W---X---Y---Z---a---b
  14794. 10 P-R          / \ / \ / \ / \
  14795. 11 Z-N         c---d---e---f---g
  14796. 12 Y-K          \ / \ / \ / \ /
  14797. 13 h-Y           h - i   j - k
  14798. 14 k-Z
  14799.  
  14800. The board should now be in the snowflake pattern, i.e. look like
  14801.  
  14802.          o - *   * - o
  14803.         / \ / \ / \ / \
  14804.        *---o---*---o---*
  14805.         \ / \ / \ / \ /
  14806.      *---*---*---*---*---*
  14807.     / \ / \ / \ / \ / \ / \
  14808.    o---o---o---o---o---o---o
  14809.     \ / \ / \ / \ / \ / \ /
  14810.      *---*---*---*---*---*
  14811.         / \ / \ / \ / \
  14812.        *---o---*---o---*
  14813.         \ / \ / \ / \ /
  14814.          o - *   * - o
  14815.  
  14816. where o is empty and * is a peg.  The top and bottom can now be reduced
  14817. to single pegs individually.  For example, we could continue
  14818.  
  14819. 15 g-T
  14820. 16 Y-a
  14821. 17 i-Z
  14822. 18 T-e
  14823. 19 j-Y
  14824. 20 b-Z
  14825. 21 c-R
  14826. 22 Z-X
  14827. 23 W-Y
  14828. 24 R-e
  14829.  
  14830. which finishes the bottom.  The top can be done in a similar manner.
  14831. --
  14832. Chris Long
  14833.  
  14834. ==> games/tictactoe.p <==
  14835. In random tic-tac-toe, what is the probability that the first mover wins?
  14836.  
  14837. ==> games/tictactoe.s <==
  14838. Count cases.
  14839.  
  14840. First assume that the game goes on even after a win.  (Later figure
  14841. out who won if each player gets a row of three.)  Then there are
  14842. 9]/5]4] possible final boards, of which
  14843.  
  14844.         8*6]/2]4] - 2*6*4]/0]4] - 3*3*4]/0]4] - 1 = 98
  14845.  
  14846. have a row of three Xs.  The first term is 8 rows times (6 choose 2)
  14847. ways to put down the remaining 2 Xs.  The second term is the number
  14848. of ways X can have a diagonal row plus a horizontal or vertical row.
  14849. The third term is the number of ways X can have a vertical and a
  14850. horizontal row, and the 4th term is the number of ways X can have two
  14851. diagonal rows.  All the two-row configurations must be subtracted to
  14852. avoid double-counting.
  14853.  
  14854. There are 8*6]/1]5] = 48 ways O can get a row.  There is no double-
  14855. counting problem since only 4 Os are on the final board.
  14856.  
  14857. There are 6*2*3]/2]1] = 36 ways that both players can have a
  14858. row.  (6 possible rows for X, each leaving 2 possible rows for O
  14859. and (3 choose 2) ways to arrange the remaining row.)  These
  14860. cases need further consideration.
  14861.  
  14862. There are 98 - 36 = 62 ways X can have a row but not O.
  14863.  
  14864. There are 48 - 36 = 12 ways O can have a row but not X.
  14865.  
  14866. There are 126 - 36 - 62 - 12 = 16 ways the game can be a tie.
  14867.  
  14868. Now consider the 36 configurations in which each player has a row.
  14869. Each such can be achieved in 5]4] = 2880 orders.  There are 3*4]4]
  14870. = 1728 ways that X's last move completes his row.  In these cases O
  14871. wins.  There are 2*3*3]3] = 216 ways that Xs fourth move completes
  14872. his row and Os row is already done in three moves.  In these cases O
  14873. also wins.  Altogether, O wins 1728 + 216 = 1944 out of 2880 times
  14874. in each of these 36 configurations.  X wins the other 936 out of
  14875. 2880.
  14876.  
  14877. Altogether, the probability of X winning is ( 62 + 36*(936/2880) ) / 126.
  14878.  
  14879. win:   737 / 1260  ( 0.5849206... )
  14880. lose:  121 / 420   ( 0.2880952... )
  14881. draw:  8 / 63      ( 0.1269841... )
  14882.  
  14883. 1000000 games:  won 584865, lost 288240, tied 126895
  14884.  
  14885. Instead, how about just methodically having the program play every
  14886. possible game, tallying up who wins?
  14887.  
  14888. Wonderful idea, especially since there are only 9] ~ 1/3 million
  14889. possible games.  Of course some are identical because they end in
  14890. fewer than 8 moves.  It is clear that these should be counted
  14891. multiple times since they are more probable than games that go
  14892. longer.
  14893.  
  14894. The result:
  14895. 362880 games:  won 212256, lost 104544, tied 46080
  14896.  
  14897. #include <stdio.h>
  14898.  
  14899. int     board▌9¿;
  14900. int     N, move, won, lost, tied;
  14901.  
  14902. int     perm▌9¿ = { 0, 1, 2, 3, 4, 5, 6, 7, 8 };
  14903.  
  14904. int     rows▌8¿▌3¿ = {
  14905.   { 0, 1, 2 }, { 3, 4, 5 }, { 6, 7, 8 }, { 0, 3, 6 },
  14906.   { 1, 4, 7 }, { 2, 5, 8 }, { 0, 4, 8 }, { 2, 4, 6 }
  14907. };
  14908.  
  14909.  
  14910. main()
  14911. {
  14912.   do {
  14913.     bzero((char *)board, sizeof board);
  14914.     for ( move=0; move<9; move++ ) {
  14915.       board▌perm▌move¿¿ = (move&1) ? 4 : 1;
  14916.       if ( move >= 4 && over() )
  14917.         break;
  14918.     }
  14919.     if ( move == 9 )
  14920.       tied++;
  14921. #ifdef DEBUG
  14922.     printf("%1d%1d%1d\n%1d%1d%1d  w %d, l %d, t %d\n%1d%1d%1d\n\n",
  14923.            board▌0¿, board▌1¿, board▌2¿,
  14924.            board▌3¿, board▌4¿, board▌5¿, won, lost, tied,
  14925.            board▌6¿, board▌7¿, board▌8¿);
  14926. #endif
  14927.     N++;
  14928.   } while ( nextperm(perm, 9) );
  14929.  
  14930.   printf("%d games:  won %d, lost %d, tied %d\n", N, won, lost, tied);
  14931.   exit(0);
  14932. }
  14933.  
  14934. int     s;
  14935. int     *row;
  14936.  
  14937. over()
  14938. {
  14939.   for ( row=rows▌0¿; row<rows▌8¿; row+=3 ) {
  14940.     s = board▌row▌0¿¿ + board▌row▌1¿¿ + board▌row▌2¿¿;
  14941.     if ( s == 3 )
  14942.       return ++won;
  14943.     if ( s == 12 )
  14944.       return ++lost;
  14945.   }
  14946.   return 0;
  14947. }
  14948.  
  14949. nextperm(c, n)
  14950. int     c▌¿, n;
  14951. {
  14952.   int   i = n-2, j=n-1, t;
  14953.  
  14954.   while ( i >= 0 && c▌i¿ >= c▌i+1¿ )
  14955.     i--;
  14956.   if ( i < 0 )
  14957.     return 0;
  14958.   while ( c▌j¿ <= c▌i¿ )
  14959.     j--;
  14960.   t = c▌i¿;  c▌i¿ = c▌j¿;  c▌j¿ = t;
  14961.   i++;  j = n-1;
  14962.   while ( i < j ) {
  14963.     t = c▌i¿;  c▌i¿ = c▌j¿;  c▌j¿ = t;
  14964.     i++;  j--;
  14965.   }
  14966.   return 1;
  14967. }
  14968.  
  14969.  
  14970.  
  14971. ==> geometry/K3,3.p <==
  14972. Can three houses be connected to three utilities without the pipes crossing?
  14973.  
  14974.             _______          _______          _______
  14975.             ! oil !          !water!          ! gas !
  14976.             !_____!          !_____!          !_____!
  14977.  
  14978.  
  14979.             _______          _______          _______
  14980.             !HOUSE!          !HOUSE!          !HOUSE!
  14981.             ! one !          ! two !          !three!
  14982.  
  14983. ==> geometry/K3,3.s <==
  14984. The problem you describe is to draw a bipartite graph of 3 nodes connected
  14985. in all ways to 3 nodes, all embedded in the plane.  The graph is called K3,3.
  14986. A famous theorem of Kuratowsky says that all graphs can be embedded
  14987. in the plane, EXCEPT those containing K3,3 or K5 (the complete graph
  14988. on 5 vertices, i.e., the graph with 5 nodes and 10 edges) as a
  14989. subgraph.  So your problem is a minimal example of a graph that
  14990. cannot be embedded in the plane.
  14991.  
  14992. The proofs that K5 and K3,3 are non-planar are really quite easy, and only
  14993. depend on Euler's Theorem that F-E+V=2 for a planar graph.
  14994. For K3,3 V is 6 and E is 9, so F would have to be 5. But each face has at
  14995. least 4 edges, so E >= (F*4)/2 = 10, contradiction.
  14996. For K5 V is 5 and E is 10, so F = 7. In this case each face has at least 3
  14997. edges, so E >= (F*3)/2 = 10.5, contradiction.
  14998.  
  14999. The difficult part of Kuratowsky is the proof in the other direction]
  15000.  
  15001. A quick, informal proof by contradiction without assuming Euler's Theorem:
  15002. Using a map in which the houses are 1, 2, and 3 and the utilities are
  15003. A, B, and C, there must be continuous lines that connect the buildings and
  15004. divide the area into three sections bounded by the loops A-1-B-2-A,
  15005. A-1-B-3-A, and A-2-B-3-A.  (One of the areas is the infinite plane *around*
  15006. whichever loop is the outer edge of the network.)  C must be in one of these
  15007. three areas; whichever area it is in, either 1, or 2, or 3, is *not* part of
  15008. the loop that rings its area and hence is inaccessible to C.
  15009.  
  15010. The usual quibble is to solve the puzzle by running one of the pipes
  15011. underneath one of houses on its way to another house; the puzzle's
  15012. instructions forbid crossing other *pipes*, but not crossing other *houses*.
  15013.  
  15014. ==> geometry/bear.p <==
  15015. If a hunter goes out his front door, goes 50 miles south, then goes 50
  15016. miles west, shoots a bear, goes 50 miles north and ends up in front of
  15017. his house.  What color was the bear?
  15018.  
  15019. ==> geometry/bear.s <==
  15020. The hunter's door is in one of two locations.  One is a foot or so from the
  15021. North Pole, facing north, such that his position in front of the door is
  15022. precisely upon the North Pole.  Since that's a ridiculous place to build a
  15023. house and since bears do not roam within fifty miles of the pole, the bear
  15024. is either imaginary or imported, and there is no telling what color it is.
  15025.  
  15026. There is another place (actually a whole set) on earth from which one can go
  15027. fifty miles south, fifty miles west, and fifty miles north and end up where
  15028. one started.  Consider the parallel of latitude close enough to the South
  15029. Pole that the circumference of the earth at that latitude is 50/n miles,
  15030. for some integer n.
  15031.  
  15032. Take any point on that parallel of latitude and pick the point fifty miles
  15033. north of it.  Situate the hunter's front porch there.  The hunter goes fifty
  15034. miles south from his porch and is at a point we'll call A.  He travels fifty
  15035. miles west, going n times around the earth, and is at A again, where he shoots
  15036. the bear.  Fifty miles north from A he is back home.  Since bears are not
  15037. indigenous to the Antarctic, again the bear is either imaginary or imported
  15038. and there is no telling what color it might be.
  15039.  
  15040. ==> geometry/bisector.p <==
  15041. If two angle bisectors of a triangle are equal, then the triangle is
  15042. isosceles (more specifically, the sides opposite to the two angles
  15043. being bisected are equal).
  15044.  
  15045. ==> geometry/bisector.s <==
  15046. The following proof is probably from Altshiller-Court's College
  15047. Geometry, since that's where I first saw the problem.
  15048.  
  15049. Let the triangle be ABC, with angle bisectors BE and CD.
  15050. Let F be such that BEFD is a parallelagram.
  15051. Let x  = measure of angle CBE = angle DBE,
  15052.     y  = measure of angle BCD = angle DCE,
  15053.     x' = measure of angle EFC,
  15054.     y' = measure of angle ECF.
  15055. (You will probably want to draw a picture.)
  15056.  
  15057. Suppose x > y.  Consider the triangles EBC and DCB.  Since BC = BC and
  15058. BE = CD, we must have CE > BD.  Now, since BD = EF, we have that CE >
  15059. EF, so that x' > y'.  Thus x+x' > y+y'.  But, triangle FDC is
  15060. isosceles, since DF = BE = DC, so x+x' = y+y', a contradiction.
  15061. Similarly, we cannot have x < y.  Therefore the base angles of ABC are
  15062. equal, making ABC an isosceles triangle. QED
  15063.  
  15064.  
  15065.  
  15066.  
  15067. ==> geometry/calendar.p <==
  15068. Build a calendar from two sets of cubes.  On the first set,
  15069. spell the months with a letter on each face of three cubes.
  15070. Use lowercase three-letter abbreviations for the names of all
  15071. twelve months (e.g., "jan", "feb", "mar").  On the second set,
  15072. number the days with a digit on each face of two cubes (e.g.,
  15073. "01", "02", etc.).
  15074.  
  15075. ==> geometry/calendar.s <==
  15076.         First note that there are *nineteen* different letters in the
  15077. month abbreviations (abcdef gjlmno prstuv y) so to get them all on the
  15078. eighteen faces of 3 cubes, you know right away you're going to have to
  15079. resort to trickery.
  15080.  
  15081.         So I wrote them all down and looked at which ones could be
  15082. reversed to make another letter in the set.  The only pair that jumped
  15083. out at me was the d/p pair.  Now I knew that it was at least feasible,
  15084. as long as it wasn't necessary to duplicate any letters.
  15085.  
  15086.         Then I scanned the abbreviations to find ones that had a lot of
  15087. common letters.  The jan-jun-jul series looked like a good place to
  15088. start:
  15089.         j       a       n
  15090.                 u       l
  15091.                                 was a good beginning but I realized
  15092. right away that I had no room for duplicate letters and the second cube
  15093. had both a and u so aug was going to be impossible.  In fact I almost
  15094. posted that answer.  Then I realized that if Martin Gardner wrote about
  15095. it, it must have a solution.  :-)  So I went back to the letter list.
  15096.  
  15097.         I don't put tails on my u's so it didn't strike me the first
  15098. time through that n and u could be combined.
  15099.       Cube 1  Cube 2  Cube 3
  15100.         j       a       n/u
  15101.                 n/u     l
  15102.                                 would let me get away with putting the g
  15103. on the first cube to get aug, so I did.
  15104.         j       a       n/u
  15105.         g       n/u     l       (1)
  15106.  
  15107.         Now came the fun part.  The a was placed so I had to work around
  15108. it for the other months that had an a in them (mar, apr, may).
  15109.         m       a       r
  15110.         d/p             y       (2)
  15111.  
  15112.         Now the d/p was placed so I had to work around that for sep and dec.
  15113. This one was easy since they shared an e as well.
  15114.         d/p     e       s
  15115.                         c       (3)
  15116.  
  15117.         Now the e was placed so feb had to be worked in.
  15118.         f       e       b       (4)
  15119.  
  15120.         The two months left (oct, nov) were far more complex.  Not only
  15121. did they have two "set" letters (c, n/u), there were two possible n/u's
  15122. to be set with.  That's why I left them for last.
  15123.         o       t       c
  15124.                 n/u     v       (5)
  15125.  
  15126.         So now I had five pieces to fit together, so that no set would
  15127. have more than six letters in it.  Trial and error provided:
  15128.  
  15129.         j       a       n/u                     a       b       e
  15130.         g       n/u     l       or,             c       d/p     g
  15131.         r       s       m       alphabetically: f       l       j
  15132.         y       c       d/p                     n/u     m       o
  15133.         e       v       t                       s       n/u     r
  15134.         o       f       b                       v       t       y
  15135.  
  15136.  
  15137.   Without some gimmick the days cannot be done.  Because of the dates 11 and
  15138. 22, there must be a 1 and a 2 on each cube. Thus there are 8 remaining spaces
  15139. for the 8 remaining numbers, and because of 30, we put 3 and 0 on different
  15140. cubes. I don't think the way you allocate the others matter. Now 6 numbers on
  15141. each cube can produce at most 36 distinct pairs, and we need 31 distinct pairs
  15142. to represent all possible dates. But since 3 each of {4,5,6,7,8,9} are on each
  15143. cube, there are at least 9 representable numbers which can't be dates.
  15144. Therefore there are at most 27 distinct numbers which are dates on the two
  15145. cubes, and it can't be done. In particular, not all of {04,05,06,07,08,09} can
  15146. be represented.
  15147.  
  15148.   The gimmick solution would be to represent the numbers in a stylised format
  15149. (like say, on a digital clock or on a computer screen) such that the 6 can be
  15150. turned upside down to be a 9. Then you can have 012 on both cubes, and three
  15151. each of {3,4,5,6,7,8} on the other faces. Done.
  15152.  
  15153.   Example: 012468 012357
  15154.  
  15155. ==> geometry/circles.and.triangles.p <==
  15156. Find the radius of the inscribed and circumscribed circles for a triangle.
  15157.  
  15158. ==> geometry/circles.and.triangles.s <==
  15159. Let a, b, and c be the sides of the triangle.  Let s be the
  15160. semiperimeter, i.e. s = (a + b + c) / 2.  Let A be the area
  15161. of the triangle, and let x be the radius of the incircle.
  15162.  
  15163. Divide the triangle into three smaller triangles by drawing
  15164. a line segment from each vertex to the incenter.  The areas
  15165. of the smaller triangles are ax/2, bx/2, and cx/2.  Thus,
  15166. A = ax/2 + bx/2 + cx/2, or A = sx.
  15167.  
  15168. We use Heron's formula, which is A = sqrt(s(s-a)(s-b)(s-c)).
  15169. This gives us x = sqrt((s-a)(s-b)(s-c)/s).
  15170.  
  15171. The radius of the circumscribed circle is given by R = abc/4A.
  15172.  
  15173. ==> geometry/coloring/cheese.cube.p <==
  15174. A cube of cheese is divided into 27 subcubes.  A mouse starts at one
  15175. corner and eats through every subcube.  Can it finish in the middle?
  15176.  
  15177. ==> geometry/coloring/cheese.cube.s <==
  15178. Give the subcubes a checkerboard-like coloring so that no two adjacent
  15179. subcubes have the same color.  If the corner subcubes are black, the
  15180. cube will have 14 black subcubes and 13 white ones.  The mouse always
  15181. alternates colors and so must end in a black subcube.  But the center
  15182. subcube is white, so the mouse can't end there.
  15183.  
  15184. ==> geometry/coloring/dominoes.p <==
  15185. There is a chess board (of course with 64 squares). You are given
  15186. 21 dominoes of size 3-by-1 (the size of an individual square on
  15187. a chess board is 1-by-1). Which square on the chess board can
  15188. you cut out so that the 21 dominoes exactly cover the remaining
  15189. 63 squares? Or is it impossible?
  15190.  
  15191. ==> geometry/coloring/dominoes.s <==
  15192. !!!!!!!!
  15193. !!!!!!!!
  15194. !!!!!!!!
  15195. ---***+*
  15196. ---...+*
  15197. ---*+O+*
  15198. ---*+...
  15199. ---*+***
  15200.  
  15201. There is only one way to remove a square, aside from rotations and
  15202. reflections.  To see that there is at most one way, do this:  Label
  15203. all the squares of the chessboard with A, B or C in sequence by rows
  15204. starting from the top:
  15205.  
  15206.                 ABCABCAB
  15207.                 CABCABCA
  15208.                 BCABCABC
  15209.                 ABCABCAB
  15210.                 CABCABCA
  15211.                 BCABCABC
  15212.                 ABCABCAB
  15213.                 CABCABCA
  15214.  
  15215. Every trimino must cover one A, one B and one C.  There is one extra
  15216. A square, so an A must be removed.  Now label the board again by
  15217. rows starting from the bottom:
  15218.  
  15219.                 CABCABCA
  15220.                 ABCABCAB
  15221.                 BCABCABC
  15222.                 CABCABCA
  15223.                 ABCABCAB
  15224.                 BCABCABC
  15225.                 CABCABCA
  15226.                 ABCABCAB
  15227.  
  15228. The square removed must still be an A.  The only squares that got
  15229. marked with A both times are these:
  15230.  
  15231.                 ........
  15232.                 ........
  15233.                 ..A..A..
  15234.                 ........
  15235.                 ........
  15236.                 ..A..A..
  15237.                 ........
  15238.                 ........
  15239.  
  15240. ==> geometry/construction/4.triangles.6.lines.p <==
  15241. Can you construct 4 equilateral triangles with 6 toothpicks?
  15242.  
  15243. ==> geometry/construction/4.triangles.6.lines.s <==
  15244. Use the toothpicks as the edges of a tetrahedron.
  15245.  
  15246. ==> geometry/construction/5.lines.with.4.points.p <==
  15247. Arrange 10 points so that they form 5 rows of 4 each.
  15248.  
  15249. ==> geometry/construction/5.lines.with.4.points.s <==
  15250. Draw a 5 pointed star, put a point where any two lines meet.
  15251.  
  15252. ==> geometry/construction/square.with.compass.p <==
  15253. Construct a square with only a compass and a straight edge.
  15254.  
  15255. ==> geometry/construction/square.with.compass.s <==
  15256. Draw a circle (C1 at P1).  Now draw a diameter D1 (intersects
  15257. at P2 and P3).  Set the compass larger than before.  From points P2
  15258. and P3 draw another larger circle (C2 and C3).  Where these two
  15259. circles cross, draw a line (D2).  This line should go the center of
  15260. circle C1 at a rt angle to the original diameter line.  This line
  15261. should cross circle C1 at P4 and P5
  15262.  
  15263. Reset the compass to its original size.  From P2 and P4 draw a circle
  15264. (C4 and C5).  These circles intersect at P6 and P1.  Connect P6, P2,
  15265. P1, P4 for a square.
  15266.  
  15267. ==> geometry/cover.earth.p <==
  15268. A thin membrane covers the surface of the earth.  One square meter is
  15269. added to the area of this membrane.  How much is added to the radius and
  15270. volume of this membrane?
  15271.  
  15272. ==> geometry/cover.earth.s <==
  15273. We know that V = (4/3)*pi*r^3 and A = 4*pi*r^2.
  15274. We need to find out how much V increases if A increases by 1 m^2.
  15275.  
  15276.   dV / dr = 4 * pi * r^2
  15277.   dA / dr = 8 * pi * r
  15278.   dV / dA = (dV / dr) / (dA / dr)
  15279.           = (4 * pi * r^2) / (8 * pi * r)
  15280.           = r/2
  15281.           = 3,250,000 m
  15282.  
  15283. If the area of the cover is increased by 1 square meter,
  15284. then the volume it contains is increased by about 3.25 million cubic meters.
  15285.  
  15286. We seem to be getting a lot of mileage out of such a small square of cotton.
  15287. However, the new cover would not be very high above the surface of the
  15288. planet -- about 6 nanometers (calculate dr/dA).
  15289.  
  15290. ==> geometry/dissections/circle.p <==
  15291. Can a circle be cut into similar pieces without point symmetry
  15292. about the midpoint?  Can it be done with a finite number of pieces?
  15293.  
  15294. ==> geometry/dissections/circle.s <==
  15295. Yes.  Draw a circle inside the original circle, sharing a common point
  15296. on the right.  Now draw another circle inside the second, sharing a
  15297. point at the left.  Now draw another inside the third, sharing a point
  15298. at the right.  Continue in this way, coloring in every other region
  15299. thus generated.  Now, all the colored regions touch, so count this as
  15300. one piece and the uncolored regions as a second piece.  So the circle
  15301. has been divided into two similar pieces and there is no point
  15302. symmetry about the midpoint.  Maybe it is cheating to call these
  15303. single pieces, though.
  15304.  
  15305. ==> geometry/dissections/hexagon.p <==
  15306. Divide the hexagon into:
  15307. 1) 3 indentical rhombuses.
  15308. 2) 6 indentical kites(?).
  15309. 3) 4 indentical trapezoids.
  15310. 4) 8 indentcal shapes (any shape).
  15311. 5) 12 identical shapes (any shape).
  15312.  
  15313. ==> geometry/dissections/hexagon.s <==
  15314. What is considered 'identical' for these questions?  If mirror-image shapes
  15315. are allowed, these are all pretty trivial.  If not, the problems are rather
  15316. more difficult...
  15317.  
  15318.         1. Connect the center to every second vertex.
  15319.         2. Connect the center to the midpoint of each side.
  15320.         3. This is the hard one.  If you allow mirror images, it's trivial:
  15321.            bisect the hexagon from vertex to vertex, then bisect with a
  15322.            perpendicular to that, from midpoint of side to midpoint of side.
  15323.         4. This one's neat.  Let the side length of the hexagon be 2 (WLOG).
  15324.            We can easily partition the hexagon into equilateral triangles
  15325.            with side 2 (6 of them), which can in turn be quartered into
  15326.            equilateral triangles with side 1.  Thus, our original hexagon
  15327.            is partitioned into 24 unit equilateral triangles.  Take the
  15328.            trapezoid formed by 3 of these little triangles.  Place one such
  15329.            trapezoid on the inside of each face of the original hexagon, so
  15330.            that the long side of the trapezoid coincides with the side of the
  15331.            hexagon.  This uses 6 trapezoids, and leaves a unit hexagon in the
  15332.            center as yet uncovered.  Cover this little hexagon with two of
  15333.            the trapezoids.  Voila.  An 8-identical-trapezoid partition.
  15334.         5. Easy.  Do the rhombus partition in #1.  Quarter each rhombus by
  15335.            connecting midpoints of opposite sides.  This produces 12 small
  15336.            rhombi, each of which is equivalent to two adjacent small triangles
  15337.            as in #4.
  15338.  
  15339. Except for #3, all of these partitions can be achieved by breaking up the
  15340. hexagon into unit equilateral triangles, and then building these into the
  15341. shapes desired.  For #3, though, this would require (since there are 24 small
  15342. triangles) trapezoids formed from 6 triangles each.  The only trapezoid that
  15343. can be built from 6 identical triangles is a parallelogram; I assume that the
  15344. poster wouldn't have asked for a trapezoid if you could do it with a special
  15345. case of trapezoid.  At any rate, that parallelogram doesn't work.
  15346.  
  15347. ==> geometry/dissections/square.70.p <==
  15348. Since 1^2 + 2^2 + 3^2 + ... + 24^2 = 70^2, can a 70x70 sqaure be dissected into
  15349. 24 squares of size 1x1, 2x2, 3x3, etc.?
  15350.  
  15351. ==> geometry/dissections/square.70.s <==
  15352. Martin Gardner asked this in his Mathematical Games column in the
  15353. September 1966 issue of Scientific American.  William Cutler was the first
  15354. of 24 readers who reduced the uncovered area to 49, using all but the 7x7
  15355. square.  All the patterns were the same except for interchanging the
  15356. squares of orders 17 and 18 and rearranging the squares of orders 1, ...,
  15357. 6, 8, 9, and 10.  Nobody proved that the solution is minimal.
  15358.  
  15359. +----------------+-------------+----------------------+---------------------+
  15360. !                !             !                      !                     !
  15361. !                !             !                      !                     !
  15362. !                !      11     !                      !                     !
  15363. !                !             !                      !                     !
  15364. !       16       !             !                      !                     !
  15365. !                +-----+--+----+         22           !         21          !
  15366. !                !     ! 2!    !                      !                     !
  15367. !                !  5  +--+----+                      !                     !
  15368. !                !     !       !                      !                     !
  15369. +----------------+--+--+   6   !                      !                     !
  15370. !                   ! 3!       !                      !                     !
  15371. !                   ++-+-------+                      !                     !
  15372. !                   !!         !                      ++--------------------+
  15373. !                   !!    8    +----------------------++                    !
  15374. !        18         !!         !                       !                    !
  15375. !                   !!         !                       !                    !
  15376. !                   ++---------+                       !                    !
  15377. !                   !          !                       !         20         !
  15378. !                   !     9    !                       !                    !
  15379. +------------------++          !          23           !                    !
  15380. !                  !!          !                       !                    !
  15381. !                  ++----------+                       !                    !
  15382. !                  !           !                       +---++---------------+
  15383. !                  !           !                       !   !!               !
  15384. !        17        !     10    !                       ! 4 !!               !
  15385. !                  !           +---------------+-------+---++               !
  15386. !                  +-+---------+---------------+            !      15       !
  15387. !                  ! !                         !            !               !
  15388. !                  ! !                         !     12     !               !
  15389. +------------------+-+                         !            +-+-------------+
  15390. !                    !                         !            !1!             !
  15391. !                    !                         +------------+-+             !
  15392. !                    !           24            !              !             !
  15393. !                    !                         !              !             !
  15394. !        19          !                         !     13       !     14      !
  15395. !                    !                         !              !             !
  15396. !                    !                         !              !             !
  15397. !                    !                         !              !             !
  15398. +--------------------+-------------------------+--------------+-------------+
  15399.  
  15400. ==> geometry/dissections/square.five.p <==
  15401. Can you dissect a square into 5 parts of equal area with just a straight edge?
  15402.  
  15403. ==> geometry/dissections/square.five.s <==
  15404. 1. Prove you can reflect points which lie on the sides of the square
  15405. about the diagonals.
  15406.  
  15407. 2. Construct two different rectangles whose vertices lie on the square
  15408. and whose sides are parallel to the diagonals.
  15409.  
  15410. 3. Construct points A, A', B, B' on one (extended) side of the square
  15411. such that A/A' and B/B' are mirror image pairs with respect to another
  15412. side of the square.
  15413.  
  15414. 4. Construct the mirror image of the center of the square in one
  15415. of the sides.
  15416.  
  15417. 5. Divide the original square into 4 equal squares whose sides are
  15418. parallel to the sides of the original square.
  15419.  
  15420. 6. Divide one side of the square into 8 equal segments.
  15421.  
  15422. 7. Construct a trapezoid in which one base is a square side and one
  15423. base is 5/8 of the opposite square side.
  15424.  
  15425. 8. Divide one side of the square into 5 equal segments.
  15426.  
  15427. 9. Divide the square into 5 equal rectangles.
  15428.  
  15429. ==> geometry/duck.and.fox.p <==
  15430. A duck is swimming about in a circular pond.  A ravenous fox (who cannot
  15431. swim) is roaming the edges of the pond, waiting for the duck to come close.
  15432. The fox can run faster than the duck can swim.  In order to escape,
  15433. the duck must swim to the edge of the pond before flying away.  Assume that
  15434. the duck can't fly until it has reached the edge of the pond.
  15435.  
  15436. How much faster must the fox run that the duck swims in order to be always
  15437. able to catch the duck?
  15438.  
  15439. ==> geometry/duck.and.fox.s <==
  15440. Assume the ratio of the fox's speed to the duck's is a, and the radius
  15441. of the pond is r.  The duck's best strategy is:
  15442.  
  15443. 1.  Swim around a circle of radius (r/a - delta) concentric with the
  15444. pond until you are diametrically opposite the fox (you, the fox, and
  15445. the center of the pond are colinear).
  15446.  
  15447. 2.  Swim a distance delta along a radial line toward the bank opposite
  15448. the fox.
  15449.  
  15450. 3.  Observe which way the fox has started to run around the circle.
  15451. Turn at a RIGHT ANGLE in the opposite direction (i.e. if you started
  15452. swimming due south in step 2 and the fox started running to the east,
  15453. i.e. clockwise around the pond, then start swimming due west).  (Note:
  15454. If at the beginning of step 3 the fox is still in the same location as
  15455. at the start of step 2, i.e.  directly opposite you, repeat step 2
  15456. instead of turning.)
  15457.  
  15458. 4.  While on your new course, keep track of the fox.  If the fox slows
  15459. down or reverses direction, so that you again become diametrically
  15460. opposite the fox, go back to step 2.  Otherwise continue in a straight
  15461. line until you reach the bank.
  15462.  
  15463. 5.  Fly away.
  15464.  
  15465. The duck should make delta as small as necessary in order to be able
  15466. to escape the fox.
  15467.  
  15468. The key to this strategy is that the duck initially follows a
  15469. radial path away from the fox until the fox commits to running either
  15470. clockwise or counterclockwise around the pond.  The duck then turns onto
  15471. a new course that intersects the circle at a point MORE than halfway
  15472. around the circle from the fox's starting position.  In fact, the duck
  15473. swims along a tangent of the circle of radius r/a.  Let
  15474.  
  15475.   theta = arc cos (1/a)
  15476.  
  15477. then the duck swims a path of length
  15478.  
  15479.   r sin theta + delta
  15480.  
  15481. but the fox has to run a path of length
  15482.  
  15483.   r*(pi + theta) - a*delta
  15484.  
  15485. around the circle.  In the limit as delta goes to 0, the duck will
  15486. escape as long as
  15487.  
  15488.   r*(pi + theta) < a*r sin theta
  15489.  
  15490. that is,
  15491.  
  15492.   pi + arc cos (1/a) - a * sqrt(a^2 - 1) < 0
  15493.  
  15494. Maximize a in the above:  a = 4.6033388487517003525565820291030165130674...
  15495. The fox can catch the duck as long as he can run about 4.6 times as fast as
  15496. the duck can swim.
  15497.  
  15498. "But wait," I hear you cry, "When the duck heads off to that spot
  15499. 'more than halfway' around the circle, why doesn't the fox just double
  15500. back?  That way he'll reach that spot much quicker."  That is why the
  15501. duck's strategy has instructions to repeat step 2 under certain
  15502. circumstances.  Note that at the end of step 2, if the fox has started
  15503. to run to head off the duck, say in a clockwise direction, he and the
  15504. duck are now on the same side of some diameter of the circle.  This
  15505. continues to be true as long as both travel along their chosen paths
  15506. at full speed.  But if the fox were now to try to reach the duck's
  15507. destination in a counterclockwise direction, then at some instant he
  15508. and the duck must be on a diameter of the pond.  At that instant, they
  15509. have exactly returned to the situation that existed at the end of step
  15510. 1, except that the duck is a little closer to the edge than she was
  15511. before.  That's why the duck always repeats step 2 if the fox is ever
  15512. diametrically opposite her.  Then the fox must commit again to go one
  15513. way or the other.  Every time the fox fails to commit, or reverses his
  15514. commitment, the duck gets a distance delta closer to the edge.  This
  15515. is a losing strategy for the fox.
  15516.  
  15517. The limiting ratio of velocities that this strategy works against
  15518. cannot be improved by any other strategy, i.e., if the ratio of
  15519. the duck's speed to the fox's speed is less than a then the duck
  15520. cannot escape given the best fox strategy.
  15521.  
  15522. Given a ratio R of speeds less than the above a, the fox is sure to
  15523. catch the duck (or keep it in water indefinitely) by pursuing the
  15524. following strategy:
  15525. Do nothing so long as the duck is in a radius of R around the centre.
  15526. As soon as it emerges from this circle, run at top speed around the
  15527. circumference. If the duck is foolish enough not to position itself
  15528. across from the center when it comes out of this circle, run "the short
  15529. way around", otherwise run in either direction.
  15530.  
  15531. To see this it is enough to verify that at the circumference of the
  15532. circle of radius R, all straight lines connecting the duck to points
  15533. on the circumference (in the smaller segment of the circle cut out
  15534. by the tangent to the smaller circle) bear a ratio greater than R
  15535. with the corresponding arc the fox must follow. That this is enough
  15536. follows from the observation that the shortest curve from a point on
  15537. a circle to a point on a larger concentric circle (shortest among all
  15538. curves that don't intersect the interior of the smaller circle) is
  15539. either a straight line or an arc of the smaller circle followed by a
  15540. tangential straight line.
  15541.  
  15542. ==> geometry/earth.band.p <==
  15543. How much will a band around the equator rise above the surface if it
  15544. is made one meter longer?
  15545.  
  15546. ==> geometry/earth.band.s <==
  15547. The formula for the circumference of a circle is 2 * pi * radius.  Therefore,
  15548. if you increase the circumference by 1 meter, you increase the radius by
  15549. 1/(2 * pi) meters, or about 0.16 meters.
  15550.  
  15551. ==> geometry/ham.sandwich.p <==
  15552. Consider a ham sandwich, consisting of two pieces of bread and one of
  15553. ham.  Suppose the sandwich was dropped into a machine and spindled,
  15554. torn and mutiliated.  Is it still possible to divide the ham sandwich
  15555. with a straight knife cut such that both the ham and the bread are
  15556. divided in two parts of equal volume?
  15557.  
  15558. ==> geometry/ham.sandwich.s <==
  15559. Yes.  There is a theorem in topology called the Ham Sandwich Theorem,
  15560. which says: Given 3 (finite) volumes (each may be of any shape, and in
  15561. several pieces), there is a plane that cuts each volume in half.  One
  15562. would learn about it typically in a first course in algebraic topology,
  15563. or maybe in a course on introductory topology (if you studied the
  15564. fundamental group).
  15565.  
  15566. ==> geometry/hike.p <==
  15567. You are hiking in a half-planar woods, exactly 1 mile from the edge,
  15568. when you suddenly trip and lose your sense of direction.  What's the
  15569. shortest path that's guaranteed to take you out of the woods?  Assume
  15570. that you can navigate perfectly relative to your current location and
  15571. (unknown) heading.
  15572.  
  15573. ==> geometry/hike.s <==
  15574. Go 2/sqrt(3) away from the starting point, turn 120 degrees and head
  15575. 1/sqrt(3) along a tangent to the unit circle, then traverse an arc of
  15576. length 7*pi/6 along this circle, then head off on a tangent 1 mile.
  15577.  
  15578. This gives a minimum of sqrt(3) + 7*pi/6 + 1 = 6.397...
  15579.  
  15580. It remains to prove this is the optimal answer.
  15581.  
  15582. ==> geometry/hole.in.sphere.p <==
  15583. Old Boniface he took his cheer,
  15584. Then he bored a hole through a solid sphere,
  15585. Clear through the center, straight and strong,
  15586. And the hole was just six inches long.
  15587.  
  15588. Now tell me, when the end was gained,
  15589. What volume in the sphere remained?
  15590. Sounds like I haven't told enough,
  15591. But I have, and the answer isn't tough]
  15592.  
  15593. ==> geometry/hole.in.sphere.s <==
  15594. The volume of the leftover material is equal to the volume of a 6" sphere.
  15595.  
  15596. First, lets look at the 2 dimensional equivalent of this problem.
  15597. Two concentric circles where the chord of the outer circle that is
  15598. tangent to the inner circle has length D. What is the area of the "doughnut"
  15599. area between the circles?
  15600.  
  15601. It is pi * (D/2)^2. The same area as a circle with that diameter.
  15602. Proof:
  15603.         big circle radius is R
  15604.         little circle radius is r
  15605.  
  15606.                               2          2
  15607.         area of donut = pi * R   - pi * r
  15608.  
  15609.                                2    2
  15610.         =               pi * (R  - r )
  15611.  
  15612.  
  15613. Draw a right triangle and apply the Pythagorean Theorem to see that
  15614.                  2      2          2
  15615.                 R  -   r   =  (D/2)
  15616. so the area is
  15617.                                   2
  15618.         =               pi * (D/2)
  15619.  
  15620.  
  15621. Start with a sphere of radius R (where R > 6"), drill out the 6"
  15622. high hole.  We will now place this large "ring" on a plane.  Next to it
  15623. place a 6" high sphere.  By Archemedes' theorem, it suffices
  15624. to show that for any plane parallel to the base plane, the cross-
  15625. sectional area of these two solids is the same.
  15626.  
  15627. Take a general plane at height h above (or below) the center
  15628. of the solids. The radius of the circle of intersection on the sphere is
  15629.  
  15630.         radius = srqt(3^2 - h^2)
  15631.  
  15632. so the area is
  15633.  
  15634.         pi * ( 3^2  - h^2 )
  15635.  
  15636.  
  15637. For the ring, once again we are looking at the area between two concentric
  15638. circles.  The outer circle has radius sqrt(R^2 - h^2),
  15639. The area of the outer circle is therefore
  15640.  
  15641.                 pi (R^2 - h^2)
  15642.  
  15643. The inner circle has
  15644. radius sqrt(R^2 - 3^2).  So the area  of the inner circle is
  15645.  
  15646.         pi * ( R^2  - 3^2 )
  15647.  
  15648. the area of the doughnut is therefore
  15649.  
  15650.                 pi(R^2 - h^2)  - pi( R^2  - 3^2 )
  15651.  
  15652.         =       pi (R^2 - h^2 - R^2 + 3^2)
  15653.  
  15654.         =       pi (3^2  - h^2)
  15655.  
  15656. Therefore the areas are the same for every plane intersecting the solids.
  15657. Therefore their volumes are the same.
  15658. QED
  15659.  
  15660. ==> geometry/ladders.p <==
  15661. Two ladders form a rough X in an alley.  The ladders are 11 and 13 meters
  15662. long and they cross 4 meters off the ground.  How wide is the alley?
  15663.  
  15664. ==> geometry/ladders.s <==
  15665. Ladders 1 and 2, denoted L1 and L2, respectively, will rest along two
  15666. walls (taken to be perpendicular to the ground), and they will
  15667. intersect at a point O = (a,s), a height s from the ground.  Find the
  15668. largest s such that this is possible.  Then find the width of the
  15669. alley, w = a+b, in terms of L1, L2, and s.  This diagram is not to
  15670. scale.
  15671.  
  15672.                  B                     D
  15673.                   !\ L1           L2 /!
  15674.                   !  \             /  !           BC = length of L1
  15675.                   !    \         /    !           AD = length of L2
  15676.                   !      \  O  /      !            s = height of intersection
  15677.                  x!        \ /        !y           A = (0,0)
  15678.                   !        /!\        !           AE = a
  15679.                   !    m /  !  \ n    !           EC = b
  15680.                   !    /    !s   \    !           AO = m
  15681.                   !  /      !      \  !           CO = n
  15682.                   !/________!________\!
  15683.          (0,0) = A    a     E    b     C
  15684.  
  15685. -----------------------------------------------------------------------------
  15686. Without loss of generality, let L2 >= L1.
  15687.  
  15688. Observe that triangles AOB and DOC are similar.  Let r be the ratio of
  15689. similitude, so that x=ry.  Consider right triangles CAB and ACD.  By
  15690. the Pythagorean theorem, L1^2 - x^2 = L2^2 - y^2.  Substituting x=ry,
  15691. this becomes y^2(1-r^2) = L2^2 - L1^2.  Letting L= L2^2 -L1^2 (L>=0),
  15692. and factoring, this becomes
  15693.  
  15694.     (*)   y^2 (1+r)(1-r) = L
  15695.  
  15696. Now, because parallel lines cut L1 (a transversal) in proportion, r =
  15697. x/y = (L1-n)/n, and so  L1/n = r+1.  Now, x/s = L1/n = r+1, so ry = x =
  15698. s(r+1).  Solving for r, one obtains the formula r = s/(y-s).
  15699. Substitute this into (*) to get
  15700.  
  15701.     (**)  y^2 (y) (y-2s) = L (y-s)^2
  15702.  
  15703. NOTE:  Observe that, since L>=0, it must be true that y-2s>=0.
  15704.  
  15705. Now, (**) defines a fourth degree polynomial in y.  It can be written in the
  15706. form (by simply expanding (**))
  15707.  
  15708.     (***)  y^4 - 2s_y^3 - L_y^2 + 2sL_y - Ls^2 = 0
  15709.  
  15710. L1 and L2 are given, and so L is a constant.  How large can s be?  Given L,
  15711. the value s=k is possible if and only if there exists a real solution, y',
  15712. to (***), such that 2k <= y' < L2.  Now that s has been chosen, L and s are
  15713. constants, and (***) gives the desired value of y.  (Make sure to choose the
  15714. value satisfying 2s <= y' < L2.  If the value of s is "admissible" (i.e.,
  15715. feasible), then there will exist exactly one such solution.)
  15716. Now, w = sqrt(L2^2 - y^2), so this concludes the solution.
  15717.  
  15718. L1 = 11, L2 = 13, s = 4.  L = 13^2-11^2 = 48, so (***) becomes
  15719.  
  15720.            y^4 - 8_y^3 - 48_y^2 + 384_y - 768 = 0
  15721.  
  15722. Numerically find root y ~= 9.70940555, which yields w ~= 8.644504.
  15723.  
  15724. ==> geometry/lattice/area.p <==
  15725. Prove that the area of a triangle formed by three lattice points is integer/2.
  15726.  
  15727. ==> geometry/lattice/area.s <==
  15728. The formula for the area is
  15729.  
  15730.         A = ! x1*y2 + x2*y3 + x3*y1 - x1*y3 - x2*y1 - x3*y2 ! / 2
  15731.  
  15732. If the xi and yi are integers, A is of the form (integer/2)
  15733.  
  15734. ==> geometry/lattice/equilateral.p <==
  15735. Can an equlateral triangle have vertices at integer lattice points?
  15736.  
  15737. ==> geometry/lattice/equilateral.s <==
  15738. No.
  15739.  
  15740. Suppose 2 of the vertices are (a,b) and (c,d), where a,b,c,d are integers.
  15741. Then the 3rd vertex lies on the line defined by
  15742.  
  15743.         (x,y) = 1/2 (a+c,b+d) + t ((d-b)/(c-a),-1)    (t any real number)
  15744.  
  15745. and since the triangle is equilateral, we must have
  15746.  
  15747.         !!t ((d-b)/(c-a),-1)!! = sqrt(3)/2 !!(c,d)-(a,b)!!
  15748.  
  15749. which yields t = +/- sqrt(3)/2 (c-a).  Thus the 3rd vertex is
  15750.  
  15751.         1/2 (a+c,b+d) +/- sqrt(3)/2 (d-b,a-c)
  15752.  
  15753. which must be irrational in at least one coordinate.
  15754.  
  15755. ==> geometry/rotation.p <==
  15756. What is the smallest rotation that returns an object to its original state?
  15757.  
  15758. ==> geometry/rotation.s <==
  15759. 720 degrees.
  15760.  
  15761. Objects are made of bosons (integer-spin particles) and fermions
  15762. (half-odd-integer spin particles), and the wave function of a fermion
  15763. changes sign upon being rotated by 360 degrees.  To get it back to its
  15764. original state you must rotate by another 360 degrees, for a total of
  15765. 720 degrees.  This fact is the basis of Fermi-Dirac statistics, the
  15766. Pauli Exclusion Principle, electron orbits, chemistry, and life.
  15767.  
  15768. Mathematically, this is due to the continuous double cover of SO(2) by
  15769. SO(3), where SO(2) is the internal symmetry group of fermions and SO(3)
  15770. is the group of rotations in three dimensional space.
  15771.  
  15772. You can demonstrate this with a tray, which you hold in your right hand
  15773. with the arm lowered, then rotate twice as you raise your arm and end
  15774. up with the tray above your head, rotated twice about its vertical
  15775. axis, but without having twisted your arm.
  15776.  
  15777. Also, by attaching strings to a sphere, it is possible to see that a
  15778. 360 degree rotation will entangle the strings, which another 360 degree
  15779. rotation will disentangle.
  15780.  
  15781. Hospitals have machines which take out your blood, centrifuge it to take out
  15782. certain parts, then return it to your veins. Because of AIDS they must never
  15783. let your blood touch the inside of the machine which has touched others'
  15784. blood. So the inside is lined with a single piece of disposable branched
  15785. plastic tubing. This tube must rotate rapidly in the centrifuge where
  15786. several branches come out. Thus the tube should twist and tangle up the
  15787. branches. But the machine untwists the branches as in the above discussion.
  15788. At several hundred rounds per minute]
  15789.  
  15790. References
  15791.     P. A. M. Dirac's "scissors demonstration"
  15792.     R. Penrose and W.  Rindler
  15793.     Spinors and Space-time, vol. 1, p. 43
  15794.     Cambridge University Press, 1984,
  15795.  
  15796.     R. Feynman and S. Weinberg
  15797.     Elementary Particles and the Laws of Physics, p. 29
  15798.     Cambridge University Press, 1987
  15799.  
  15800. ==> geometry/smuggler.p <==
  15801. Somewhere on the high sees smuggler S is attempting, without much
  15802. luck, to outspeed coast guard G, whose boat can go faster than S's. G
  15803. is one mile east of S when a heavy fog descends. It's so heavy that
  15804. nobody can see or hear anything further than a few feet. Immediately
  15805. after the fog descends, S changes course and attempts to escape at
  15806. constant speed under a new, fixed course. Meanwhile, G has lost track
  15807. of S. But G happens to know S's speed, that it is constant, and that S
  15808. is sticking to some fixed heading, unknown to G.
  15809.  
  15810. How does G catch S?
  15811.  
  15812. G may change course and speed at will. He knows his own speed and
  15813. course at all times. There is no wind, G does not have radio or radar,
  15814. there is enough space for maneuvering, etc.
  15815.  
  15816. ==> geometry/smuggler.s <==
  15817. One way G can catch S is as follows (it is not the fastest way).
  15818.  
  15819. G waits until he knows that S has traveled for one mile. At that time, both
  15820. S and G are somewhere on a circle with radius one mile, and with its center
  15821. at the original position of S. G then begins to travel with a velocity that
  15822. has a radially outward component equal to that of S, and with a tangential
  15823. component as large as possible, given G's own limitation of total speed. By
  15824. doing so, G and S will always both be on an identical circle having its
  15825. center at the original position of S. Because G has a tangential component
  15826. whereas S does not, G will always catch S (actually, this is not proven
  15827. until you solve the o.d.e. associated with the problem).
  15828.  
  15829. If G can go at 40 mph and S goes at 20 mph, you can work out that it will
  15830. take G at most 1h 49m 52s to catch S.  On average, G will catch S in:
  15831.  
  15832. ( -2pi + sqrt(3) ( exp(2pi/sqrt(3)) - 1 )) / 40pi    hours,
  15833.  
  15834. which is, 27 min and 17 sec.
  15835.  
  15836. ==> geometry/table.in.corner.p <==
  15837. Put a round table into a (perpendicular) corner so that the table top
  15838. touches both walls and the feet are firmly on the ground.  If there is
  15839. a point on the perimeter of the table, in the quarter circle between
  15840. the two points of contact, which is 10 cm from one wall and 5 cm from
  15841. the other, what's the diameter of the table?
  15842.  
  15843. ==> geometry/table.in.corner.s <==
  15844. Consider the +X axis and the +Y axis to be the corner.  The table has
  15845. radius r which puts the center of the circle at (r,r) and makes the
  15846. circle tangent to both axis.  The equation of the circle (table's
  15847. perimeter) is
  15848.  
  15849.     (x-r)^2 + (y-r)^2 = r^2 .
  15850.  
  15851. This leads to
  15852.  
  15853.      r^2 - 2(x+y) + x^2 + y^2 = 0
  15854.  
  15855. Using x = 10, y = 5 we get the solutions 25 and 5.  The former is the
  15856. radius of the table.  It's diameter is 50 cm.
  15857.  
  15858. The latter number is the radius of a table that has a point which
  15859. satisfies the conditions but is on the outside edge of the table.
  15860.  
  15861. ==> geometry/tesseract.p <==
  15862. If you suspend a cube by one corner and slice it in half with a
  15863. horizontal plane through its centre of gravity, the section face is a
  15864. hexagon.  Now suspend a tesseract (a four dimensional hypercube) by one
  15865. corner and slice it in half with a hyper-horizontal hyperplane through
  15866. its centre of hypergravity.  What is the shape of the section
  15867. hyper-face?
  15868.  
  15869. ==> geometry/tesseract.s <==
  15870. The 4-cube is the set of all points in ▌-1,1¿^4 .
  15871. The hyperplane { (x,y,z,w) : x + y + z + w = 0 } cuts the 4-cube
  15872. in the desired manner.
  15873.  
  15874. Now, { (.5,.5,-.5,-.5), (.5,-.5,.5,-.5), (.5,-.5,-.5,.5) } is an
  15875. orthonormal basis for the hyperplane.  Let (a,b,c) be a point on the
  15876. hyperplane with respect to this basis.  (a,b,c) is in the 4-cube if and
  15877. only if !a! + !b! + !c! <= 2.   The shape of the intersection is a
  15878. regular octahedron.
  15879.  
  15880. ==> geometry/tetrahedron.p <==
  15881. Suppose you have a sphere of radius R and you have four planes that are
  15882. all tangent to the sphere such that they form an arbitrary tetrahedron
  15883. (it can be irregular).  What is the ratio of the surface area of the
  15884. tetrahedron to its volume?
  15885.  
  15886. ==> geometry/tetrahedron.s <==
  15887. For each face of the tetrahedron, construct a new tetrahedron with that
  15888. face as the base and the center of the sphere as the fourth vertex.
  15889. Now the original tetrahedron is divided into four smaller ones, each of
  15890. height R.  The volume of a tetrahedron is Ah/3 where A is the area of
  15891. the base and h the height; in this case h=R.  Combine the four
  15892. tetrahedra algebraically to find that the volume of the original
  15893. tetrahedron is R/3 times its surface area.
  15894.  
  15895. ==> geometry/tiling/rational.sides.p <==
  15896. A rectangular region R is divided into rectangular areas.  Show that if
  15897. each of the rectangles in the region has at least one side with
  15898. rational length then the same can be said of R.
  15899.  
  15900. ==> geometry/tiling/rational.sides.s <==
  15901. "Fourteen proofs of a result about tiling a rectangle" (Stan Wagon)
  15902. _The American Mathematical Monthly_, Aug-Sep 1987, Vol 94 #7.  There
  15903. was also a fifteenth proof published a few issues later, attributed to
  15904. a (University of Kentucky?) student.
  15905.  
  15906. ==> geometry/tiling/rectangles.with.squares.p <==
  15907. Given two sorts of squares, (axa) and (bxb), what rectangles can be tiled?
  15908.  
  15909. ==> geometry/tiling/rectangles.with.squares.s <==
  15910. A rectangle can be tiled with (axa) and (bxb) squares,   iff
  15911.  
  15912. (i) gcd(a,b)=1 , and any of the following hold:
  15913.  
  15914. either:  both sides of the rectangle are multiples of a;
  15915.     or:  both sides of the rectangle are multiples of b;
  15916.     or:  one side is a multiple of (ab), and the other is any length EXCEPT
  15917.          one of a finite number of "bad" lengths: those numbers which are
  15918.          NOT positive integer combinations of a & b. { By Sylvester's theorem
  15919.          there are (a-1)(b-1)/2 of these, the largest being (a-1)(b-1)-1. }
  15920.  
  15921. (ii) gcd(a,b) = d .
  15922.      Then merely apply (i) to the problem with a,b replaced
  15923.      by a/d, b/d  and the rectangle lengths also divided by d.
  15924.      i.e.  all cells must appear in (dxd) subsquares.
  15925.  
  15926. ------
  15927. PROOF
  15928. It is clear that (ii) follows from (i), and that simple constructions give
  15929. the "if" part of (i). For the "only if" part, we prove that...
  15930.  
  15931. (S) If one side of the rectangle is not divisible by a, and the other is
  15932.     not divisible by b, then the tiling is impossible.
  15933.  
  15934. The results in (i) follow immediately from (S).
  15935.  
  15936. To prove (S):  ( Chakraborty-Hoey style ).
  15937.                  ~~~~~~~~~~~~~~~~
  15938. Let the width of the rectangle be a NON-(a-multiple). Then the number of
  15939. bxb squares starting (i.e. top edge) at row 1 must be a NON-a-multiple.
  15940. Thus the number of bxb starting at row 2 must BE an a-multiple. Similarly
  15941. for the number starting at rows 3,4,...,b . Then the number starting at
  15942. row (b+1) must be a NON-a-multiple again.
  15943.  
  15944. Similarly the number starting at rows (2b+1), (3b+1), (4b+1),... must all be
  15945. non-a-multiples. So if the number of rows is NOT a multiple of b, (call it
  15946. bx+r), then row (bx+1) must have a NON-a-multiple of bxb squares starting
  15947. there, i.e. at least one, and there is no room left to squeeze it in.     ▌QED¿
  15948. ----
  15949.  
  15950. A Rickard-style proof of (S) is    ..BBB....BBWWW...WBBB....BBWWW...W(..etc)
  15951.   ~~~~~~~   also possible, by      ..BBB....BBWWW...WBBB....BBWWW...W
  15952. coloring the rectangle in          ..BBB....BBWWW...WBBB....BBWWW...W
  15953. vertical strips as shown here:       <-  a  ->< b-a ><-  a  ->< b-a >
  15954.  
  15955. Every square tile covers an a-multiple of black squares. But if the width
  15956. is a NON-b-multiple, and the number of rows is a NON-a-multiple, then there
  15957. are a NON-a-multiple of black squares in total.    ▌QED¿
  15958.  
  15959. (Note: the coloring must have 1 column of blacks on the right, and any
  15960.  ====     spare columns of whites on the left.)
  15961.  
  15962. ===================
  15963.  
  15964. Bill Taylor.            wft@math.canterbury.ac.nz
  15965.  
  15966. >A Rickard-style proof of (S) is    ..BBB....BBWWW...WBBB....BBWWW...W(..etc)
  15967. >  ~~~~~~~   also possible, by      ..BBB....BBWWW...WBBB....BBWWW...W
  15968. >coloring the rectangle in          ..BBB....BBWWW...WBBB....BBWWW...W
  15969. >vertical strips as shown here:       <-  a  ->< b-a ><-  a  ->< b-a >
  15970. >
  15971. >Every square tile covers an a-multiple of black squares. But if the width
  15972. >is a NON-b-multiple, and the number of rows is a NON-a-multiple, then there
  15973. >are a NON-a-multiple of black squares in total.    ▌QED¿
  15974. >
  15975. >(Note: the coloring must have 1 column of blacks on the right, and any
  15976. > ====     spare columns of whites on the left.)
  15977.  
  15978. This statement of how to position the colouring isn't good enough, I'm
  15979. afraid. Take a=4, b=7 and consider e.g. a 19x10 rectangle. Coloured your
  15980. way, you get:
  15981.  
  15982.     BWWWBBBBWWWBBBBWWWB
  15983.     BWWWBBBBWWWBBBBWWWB
  15984.     :::::::::::::::::::
  15985.     BWWWBBBBWWWBBBBWWWB
  15986.     BWWWBBBBWWWBBBBWWWB
  15987.  
  15988. The result has 10*10=100 black squares in it, which *is* a multiple of a=4,
  15989. despite the fact that 19 is not a multiple of 7 and 10 is not a multiple of
  15990. 4.
  15991.  
  15992. Of course, there is an alternative offset for the pattern that does give you
  15993. the result you want:
  15994.  
  15995.     WWBBBBWWWBBBBWWWBBB
  15996.     WWBBBBWWWBBBBWWWBBB
  15997.     :::::::::::::::::::
  15998.     WWBBBBWWWBBBBWWWBBB
  15999.     WWBBBBWWWBBBBWWWBBB
  16000.  
  16001. To show this happens in general: because the width of the rectangle is a
  16002. non-multiple of b, it is possible to position it on the pattern so that the
  16003. leftmost column in the rectangle is white and the column just right of the
  16004. right edge of the rectangle is black. Suppose N columns are black with this
  16005. positioning. Then the rectangle contains N*H black cells, where H is the
  16006. height of the rectangle.
  16007.  
  16008. If we then shift the rectangle right by one, the number of black columns
  16009. increases by 1 and it contains (N+1)*H black cells. The difference between
  16010. these two numbers of black cells is H, which is not a multiple of a.
  16011. Therefore N*H and (N+1)*H cannot both be multiples of a, and so one of these
  16012. two positionings of the pattern will suit your purposes.
  16013.  
  16014. David Seal
  16015. dseal@armltd.co.uk
  16016.  
  16017. ==> geometry/tiling/scaling.p <==
  16018. A given rectangle can be entirely covered (i.e. concealed) by an
  16019. appropriate arrangement of 25 disks of unit radius.
  16020.  
  16021. Can the same rectangle be covered by 100 disks of 1/2 unit radius?
  16022.  
  16023. ==> geometry/tiling/scaling.s <==
  16024. Yes.  The same configuration of circles, when every distance is reduced
  16025. by half (including the diameters), will cover a similar rectangle whose
  16026. sides are one half of the original one.  The original rectangle is the
  16027. union of four such rectangles.
  16028.  
  16029. ==> geometry/tiling/seven.cubes.p <==
  16030. Consider 7 cubes of equal size arranged as follows. Place 5 cubes so
  16031. that they form a Swiss cross or a + (plus). ( 4 cubes on the sides and
  16032. 1 in the middle). Now place one cube on top of the middle cube and the
  16033. seventh below the middle cube, to effectively form a 3-dimensional
  16034. swiss cross.
  16035.  
  16036. Can a number of such blocks (of 7 cubes each) be arranged so that they
  16037. are able to completely fill up a big cube (say 10 times the size of
  16038. the small cubes)? It is all right if these blocks project out of the
  16039. big cube, but there should be no holes or gaps.
  16040.  
  16041. ==> geometry/tiling/seven.cubes.s <==
  16042. Let n be a positive integer.  Define the function f from Z^n to Z by
  16043. f(x) = x_1+2x_2+3x_3+...+nx_n.  For x in Z^n, say y is a neighbor of x
  16044. if y and x differ by one in exactly one coordinate.  Let S(x) be the
  16045. set consisting of x and its 2n neighbors.  It is easy to check that
  16046. the values of f(y) for y in S(x) are congruent to 0,1,2,...,2n+1 (mod
  16047. 2n+1) in some order.  Using this, it is easy to check that every y in
  16048. Z^n is a neighbor of one and only one x in Z^n such that f(x) is
  16049. congruent to 0 (mod 2n+1).  So Z^n can be tiled by clusters of the
  16050. form S(x), where f(x) is congruent to 0 mod 2n+1.
  16051.  
  16052. ==> group/group.01.p <==
  16053. AEFHIKLMNTVWXYZ BCDGJOPQRSU
  16054.  
  16055. ==> group/group.01.s <==
  16056. AEFHIKLMNTVWXYZ drawn with straight lines
  16057. BCDGJOPQRSU     not drawn with straight lines
  16058.  
  16059. ==> group/group.01a.p <==
  16060. 147 0235689
  16061.  
  16062. ==> group/group.01a.s <==
  16063. 147     drawn with straight lines
  16064. 0235689 not drawn with straight lines
  16065.  
  16066. ==> group/group.02.p <==
  16067. ABEHIKMNOPTXZ CDFGJLQRSUVWY
  16068.  
  16069. ==> group/group.02.s <==
  16070. ABEHIKMNOPTXZ   resembles Greek letter
  16071. CDFGJLQRSUVWY   does not resemble Greek letter
  16072.  
  16073. ==> group/group.03.p <==
  16074. BEJQXYZ DFGHLPRU KSTV CO AIW MN
  16075.  
  16076. ==> group/group.03.s <==
  16077.  
  16078. BEJQXYZ                 no state starting with this letter
  16079. DFGHLPRU                one state starting with this letter
  16080. KSTV                    two states starting with this letter
  16081. CO                      three states starting with this letter
  16082. AIW                     four states starting with this letter
  16083.                         five states starting with this letter
  16084.                         six states starting with this letter
  16085.                         seven states starting with this letter
  16086. MN                      eight states starting with this letter
  16087.  
  16088. ==> group/group.04.p <==
  16089. BDO P ACGJLMNQRSUVWZ EFTY HIKX
  16090.  
  16091. ==> group/group.04.s <==
  16092. BDO             no endpoint
  16093. P               one endpoint
  16094. ACGJLMNQRSUVWZ  two endpoints
  16095. EFTY            three endpoints
  16096. HIKX            four endpoints
  16097.  
  16098. ==> group/group.05.p <==
  16099. CEFGHIJKLMNSTUVWXYZ ADOPQR B
  16100.  
  16101. ==> group/group.05.s <==
  16102. CEFGHIJKLMNSTUVWXYZ     no enclosed area
  16103. ADOPQR                  one enclosed area
  16104. B                       two enclosed areas
  16105.  
  16106. ==> group/group.06.p <==
  16107. BCEGKMQSW DFHIJLNOPRTUVXYZ
  16108.  
  16109. ==> group/group.06.s <==
  16110. BCEGKMQSW               prime numbers
  16111. DFHIJLNOPRTUVXYZ        composi
  16112.  
  16113. ==> induction/hanoi.p <==
  16114. Is there an algorithom for solving the hanoi tower puzzle for any number
  16115. of towers?  Is there an equation for determining the minimum number of
  16116. moves required to solve it, given a variable number of disks and towers?
  16117.  
  16118. ==> induction/hanoi.s <==
  16119. The best way of thinking of the Towers of Hanoi problem is inductively.
  16120. To move n disks from post 1 to post 2, first move (n-1) disks
  16121. from post 1 to post 3, then move disk n from post 1 to post 2, then move
  16122. (n-1) disks from post 3 to post 2 (same procedure as moving (n-1) disks
  16123. from post 1 to post 3).  In order to figure out how to move (n-1) disks
  16124. from post 1 to post 3, first move (n-2) disks . . . .
  16125.  
  16126. As far as an algorithm which straightens out any legal position is
  16127. concerned, the algorithm would go something like this:
  16128.  
  16129. 1.  Find the smallest disk.  Call the post that it's on post 1.
  16130.  
  16131. 2.  Find the smallest disk which is not on post 1.  This disk is, say,
  16132. size k.  (I am calling the smallest disk size 1 here.)  Call the post
  16133. that disk k is on post 2.  Disks 1 through (k-1) are then all stacked up
  16134. correctly on post 1 disk k is on top of post 2.  This follow from the
  16135. fact that the disks are in a legal postition.
  16136.  
  16137. 3.  Move disks 1 through (k-1) from post 1 to post 2, ignoring the other
  16138. disks.  This is just the standard Tower of Hanoi problem for (k-1)
  16139. disks.
  16140.  
  16141. 4.  If the disks are not yet correctly arranged, repeat from step 1.
  16142.  
  16143. In fact, this gives the straightening with the fewest number of moves.
  16144.  
  16145. With 3 towers, for N number of disks, the formula for the minimum number of
  16146. moves to complete the puzzle correctly is:
  16147.                 (2^N) - 1
  16148.  
  16149. This bit of ancient folklore was invented by De Parville in 1884.
  16150.  
  16151. ``In the great temple at Benares, says he, beneath the dome which
  16152.   marks the centre of the world, rests a brass plate in which are
  16153.   fixed three diamond needles, each a cubit high and as thick as
  16154.   the body of a bee.  On one of these needles, at the creation,
  16155.   God placed sixty-four discs of pure gold, the largest disc resting
  16156.   on the brass plate, and the others getting smaller and smaller
  16157.   up to the top one.  This is the Tower of Bramah.  Day and night
  16158.   unceasingly the priests transfer the discs from one diamond needle
  16159.   to another according to the fixed and immutable laws of Bramah,
  16160.   which require that the priest on duty must not move more than one
  16161.   disc at a time and that he must place this disc on a needle so
  16162.   that there is no smaller disc below it.  When the sixty-four
  16163.   discs shall have been thus transferred from the needle on which
  16164.   at the creation God placed them to one of the other needles,
  16165.   tower, temple, and Brahmins alike will crumble into dust, and
  16166.   with a thunderclap the world will vanish.''  (W W R Ball,
  16167.   MATHEMATICAL RECREATIONS AND ESSAYS, p. 304)
  16168.  
  16169. This has been discussed by several authors, e.g.
  16170.  
  16171.     Er, Info Sci 42 (1987) 137-141.
  16172.     Graham, Knuth and Patashnik, _Concrete_Mathematics_.
  16173.  
  16174. There are many papers claiming to solve this, and they are probably
  16175. all correct but they rely on the unproven "Frame's conjecture".
  16176. In particular, for the 4 peg case the conjecture states that an optimal
  16177. solution begins by forming a substack of the k smallest discs, then moving
  16178. the rest, and then moving those k again; k to be determined.
  16179.  
  16180. Here is a extensible bc program that does the same work. The output
  16181. format is not that great. We get 300 numbers as output. The first
  16182. hundred represent N, the next 100 represent f(N) and the last hundred
  16183. represent i, which is the number of discs to move to tmp1 using f(N).
  16184. For convenience, I have here some values for N <= 48. Enjoy.
  16185.  
  16186. Sharma
  16187.  
  16188.  
  16189. N        1   2   3  4  5   6  7  8  9  10  11 12 13  14  15  16  17  18  19
  16190. f(N)     1   3   5  9 13  17 25 33 41  49  65 81 97 113 129 161 193 225 257
  16191. i        0   1   1  2  2   3  3  4  5   6   6  7  8   9  10  10  11  12  13
  16192.  
  16193.  
  16194. N       20   21  22  23  24  25  26  27  28  29  30   31   32   33   34   35
  16195. f(N)    289 321 385 449 513 577 641 705 769 897 1025 1153 1281 1409 1537 1665
  16196. i       14   15  15  16  17  18  19  20  21  21   22   23   24   25   26   27
  16197.  
  16198. N       36    37    38    39     40    41   42   43   44   45   46   47   48
  16199. f(N)    1793 2049  2305  2561   2817  3073 3329 3585 3841 4097 4609 5121 5633
  16200. i       28    28    29    30     31    32    33   34   35   36  36   37   38
  16201.  
  16202.  
  16203. /* This is the bc program that gives f(N) for 4 peg case */
  16204.  
  16205. w = 101; /* This represents the number of disks */
  16206.  
  16207. m▌0¿ = 0;
  16208. m▌1¿ = 1;
  16209. m▌2¿ = 3;
  16210. m▌3¿ = 5;
  16211. m▌4¿ = 9;
  16212. m▌5¿ = 13;
  16213. m▌6¿ = 17;
  16214.  
  16215. /* f(n) is the function that gives the min # of moves for 4 peg case */
  16216. define f(n) {
  16217.   return (m▌n¿);
  16218. }
  16219.  
  16220. /* g(n) is the function that fives the min # of moves for 3 peg case */
  16221. define g(n) {
  16222.   return (2^n - 1);
  16223. }
  16224.  
  16225. /* x(n) is the Optimization Routine */
  16226.  
  16227. define x(n) {
  16228.   auto j
  16229.   auto r
  16230.   auto i
  16231.  
  16232.   if(n == 1) return (1);
  16233.   j = f(1) + g(n-1);
  16234.  
  16235.   for(i = 2; i < n; i++) {
  16236.     r = f(i) + g(n-i);
  16237.     if(r < j) { j = r; d = i; }
  16238.   }
  16239.   return (j);
  16240. }
  16241.  
  16242. /* main program */
  16243. for(q = 4; q < w; q++) {
  16244.   t = x(q-1);
  16245.   m▌q¿ = 2 * t + 1;
  16246.   d▌q¿ = d;
  16247. };
  16248.  
  16249.  
  16250. /*This for loop prints the number of discs from 1 <= n <=  w*/
  16251.  
  16252. for(q = 1; q < w; q++) {
  16253. q;
  16254. }
  16255.  
  16256. /*This for loop prints f(n) for 1 <= n <= w */
  16257.  
  16258. for(q = 1; q < w; q++) {
  16259. m▌q¿;
  16260. }
  16261.  
  16262. /*This for loop prints i for 1 <= n <= w
  16263. i represents the number of disks to be moved to tmp location using f(n)
  16264. N-i-1 will be moved using g(n) */
  16265.  
  16266. for(q = 1; q < w; q++) {
  16267. d▌q¿;
  16268. }
  16269.  
  16270. --
  16271. sharma@sharma.warren.mentorg.com
  16272.  
  16273. ==> induction/n-sphere.p <==
  16274. With what odds do three random points on an n-sphere form an acute triangle?
  16275.  
  16276. ==> induction/n-sphere.s <==
  16277. Select three points a, b, and c, randomly with respect to the surface of an
  16278. n-sphere.  These three points determine a fourth, x, which is the intersection
  16279. of the sphere with the axis perpendicular to the abc plane.  (Choose the pole
  16280. nearest the plane.) I could have, just as easily, selected x, a distance d
  16281. from x, and three points d units away from x.  The distribution of d is not
  16282. uniform, but that's ok.  For every x and d, the three points abc form an acute
  16283. triangle with probability p▌n-1¿.  By induction, p▌n¿ = 1/4.
  16284.  
  16285. ==> induction/paradox.p <==
  16286. What simple property holds for the first 10,000 integers, then fails?
  16287.  
  16288. ==> induction/paradox.s <==
  16289. Consider the sequences defined by:
  16290. s(1) = a; s(2) = b; s(n) = least integer such that s(n)/s(n-1) > s(n-1)/s(n-2).
  16291. In other words, s(n) = 1+floor(s(n-1)^2/s(n-2)) for n >= 3.  These
  16292. sequences are similar in some ways to the classically-studied Pisot
  16293. sequences.  For example, if a = 1, b = 2, then we get the odd-indexed
  16294. Fibonacci numbers.
  16295.  
  16296. D. Boyd of UBC, an expert in Pisot sequences, pointed out the following.
  16297. If we let a = 8, b = 55 in the definition above, then the resulting
  16298. sequence s(n) appears to satisfy the following linear recurrence
  16299. of order 4:
  16300.  
  16301.         s(n) = 6s(n-1) + 7s(n-2) - 5s(n-3) - 6s(n-4)
  16302.  
  16303. Indeed, it does satisfy this linear recurrence for the
  16304. first 11,056 terms.  However, it fails at the 11,057th term]
  16305. And s(11057) is a 9270 digit number.
  16306. (The reason for this coincidence depends on a remarkable fact
  16307. about the absolute values of the roots of the polynomial
  16308. x^4 - 6x^3 - 7x^2 + 5x + 6.)
  16309.  
  16310. ==> induction/party.p <==
  16311. You're at a party.  Any two (different) people at the party have exactly one
  16312. friend in common (the friend is also at the party).  Prove that there is at
  16313. least one person at the party who is a friend of everyone else.  Assume that
  16314. the friendship relation is symmetric and not reflexive.
  16315.  
  16316. ==> induction/party.s <==
  16317. Here is an easy solution by induction. Let P be the set of people in the
  16318. party, and n the size of P. If n=2 or 3, the result is trivial. Suppose now
  16319. that n>3 and that the result is true for n-1.
  16320.  
  16321. For any two distinct x,y in P, write x & y to mean that `x is a friend of y',
  16322. and x ~& y to mean that `x is not a friend of y'.
  16323.  
  16324. Take q in P. The hypothesis on the relation & is still satisfied on P-{q}; by
  16325. induction, the result is thus true for P-{q}, and there is some p in P-{q}
  16326. such that p & x for any x in P-{p,q}. We have two cases:
  16327.  
  16328. a) p & q. Then the result holds for P with p.
  16329.  
  16330. b) p ~& q. By hypothesis, there is a unique r in P-{p,q} such that p & r & q.
  16331. For any x in P-{p,q}, if x & q, then p & x & q, and so x=r. Thus r is the
  16332. unique friend of q. Now for any s in P-{q,r} there exists some x such that s &
  16333. x & q, and so x=r. This means that r & s for any s in P-{q,r}, and as r & q,
  16334. the results holds in P with r.
  16335.  
  16336. The problem can also be solved by applying the spectral theory of graphs
  16337. (see for instance Bollobas' excellent book, _Extremal Graph Theory_).
  16338.  
  16339. The problem's condition is vacuous if there is only N=1 person at the "party",
  16340. impossible if N=2 (If you aren't your own friend, nor I mine, somebody *else*
  16341. must be our mutual friend), and trivial if N=3 (everybody must be everyone
  16342. else's friend).  Henceforth assume N>3.
  16343.  
  16344. Let A,B be two friends, and C their mutual friend.  Let a be the number
  16345. of A's friends other than B and C, and likewise b,c.  Each of A's friends
  16346. is also friendly with exactly one other of A's friends, and with none of
  16347. B and C's other friends (if A1,B1 are friends of A,B resp. and of each other
  16348. then A1 and B have more than one mutual friend); likewise for B and C.
  16349. Let M=N-(a+b+c+3) be the number of people not friendly with any of A,B,C.
  16350. Each of them is friendly with exactly one of A's and one of B's friends;
  16351. and each pair of a friend of A and a friend of B must have exactly
  16352. one of them as a mutual friend.  Thus M=ab; likewise M=ab=ac=bc. Thus
  16353. either M and two of a,b,c vanish, or a=b=c=k (say), M=k^2, and N=k^3+3k+3.
  16354. In the first case, say b=c=0; necessarily a is even, and A is a friend of
  16355. everybody else at the party, each of whom is friendly with exactly one other
  16356. person; clearly any such configuration (a graph of k/2+1 triangles with a
  16357. common vertex) satisfies the problem's conditions).
  16358.  
  16359. It remains to show that the second case is impossible.  Since N=k^2+3k+3
  16360. does not depend on A,B,C, neither does k, and it quickly follows that the
  16361. party's friendship graph is regular with reduced matrix
  16362.  
  16363.              ▌  0   k+2   0  ¿
  16364.              ▌  1    1    k  ¿
  16365.              ▌  0    1   k+1 ¿
  16366.  
  16367. and eigenvalues k+2 and +-sqrt(k+1) and multiplicities 1,m1,m2 for some
  16368. *integers* m1 and m2 such that (m1-m2)*sqrt(k+1)=-(k+2) (because the graph's
  16369. matrix has trace zero).  Thus sqrt(k+1) divides k+2 and k+1 divides
  16370.  
  16371.             (k+2)^2=(k+1)(k+3)+1
  16372.  
  16373. which is only possible if k=0,  Q.E.D.
  16374.  
  16375. ==> induction/roll.p <==
  16376. An ordinary die is thrown until the running total of the throws first
  16377. exceeds 12.  What is the most likely final total that will be obtained?
  16378.  
  16379. ==> induction/roll.s <==
  16380. Claim: If you throw a die until the running total exceeds n>=5, a final
  16381. outcome of n+1 is more likely than any other.
  16382.  
  16383. Assume we throw an m for a total n+k>n+1, and assume m-k>=0.  Now, it
  16384. is just as likely to throw an m as an m-k+1, which means that the sum
  16385. n+1 is just as likely as any other.  Now consider the series of throws
  16386. consisting of n-5 1's followed by a 6 and note that we cannot achieve
  16387. more than an n+1 by changing the last die roll.  Hence, a total of n+1
  16388. is more likely than any other.
  16389.  
  16390. ==> induction/takeover.p <==
  16391. After graduating from college, you have taken an important managing position
  16392. in the prestigious financial firm of "Mary and Lee".
  16393. You are responsable for all the decisions concerning take-over bids.
  16394. Your immediate concern is whether to take over "Financial Data".
  16395. There is no doubt that you will be successful if you are the first to
  16396. bid and that this will be profitable for the firm and you in the long
  16397. run.  However, you know that there exist another n financial firms,
  16398. similar to "Mary and Lee", that are also considering the possibility.
  16399. Although you are likely to be the first one to move, you know that
  16400. just after a take-over there is a lot of adjustment that needs to be
  16401. done.  In fact, for a period of time following any take-over the
  16402. successful firm becomes a prime candidate for a take-over which will
  16403. cost the job of whoever is responsable for take-overs.  Among all
  16404. financial firms it is common knowledge that the managers responsable
  16405. for take-overs are rational and intelligent.  What is your best response?
  16406.  
  16407. ==> induction/takeover.s <==
  16408. Assume the takeover is wise for n.  The takeover is then unwise for
  16409. n+1, as the other companies now find themselves in the same situation
  16410. as you for n.  If the decision is unwise for n, by similar reasoning
  16411. it is wise to takeover FD for n+1.  Now note that for n=1 the takeover
  16412. decision is clearly unwise, hence by induction you should takeover
  16413. FD iff n is even.
  16414.  
  16415. ==> logic/29.p <==
  16416. Three people check into a hotel.  They pay $30 to the manager and go
  16417. to their room.  The manager finds out that the room rate is $25 and
  16418. gives $5 to the bellboy to return.  On the way to the room the bellboy
  16419. reasons that $5 would be difficult to share among three people so
  16420. he pockets $2 and gives $1 to each person.
  16421.  
  16422. Now each person paid $10 and got back $1.  So they paid $9 each,
  16423. totalling $27.  The bellboy has $2, totalling $29.
  16424.  
  16425. Where is the remaining dollar?
  16426.  
  16427. ==> logic/29.s <==
  16428. Each person paid $9, totalling $27.  The manager has $25 and the bellboy $2.
  16429. The bellboy's $2 should be added to the manager's $25 or subtracted from
  16430. the tenants' $27, not added to the tenants' $27.
  16431.  
  16432. ==> logic/ages.p <==
  16433. 1) Ten years from now Tim will be twice as old as Jane was when Mary was
  16434.    nine times as old as Tim.
  16435.  
  16436. 2) Eight years ago, Mary was half as old as Jane will be when Jane is one year
  16437.    older than Tim will be at the time when Mary will be five times as old as
  16438.    Tim will be two years from now.
  16439.  
  16440. 3) When Tim was one year old, Mary was three years older than Tim will be when
  16441.    Jane is three times as old as Mary was six years before the time when Jane
  16442.    was half as old as Tim will be when Mary will be ten years older than Mary
  16443.    was when Jane was one-third as old as Tim will be when Mary will be three
  16444.    times as old as she was when Jane was born.
  16445.  
  16446.                              HOW OLD ARE THEY NOW?
  16447.  
  16448. ==> logic/ages.s <==
  16449. The solution: Tim is 3, Jane is 8, and Mary is 15.  A little grumbling
  16450. is in order here, as clue number 1 leads to the situation a year and a
  16451. half ago, when Tim was 1 1/2, Jane was 6 1/2, and Mary was 13 1/2.
  16452.  
  16453. This sort of problem is easy if you write down a set of equations.  Let
  16454. t be the year that Tim was born, j be the year that Jane was born, m be
  16455. the year that Mary was born, and y be the current year.  As indefinite
  16456. years come up, let y1, y2, ... be the indefinite years.  Then the
  16457. equations are
  16458.  
  16459.  
  16460. y + 10 - t = 2 (y1 - j)
  16461. y1 - m = 9 (y1 - t)
  16462.  
  16463. y - 8 - m = 1/2 (y2 - j)
  16464. y2 - j = 1 + y3 - t
  16465. y3 - m = 5 (y + 2 - t)
  16466.  
  16467. t + 1 - m = 3 + y4 - t
  16468. y4 - j = 3 (y5 - 6 - m)
  16469. y5 - j = 1/2 (y6 - t)
  16470. y6 - m = 10 + y7 - m
  16471. y7 - j = 1/3 (y8 - t)
  16472. y8 - m = 3 (j - m)
  16473.  
  16474. t = y - 3
  16475. j = y - 8
  16476. m = y - 15
  16477.  
  16478. ==> logic/bookworm.p <==
  16479. A bookworm eats from the first page of an encyclopedia to the last page.
  16480. The bookworm eats in a straight line.  The encyclopedia consists of ten
  16481. 1000-page volumes.  Not counting covers, title pages, etc., how many pages
  16482. does the bookworm eat through?
  16483.  
  16484. ==> logic/bookworm.s <==
  16485. On a book shelf the first page of the first volume is on the "inside"
  16486.   __                             __
  16487. B!  !                           !  !F
  16488. A!1 !...........................!10!R
  16489. C!  !                           !  !O
  16490. K!  !                           !  !N
  16491.  !  !                           !  !T
  16492.   ----------------------------------
  16493. so the bookworm eats only through the cover of the first volume, then 8 times
  16494. 1000 pages of Volumes 2 - 9, then through the cover to the 1st page of Vol 10.
  16495. He eats 8,000 pages.
  16496.  
  16497. ==> logic/boxes.p <==
  16498. Which Box Contains the Gold?
  16499.  
  16500. Two boxes are labeled "A" and "B".  A sign on box A says "The sign
  16501. on box B is true and the gold is in box A".  A sign on box B says
  16502. "The sign on box A is false and the gold is in box A".  Assuming there
  16503. is gold in one of the boxes, which box contains the gold?
  16504.  
  16505. ==> logic/boxes.s <==
  16506. The problem cannot be solved with the information given.
  16507.  
  16508. The sign on box A says "The sign on box B is true and the gold is in box A".
  16509. The sign on box B says "The sign on box A is false and the gold is in box A".
  16510. The following argument can be made: If the statement on box A is true, then
  16511. the statement on box B is true, since that is what the statement on box A
  16512. says.  But the statement on box B states that the statement on box A is false,
  16513. which contradicts the original assumption.  Therefore, the statement on box A
  16514. must be false.  This implies that either the statement on box B is false or
  16515. that the gold is in box B.  If the statement on box B is false, then either
  16516. the statement on box A is true (which it cannot be) or the gold is in box B.
  16517. Either way, the gold is in box B.
  16518.  
  16519. However, there is a hidden assumption in this argument: namely, that
  16520. each statement must be either true or false.  This assumption leads to
  16521. paradoxes, for example, consider the statement: "This statement is
  16522. false."  If it is true, it is false; if it is false, it is true.  The
  16523. only way out of the paradox is to deny that the statement is either true
  16524. or false and label it meaningless instead.  Both of the statements on the
  16525. boxes are therefore meaningless and nothing can be concluded from them.
  16526.  
  16527. In general, statements about the truth of other statements lead to
  16528. contradictions.  Tarski invented metalanguages to avoid this problem.
  16529. To avoid paradox, a statement about the truth of a statement in a language
  16530. must be made in the metalanguage of the language.
  16531.  
  16532. Common sense dictates that this problem cannot be solved with the information
  16533. given.  After all, how can we deduce which box contains the gold simply by
  16534. reading statements written on the outside of the box?  Suppose we deduce that
  16535. the gold is in box B by whatever line of reasoning we choose.  What is to stop
  16536. us from simply putting the gold in box A, regardless of what we deduced?
  16537. (cf. Smullyan, "What Is the Name of This Book?", Prentice-Hall, 1978,   #70)
  16538.  
  16539. ==> logic/calibans.will.p <==
  16540.         ----------------------------------------------
  16541.         !       Caliban's Will by M.H. Newman        !
  16542.         ----------------------------------------------
  16543.  
  16544. When Caliban's will was opened it was found to contain the following
  16545. clause:
  16546.  
  16547. "I leave ten of my books to each of Low, Y.Y., and 'Critic,' who are
  16548. to choose in a certain order.
  16549.  
  16550. No person who has seen me in a green tie is to choose before Low.
  16551.  
  16552. If Y.Y. was not in Oxford in 1920 the first chooser never lent me
  16553. an umbrella.
  16554.  
  16555. If Y.Y. or 'Critic' has second choice, 'Critic' comes before the one
  16556. who first fell in love."
  16557.  
  16558. Unfortunately Low, Y.Y., and 'Critic' could not remember any of the
  16559. relevant facts; but the family solicitor pointed out that, assuming the
  16560. problem to be properly constructed (i.e. assuming it to contain no
  16561. statement superfluous to its solution) the relevant data and order
  16562. could be inferred.
  16563.  
  16564. What was the prescribed order of choosing; and who lent Caliban an
  16565. umbrella?
  16566.  
  16567. ==> logic/calibans.will.s <==
  16568. Let T be "person who saw Caliban in a green tie."
  16569. Let U be "person who lent Caliban an umbrella."
  16570. Then the data are:
  16571. (1) No T chooses before Low.
  16572. (2) Either Y.Y. was in Oxford in March 1920 or the first chooser is not
  16573.     a U.
  16574. (3) Either Low is second or Critic is not last.
  16575.  
  16576. Consider first (3)
  16577. If it could be shown that Low is first, then from (3), Critic is not
  16578. last and therefore is second; i.e. the order is Low, Critic, Y.Y.
  16579.  
  16580. Next (1)
  16581. If both Critic and Y.Y. were T's would require Low first and (3) then
  16582. gives the order Low-Critic-Y.Y., ie. (2) would be superfluous.  Hence
  16583. Critic and Y.Y. are not both T's.
  16584.  
  16585. If neither Critic nor Y.Y. were a T, (1) would be trivially true for
  16586. any ordering and therefore would give no information, i.e. would be
  16587. superfluous.  Hence just one of Y.Y. and Critic is a T.  It follows
  16588. that the only possible order in which Low is not first is:
  16589.  
  16590.         Not T, Low, T
  16591.  
  16592. Now (2)
  16593. First if Y.Y was in Oxford in March 1920, nothing follows from (2)
  16594. about the order and (2) is superfluous.  Hence Y.Y. was not in Oxford.
  16595. If Low were a U he would not, by (2) come first, and so by (1) the
  16596. order would be:
  16597.  
  16598.         Not T, Low, T
  16599.  
  16600. i.e. (1) and (2) alone would fix an order, and (3) would be superfluous.
  16601. Hence Low is not a U.
  16602.  
  16603. It now follows, by the arguments just given for T's under (1) that just
  16604. one of Y.Y. and Critic is a U.  If the same one is the T and the U (2)
  16605. follows from (1) (since Low is not a U); i.e (2) is superfluous.  The
  16606. situation is therefore:
  16607.         T's: just one of Y.Y. and Critic; not Low
  16608.         U's: the other one of Y.Y; not Low
  16609. It now follows that "not T, Low, T" is impossible, for the "not T" is
  16610. the "U" and therefore, by (2), is not first.  Hence Low is first, and
  16611. (3) gives the order:
  16612.         Low, Critic, Y.Y.
  16613.  
  16614. Finally, Y.Y. is a T, and Critic is a U.  For if Critic is a T, then
  16615. by (1) Low precedes Critic and hence (3) allows only "Low, Critic, Y.Y";
  16616. (2) is superfluous.  I.e. Critic (only) lent Caliban an umbrella.
  16617.  
  16618. The problem is from _Problems Omnibus_ by Hubert Phillips,
  16619. Arco Publications, London, 1960.  Hubert Phillips was a noted puzzelist
  16620. who contributed under his own name and the pseudonyms of "Caliban",
  16621. "T.O. Hare", and "The Doc".
  16622.  
  16623. ==> logic/camel.p <==
  16624. An Arab sheikh tells his two sons that are to race their camels to a
  16625. distant city to see who will inherit his fortune.  The one whose camel
  16626. is slower will win.  The brothers, after wandering aimlessly for days,
  16627. ask a wiseman for advise.  After hearing the advice they jump on the
  16628. camels and race as fast as they can to the city.  What did the wiseman
  16629. say?
  16630.  
  16631. ==> logic/camel.s <==
  16632. The wiseman tells them to switch camels.
  16633.  
  16634. ==> logic/centrifuge.p <==
  16635. You are a biochemist, working with a 12-slot centrifuge.  This is a gadget
  16636. that has 12 equally spaced slots around a central axis, in which you can
  16637. place chemical samples you want centrifuged.  When the machine is turned on,
  16638. the samples whirl around the central axis and do their thing.
  16639.  
  16640. To ensure that the samples are evenly mixed, they must be distributed in the
  16641. 12 slots such that the centrifuge is balanced evenly.  For example, if you
  16642. wanted to mix 4 samples, you could place them in slots 12, 3, 6 and 9
  16643. (assuming the slots are numbered from 1 to 12 like a clock).
  16644.  
  16645. Problem:  Can you use the centrifuge to mix 5 samples?
  16646.  
  16647. ==> logic/centrifuge.s <==
  16648. The superposition of any two solutions is yet another solution, so given
  16649. that the factors > 1 of 12 (2, 3, 4, 6, 12) are all solutions, the
  16650. only thing to check about, for example, the proposed solution 2+3 is
  16651. that not all ways of combining 2 & 3 would have centrifuge tubes
  16652. from one subsolution occupying the slot for one of the tubes in
  16653. another solution.  For the case 2+3, there is no problem:  Place 3
  16654. tubes, one in every 4th position, then place the 4th and 5th
  16655. diametrically opposed (each will end up in a slot adjacent to one of
  16656. the first 3 tubes).  The obvious generalization is, what are the
  16657. numbers of tubes that cannot be balanced?  Observing that there are
  16658. solutions for 2,3,4,5,6 tubes and that if X has a solution, 12-X has
  16659. also one (obtained by swapping tubes and holes), it is obvious that
  16660. 1 and 11 are the only cases without solutions.
  16661.  
  16662. Here is how this problem is often solved in practice:  A dummy tube
  16663. is added to produce a total number of tubes that is easy to balance.
  16664. For example, if you had to centrifuge just one sample, you'd add a
  16665. second tube opposite it for balance.
  16666.  
  16667. ==> logic/children.p <==
  16668. A man walks into a bar, orders a drink, and starts chatting with the
  16669. bartender.  After a while, he learns that the bartender has three
  16670. children.  "How old are your children?" he asks.  "Well," replies the
  16671. bartender, "the product of their ages is 72."  The man thinks for a
  16672. moment and then says, "that's not enough information."  "All right,"
  16673. continues the bartender, "if you go outside and look at the building
  16674. number posted over the door to the bar, you'll see the sum of the
  16675. ages."  The man steps outside, and after a few moments he reenters and
  16676. declares, "Still not enough]"  The bartender smiles and says, "My
  16677. youngest just loves strawberry ice cream."
  16678.  
  16679. How old are the children?
  16680.  
  16681. A variant of the problem is for the sum of the ages to be 13 and the
  16682. product of the ages to be the number posted over the door.  In this
  16683. case, it is the oldest that loves ice cream.
  16684.  
  16685. Then how old are they?
  16686.  
  16687.  
  16688. ==> logic/children.s <==
  16689. First, determine all the ways that three ages can multiply together to get 72:
  16690.  
  16691. 72  1  1        (quite a feat for the bartender)
  16692. 36  2  1
  16693. 24  3  1
  16694. 18  4  1
  16695. 18  2  2
  16696. 12  6  1
  16697. 12  3  2
  16698. 9  4  2
  16699. 9  8  1
  16700. 8  3  3
  16701. 6  6  2
  16702. 6  4  3
  16703.  
  16704. As the man says, that's not enough information; there are many possibilities.
  16705. So the bartender tells him where to find the sum of the ages--the man now knows
  16706. the sum even though we don't.  Yet he still insists that there isn't enough
  16707. info.  This must mean that there are two permutations with the same sum;
  16708. otherwise the man could have easily deduced the ages.
  16709.  
  16710. The only pair of permutations with the same sum are 8 3 3 and 6 6 2, which both
  16711. add up to 14 (the bar's address).  Now the bartender mentions his
  16712. "youngest"--telling us that there is one child who is younger than the other
  16713. two.  This is impossible with 8 3 3--there are two 3 year olds.  Therefore the
  16714. ages of the children are 6, 6, and 2.
  16715.  
  16716. Pedants have objected that the problem is insoluble because there could be
  16717. a youngest between two three year olds (even twins are not born exactly at
  16718. the same time).  However, the word "age" is frequently used to denote the
  16719. number of years since birth.  For example, I am the same age as my wife,
  16720. even though technically she is a few months older than I am.  And using the
  16721. word "youngest" to mean "of lesser age" is also in keeping with common parlance.
  16722. So I think the solution is fine as stated.
  16723.  
  16724. In the sum-13 variant, the possibilities are:
  16725.  
  16726. 11  1  1
  16727. 10  2  1
  16728. 9  3  1
  16729. 9  2  2
  16730. 8  4  1
  16731. 8  3  2
  16732. 7  5  1
  16733. 7  4  2
  16734. 7  3  3
  16735. 6  6  1
  16736. 6  5  2
  16737. 6  4  3
  16738.  
  16739. The two that remain are 9 2 2 and 6 6 1 (both products equal 36).  The
  16740. final bit of info (oldest child) indicates that there is only one
  16741. child with the highest age.  This cancels out the 6 6 1 combination, leaving
  16742. the childern with ages of 9, 2, and 2.
  16743.  
  16744. ==> logic/condoms.p <==
  16745. How can you have mutually safe sex with three women with only two condoms?
  16746.  
  16747. ==> logic/condoms.s <==
  16748. Use both condoms on the first woman.  Take off the outer condom (turning it
  16749. inside-out in the process) and set it aside.  Use the inner condom alone on
  16750. the second woman.  Put the outer condom back on.  Use it on the third woman.
  16751.  
  16752. ==> logic/dell.p <==
  16753. How can I solve logic puzzles (e.g., as published by Dell) automatically?
  16754.  
  16755. ==> logic/dell.s <==
  16756. #include        <setjmp.h>
  16757.  
  16758. #define EITHER          if (S▌1¿ = S▌0¿, ] setjmp((S++)->jb)) {
  16759. #define OR              } else EITHER
  16760. #define REJECT          longjmp((--S)->jb, 1)
  16761. #define END_EITHER      } else REJECT;
  16762.  
  16763. /* values in tmat: */
  16764. #define T_UNK   0
  16765. #define T_YES   1
  16766. #define T_NO    2
  16767.  
  16768. #define Val(t1,t2)      (S->tmat▌t1¿▌t2¿)
  16769. #define CLASS(x)                \
  16770.                 (((x) / NUM_ITEM) * NUM_ITEM)
  16771. #define EVERY_TOKEN(x)          \
  16772.                 (x = 0; x < TOT_TOKEN; x++)
  16773. #define EVERY_ITEM(x, class)    \
  16774.                 (x = CLASS(class); x < CLASS(class) + NUM_ITEM; x++)
  16775.  
  16776. #define BEGIN                                           \
  16777. struct state {                                          \
  16778.         char    tmat▌TOT_TOKEN¿▌TOT_TOKEN¿;             \
  16779.         jmp_buf jb;                                     \
  16780. } States▌100¿, *S = States;                             \
  16781.                                                         \
  16782. main()                                                  \
  16783. {                                                       \
  16784.         int     token;                                  \
  16785.                                                         \
  16786.         for EVERY_TOKEN(token)                          \
  16787.                 yes(token, token);                      \
  16788.         EITHER
  16789.  
  16790. /* Here is the problem-specific data */
  16791. #define NUM_ITEM        5
  16792. #define NUM_CLASS       6
  16793. #define TOT_TOKEN       (NUM_ITEM * NUM_CLASS)
  16794.  
  16795. #define HOUSE_0         0
  16796. #define HOUSE_1         1
  16797. #define HOUSE_2         2
  16798. #define HOUSE_3         3
  16799. #define HOUSE_4         4
  16800.  
  16801. #define ENGLISH         5
  16802. #define SPANISH         6
  16803. #define NORWEG          7
  16804. #define UKRAIN          8
  16805. #define JAPAN           9
  16806.  
  16807. #define GREEN           10
  16808. #define RED             11
  16809. #define IVORY           12
  16810. #define YELLOW          13
  16811. #define BLUE            14
  16812.  
  16813. #define COFFEE          15
  16814. #define TEA             16
  16815. #define MILK            17
  16816. #define OJUICE          18
  16817. #define WATER           19
  16818.  
  16819. #define DOG             20
  16820. #define SNAIL           21
  16821. #define FOX             22
  16822. #define HORSE           23
  16823. #define ZEBRA           24
  16824.  
  16825. #define OGOLD           25
  16826. #define PLAYER          26
  16827. #define CHESTER         27
  16828. #define LSTRIKE         28
  16829. #define PARLIA          29
  16830.  
  16831. char *names▌¿ = {
  16832.         "HOUSE_0", "HOUSE_1", "HOUSE_2", "HOUSE_3", "HOUSE_4",
  16833.         "ENGLISH", "SPANISH", "NORWEG", "UKRAIN", "JAPAN",
  16834.         "GREEN", "RED", "IVORY", "YELLOW", "BLUE",
  16835.         "COFFEE", "TEA", "MILK", "OJUICE", "WATER",
  16836.         "DOG", "SNAIL", "FOX", "HORSE", "ZEBRA",
  16837.         "OGOLD", "PLAYER", "CHESTER", "LSTRIKE", "PARLIA",
  16838. };
  16839.  
  16840.         BEGIN
  16841.  
  16842.         yes(ENGLISH, RED);      /* Clue 1 */
  16843.         yes(SPANISH, DOG);      /* Clue 2 */
  16844.         yes(COFFEE, GREEN);     /* Clue 3 */
  16845.         yes(UKRAIN, TEA);       /* Clue 4 */
  16846.  
  16847.         EITHER                  /* Clue 5 */
  16848.                 yes(IVORY, HOUSE_0);
  16849.                 yes(GREEN, HOUSE_1);
  16850.         OR
  16851.                 yes(IVORY, HOUSE_1);
  16852.                 yes(GREEN, HOUSE_2);
  16853.         OR
  16854.                 yes(IVORY, HOUSE_2);
  16855.                 yes(GREEN, HOUSE_3);
  16856.         OR
  16857.                 yes(IVORY, HOUSE_3);
  16858.                 yes(GREEN, HOUSE_4);
  16859.         END_EITHER
  16860.  
  16861.         yes(OGOLD, SNAIL);      /* Clue 6 */
  16862.         yes(PLAYER, YELLOW);    /* Clue 7 */
  16863.         yes(MILK, HOUSE_2);     /* Clue 8 */
  16864.         yes(NORWEG, HOUSE_0);   /* Clue 9 */
  16865.  
  16866.         EITHER                  /* Clue 10 */
  16867.                 yes(CHESTER, HOUSE_0);
  16868.                 yes(FOX, HOUSE_1);
  16869.         OR
  16870.                 yes(CHESTER, HOUSE_4);
  16871.                 yes(FOX, HOUSE_3);
  16872.         OR
  16873.                 yes(CHESTER, HOUSE_1);
  16874.                 EITHER  yes(FOX, HOUSE_0);
  16875.                 OR      yes(FOX, HOUSE_2);
  16876.                 END_EITHER
  16877.         OR
  16878.                 yes(CHESTER, HOUSE_2);
  16879.                 EITHER  yes(FOX, HOUSE_1);
  16880.                 OR      yes(FOX, HOUSE_3);
  16881.                 END_EITHER
  16882.         OR
  16883.                 yes(CHESTER, HOUSE_3);
  16884.                 EITHER  yes(FOX, HOUSE_2);
  16885.                 OR      yes(FOX, HOUSE_4);
  16886.                 END_EITHER
  16887.         END_EITHER
  16888.  
  16889.         EITHER                  /* Clue 11 */
  16890.                 yes(PLAYER, HOUSE_0);
  16891.                 yes(HORSE, HOUSE_1);
  16892.         OR
  16893.                 yes(PLAYER, HOUSE_4);
  16894.                 yes(HORSE, HOUSE_3);
  16895.         OR
  16896.                 yes(PLAYER, HOUSE_1);
  16897.                 EITHER  yes(HORSE, HOUSE_0);
  16898.                 OR      yes(HORSE, HOUSE_2);
  16899.                 END_EITHER
  16900.         OR
  16901.                 yes(PLAYER, HOUSE_2);
  16902.                 EITHER  yes(HORSE, HOUSE_1);
  16903.                 OR      yes(HORSE, HOUSE_3);
  16904.                 END_EITHER
  16905.         OR
  16906.                 yes(PLAYER, HOUSE_3);
  16907.                 EITHER  yes(HORSE, HOUSE_2);
  16908.                 OR      yes(HORSE, HOUSE_4);
  16909.                 END_EITHER
  16910.         END_EITHER
  16911.  
  16912.         yes(LSTRIKE, OJUICE);   /* Clue 12 */
  16913.         yes(JAPAN, PARLIA);     /* Clue 13 */
  16914.  
  16915.         EITHER                  /* Clue 14 */
  16916.                 yes(NORWEG, HOUSE_0);
  16917.                 yes(BLUE, HOUSE_1);
  16918.         OR
  16919.                 yes(NORWEG, HOUSE_4);
  16920.                 yes(BLUE, HOUSE_3);
  16921.         OR
  16922.                 yes(NORWEG, HOUSE_1);
  16923.                 EITHER  yes(BLUE, HOUSE_0);
  16924.                 OR      yes(BLUE, HOUSE_2);
  16925.                 END_EITHER
  16926.         OR
  16927.                 yes(NORWEG, HOUSE_2);
  16928.                 EITHER  yes(BLUE, HOUSE_1);
  16929.                 OR      yes(BLUE, HOUSE_3);
  16930.                 END_EITHER
  16931.         OR
  16932.                 yes(NORWEG, HOUSE_3);
  16933.                 EITHER  yes(BLUE, HOUSE_2);
  16934.                 OR      yes(BLUE, HOUSE_4);
  16935.                 END_EITHER
  16936.         END_EITHER
  16937.  
  16938. /* End of problem-specific data */
  16939.  
  16940.                 solveit();
  16941.         OR
  16942.                 printf("All solutions found\n");
  16943.                 exit(0);
  16944.         END_EITHER
  16945. }
  16946.  
  16947. no(a1, a2)
  16948. {
  16949.         int     non1, non2, token;
  16950.  
  16951.         if (Val(a1, a2) == T_YES)
  16952.                 REJECT;
  16953.         else if (Val(a1, a2) == T_UNK) {
  16954.                 Val(a1, a2) = T_NO;
  16955.                 no(a2, a1);
  16956.                 non1 = non2 = -1;
  16957.  
  16958.                 for EVERY_ITEM(token, a1)
  16959.                         if (Val(token, a2) ]= T_NO)
  16960.                                 if (non1 == -1)
  16961.                                         non1 = token;
  16962.                                 else
  16963.                                         break;
  16964.                 if (non1 == -1)
  16965.                         REJECT;
  16966.                 else if (token == CLASS(a1) + NUM_ITEM)
  16967.                         yes(non1, a2);
  16968.  
  16969.                 for EVERY_TOKEN(token)
  16970.                         if (Val(token, a1) == T_YES)
  16971.                                 no(a2, token);
  16972.         }
  16973. }
  16974.  
  16975. yes(a1, a2)
  16976. {
  16977.         int     token;
  16978.  
  16979.         if (Val(a1, a2) == T_NO)
  16980.                 REJECT;
  16981.         else if (Val(a1, a2) == T_UNK) {
  16982.                 Val(a1, a2) = T_YES;
  16983.                 yes(a2, a1);
  16984.                 for EVERY_ITEM(token, a1)
  16985.                         if (token ]= a1)
  16986.                                 no(token, a2);
  16987.                 for EVERY_TOKEN(token)
  16988.                         if (Val(token, a1) == T_YES)
  16989.                                 yes(a2, token);
  16990.                         else if (Val(token, a1) == T_NO)
  16991.                                 no(a2, token);
  16992.         }
  16993. }
  16994.  
  16995. solveit()
  16996. {
  16997.         int     token, tok2;
  16998.  
  16999.         for EVERY_TOKEN(token)
  17000.                 for (tok2 = token; tok2 < TOT_TOKEN; tok2++)
  17001.                         if (Val(token, tok2) == T_UNK) {
  17002.                                 EITHER
  17003.                                         yes(token, tok2);
  17004.                                 OR
  17005.                                         no(token, tok2);
  17006.                                 END_EITHER;
  17007.                                 return solveit();
  17008.                         }
  17009.         printf("Solution:\n");
  17010.         for EVERY_ITEM(token, 0) {
  17011.                 for (tok2 = NUM_ITEM; tok2 < TOT_TOKEN; tok2++)
  17012.                         if (Val(token, tok2) == T_YES)
  17013.                                 printf("\t%s %s\n",names▌token¿,names▌tok2¿);
  17014.                 printf("\n");
  17015.         }
  17016.         REJECT;
  17017. }
  17018.  
  17019. ---
  17020. james@crc.ricoh.com (James Allen)
  17021.  
  17022. ==> logic/elimination.p <==
  17023. 97 baseball teams participate in an annual state tournament.
  17024. The way the champion is chosen for this tournament is by the same old
  17025. elimination schedule. That is, the 97 teams are to be divided into
  17026. pairs, and the two teams of each pair play against each other.
  17027. After a team is eliminated from each pair, the winners would
  17028. be again divided into pairs, etc.  How many games must be played
  17029. to determine a champion?
  17030.  
  17031. ==> logic/elimination.s <==
  17032. In order to determine a winner all but one team must lose.
  17033. Therefore there must be at least 96 games.
  17034.  
  17035. ==> logic/family.p <==
  17036. Suppose that it is equally likely for a pregnancy to deliver
  17037. a baby boy as it is to deliver a baby girl.  Suppose that for a
  17038. large society of people, every family continues to have children
  17039. until they have a boy, then they stop having children.
  17040. After 1,000 generations of families, what is the ratio of males
  17041. to females?
  17042.  
  17043. ==> logic/family.s <==
  17044. The ratio will be 50-50 in both cases.  We are not killing off any
  17045. fetuses or babies, and half of all conceptions will be male, half
  17046. female.  When a family decides to stop does not affect this fact.
  17047.  
  17048. ==> logic/flip.p <==
  17049. How can a toss be called over the phone (without requiring trust)?
  17050.  
  17051. ==> logic/flip.s <==
  17052. A flips a coin.  If the result is heads, A multiplies 2 90-digit prime
  17053. numbers; if the result is tails, A multiplies 3 60-digit prime
  17054. numbers.  A tells B the result of the multiplication.  B now calls
  17055. either heads or tails and tells A.  A then supplies B with the
  17056. original numbers to verify the flip.
  17057.  
  17058. ==> logic/friends.p <==
  17059. Any group of 6 or more contains either 3 mutual friends or 3 mutual strangers.
  17060. Prove it.
  17061.  
  17062. ==> logic/friends.s <==
  17063. Take a person X.  Of the five other people, there must either be at least three
  17064. acquaintances of X or at least three strangers of X.  Assume wlog that X has
  17065. three strangers A,B,C.  Unless A,B,C is the required triad of acquaintances,
  17066. they must include a pair of strangers, wlog A,B.  Then X,A,B is the required
  17067. triad of strangers, QED.
  17068.  
  17069. ==> logic/hundred.p <==
  17070. A sheet of paper has statements numbered from 1 to 100.  Statement n says
  17071. "exactly n of the statements on this sheet are false."  Which statements are
  17072. true and which are false?  What if we replace "exactly" by "at least"?
  17073.  
  17074. ==> logic/hundred.s <==
  17075. It is tempting to argue as follows:
  17076.  
  17077. Since only one statement can be true (they are mutually contradictory),
  17078. therefore 99 are false. So, all are false except for statement 99.
  17079.  
  17080. If replaced by "at least", and the "real" number of false statements is
  17081. x, then statements x+1 to 100 will be false (since they falsely claim
  17082. that there are more false statements than there actually are). So, 100-x are
  17083. false, ie.  x=100-x, so x=50. The first 50 statements are true, and statements
  17084. 51 to 100 are false.
  17085.  
  17086. However, there is a hidden and incorrect assumption in this argument.
  17087. To see this, suppose that there is one statement on the sheet and it
  17088. says "One statement is false"  or "At least one statement is false,"
  17089. either way it implies "this statement is false," which is a familiar
  17090. paradoxical statement.  We have learned that this paradox arises because
  17091. of the false assumption that all statements are either true or false.
  17092. This is the hidden assumption in the above reasoning.
  17093.  
  17094. If it is acknowledged that some of the statements on the page may be
  17095. neither true nor false (i.e., meaningless), then nothing whatsoever can
  17096. be concluded about which statements are true or false.
  17097.  
  17098. This problem has been carefully contrived to appear to be solvable (like
  17099. the vacuous statement "this statement is true").  By changing the
  17100. numbers in some statements and changing "true" to "false," various
  17101. circular forms of the liar's paradox can be constructed.
  17102.  
  17103. From _Litton's Problematical Recreations_
  17104.  
  17105. ==> logic/inverter.p <==
  17106. Can a digital logic circuit with two inverters invert N independent inputs?
  17107. The circuit may contain any number of AND or OR gates.
  17108.  
  17109. ==> logic/inverter.s <==
  17110. It can be shown that N inverters can invert 2N-1 independent inputs, given
  17111. an unlimited supply of AND and OR gates. The classic version of this
  17112. puzzle is to invert 3 independent inputs using AND gates, OR gates, and
  17113. only 2 inverters.
  17114.  
  17115. So, start with N inverters.  Replace 3 of them with 2.
  17116. Keep doing that until you're down to 2 inverters.
  17117.  
  17118. I was skeptical at first, because such a design requires so much feedback
  17119. that I was sure the system would oscillate when switching between two
  17120. particular states.  But after writing a program to test every possible state
  17121. change (32^2), it appears that this system settles after a maximum of
  17122. 3 feedback logic iterations. I did not include gate delays in the simulation,
  17123. however, which could increase the number of iterations before the system
  17124. settles.
  17125.  
  17126. In any case, it appears that the world needs only 2 inverters] :-)
  17127.  
  17128. ==> logic/josephine.p <==
  17129. The recent expedition to the lost city of Atlantis discovered scrolls
  17130. attributted to the great poet, scholar, philosopher Josephine. They
  17131. number eight in all, and here is the first.
  17132.  
  17133. THE KINGDOM OF MAMAJORCA, WAS RULED BY QUEEN HENRIETTA I. IN MAMAJORCA
  17134. WOMEN HAVE TO PASS AN EXTENSIVE LOGIC EXAM BEFORE THEY ARE ALLOWED TO
  17135. GET MARRIED. QUEENS DO NOT HAVE TO TAKE THIS EXAM. ALL THE WOMEN IN
  17136. MAMAJORCA ARE LOYAL TO THEIR QUEEN AND DO WHATEVER SHE TELLS THEM TO.
  17137. THE QUEENS OF MAMAJORCA ARE TRUTHFUL. ALL SHOTS FIRED IN MAMAJORCA CAN
  17138. BE HEARD IN EVERY HOUSE. ALL ABOVE FACTS ARE KNOWN TO BE COMMON
  17139. KNOWLEDGE.
  17140.  
  17141. HENRIETTA WAS WORRIED ABOUT THE INFIDELITY OF THE MARRIED MEN IN
  17142. MAMAJORCA.  SHE SUMMONED ALL THE WIVES TO THE TOWN SQUARE, AND MADE
  17143. THE FOLLOWING ANNOUNCEMENT. "THERE IS AT LEAST ONE UNFAITHFUL HUSBAND
  17144. IN MAMAJORCA. ALL WIVES KNOW WHICH HUSBANDS ARE UNFAITHFUL, BUT HAVE
  17145. NO KNOWLEDGE ABOUT THE FIDELITY OF THEIR OWN HUSBAND. YOU ARE
  17146. FORBIDDEN TO DISCUSS YOUR HUSBAND'S FAITHFULNESS WITH ANY OTHER WOMAN.
  17147. IF YOU DISCOVER THAT YOUR HUSBAND IS UNFAITHFUL, YOU MUST SHOOT HIM AT
  17148. PRECISELY MIDNIGHT OF THE DAY YOU FIND THAT OUT."
  17149.  
  17150. THIRTY-NINE SILENT NIGHTS FOLLOWED THE QUEEN'S ANNOUNCEMENT. ON THE
  17151. FORTIETH NIGHT, SHOTS WERE HEARD. QUEEN HENRIETTA I IS REVERED IN
  17152. MAMAJORCAN HISTORY.
  17153.  
  17154. As with all philosophers Josephine doesn't provide the question, but leaves
  17155. it implicit in his document. So figure out the questions - there are two -
  17156. and answer them.
  17157.  
  17158. Here is Josephine's second scroll.
  17159.  
  17160. QUEEN HENRIETTA I WAS SUCCEEDED BY DAUGHTER QUEEN HENRIETTA II. AFTER
  17161. A WHILE HENRIETTA LIKE HER FAMOUS MOTHER BECAME WORRIED ABOUT THE
  17162. INFIDELITY PROBLEM. SHE DECIDED TO ACT, AND SENT A LETTER TO HER
  17163. SUBJECTS (WIVES) THAT CONTAINED THE EXACT WORDS OF HENRIETTA I'S
  17164. FAMOUS SPEECH.  SHE ADDED THAT THE LETTERS WERE GUARENTEED TO REACH
  17165. ALL WIVES EVENTUALLY.
  17166.  
  17167. QUEEN HENRIETTA II IS REMEMBERED AS A FOOLISH AND UNJUST QUEEN.
  17168.  
  17169. What is the question and answer implied by this scroll?
  17170.  
  17171. ==> logic/josephine.s <==
  17172. The two questions for scroll #1 were:
  17173.  
  17174. 1. How many husbands were shot on that fateful night?
  17175. 2. Why is Queen Henrietta I revered in Mamajorca?
  17176.  
  17177. The answers are:
  17178.  
  17179. If there are n unfaithful husbands (UHs), every wife of an UH knows of
  17180. n-1 UH's while every wife of a faithful husband knows of n UHs.  ▌this
  17181. because everyone has perfect information about everything except the
  17182. fidelity of their own husband¿.  Now we do a simple induction: Assume
  17183. that there is only one UH.  Then all the wives but one know that there
  17184. is just one UH, but the wife of the UH thinks that everyone is
  17185. faithful.  Upon hearing that "there is at least one UH", the wife
  17186. realizes that the only husband it can be is her own, and so shoots
  17187. him.  Now, imagine that there are just two UH's.  Each wife of an UH
  17188. assumes that the situation is "only one UH in town" and so waits to
  17189. hear the other wife (she knows who it is, of course) shoot her husband
  17190. on the first night.  When no one is shot, that can only be because her
  17191. OWN husband was a second UH.  The wife of the second UH makes the same
  17192. deduction when no shot is fired the first night (she was waiting, and
  17193. expecting the other to shoot, too).  So they both figure it out after
  17194. the first night, and shoot their husbands the second night.  It is
  17195. easy to tidy up the induction to show that the n UHs will all be shot
  17196. just on the n'th midnight.
  17197.  
  17198. The question for scroll #2 is:
  17199.  
  17200. 3. Why is Queen Henrietta II not?
  17201.  
  17202. The answer is:
  17203.  
  17204. The problem now is that QHII didn't realize that it is *critical* that
  17205. all of the wives, of faithful and UH's alike, to
  17206. *BEGIN*AT*THE*SAME*MOMENT*.  The uncertainty of having a particular
  17207. wife's notice come a day or two late makes the whole logic path fall
  17208. apart.  That's why she's foolish.  She is unjust, because some wives,
  17209. honed and crack logicians all, remember, will *incorrectly* shoot
  17210. faithful husbands.  Let us imagine the situation with just a SINGLE UH
  17211. in the whole country.  And, wouldn't you know it, the notice to the
  17212. wife of the UH just happens to be held up a day, whereas everyone
  17213. else's arrived the first day.  Now, all of the wives that got the
  17214. notice the first day know that there is just one UH in the country.
  17215. And they know that the wife of that UH will think that everyone is
  17216. faithful, and so they'll expect her to figure it out and shoot her
  17217. husband the first night.  BUT SHE DIDN"T GET THE NOTICE THE FIRST
  17218. NIGHT....  BUT THE OTHER WIVES HAVE NO WAY OF KNOWING THAT.  So, the
  17219. wife of the UH doesn't know that anything is going on and so (of
  17220. course) doesn't do anything the first night.  The next day she gets
  17221. the notice, figures it all out, and her husband will be history come
  17222. that midnight.  BUT... *every* other wife thought that there should
  17223. have been a shooting the first night, and since there wasn't there
  17224. must have been an additional UH, and it can only have been _her_
  17225. husband.  So on the second night **ALL** of the husbands are shot.
  17226. Things are much more complicated if the mix of who gets the notice
  17227. when is less simple than the one I mentioned above, but it is always
  17228. wrong and/or tragic.
  17229.  
  17230. NOTE: if the wives *know* that the country courier service (or however
  17231. these things get delivered) is flaky, then they can avoid the
  17232. massacre, but unless the wives exchange notes no one will ever be shot
  17233. (since there is always a chance that rather than _your_ husband being
  17234. an UH, you could reason that it might be that the wife of one of the
  17235. UH's that you know about just hasn't gotten her copy of the scroll
  17236. yet).  I guess you could call this case "unjust", too, since the UH's
  17237. evade punishment, despite the perfect logic of the wives.
  17238.  
  17239. ==> logic/locks.and.boxes.p <==
  17240. You want to send a valuable object to a friend.  You have a box which
  17241. is more than large enough to contain the object.  You have several
  17242. locks with keys.  The box has a locking ring which is more than large enough
  17243. to have a lock attached.  But your friend does not have the key to any
  17244. lock that you have.  How do you do it?
  17245.  
  17246.  
  17247. ==> logic/locks.and.boxes.s <==
  17248. Attach a lock to the ring.  Send it to her.  She attaches her own lock
  17249. and sends it back.  You remove your lock and send it back to her.  She
  17250. removes her lock.
  17251.  
  17252. ==> logic/mixing.p <==
  17253. Start with a half cup of tea and a half cup of coffee. Take one tablespoon
  17254. of the tea and mix it in with the coffee. Take one tablespoon of this mixture
  17255. and mix it back in with the tea. Which of the two cups contains more of its
  17256. original contents?
  17257.  
  17258. ==> logic/mixing.s <==
  17259. Mixing Liquids
  17260.  
  17261. The two cups end up with the same volume of liquid they started with. The same
  17262. amount of tea was moved to the coffee cup as coffee to the teacup. Therefore
  17263. each cup contains the same amount of its original contents.
  17264.  
  17265. ==> logic/number.p <==
  17266. Mr. S. and Mr. P. are both perfect logicians, being able to correctly deduce
  17267. any truth from any set of axioms.  Two integers (not necessarily unique) are
  17268. somehow chosen such that each is within some specified range.  Mr. S.
  17269. is given the sum of these two integers; Mr. P. is given the product of these
  17270. two integers.  After receiving these numbers, the two logicians do not
  17271. have any communication at all except the following dialogue:
  17272. <<1>>   Mr. P.:  I do not know the two numbers.
  17273. <<2>>   Mr. S.:  I knew that you didn't know the two numbers.
  17274. <<3>>   Mr. P.:  Now I know the two numbers.
  17275. <<4>>   Mr. S.:  Now I know the two numbers.
  17276.  
  17277. Given that the above statements are absolutely truthful, what are the two
  17278. numbers?
  17279.  
  17280. ==> logic/number.s <==
  17281. The answer depends upon the ranges from which the numbers are chosen.
  17282.  
  17283. The unique solution for the ranges ▌2,62¿ through ▌2,500+¿ is:
  17284.  
  17285.   SUM   PRODUCT   X   Y
  17286.    17      52     4  13
  17287.  
  17288. The unique solution for the ranges ▌3,94¿ through ▌3,500+¿ is:
  17289.  
  17290.   SUM   PRODUCT   X   Y
  17291.    29     208    13  16
  17292.  
  17293. There are no unique solutions for the ranges starting with 1,
  17294. and there are no solutions for ranges starting with numbers above 3.
  17295.  
  17296. A program to compute the possible pairs is included below.
  17297.  
  17298. #include <stdio.h>
  17299.  
  17300. /*
  17301.  
  17302. BEGINNING OF PROBLEM STATEMENT:
  17303. Mr. S. and Mr. P. are both perfect logicians, being able to correctly deduce
  17304. any truth from any set of axioms.  Two integers (not necessarily unique) are
  17305. somehow chosen such that each is within some specified range.  Mr. S.
  17306. is given the sum of these two integers; Mr. P. is given the product of these
  17307. two integers.  After receiving these numbers, the two logicians do not
  17308. have any communication at all except the following dialogue:
  17309. <<1>>   Mr. P.:  I do not know the two numbers.
  17310. <<2>>   Mr. S.:  I knew that you didn't know the two numbers.
  17311. <<3>>   Mr. P.:  Now I know the two numbers.
  17312. <<4>>   Mr. S.:  Now I know the two numbers.
  17313.  
  17314. Given that the above statements are absolutely truthful, what are the two
  17315. numbers?
  17316.  
  17317. END OF PROBLEM STATEMENT
  17318.  
  17319.  */
  17320.  
  17321. #define SMALLEST_MIN    1
  17322. #define LARGEST_MIN     10
  17323. #define SMALLEST_MAX    50
  17324. #define LARGEST_MAX     500
  17325.  
  17326. long P▌(LARGEST_MAX + 1) * (LARGEST_MAX + 1)¿;          /* products */
  17327. long S▌(LARGEST_MAX + 1) + (LARGEST_MAX + 1)¿;          /*   sums   */
  17328.  
  17329. find(long min, long max)
  17330. {
  17331.         long i, j;
  17332.         /*
  17333.          *      count factorizations in P▌¿
  17334.          *      all P▌n¿ > 1 satisfy <<1>>.
  17335.          */
  17336.         for(i = 0; i <= max * max; ++i)
  17337.                 P▌i¿ = 0;
  17338.  
  17339.         for(i = min; i <= max; ++i)
  17340.                 for(j = i; j <= max; ++j)
  17341.                         ++P▌i * j¿;
  17342.  
  17343.         /*
  17344.          *      decompose possible SUMs and check factorizations
  17345.          *              all S▌n¿ == min - 1 satisfy <<2>>.
  17346.          */
  17347.         for(i = min + min; i <= max + max; ++i) {
  17348.  
  17349.                 for(j = i / 2; j >= min; --j)
  17350.                         if(P▌j * (i - j)¿ < 2)
  17351.                                 break;
  17352.  
  17353.                 S▌i¿ = j;
  17354.         }
  17355.  
  17356.         /*
  17357.          *      decompose SUMs which satisfy <<2>> and see which products
  17358.          *      they produce.  All (P▌n¿ / 1000 == 1) satisfy <<3>>.
  17359.          */
  17360.         for(i = min + min; i <= max + max; ++i)
  17361.                 if(S▌i¿ == min - 1)
  17362.                         for(j = i / 2; j >= min; --j)
  17363.                                 if(P▌j * (i - j)¿ > 1)
  17364.                                         P▌j * (i - j)¿ += 1000;
  17365.         /*
  17366.          *      decompose SUMs which satisfy <<2>> again and see which products
  17367.          *      satisfy <<3>>.  Any (S▌n¿ == 999 + min) satisfies <<4>>
  17368.          */
  17369.         for(i = min + min; i <= max + max; ++i)
  17370.                 if(S▌i¿ == min - 1)
  17371.                         for(j = i / 2; j >= min; --j)
  17372.                                 if(P▌j * (i - j)¿ / 1000 == 1)
  17373.                                         S▌i¿ += 1000;
  17374.         /*
  17375.          *      find the answer(s) and print them
  17376.          */
  17377.         printf("▌%d,%d¿\n",min,max);
  17378.         for(i = min + min; i <= max + max; ++i)
  17379.                 if(S▌i¿ == 999 + min)
  17380.                         for(j = i / 2; j >= min; --j)
  17381.                                 if(P▌j * (i - j)¿ / 1000 == 1)
  17382.                                         printf("{ %d %d }: S = %d, P = %d\n",
  17383.                                                 i - j, j, i, (i - j)  * j);
  17384. }
  17385.  
  17386. main()
  17387. {
  17388.         long min, max;
  17389.  
  17390.         for (min = SMALLEST_MIN; min <= LARGEST_MIN; min ++)
  17391.             for (max = SMALLEST_MAX; max <= LARGEST_MAX; max++)
  17392.                 find(min,max);
  17393. }
  17394.  
  17395. -------------------------------------------------------------------------
  17396. =                       Jeff Kenton     (617) 894-4508                  =
  17397. =                           jkenton@world.std.com                       =
  17398. -------------------------------------------------------------------------
  17399.  
  17400. ==> logic/riddle.p <==
  17401. Who makes it, has no need of it.  Who buys it, has no use for it.  Who
  17402. uses it can neither see nor feel it.
  17403.  
  17404. Tell me what a dozen rubber trees with thirty boughs on each might be?
  17405.  
  17406. As I went over London Bridge
  17407. I met my sister Jenny
  17408. I broke her neck and drank her blood
  17409. And left her standing empty
  17410.  
  17411. It is said among my people that some things are improved by death.
  17412. Tell me, what stinks while living, but in death, smells good?
  17413.  
  17414. All right.  Riddle me this:  what goes through the door without
  17415. pinching itself?  What sits on the stove without burning itself?  What
  17416. sits on the table and is not ashamed?
  17417.  
  17418. What work is it that the faster you work, the longer it is before
  17419. you're done, and the slower you work, the sooner you're finished?
  17420.  
  17421. Whilst I was engaged in sitting I spied the dead carrying the living.
  17422.  
  17423. I know a word of letters three.  Add two, and fewer there will be.
  17424.  
  17425. I give you a group of three.  One is sitting down, and will never get
  17426. up.  The second eats as much as is given to him, yet is always hungry.
  17427. The third goes away and never returns.
  17428.  
  17429. Whoever makes it, tells it not.  Whoever takes it, knows it not.  And
  17430. whoever knows it wants it not.
  17431.  
  17432. Two words, my answer is only two words.
  17433. To keep me, you must give me.
  17434.  
  17435. Sir, I bear a rhyme excelling
  17436. In mystic force and magic spelling
  17437. Celestial sprites elucidate
  17438. All my own striving can't relate
  17439.  
  17440. There is not wind enough to twirl
  17441. That one red leaf, nearest of its clan,
  17442. Which dances as often as dance it can.
  17443.  
  17444. Half-way up the hill, I see thee at last
  17445. Lying beneath me with thy sounds and sights --
  17446. A city in the twilight, dim and vast,
  17447. With smoking roofs, soft bells, and gleaming lights.
  17448.  
  17449. I am, in truth, a yellow fork
  17450. From tables in the sky
  17451. By inadvertent fingers dropped
  17452. The awful cutlery.
  17453. Of mansions never quite disclosed
  17454. And never quite concealed
  17455. The apparatus of the dark
  17456. To ignorance revealed.
  17457.  
  17458. Many-maned scud-thumper,
  17459. Maker of worn wood,
  17460. Shrub-ruster,
  17461. Sky-mocker,
  17462. Rave]
  17463.  
  17464. Make me thy lyre, even as the forests are.
  17465. What if my leaves fell like its own --
  17466. The tumult of thy mighty harmonies
  17467. Will take from both a deep autumnal tone.
  17468.  
  17469. This darksome burn, horseback brown,
  17470. His rollock highroad roaring down,
  17471. In coop and in comb the fleece of his foam
  17472. Flutes and low to the body falls home.
  17473.  
  17474. I've measured it from side to side,
  17475. 'Tis three feet long and two feet wide.
  17476. It is of compass small, and bare
  17477. To thirsty suns and parching air.
  17478.  
  17479. My love, when I gaze on thy beautiful face,
  17480. Careering along, yet always in place --
  17481. The thought has often come into my mind
  17482. If I ever shall see thy glorious behind.
  17483.  
  17484. Then all thy feculent majesty recalls
  17485. The nauseous mustiness of forsaken bowers,
  17486. The leprous nudity of deserted halls --
  17487. The positive nastiness of sullied flowers.
  17488. And I mark the colours, yellow and black,
  17489. That fresco thy lithe, dictatorial thighs.
  17490.  
  17491. When young, I am sweet in the sun.
  17492. When middle-aged, I make you gay.
  17493. When old, I am valued more than ever.
  17494.  
  17495. I am always hungry,
  17496. I must always be fed,
  17497. The finger I lick
  17498. Will soon turn red.
  17499.  
  17500. All about, but cannot be seen,
  17501. Can be captured, cannot be held,
  17502. No throat, but can be heard.
  17503.  
  17504. I am only useful
  17505. When I am full,
  17506. Yet I am always
  17507. Full of holes.
  17508.  
  17509. If you break me
  17510. I do not stop working,
  17511. If you touch me
  17512. I may be snared,
  17513. If you lose me
  17514. Nothing will matter.
  17515.  
  17516. If a man carried my burden
  17517. He would break his back.
  17518. I am not rich,
  17519. But leave silver in my track.
  17520.  
  17521. Until I am measured
  17522. I am not known,
  17523. Yet how you miss me
  17524. When I have flown.
  17525.  
  17526. I drive men mad
  17527. For love of me,
  17528. Easily beaten,
  17529. Never free.
  17530.  
  17531. When set loose
  17532. I fly away,
  17533. Never so cursed
  17534. As when I go astray.
  17535.  
  17536. I go around in circles
  17537. But always straight ahead,
  17538. Never complain
  17539. No matter where I am led.
  17540.  
  17541. Lighter than what
  17542. I am made of,
  17543. More of me is hidden
  17544. Than is seen.
  17545.  
  17546. I turn around once,
  17547. What is out will not get in.
  17548. I turn around again,
  17549. What is in will not get out.
  17550.  
  17551. Each morning I appear
  17552. To lie at your feet,
  17553. All day I will follow
  17554. No matter how fast you run,
  17555. Yet I nearly perish
  17556. In the midday sun.
  17557.  
  17558. Weight in my belly,
  17559. Trees on my back,
  17560. Nails in my ribs,
  17561. Feet I do lack.
  17562.  
  17563. Bright as diamonds,
  17564. Loud as thunder,
  17565. Never still,
  17566. A thing of wonder.
  17567.  
  17568. My life can be measured in hours,
  17569. I serve by being devoured.
  17570. Thin, I am quick
  17571. Fat, I am slow
  17572. Wind is my foe.
  17573.  
  17574. To unravel me
  17575. You need a simple key,
  17576. No key that was made
  17577. By locksmith's hand,
  17578. But a key that only I
  17579. Will understand.
  17580.  
  17581. I am seen in the water
  17582. If seen in the sky,
  17583. I am in the rainbow,
  17584. A jay's feather,
  17585. And lapis lazuli.
  17586.  
  17587. Glittering points
  17588. That downward thrust,
  17589. Sparkling spears
  17590. That never rust.
  17591.  
  17592. You heard me before,
  17593. Yet you hear me again,
  17594. Then I die,
  17595. 'Till you call me again.
  17596.  
  17597. Three lives have I.
  17598. Gentle enough to soothe the skin,
  17599. Light enough to caress the sky,
  17600. Hard enough to crack rocks.
  17601.  
  17602. You can see nothing else
  17603. When you look in my face,
  17604. I will look you in the eye
  17605. And I will never lie.
  17606.  
  17607. Lovely and round,
  17608. I shine with pale light,
  17609. grown in the darkness,
  17610. A lady's delight.
  17611.  
  17612. At the sound of me, men may dream
  17613. Or stamp their feet
  17614. At the sound of me, women may laugh
  17615. Or sometimes weep
  17616.  
  17617. When I am filled
  17618. I can point the way,
  17619. When I am empty
  17620. Nothing moves me,
  17621. I have two skins
  17622. One without and one within.
  17623.  
  17624. My tines be long,
  17625. My tines be short
  17626. My tines end ere
  17627. My first report.
  17628. What am I?
  17629.  
  17630. ==> logic/riddle.s <==
  17631. Who makes it, has no need of it.  Who buys it, has no use for it.  Who
  17632. uses it can neither see nor feel it.
  17633.  
  17634. coffin
  17635.  
  17636. Tell me what a dozen rubber trees with thirty boughs on each might be?
  17637.  
  17638. months of the year
  17639.  
  17640. As I went over London Bridge
  17641. I met my sister Jenny
  17642. I broke her neck and drank her blood
  17643. And left her standing empty
  17644.  
  17645. gin
  17646.  
  17647. It is said among my people that some things are improved by death.
  17648. Tell me, what stinks while living, but in death, smells good?
  17649.  
  17650. pig
  17651.  
  17652. All right.  Riddle me this:  what goes through the door without
  17653. pinching itself?  What sits on the stove without burning itself?  What
  17654. sits on the table and is not ashamed?
  17655.  
  17656. the sun
  17657.  
  17658. What work is it that the faster you work, the longer it is before
  17659. you're done, and the slower you work, the sooner you're finished?
  17660.  
  17661. roasting meat on a spit
  17662.  
  17663. Whilst I was engaged in sitting I spied the dead carrying the living.
  17664.  
  17665. a ship
  17666.  
  17667. I know a word of letters three.  Add two, and fewer there will be.
  17668.  
  17669. 'few'
  17670.  
  17671. I give you a group of three.  One is sitting down, and will never get
  17672. up.  The second eats as much as is given to him, yet is always hungry.
  17673. The third goes away and never returns.
  17674.  
  17675. stove, fire, and smoke
  17676.  
  17677. Whoever makes it, tells it not.  Whoever takes it, knows it not.  And
  17678. whoever knows it wants it not.
  17679.  
  17680. counterfeit money
  17681.  
  17682. Two words, my answer is only two words.
  17683. To keep me, you must give me.
  17684.  
  17685. your word
  17686.  
  17687. Sir, I bear a rhyme excelling
  17688. In mystic force and magic spelling
  17689. Celestial sprites elucidate
  17690. All my own striving can't relate
  17691.  
  17692. ???
  17693.  
  17694. There is not wind enough to twirl
  17695. That one red leaf, nearest of its clan,
  17696. Which dances as often as dance it can.
  17697.  
  17698. the sun, Samuel Taylor Coleridge
  17699.  
  17700. Half-way up the hill, I see thee at last
  17701. Lying beneath me with thy sounds and sights --
  17702. A city in the twilight, dim and vast,
  17703. With smoking roofs, soft bells, and gleaming lights.
  17704.  
  17705. the past, Longfellow
  17706.  
  17707. I am, in truth, a yellow fork
  17708. From tables in the sky
  17709. By inadvertent fingers dropped
  17710. The awful cutlery.
  17711. Of mansions never quite disclosed
  17712. And never quite concealed
  17713. The apparatus of the dark
  17714. To ignorance revealed.
  17715.  
  17716. lightning, Emily Dickinson
  17717.  
  17718. Many-maned scud-thumper,
  17719. Maker of worn wood,
  17720. Shrub-ruster,
  17721. Sky-mocker,
  17722. Rave]
  17723. Portly pusher,
  17724. Wind-slave.
  17725.  
  17726. the ocean, John Updike
  17727.  
  17728. Make me thy lyre, even as the forests are.
  17729. What if my leaves fell like its own --
  17730. The tumult of thy mighty harmonies
  17731. Will take from both a deep autumnal tone.
  17732.  
  17733. the west wind, Percy Bysshe Shelley
  17734.  
  17735. This darksome burn, horseback brown,
  17736. His rollock highroad roaring down,
  17737. In coop and in comb the fleece of his foam
  17738. Flutes and low to the body falls home.
  17739.  
  17740. river, Gerard Manley Hopkins
  17741.  
  17742. I've measured it from side to side,
  17743. 'Tis three feet long and two feet wide.
  17744. It is of compass small, and bare
  17745. To thirsty suns and parching air.
  17746.  
  17747. the grave of a child, Wordsworth
  17748.  
  17749. My love, when I gaze on thy beautiful face,
  17750. Careering along, yet always in place --
  17751. The thought has often come into my mind
  17752. If I ever shall see thy glorious behind.
  17753.  
  17754. the moon, Sir Edmund Gosse
  17755.  
  17756. Then all thy feculent majesty recalls
  17757. The nauseous mustiness of forsaken bowers,
  17758. The leprous nudity of deserted halls --
  17759. The positive nastiness of sullied flowers.
  17760. And I mark the colours, yellow and black,
  17761. That fresco thy lithe, dictatorial thighs.
  17762.  
  17763. spider, Francis Saltus Saltus
  17764.  
  17765. When young, I am sweet in the sun.
  17766. When middle-aged, I make you gay.
  17767. When old, I am valued more than ever.
  17768.  
  17769. wine
  17770.  
  17771. I am always hungry,
  17772. I must always be fed,
  17773. The finger I lick
  17774. Will soon turn red.
  17775.  
  17776. fire
  17777.  
  17778. All about, but cannot be seen,
  17779. Can be captured, cannot be held,
  17780. No throat, but can be heard.
  17781.  
  17782. wind
  17783.  
  17784. I am only useful
  17785. When I am full,
  17786. Yet I am always
  17787. Full of holes.
  17788.  
  17789. sieve (or sponge)
  17790.  
  17791. If you break me
  17792. I do not stop working,
  17793. If you touch me
  17794. I may be snared,
  17795. If you lose me
  17796. Nothing will matter.
  17797.  
  17798. heart
  17799.  
  17800. If a man carried my burden
  17801. He would break his back.
  17802. I am not rich,
  17803. But leave silver in my track.
  17804.  
  17805. snail
  17806.  
  17807. Until I am measured
  17808. I am not known,
  17809. Yet how you miss me
  17810. When I have flown.
  17811.  
  17812. time
  17813.  
  17814. I drive men mad
  17815. For love of me,
  17816. Easily beaten,
  17817. Never free.
  17818.  
  17819. gold
  17820.  
  17821. When set loose
  17822. I fly away,
  17823. Never so cursed
  17824. As when I go astray.
  17825.  
  17826. ?
  17827.  
  17828. I go around in circles
  17829. But always straight ahead,
  17830. Never complain
  17831. No matter where I am led.
  17832.  
  17833. wagon wheel
  17834.  
  17835. Lighter than what
  17836. I am made of,
  17837. More of me is hidden
  17838. Than is seen.
  17839.  
  17840. iceberg
  17841.  
  17842. I turn around once,
  17843. What is out will not get in.
  17844. I turn around again,
  17845. What is in will not get out.
  17846.  
  17847. stopcock
  17848.  
  17849. Each morning I appear
  17850. To lie at your feet,
  17851. All day I will follow
  17852. No matter how fast you run,
  17853. Yet I nearly perish
  17854. In the midday sun.
  17855.  
  17856. shadow
  17857.  
  17858. Weight in my belly,
  17859. Trees on my back,
  17860. Nails in my ribs,
  17861. Feet I do lack.
  17862.  
  17863. ship
  17864.  
  17865. Bright as diamonds,
  17866. Loud as thunder,
  17867. Never still,
  17868. A thing of wonder.
  17869.  
  17870. waterfall? (fireworks?)
  17871.  
  17872. My life can be measured in hours,
  17873. I serve by being devoured.
  17874. Thin, I am quick
  17875. Fat, I am slow
  17876. Wind is my foe.
  17877.  
  17878. candle
  17879.  
  17880. To unravel me
  17881. You need a simple key,
  17882. No key that was made
  17883. By locksmith's hand,
  17884. But a key that only I
  17885. Will understand.
  17886.  
  17887. cipher
  17888.  
  17889. I am seen in the water
  17890. If seen in the sky,
  17891. I am in the rainbow,
  17892. A jay's feather,
  17893. And lapis lazuli.
  17894.  
  17895. blue
  17896.  
  17897. Glittering points
  17898. That downward thrust,
  17899. Sparkling spears
  17900. That never rust.
  17901.  
  17902. icicle
  17903.  
  17904. You heard me before,
  17905. Yet you hear me again,
  17906. Then I die,
  17907. 'Till you call me again.
  17908.  
  17909. echo
  17910.  
  17911. Three lives have I.
  17912. Gentle enough to soothe the skin,
  17913. Light enough to caress the sky,
  17914. Hard enough to crack rocks.
  17915.  
  17916. water
  17917.  
  17918. You can see nothing else
  17919. When you look in my face,
  17920. I will look you in the eye
  17921. And I will never lie.
  17922.  
  17923. your reflection
  17924.  
  17925. Lovely and round,
  17926. I shine with pale light,
  17927. grown in the darkness,
  17928. A lady's delight.
  17929.  
  17930. pearl
  17931.  
  17932. At the sound of me, men may dream
  17933. Or stamp their feet
  17934. At the sound of me, women may laugh
  17935. Or sometimes weep
  17936.  
  17937. music
  17938.  
  17939. When I am filled
  17940. I can point the way,
  17941. When I am empty
  17942. Nothing moves me,
  17943. I have two skins
  17944. One without and one within.
  17945.  
  17946. sails?
  17947.  
  17948. My tines be long,
  17949. My tines be short
  17950. My tines end ere
  17951. My first report.
  17952. What am I?
  17953.  
  17954. lightning
  17955.  
  17956. ==> logic/river.crossing.p <==
  17957. Three humans, one big monkey and two small monkeys are to cross a river:
  17958.         a) Only humans and the big monkey can row the boat.
  17959.         b) At all times, the number of human on either side of the
  17960.            river must be GREATER OR EQUAL to the number of monkeys
  17961.            on THAT side. ( Or else the humans will be eaten by the monkeys])
  17962.  
  17963. ==> logic/river.crossing.s <==
  17964. The three columns represent the left bank, the boat, and the right bank
  17965. respectively. The < or > indicates the direction of motion of the boat.
  17966.  
  17967. HHHMmm  .       .
  17968. HHHm    Mm>     .
  17969. HHHm    <M      m
  17970. HHH     Mm>     m
  17971. HHH     <M      mm
  17972. HM      HH>     mm
  17973. HM      <Hm     Hm
  17974. Hm      HM>     Hm
  17975. Hm      <Hm     HM
  17976. mm      HH>     HM
  17977. mm      <M      HHH
  17978. m       Mm>     HHH
  17979. m       <M      HHHm
  17980. .       Mm>     HHHm
  17981. .       .       HHHMmm
  17982.  
  17983. ==> logic/ropes.p <==
  17984. Two fifty foot ropes are suspended from a forty foot ceiling, about
  17985. twenty feet apart.  Armed with only a knife, how much of the rope can
  17986. you steal?
  17987.  
  17988. ==> logic/ropes.s <==
  17989. Almost all of it.  Tie the ropes together.  Climb up one of them.  Tie
  17990. a loop in it as close as possible to the ceiling.  Cut it below the
  17991. loop.  Run the rope through the loop and tie it to your waist.  Climb
  17992. the other rope (this may involve some swinging action).  Pull the rope
  17993. going through the loop tight and cut the other rope as close as
  17994. possible to the ceiling.  You will swing down on the rope through the
  17995. loop.  Lower yourself to the ground by letting out rope.  Pull the
  17996. rope through the loop.
  17997.  
  17998. ==> logic/same.street.p <==
  17999. Sally and Sue have a strong desire to date Sam.  They all live on the
  18000. same street yet neither Sally or Sue know where Sam lives.  The houses
  18001. on this street are numbered 1 to 99.
  18002.  
  18003. Sally asks Sam "Is your house number a perfect square?".  He answers.
  18004. Then Sally asks "Is is greater than 50?".  He answers again.
  18005.  
  18006. Sally thinks she now knows the address of Sam's house and decides to
  18007. visit.
  18008.  
  18009. When she gets there, she finds out she is wrong.  This is not
  18010. surprising, considering Sam answered only the second question
  18011. truthfully.
  18012.  
  18013. Sue, unaware of Sally's conversation, asks Sam two questions.
  18014. Sue asks "Is your house number a perfect cube?".  He answers.
  18015. She then asks "Is it greater than 25?".  He answers again.
  18016.  
  18017. Sue thinks she knows where Sam lives and decides to pay him a visit.
  18018. She too is mistaken as Sam once again answered only the second
  18019. question truthfully.
  18020.  
  18021. If I tell you that Sam's number is less than Sue's or Sally's,
  18022. and that the sum of their numbers is a perfect square multiplied
  18023. by two, you should be able to figure out where all three of them
  18024. live.
  18025.  
  18026. ==> logic/same.street.s <==
  18027. Sally and Sue have a strong desire to date Sam.  They all live on the
  18028. same street yet neither Sally or Sue know where Sam lives.  The houses
  18029. on this street are numbered 1 to 99.
  18030.  
  18031. Sally asks Sam "Is your house number a perfect square?".  He answers.
  18032. Then Sally asks "Is is greater than 50?".  He answers again.
  18033.  
  18034. Sally thinks she now knows the address of Sam's house and decides to
  18035. visit.
  18036.  
  18037.         Since Sally thinks that she has enough information,
  18038.         I deduce that Sam answered that his house number was
  18039.         a perfect square greater than 50.  There are two
  18040.         of these {64,81} and Sally must live in one of them in
  18041.         order to have decided she knew where Sam lives.
  18042.  
  18043. When she gets there, she finds out she is wrong.  This is not
  18044. surprising, considering Sam answered only the second question
  18045. truthfully.
  18046.  
  18047.         So Sam's house number is greater than 50, but not
  18048.         a perfect square.
  18049.  
  18050. Sue, unaware of Sally's conversation, asks Sam two questions.
  18051. Sue asks "Is your house number a perfect cube?".  He answers.
  18052. She then asks "Is it greater than 25?".  He answers again.
  18053.  
  18054.         Observation: perfect cubes greater than 25 are
  18055.         {27, 64}, less than 25 are {1,8}.
  18056.  
  18057. Sue thinks she knows where Sam lives and decides to pay him a visit.
  18058. She too is mistaken as Sam once again answered only the second
  18059. question truthfully.
  18060.  
  18061.         Since Sam's house number is greater than 50, he told Sue that it
  18062.         was greater than 25 as well.  Since Sue thought she knew which house
  18063.         was his, she must live in either of {27,64}.
  18064.  
  18065. If I tell you that Sam's number is less than Sue's or Sally's,
  18066.  
  18067.         Since Sam's number is greater than 50, and Sue's is even
  18068.         bigger, she must live in 64.  Assuming Sue and Sally are
  18069.         not roommates (although awkward social situations of this
  18070.         kind are not without precedent), Sally lives in 81.
  18071.  
  18072. and that the sum of their numbers is a perfect square multiplied
  18073. by two, you should be able to figure out where all three of them
  18074. live.
  18075.  
  18076.         Sue + Sally + Sam = 2 p^2        for p an integer
  18077.         64  + 81    + Sam = 2 p^2
  18078.  
  18079.         Applying the constraint 50 < Sam < 64, looks like Sam = 55 (p = 10).
  18080.  
  18081.         In summary,
  18082.                 Sam = 55
  18083.                 Sue = 64
  18084.                 Sally = 81
  18085.  
  18086.     -- Tom Smith <tom@ulysses.att.com>
  18087.  
  18088. ==> logic/self.ref.p <==
  18089. Find a number ABCDEFGHIJ such that A is the count of how many 0's are in the
  18090. number, B is the number of 1's, and so on.
  18091.  
  18092. ==> logic/self.ref.s <==
  18093. 6210001000
  18094.  
  18095. For other numbers of digits:
  18096.  
  18097. n=1:    no sequence possible
  18098. n=2:    no sequence possible
  18099. n=3:    no sequence possible
  18100. n=4:    1210, 2020
  18101. n=5:    21200
  18102. n=6:    no sequence possible
  18103. n=7:    3211000
  18104. n=8:    42101000
  18105. n=9:    521001000
  18106. n=10:   6210001000
  18107. n>10:   (n-4), 2, 1, 0 * (n-7), 1, 0, 0, 0
  18108.  
  18109. No 1, 2, or 3 digit numbers are possible.  Letting x_i be the ith
  18110. digit, starting with 0, we see that (1) x_0 + ... + x_n = n+1 and
  18111. (2) 0*x_0 + ... + n*x_n = n+1, where n+1 is the number of digits.
  18112.  
  18113. I'll first prove that x_0 > n-3 if n>4.  Assume not, then this
  18114. implies that at least four of the x_i with i>0 are non-zero.  But
  18115. then we would have \sum_i i*x_i >= 10 by (2), impossible unless n=9,
  18116. but it isn't possible in this case (51111100000 isn't valid).
  18117.  
  18118. Now I'll prove that x_0 < n-1.  x_0 clearly can't equal n; assume
  18119. x_0 = n-1 ==> x_{n-1} = 1 by (2) if n>3.  Now only one of the
  18120. remaining x_i may be non-zero, and we must have that x_0 + ... + x_n
  18121. = n+1, but since x_0 + x_{n-1} = n ==> the remaining x_i = 1 ==> by
  18122. (2) that x_2 = 1.  But this can't be, since x_{n-1} = 1 ==> x_1>0.
  18123. Now assuming x_0 = n-2 we conclude that x_{n-2} = 1 by (2) if n>5
  18124. ==> x_1 + ... + x_{n-3} + x_{n-1} + x_n = 2 and 1*x_1 + ... +
  18125. (n-3)*x_{n-3} + (n-1)*x_{n-1} + n*x_n = 3 ==> x_1=1 and x_2=1,
  18126. contradiction.
  18127.  
  18128. Case n>5:
  18129.  
  18130. We have that x_0 = n-3 and if n>=7 ==> x_{n-3}=1 ==> x_1=2 and
  18131. x_2=1 by (1) and (2).  For the case n=6 we see that x_{n-3}=2
  18132. leads to an easy contradiction, and we get the same result.  The
  18133. cases n=4,5 are easy enough to handle, and lead to the two solutions
  18134. above.
  18135. --
  18136.     -- clong@romulus.rutgers.edu (Chris Long)
  18137.  
  18138. ==> logic/situation.puzzles.outtakes.p <==
  18139. The following puzzles have been removed from my situation puzzles list,
  18140. or never made it onto the list in the first place.  There are a wide
  18141. variety of reasons for the non-inclusion: some I think are obvious,
  18142. some don't have enough of a story, some involve gimmicks that annoy me,
  18143. some I think are riddles rather than situation puzzles, and some are
  18144. so contrary to reality as to be unplayable.  Basically, what it comes
  18145. down to is that I don't like these enough to put them on my list.  If
  18146. you think of ways to make any of them more palatable to me, or to
  18147. reorganize my entire list, or if you just want to chat, by all means
  18148. contact me at zorn@apple.com.
  18149. --jed e. hartman, 5/5/92
  18150.  
  18151. -----------------------------------------
  18152.  
  18153. Contra-reality puzzles, or, "That's not the way it works]"
  18154.  
  18155. 2.10.  A man is sitting in a train compartment.  He sees a three-
  18156. fingered hand through the compartment window, in the hallway of the
  18157. train.  He opens the compartment door and shoots the person with the
  18158. three-fingered hand, but he goes free.  (Michael Bernstein)
  18159.  
  18160. 2.61.  A man ran into a fire, and lived.  A man stayed where there was
  18161. no fire, and died.  (Eric Wang original)
  18162.  
  18163. 2.50.  The pope is giving a speech.  A man in the audience shoots the
  18164. mayor who is behind the pope.  (PRO)
  18165.  
  18166. Date: 2 Feb 92 23:05:11 GMT
  18167. In article <64023@netnews.upenn.edu>, weemba@libra (Matthew P Wiener) writes:
  18168. >Here's ▌one¿ I made up years ago: "She stopped having sex. She died."
  18169.  
  18170. 1.37.  A holy man is dead in a room.  (Perry Deess original)
  18171.  
  18172. -----------------------------------------
  18173.  
  18174. Clocks, calendars, money, and other numerical trivia:
  18175.  
  18176. 2.15.  Two people are talking long distance on the phone; one is in an
  18177. East-Coast state, the other is in a West-Coast state.  The first asks
  18178. the other "What time is it?", hears the answer, and says, "That's funny.
  18179. It's the same time here]"  (EMS)
  18180.  
  18181. 2.19.  A woman goes into a convenience store to buy a can of Coke.  She
  18182. pays for it with a $20 bill and receives $22 in change.  (MI; partial MB
  18183. wording)
  18184.  
  18185. 2.20.  A newspaper reported that Jacques Dubois finished first in the
  18186. walking race held in Paris.  The number of miles he walked was given
  18187. as 62,137.  The article was not in error.  (AR, quoting Richard Fowell;
  18188. MB wording)
  18189.  
  18190. Organization: Penn State University
  18191. Date: Tuesday, 4 Dec 1990 20:08:00 EST
  18192. From: SCOTT MATTHEWS <SDM119@psuvm.psu.edu>
  18193.    A man goes to a hardware store to buy a certain item.  He asks the salesman
  18194. how much this item costs to which he answers, "They are 3 for $1.00."  The man
  18195. say, "Okay I'll take 100," to which the salesman correctly replies, "That will
  18196. be $1.00."  The man pays $1.00 and leaves satisfied.  What is the item.
  18197.  
  18198. >"A man, his son, and his grandson had their first birthday together."
  18199. (Matthew P Wiener original)
  18200.  
  18201. -----------------------------------------
  18202.  
  18203. Just too weird and/or random and/or silly for me:
  18204.  
  18205. 2.17.  A woman walks up to a door and knocks.  Another woman answers the
  18206. door.  The woman outside kills the woman inside.  (GH)
  18207.  
  18208. 2.59.  A man is lying dead in a pool of blood and glass.  (PRO)
  18209.  
  18210. 2.60.  The seals came up to do their show but immediately dove back into
  18211. the water.  (PRO)
  18212.  
  18213. 2.58.  A raft carrying passengers took a trip down a river.  None of the
  18214. passengers made it home alive.  (CR; partial JM wording)
  18215.  
  18216. -----------------------------------------
  18217.  
  18218. Confusing the map with the territory, or, call by reference:
  18219.  
  18220. 2.22.  In his own home a man watches as a woman dies, yet does nothing
  18221. to save her.  (MN)
  18222.  
  18223. 2.39.  King Henry VIII is lying at the bottom of the stairs with a gash
  18224. across his face.  (PRO)
  18225.  
  18226. 2.40.  A man travels to twenty countries and stays in each country for a
  18227. month.  During this time he never sees the light of day.  (PRO)
  18228.  
  18229.  
  18230. -----------------------------------------
  18231.  
  18232. How to prove your audience are sexists:
  18233.  
  18234. 2.48.  A boy and his father are injured in a car accident.  Both are
  18235. taken to a hospital.  The father dies at arrival, but the boy lives
  18236. and is taken to surgery.  A grey-haired, bespectacled surgeon looks at
  18237. the boy and says, "I cannot operate on this boy -- he's my son."  (JV)
  18238.  
  18239. 2.49.  A husband coming home hears his wife call "Bill, don't kill me]".
  18240. He walks in and finds his wife dead.  Inside are a postman, a doctor,
  18241. and a lawyer, none of whom the husband knows.  The husband immediately
  18242. realizes the postman killed his wife.  (EMS; partial JM wording)
  18243.  
  18244. -----------------------------------------
  18245.  
  18246. Need some work:
  18247.  
  18248. 2.56.  She said "I love you," and died.  (EMS)
  18249.  
  18250. Q.  A woman gets up, drives to town, buys a gun, and shoots her husband.
  18251.  
  18252. >  >"He opens his mouth and she dies."  (Ivan A Derzhanski)
  18253.  
  18254. >  >"He comes home, undresses, turns the light off and goes to bed. After a few
  18255. >  >minutes he springs up and says, `There's a corpse under my bed]'"
  18256. (Ivan A Derzhanski)
  18257.  
  18258. 2.34.  A man is holding a box.  Though he cannot see into it, he knows
  18259. what's inside.  (Eric Stephan original)
  18260.  
  18261. -----------------------------------------
  18262.  
  18263. Miscellaneous others:
  18264.  
  18265. 2.24.  The telephone rang in the middle of the night and the woman woke
  18266. up.  When she answered it the caller hung up.  The caller felt better.
  18267. (Sasan Soltani)
  18268.  
  18269. 2.27.  A man called to a waiter in a restaurant, "There's a fly in my
  18270. tea]"  "I will bring you a fresh cup of tea," said the waiter.  After a
  18271. few moments, the man called out, "This is the same cup of tea]"  How did
  18272. he know?  (PRO)
  18273.  
  18274. 2.28.  A man drives over a broken glass bottle.  He travels the last 100
  18275. miles of the Sahara 5000 roadrace with a flat tire.  (EMS)
  18276.  
  18277. 2.35.  A man was walking along some railroad tracks when he noticed that
  18278. a train was coming.  He ran toward the train before stepping aside. (RM)
  18279.  
  18280. 2.41.  A man puts a quarter down, and leaves.  (PRO)
  18281.  
  18282. 2.44.  A dish moves, a scientist makes a discovery.  (MN)
  18283.  
  18284. 2.45.  An Arab sheikh tells his two sons that are to race their camels
  18285. to a distant city to see who will inherit his fortune.  The one whose
  18286. camel arrives last will win.  The brothers, after wandering aimlessly
  18287. for days, ask a wise man for advise.  After hearing the advice they jump
  18288. on the camels and race as fast as they can to their destination.  (PRO)
  18289.  
  18290. 2.46.  Two children born in the same hospital, in the same hour, day,
  18291. and year, have the same mother and father, but are not twins.  (Sasan
  18292. Soltani)
  18293.  
  18294. 2.47.  A couple will build a square house.  In each wall they'll have a
  18295. window, and each window will face north.  (Sasan Soltani)
  18296.  
  18297. The man who built it didn't use it, the man who used it didn't want it,
  18298. etc.
  18299.  
  18300. 2.52.  A man pleads with his boss not to fly to Chicago.  The boss goes
  18301. anyway, and when he returns, he fires the man.  (EMS)
  18302.  
  18303. 2.53.  On an archeological dig, the frozen remains of a man and woman
  18304. are found.  Immediately, the archeologists realize that the remains are
  18305. those of Adam and Eve.  (EMS)
  18306.  
  18307. 2.54.  A man carrying an attache case full of $20 bills falls on the way
  18308. to the bank and is never seen again.  (PRO)
  18309.  
  18310. Q.  A man sees his wife, and later kills her
  18311.  
  18312. >From klkarp@remus.rutgers.edu Mon Dec 17 22:04:57 1990
  18313. Date: Mon, 17 Dec 90 22:07:25 EST
  18314. (Karen Karp)
  18315. 4)  A guy is trapped in a room with a bed, a calender, a saw and a
  18316. table.  There are no windows or doors (except a vent to breathe if you
  18317. get technical).  How does the guy live and finally escape??
  18318.  
  18319. from Joe Kincaid:
  18320.       6)  A man is found dead at his work table.  The investigating policeman
  18321.           looks the scene over and immediately declares it to be murder.
  18322.  
  18323. From: Ivan A Derzhanski <iad@cogsci.edinburgh.ac.uk>
  18324. Date: Thu, 27 Feb 92 15:03:19 GMT
  18325. Historical note: The oldest "situation puzzle" (well, kind of) I know
  18326. of is described in the Maqamat of Al-Hariri.  It is actually a puzzle
  18327. for lawyers, and it goes like this: "A man (X) had a brother (B), and
  18328. his wife had a brother (WB) too.  All of them were free Muslims by
  18329. birth.  When X died, all he left went to WB; B got nothing.  How can
  18330. thbis be lawful?"
  18331.  
  18332. Date:   Sat, 17 Nov 1990 03:14:00 -0500
  18333. From: msb@sq.com (Mark Brader)
  18334. By the way, this one reminds me of the Isaac Asimov story where an agent
  18335. is shot and gives the dying clue "the blind man".  I think that might have
  18336. been the title, too, I don't remember.
  18337.  
  18338. 1.35.  A policeman follows a burglar into a bar.  When he enters the bar
  18339. he finds two similar-looking men, dressed alike, with the loot between
  18340. them.  After several minutes he arrests one of the men.  (PRO, from
  18341. "Which is Which?" by Isaac Asimov; partial JM wording)
  18342.  
  18343. ==> logic/situation.puzzles.outtakes.s <==
  18344.  
  18345. -----------------------------------------
  18346.  
  18347. Contra-reality puzzles, or, "That's not the way it works]"
  18348.  
  18349. 2.10.  A man is sitting in a train compartment.  He sees a three-
  18350. fingered hand through the compartment window, in the hallway of the
  18351. train.  He opens the compartment door and shoots the person with the
  18352. three-fingered hand, but he goes free.  (Michael Bernstein)
  18353. 2.10.  He's with a policeman, who's taking him to jail, and he uses the
  18354. policeman's gun.  He was convicted of his wife's murder; she had framed
  18355. him for it somehow, involving cutting off two of her own fingers and
  18356. mailing them to the police.  Since he had already been convicted of her
  18357. murder, he couldn't be tried twice for the same crime, and since he
  18358. obviously hadn't actually been guilty before, he's set free.  The main
  18359. problem with this question is that as far as I know, that's NOT how the
  18360. "double jeopardy" law works, and so it couldn't happen in real life.
  18361. Nonetheless, it's a neat setup.
  18362. ▌Further info:
  18363. >From ypay@leland.Stanford.EDU Wed Feb 26 12:06:34 1992
  18364. By Cheryl Balbes:
  18365. Situation:
  18366. A woman sees a man on a train eating an orange. She shoots and kills him. She
  18367. is arrested and known to be sane and guilty but does not go to jail.
  18368. Solution:
  18369. The man and the woman were married. It was a terrible marriage - so terrible
  18370. that he wanted revenge for it. So he cut off three of his fingers and burned
  18371. down the house. The police arrived and arrested the woman for murder of her
  18372. husband, citing the fingers as evidence of his death. She was tried and
  18373. convicted and went to jail for most of her life. When she finally got out, she
  18374. took a train ride and saw a man eating an orange. When he used the orange
  18375. peeler, she could see that he was missing three fingers so she knew he was her
  18376. husband. For ruining her life, she took out a gun and shot him dead. She was
  18377. arrested and known to be guilty, but could not go to jail again for the same
  18378. crime.
  18379. Dan Cory¿
  18380. ▌Ian Collier has a slightly variant answer:
  18381. 30 years ago, the man and the woman (his wife) attempted to
  18382. defraud the insurance company by faking her death. However he was found
  18383. guilty of murdering his wife and given a long prison sentence (his wife
  18384. remained hidden and made no attempt to prevent the conviction). The man,
  18385. having just been freed from prison, summons his wife from a far city
  18386. and shoots her. He is not punished, because he has already served the
  18387. sentence.¿
  18388.  
  18389. 2.61.  A man ran into a fire, and lived.  A man stayed where there was
  18390. no fire, and died.  (Eric Wang original)
  18391. 2.61.  The two men were computer programmers working in a small room
  18392. protected by a halon gas fire extinguisher system, when a fire broke out
  18393. in an adjoining room.  One of the programmers ran through the fire and
  18394. escaped with only minor burns.  The other one stayed in the room until
  18395. the building's fire extinguishers kicked in, and died of oxygen
  18396. starvation when the halon gas combined with all of the oxygen in the
  18397. room.  I'm told by Rolf Wilson that this really isn't how halon gas
  18398. extinguishers work, so this puzzle should unfortunately be removed from
  18399. the list.
  18400.  
  18401. 2.50.  The pope is giving a speech.  A man in the audience shoots the
  18402. mayor who is behind the pope.  (PRO)
  18403. 2.50.  The pope has returned to the village where he began his
  18404. priesthood fifty years earlier.  He was late for the ceremony, so the
  18405. mayor spoke first; he claimed to be the first person to give confession
  18406. to the pope, fifty years earlier.  When the pope arrived, he related
  18407. that the first confession he had heard was that of the murder of a young
  18408. woman.  The man in the audience had a sister who was murdered at that
  18409. time.  The sanctity of the confessional is conveniently ignored.
  18410.  
  18411. Date: 2 Feb 92 23:05:11 GMT
  18412. In article <64023@netnews.upenn.edu>, weemba@libra (Matthew P Wiener) writes:
  18413. >Here's ▌one¿ I made up years ago: "She stopped having sex. She died."
  18414. A woman was a hemophiliac.  She stayed pregnant whenever possible.  When
  18415. she stopped having sex, she had her first period, and bled to death.
  18416. ▌jeh comments: there was a lot of debate on the Net about this, most of
  18417. which tended to conclude that (a) menstrual fluid isn't blood; (b) most
  18418. of the few female hemophiliacs die at birth; and (c) terminating a
  18419. pregnancy in any conceivable (heh) way is likely to result in too much
  18420. blood loss.¿
  18421.  
  18422. 1.37.  A holy man is dead in a room.  (Perry Deess original)
  18423. 1.37.  The man is a Moslem.  He was caught stealing, and so his right
  18424. hand was cut off.  However, he's very devout, and thus isn't allowed
  18425. to eat using his left hand; so he starved to death.  It could be
  18426. argued that this isn't very realistic; he could've gotten someone to
  18427. feed him, or somehow eaten without using his hands at all.  And I
  18428. don't know if those rules of Islam really interact that way.  Any real
  18429. info would be appreciated.
  18430. ▌Ivan A Derzhanski writes:
  18431. They are supposed to use their left hand.  There is more than one tale
  18432. in the _Arabian Nights_ about someone who has lost his right hand,
  18433. typically as a punishment for theft (and typically unjustly).  He
  18434. conceals this fact (long sleeves and all that), until he is served a
  18435. meal and the storyteller sees him eating with his left hand, something
  18436. which is not exactly taboo, but is not the done thing either.
  18437. Note that the punishment for theft is known to be loss of the right
  18438. hand, not death by starvation.
  18439. I would be more interested to know what a Hindu would do.  Those
  18440. people are much more careful about what is to be done with which hand.
  18441. And, of course, there are many ways to lose a hand.¿
  18442.  
  18443. -----------------------------------------
  18444.  
  18445. Clocks, calendars, money, and other numerical trivia:
  18446.  
  18447. 2.15.  Two people are talking long distance on the phone; one is in an
  18448. East-Coast state, the other is in a West-Coast state.  The first asks
  18449. the other "What time is it?", hears the answer, and says, "That's funny.
  18450. It's the same time here]"  (EMS)
  18451. 2.15.  One is in Eastern Oregon (in Mountain time), the other in
  18452. Western Florida (in Central time), and it's daylight-savings changeover
  18453. day at 1:30 AM.
  18454. 2.15a.  Variant answer: The east-coast state is in the USA, on Eastern
  18455. Daylight time.  The west-coast state is Western Australia.  There's a
  18456. twelve-hour time difference, so it's 8 o'clock in both places.  (from
  18457. Tim Lambert.)
  18458.  
  18459. 2.19.  A woman goes into a convenience store to buy a can of Coke.  She
  18460. pays for it with a $20 bill and receives $22 in change.  (MI; partial MB
  18461. wording)
  18462. 2.19.  It's in Canada; she pays in American money and receives change in
  18463. Canadian money.  Mark Brader points out that the amount of change can
  18464. vary wildly depending on the price of the drink as well as both the
  18465. official exchange rate and the actual exchange rate given.
  18466.  
  18467. 2.20.  A newspaper reported that Jacques Dubois finished first in the
  18468. walking race held in Paris.  The number of miles he walked was given
  18469. as 62,137.  The article was not in error.  (AR, quoting Richard Fowell;
  18470. MB wording)
  18471. 2.20.  The comma, in European numbers, is used the same way Americans
  18472. use a decimal point.  The man thus (Americans would say) walked 62.137
  18473. miles, or 100 km.  Mark Brader points out that no newspaper in a country
  18474. which uses the decimal comma would report the distance in miles; if
  18475. anyone can think of a way around this problem, let me know.  One
  18476. possibility is to say that he walked 42,551 km (or whatever the actual
  18477. length of a marathon is); but I don't know whether there are such things
  18478. as walking marathons, or whether they'd be the same length in Europe as
  18479. in America.
  18480.  
  18481. Organization: Penn State University
  18482. Date: Tuesday, 4 Dec 1990 20:08:00 EST
  18483. From: SCOTT MATTHEWS <SDM119@psuvm.psu.edu>
  18484.    A man goes to a hardware store to buy a certain item.  He asks the salesman
  18485. how much this item costs to which he answers, "They are 3 for $1.00."  The man
  18486. say, "Okay I'll take 100," to which the salesman correctly replies, "That will
  18487. be $1.00."  The man pays $1.00 and leaves satisfied.  What is the item.
  18488. ▌A: house numbers.¿
  18489.  
  18490. >"A man, his son, and his grandson had their first birthday together."
  18491. (Matthew P Wiener original)
  18492. David Grabiner's answer:
  18493. Sweden had no leap-year days from 1748 to 1788 in order to catch up with
  18494. the Gregorian calendar without creating excessive trouble.  (In many
  18495. other countries, people found that their loans suddenly became due
  18496. eleven days earlier.)
  18497. Thus, the grandfather was born in Sweden on February 29, 1744; the
  18498. other two were born outside Sweden on February 29, 1768 and 1788,
  18499. and returned to Sweden before their fourth birthdays.
  18500. ▌jeh comments: this is, as Matt said in his original posting, an
  18501. obscure-calendar puzzle; and the net indicated that the calendar
  18502. story may not be true anyway.¿
  18503.  
  18504. -----------------------------------------
  18505.  
  18506. Just too weird and/or random and/or silly for me:
  18507.  
  18508. 2.17.  A woman walks up to a door and knocks.  Another woman answers the
  18509. door.  The woman outside kills the woman inside.  (GH)
  18510. 2.17.  The woman outside is a psychotic librarian.  The woman inside has
  18511. an extremely overdue book.
  18512. 2.17a.  Variant answer: The woman outside is married and lived at the
  18513. home in question.  She misplaced her key, and the door was answered by
  18514. her husband's lover.  Though this answer would allow the question to be
  18515. in section 1, it's really a much-less-interesting version of #1.15, and
  18516. it seems to me that it would be a fairly obvious answer.
  18517.  
  18518. 2.59.  A man is lying dead in a pool of blood and glass.  (PRO)
  18519. 2.59.  The man caught a large fish and was so excited he went to a phone
  18520. booth to call his wife.  In trying to describe the size of the fish, he
  18521. said, "It was THIS big]" and stretched his arms wide to indicate its
  18522. length.  His arms went through the sides of the phone booth, his wrists
  18523. were sliced by broken glass, and he bled to death.
  18524. ▌Variant from Bernd Wechner:
  18525. A man makes a telephone call and dies.
  18526. Answer: The man was ringing his wife, and learned from her that he had
  18527. won the lottery.  In jumping for joy he broke through the glass wall
  18528. of the telephone booth and cut his wrists whereupon he bled to death.¿
  18529.  
  18530. 2.60.  The seals came up to do their show but immediately dove back into
  18531. the water.  (PRO)
  18532. 2.60.  The seals were frightened by an audience of nuns, who, to the
  18533. seals, looked like a herd of killer whales.
  18534.  
  18535. 2.58.  A raft carrying passengers took a trip down a river.  None of the
  18536. passengers made it home alive.  (CR; partial JM wording)
  18537. 2.58.  The raft was floating down the Amazon river when it floated under
  18538. a big tree.  A snake was hanging down out of the tree, so the people
  18539. pushed the entire raft away from the tree, where it capsized.  The
  18540. passengers were then eaten by piranha.
  18541.  
  18542. -----------------------------------------
  18543.  
  18544. Confusing the map with the territory, or, call by reference:
  18545.  
  18546. 2.22.  In his own home a man watches as a woman dies, yet does nothing
  18547. to save her.  (MN)
  18548. 2.22.  He saw it happening on TV.
  18549.  
  18550. 2.39.  King Henry VIII is lying at the bottom of the stairs with a gash
  18551. across his face.  (PRO)
  18552. 2.39.  It is a painting of Henry VIII.
  18553.  
  18554. 2.40.  A man travels to twenty countries and stays in each country for a
  18555. month.  During this time he never sees the light of day.  (PRO)
  18556. 2.40.  The man is a mummy, on tour to different museums throughout the
  18557. world.
  18558.  
  18559. ▌note similarity of type to "ship at bottom of sea" and "husband who'd
  18560. blown his brains out."¿
  18561.  
  18562. -----------------------------------------
  18563.  
  18564. How to prove your audience are sexists:
  18565.  
  18566. 2.48.  A boy and his father are injured in a car accident.  Both are
  18567. taken to a hospital.  The father dies at arrival, but the boy lives
  18568. and is taken to surgery.  A grey-haired, bespectacled surgeon looks at
  18569. the boy and says, "I cannot operate on this boy -- he's my son."  (JV)
  18570. 2.48.  The surgeon is the boy's mother.  As with #2.45, #2.46, and
  18571. #2.47, I've frequently heard this as presented as a riddle; the
  18572. attributions for these indicate the first person to tell them to me as
  18573. mystery questions.
  18574.  
  18575. 2.49.  A husband coming home hears his wife call "Bill, don't kill me]".
  18576. He walks in and finds his wife dead.  Inside are a postman, a doctor,
  18577. and a lawyer, none of whom the husband knows.  The husband immediately
  18578. realizes the postman killed his wife.  (EMS; partial JM wording)
  18579. 2.49.  The postman is a man.  The doctor and lawyer are women.
  18580.  
  18581. -----------------------------------------
  18582.  
  18583. Need some work:
  18584.  
  18585. 2.56.  She said "I love you," and died.  (EMS)
  18586. 2.56.  She was a circus performer who performed rope tricks.  During one
  18587. of them, she hung from the ceiling holding only a rope in her mouth.
  18588. The other end of the rope was held by her husband.  There's no
  18589. motivation given for her choosing to do something so stupid; if anyone
  18590. wants to twiddle this into a more reasonable question, please do.
  18591.  
  18592. Q.  A woman gets up, drives to town, buys a gun, and shoots her husband.
  18593. A.  The woman suspects her husband of cheating on her.  She notes the mileage
  18594.         on the car each day.  The previous night, hubby worked late at the
  18595.         office, but the mileage on the car is far greater than can be
  18596.         accounted for.  (from Simon Travaglia)
  18597. ▌jeh comments: This last could make a really nice puzzle, but has too many
  18598. plotholes as it stands.  Rework it sometime.¿
  18599.  
  18600. >  >"He opens his mouth and she dies."  (Ivan A Derzhanski)
  18601. The male acrobat
  18602. hangs from the ceiling and holds the female acrobat by his teeth.  He
  18603. drops her, and she breaks her spine.
  18604. ▌jeh comments: again, this needs more of a real story for me to accept
  18605. it.¿
  18606.  
  18607. >  >"He comes home, undresses, turns the light off and goes to bed. After a few
  18608. >  >minutes he springs up and says, `There's a corpse under my bed]'"
  18609. (Ivan A Derzhanski)
  18610. He hears a watch
  18611. tick under the bed.  Why the watch has to be on the hand of someone
  18612. (and if it is, he is obviously dead, because his breath is not heard)
  18613. is left to the guessers' discretion.
  18614. ▌jeh comments: see previous comment.¿
  18615.  
  18616. 2.34.  A man is holding a box.  Though he cannot see into it, he knows
  18617. what's inside.  (Eric Stephan original)
  18618. 2.34.  He's allergic to whatever's inside the box.
  18619. ▌jeh comments: how is this different from just being able to smell it?¿
  18620.  
  18621. -----------------------------------------
  18622.  
  18623. Miscellaneous others:
  18624.  
  18625. 2.24.  The telephone rang in the middle of the night and the woman woke
  18626. up.  When she answered it the caller hung up.  The caller felt better.
  18627. (Sasan Soltani)
  18628. 2.24.  It was a husband calling from overseas to see that his wife
  18629. arrived home all right.  Hanging up before three seconds elapse results
  18630. in no charge to the calling party.  He could not call person-to-person
  18631. because the local operators did not speak English.
  18632.  
  18633. 2.27.  A man called to a waiter in a restaurant, "There's a fly in my
  18634. tea]"  "I will bring you a fresh cup of tea," said the waiter.  After a
  18635. few moments, the man called out, "This is the same cup of tea]"  How did
  18636. he know?  (PRO)
  18637. 2.27.  The man had already sugared his tea before sending it back.
  18638.  
  18639. 2.28.  A man drives over a broken glass bottle.  He travels the last 100
  18640. miles of the Sahara 5000 roadrace with a flat tire.  (EMS)
  18641. 2.28.  The flat tire is his spare.
  18642.  
  18643. 2.35.  A man was walking along some railroad tracks when he noticed that
  18644. a train was coming.  He ran toward the train before stepping aside. (RM)
  18645. 2.35.  The man was on a bridge, closer to the end the train was
  18646. approaching from.
  18647.  
  18648. 2.41.  A man puts a quarter down, and leaves.  (PRO)
  18649. 2.41.  The man has put a quarter of the cost of a new car into a down
  18650. payment; he then drives away in the car.
  18651.  
  18652. 2.44.  A dish moves, a scientist makes a discovery.  (MN)
  18653. 2.44.  The dish is a satellite dish.
  18654.  
  18655. 2.45.  An Arab sheikh tells his two sons that are to race their camels
  18656. to a distant city to see who will inherit his fortune.  The one whose
  18657. camel arrives last will win.  The brothers, after wandering aimlessly
  18658. for days, ask a wise man for advise.  After hearing the advice they jump
  18659. on the camels and race as fast as they can to their destination.  (PRO)
  18660. 2.45.  The wise man tells them to switch camels.
  18661.  
  18662. 2.46.  Two children born in the same hospital, in the same hour, day,
  18663. and year, have the same mother and father, but are not twins.  (Sasan
  18664. Soltani)
  18665. 2.46.  The children are two of a set of triplets.
  18666. ▌jeh wonders: is this fundamentally different from the people
  18667. crowding under an umbrella or the black-painted town?  Should all
  18668. three be together on one list or the other?¿
  18669.  
  18670. 2.47.  A couple will build a square house.  In each wall they'll have a
  18671. window, and each window will face north.  (Sasan Soltani)
  18672. 2.47.  The house is at the south pole.  This is much the same question
  18673. as the age-old riddle asking what color a certain dead bear is.
  18674.  
  18675. The man who built it didn't use it, the man who used it didn't want it,
  18676. etc.  (A: coffin.)  Suggested as a story riddle by Ed Wagner.
  18677. ▌jeh sez: If I included this, I'd feel obliged to include every riddle I've
  18678. ever heard.¿
  18679.  
  18680. 2.52.  A man pleads with his boss not to fly to Chicago.  The boss goes
  18681. anyway, and when he returns, he fires the man.  (EMS)
  18682. 2.52.  The man was a night watchman who told his boss that last night he
  18683. had a dream that the boss would die in a plane crash.  The boss fired
  18684. him for sleeping on the job.
  18685.  
  18686. 2.53.  On an archeological dig, the frozen remains of a man and woman
  18687. are found.  Immediately, the archeologists realize that the remains are
  18688. those of Adam and Eve.  (EMS)
  18689. 2.53.  The two bodies lacked what only Adam and Eve would lack --
  18690. bellybuttons.
  18691.  
  18692. 2.54.  A man carrying an attache case full of $20 bills falls on the way
  18693. to the bank and is never seen again.  (PRO)
  18694. 2.54.  The man falls off the river bank and drowns.
  18695.  
  18696. Q.  A man sees his wife, and later kills her
  18697. A.  The man sees his wife "performing" at a peep show.  (from Simon
  18698. Travaglia)
  18699.  
  18700. >From klkarp@remus.rutgers.edu Mon Dec 17 22:04:57 1990
  18701. Date: Mon, 17 Dec 90 22:07:25 EST
  18702. (Karen Karp)
  18703. 4)  A guy is trapped in a room with a bed, a calender, a saw and a
  18704. table.  There are no windows or doors (except a vent to breathe if you
  18705. get technical).  How does the guy live and finally escape??
  18706. 4) He eats dates from the calender, drinks water from the springs in
  18707. the bed and saws the table in half, 2 halves make a whole and he
  18708. escapes out the hole.
  18709.  
  18710. from Joe Kincaid:
  18711.       6)  A man is found dead at his work table.  The investigating policeman
  18712.           looks the scene over and immediately declares it to be murder.
  18713.               The man is a blind hemophiliac who *always* keeps his work
  18714.           table in precise order because of his condition(s).  When he
  18715.           reached for his awl, it was turned upside-down and he impaled his
  18716.           hand on it.  Being a hemophiliac, he bled to death.  This couldn't
  18717.           happen by accident.
  18718. ▌jeh comments: I suppose this *could* happen in real life; but it seems to
  18719. me that it *could* happen by accident.  Perhaps this is no less plausible
  18720. than the "blind midget"-type puzzles; I'm ambivalent about it.  But I'm
  18721. leaving it out for now.¿
  18722.  
  18723. From: Ivan A Derzhanski <iad@cogsci.edinburgh.ac.uk>
  18724. Date: Thu, 27 Feb 92 15:03:19 GMT
  18725. Historical note: The oldest "situation puzzle" (well, kind of) I know
  18726. of is described in the Maqamat of Al-Hariri.  It is actually a puzzle
  18727. for lawyers, and it goes like this: "A man (X) had a brother (B), and
  18728. his wife had a brother (WB) too.  All of them were free Muslims by
  18729. birth.  When X died, all he left went to WB; B got nothing.  How can
  18730. thbis be lawful?"  Solution: "X had married his son (S) to his
  18731. mother-in-law (WM).  S died, but left a son, who turned out to be a
  18732. brother of X's wife (being a son of her mother, WM), but at the same
  18733. time he is a grandson of X, and the grandson, as a direct
  18734. descendant, has more rights to the heritage than the brother."
  18735.  
  18736. Date:   Sat, 17 Nov 1990 03:14:00 -0500
  18737. From: msb@sq.com (Mark Brader)
  18738. By the way, this one reminds me of the Isaac Asimov story where an agent
  18739. is shot and gives the dying clue "the blind man".  I think that might have
  18740. been the title, too, I don't remember.  The solution:  the cover role of
  18741. the enemy agent who shot him was a repairman, and he got admission to
  18742. the premises to fix a broken window blind.
  18743. ▌jeh comments: reminds me of "The ▌Case of the?¿ Three Blind Mice,"
  18744. in which a dying man gasps that the one who killed him was "Mice..."
  18745. Turns out not to be "my s...ister" or "my s...on" or the character
  18746. named "Muyskins," but "my s...olicitor"; the guy is British.¿
  18747.  
  18748. 1.35.  A policeman follows a burglar into a bar.  When he enters the bar
  18749. he finds two similar-looking men, dressed alike, with the loot between
  18750. them.  After several minutes he arrests one of the men.  (PRO, from
  18751. "Which is Which?" by Isaac Asimov; partial JM wording)
  18752. 1.35.  Both men were wearing glasses.  The burglar, however, was wearing
  18753. photosensitive sunglasses; the policeman noticed them changing shade and
  18754. realized the man must have just entered.
  18755.  
  18756.  
  18757.  
  18758.  
  18759. ==> logic/situation.puzzles.p <==
  18760.                         Jed's List of Situation Puzzles
  18761.  
  18762. History:
  18763.    original compilation            11/28/87
  18764.    major revision                  08/09/89
  18765.    further additions               08/23/89 - 10/21/90
  18766.    variants added to answer list   07/04/90
  18767.    editing and renumbering         07/25/90 - 11/11/90
  18768.    items removed; title changed    09/20/90 - 11/11/90
  18769.    editing and additions           02/26/92 - 09/17/92
  18770.  
  18771.  
  18772.    "A man lies dead in a room with fifty-three bicycles in front of him.
  18773. What happened?"
  18774.  
  18775.    This is a list of what I refer to (for lack of a better name) as
  18776. situation puzzles.  In the game of situation puzzles, a situation like the
  18777. one above is presented to a group of players, who must then try to find
  18778. out more about the situation by asking further questions.  The person who
  18779. initially presented the situation can only answer "yes" or "no" to
  18780. questions (or occasionally "irrelevant" or "doesn't matter").
  18781.  
  18782.    My list has been divided into two sections.  Section 1 consists of
  18783. situation puzzles which are set in a realistic world; the situations could
  18784. all actually occur.  Section 2 consists of puzzles which involve double
  18785. meanings for one or more words and those which could not possibly take
  18786. place in reality as we know it, plus a few miscellaneous others.
  18787.  
  18788.    See the end of the list for more notes and comments.
  18789.  
  18790.  
  18791. Section 1: "Realistic" situation puzzles.
  18792.  
  18793. 1.1.  In the middle of the ocean is a yacht.  Several corpses are floating
  18794. in the water nearby.  (SJ)
  18795.  
  18796. 1.2.  A man is lying dead in a room.  There is a large pile of gold and
  18797. jewels on the floor, a chandelier attached to the ceiling, and a large
  18798. open window.  (DVS; partial JM wording)
  18799.  
  18800. 1.3.  A woman came home with a bag of groceries, got the mail, and walked
  18801. into the house.  On the way to the kitchen, she went through the living
  18802. room and looked at her husband, who had blown his brains out.  She then
  18803. continued to the kitchen, put away the groceries, and made dinner.
  18804. (partial JM wording)
  18805.  
  18806. 1.4.  A body is discovered in a park in Chicago in the middle of summer.
  18807. It has a fractured skull and many other broken bones, but the cause of
  18808. death was hypothermia.  (MI, from _Hill Street Blues_)
  18809.  
  18810. 1.5.  A man lives on the twelfth floor of an apartment building.  Every
  18811. morning he takes the elevator down to the lobby and leaves the building.
  18812. In the evening, he gets into the elevator, and, if there is someone else
  18813. in the elevator -- or if it was raining that day -- he goes back to his
  18814. floor directly.  However, if there is nobody else in the elevator and it
  18815. hasn't rained, he goes to the 10th floor and walks up two flights of
  18816. stairs to his room.  (MH)
  18817.  
  18818. 1.6.  A woman has incontrovertible proof in court that her husband was
  18819. murdered by her sister.  The judge declares, "This is the strangest case
  18820. I've ever seen.  Though it's a cut-and-dried case, this woman cannot be
  18821. punished."  (This is different from #1.43.)  (MH)
  18822.  
  18823. 1.7.  A man walks into a bar and asks for a drink.  The bartender pulls
  18824. out a gun and points it at him.  The man says, "Thank you," and walks out.
  18825. (DVS)
  18826.  
  18827. 1.8.  A man is returning from Switzerland by train.  If he had been in a
  18828. non-smoking car he would have died.  (DVS; MC wording)
  18829.  
  18830. 1.9.  A man goes into a restaurant, orders abalone, eats one bite, and
  18831. kills himself.  (TM and JM wording)
  18832.  
  18833. 1.10.  A man is found hanging in a locked room with a puddle of water
  18834. under his feet.  (This is different from #1.11.)
  18835.  
  18836. 1.11.  A man is dead in a puddle of blood and water on the floor of a
  18837. locked room.  (This is different from #1.10.)
  18838.  
  18839. 1.12.  A man is lying, dead, face down in the desert wearing a backpack.
  18840. (This is different from #1.13, #2.11, and #2.12.)
  18841.  
  18842. 1.13.  A man is lying face down, dead, in the desert, with a match near
  18843. his outstretched hand.  (This is different from #1.12, #2.11, and #2.12.)
  18844. (JH; partial JM wording)
  18845.  
  18846. 1.14.  A man is driving his car.  He turns on the radio, listens for five
  18847. minutes, turns around, goes home, and shoots his wife.  (This is different
  18848. from #1.15.)
  18849.  
  18850. 1.15.  A man driving his car turns on the radio.  He then pulls over to
  18851. the side of the road and shoots himself.  (This is different from #1.14.)
  18852.  
  18853. 1.16.  Music stops and a woman dies.  (DVS)
  18854.  
  18855. 1.17.  A man is dead in a room with a small pile of pieces of wood and
  18856. sawdust in one corner.  (from "Coroner's Inquest," by Marc Connelly)
  18857.  
  18858. 1.18.  A flash of light, a man dies.  (ST original)
  18859.  
  18860. 1.19.  A rope breaks.  A bell rings.  A man dies.  (KH)
  18861.  
  18862. 1.20.  A woman buys a new pair of shoes, goes to work, and dies.  (DM)
  18863.  
  18864. 1.21.  A man is riding a subway.  He meets a one-armed man, who pulls out
  18865. a gun and shoots him.  (SJ)
  18866.  
  18867. 1.22.  Two women are talking.  One goes into the bathroom, comes out five
  18868. minutes later, and kills the other.
  18869.  
  18870. 1.23.  A man is sitting in bed.  He makes a phone call, saying nothing,
  18871. and then goes to sleep.  (SJ)
  18872.  
  18873. 1.24.  A man kills his wife, then goes inside his house and kills himself.
  18874. (DH original, from "Nightmare in Yellow," by Fredric Brown)
  18875.  
  18876. 1.25.  Abel walks out of the ocean.  Cain asks him who he is, and Abel
  18877. answers.  Cain kills Abel.  (MWD original)
  18878.  
  18879. 1.26.  Two men enter a bar.  They both order identical drinks.  One lives;
  18880. the other dies.  (CR; partial JM wording)
  18881.  
  18882. 1.27.  Joe leaves his house, wearing a mask and carrying an empty sack.
  18883. An hour later he returns.  The sack is now full.  He goes into a room and
  18884. turns out the lights.  (AL)
  18885.  
  18886. 1.28.  A man takes a two-week cruise to Mexico from the U.S.  Shortly
  18887. after he gets back, he takes a three-day cruise which doesn't stop at any
  18888. other ports.  He stays in his cabin all the time on both cruises.  As a
  18889. result, he makes $250,000.  (MI, from "The Wager")
  18890.  
  18891. 1.29.  Hans and Fritz are German spies during World War II.  They try to
  18892. enter America, posing as returning tourists.  Hans is immediately
  18893. arrested.  (JM)
  18894.  
  18895. 1.30.  Tim and Greg were talking.  Tim said "The terror of flight."  Greg
  18896. said "The gloom of the grave."  Greg was arrested.  (MPW original, from
  18897. "No Refuge Could Save," by Isaac Asimov)
  18898.  
  18899. 1.31.  A man is found dead in his parked car.  Tire tracks lead up to the
  18900. car and away.  (SD)
  18901.  
  18902. 1.32.  A man dies in his own home.  (ME original)
  18903.  
  18904. 1.33.  A woman in Paris in 1895 is waiting for her husband to come home.
  18905. When he arrives, the house has burned to the ground and she's dead.  (JM)
  18906.  
  18907. 1.34.  A man gets onto an elevator.  When the elevator stops, he knows his
  18908. wife is dead.  (LA; partial KH wording)
  18909.  
  18910. 1.35.  Three men die.  On the pavement are pieces of ice and broken glass.
  18911. (JJ)
  18912.  
  18913. 1.36.  She lost her job when she invited them to dinner.  (DS original)
  18914.  
  18915. 1.37.  A man is running along a corridor with a piece of paper in his
  18916. hand.  The lights flicker and the man drops to his knees and cries out,
  18917. "Oh no]"  (MP)
  18918.  
  18919. 1.38.  A car without a driver moves; a man dies.  (EMS)
  18920.  
  18921. 1.39.  As I drive to work on my motorcycle, there is one corner which I go
  18922. around at a certain speed whether it's rainy or sunny.  If it's cloudy but
  18923. not raining, however, I usually go faster.  (SW original)
  18924.  
  18925. 1.40.  A woman throws something out a window and dies.  (JM)
  18926.  
  18927. 1.41.  An avid birdwatcher sees an unexpected bird.  Soon he's dead.  (RSB
  18928. original)
  18929.  
  18930. 1.42.  There are a carrot, a pile of pebbles, and a pipe lying together in
  18931. the middle of a field.  (PRO; partial JM wording)
  18932.  
  18933. 1.43.  Two brothers are involved in a murder.  Though it's clear that one
  18934. of them actually committed the crime, neither can be punished.  (This is
  18935. different from #1.6.)  (from "Unreasonable Doubt," by Stanley Ellin)
  18936.  
  18937. 1.44.  An ordinary American citizen, with no passport, visits over thirty
  18938. foreign countries in one day.  He is welcomed in each country, and leaves
  18939. each one of his own accord.  (PRO)
  18940.  
  18941. 1.45.  If he'd turned on the light, he'd have lived.  (JM)
  18942.  
  18943. 1.46.  A man is found dead on the floor in the living room.  (ME original)
  18944.  
  18945. 1.47.  A man is found dead outside a large building with a hole in him.
  18946. (JM, modified from PRO)
  18947.  
  18948. 1.48.  A man is found dead in an alley lying in a red pool with two sticks
  18949. crossed near his head.  (PRO)
  18950.  
  18951. 1.49.  A man lies dead next to a feather.  (PRO)
  18952.  
  18953. 1.50.  There is blood on the ceiling of my bedroom.  (MI original)
  18954.  
  18955. 1.51.  A man wakes up one night to get some water.  He turns off the light
  18956. and goes back to bed.  The next morning he looks out the window, screams,
  18957. and kills himself.  (CR; KK wording)
  18958.  
  18959. 1.52.  She grabbed his ring, pulled on it, and dropped it.  (JM, from
  18960. _Math for Girls_)
  18961.  
  18962. 1.53.  A man sitting on a park bench reads a newspaper article headlined
  18963. "Death at Sea" and knows a murder has been committed.
  18964.  
  18965. 1.54.  A man tries the new cologne his wife gave him for his birthday.  He
  18966. goes out to get some food, and is killed.  (RW original)
  18967.  
  18968. 1.55.  A man in uniform stands on the beach of a tropical island.  He takes
  18969. out a cigarette, lights it, and begins smoking.  He takes out a letter and
  18970. begins reading it.  The cigarette burns down between his fingers, but he
  18971. doesn't throw it away.  He cries.  (RW)
  18972.  
  18973. 1.56.  A man went into a restaurant, had a large meal, and paid nothing for
  18974. it.  (JM original)
  18975.  
  18976. 1.57.  A married couple goes to a movie.  During the movie the husband
  18977. strangles the wife.  He is able to get her body home without attracting
  18978. attention.  (from _Beyond the Easy Answer_)
  18979.  
  18980.  
  18981. Section 2: Double meanings, fictional settings, and miscellaneous others.
  18982.  
  18983. 2.1.  A man shoots himself, and dies.  (HL) (This is different from #2.2.)
  18984.  
  18985. 2.2.  A man walks into a room, shoots, and kills himself.  (HL) (This is
  18986. different from #2.1.)
  18987.  
  18988. 2.3.  Adults are holding children, waiting their turn.  The children are
  18989. handed (one at a time, usually) to a man, who holds them while a woman
  18990. shoots them.  If the child is crying, the man tries to stop the crying
  18991. before the child is shot.  (ML)
  18992.  
  18993. 2.4.  Hiking in the mountains, you walk past a large field and camp a few
  18994. miles farther on, at a stream.  It snows in the night, and the next day
  18995. you find a cabin in the field with two dead bodies inside.  (KL; KD and
  18996. partial JM wording)
  18997.  
  18998. 2.5.  A man marries twenty women in his village but isn't charged with
  18999. polygamy.
  19000.  
  19001. 2.6.  A man is alone on an island with no food and no water, yet he does
  19002. not fear for his life.  (MN)
  19003.  
  19004. 2.7.  Joe wants to go home, but he can't go home because the man in the
  19005. mask is waiting for him.  (AL wording)
  19006.  
  19007. 2.8.  A man is doing his job when his suit tears.  Fifteen minutes later,
  19008. he's dead.  (RM)
  19009.  
  19010. 2.9.  A dead man lies near a pile of bricks and a beetle on top of a book.
  19011. (MN)
  19012.  
  19013. 2.10.  At the bottom of the sea there lies a ship worth millons of dollars
  19014. that will never be recovered.  (TF original)
  19015.  
  19016. 2.11.  A man is found dead in the arctic with a pack on his back.  (This
  19017. is different from #1.12, #1.13, and #2.12.)  (PRO)
  19018.  
  19019. 2.12.  There is a dead man lying in the desert next to a rock.  (This is
  19020. different from #1.12, #1.13, and #2.11.)  (GH)
  19021.  
  19022. 2.13.  As a man jumps out of a window, he hears the telephone ring and
  19023. regrets having jumped.  (from "Some Days are Like That," by Bruce J.
  19024. Balfour; partial JM wording)
  19025.  
  19026. 2.14.  Two people are playing cards.  One looks around and realizes he's
  19027. going to die.  (JM original)
  19028.  
  19029. 2.15.  A man lies dead in a room with fifty-three bicycles in front of
  19030. him.
  19031.  
  19032. 2.16.  A horse jumps over a tower and lands on a man, who disappears.  (ES
  19033. original)
  19034.  
  19035. 2.17.  A train pulls into a station, but none of the waiting passengers
  19036. move.  (MN)
  19037.  
  19038. 2.18.  A man pushes a car up to a hotel and tells the owner he's bankrupt.
  19039. (DVS; partial AL and JM wording)
  19040.  
  19041. 2.19.  Three large people try to crowd under one small umbrella, but
  19042. nobody gets wet.  (CC)
  19043.  
  19044. 2.20.  A black man dressed all in black, wearing a black mask, stands at a
  19045. crossroads in a totally black-painted town.  All of the streetlights in
  19046. town are broken.  There is no moon.  A black-painted car without
  19047. headlights drives straight toward him, but turns in time and doesn't hit
  19048. him.  (AL and RM wording)
  19049.  
  19050. 2.21.  Bob and Carol and Ted and Alice all live in the same house.  Bob
  19051. and Carol go out to a movie, and when they return, Alice is lying dead on
  19052. the floor in a puddle of water and glass.  It is obvious that Ted killed
  19053. her but Ted is not prosecuted or severely punished.
  19054.  
  19055. 2.22.  A man rides into town on Friday.  He stays one night and leaves on
  19056. Friday.  (KK)
  19057.  
  19058. 2.23.  Bruce wins the race, but he gets no trophy.  (EMS)
  19059.  
  19060. 2.24.  A woman opens an envelope and dyes.  (AL)
  19061.  
  19062. 2.25.  A man was brought before a tribal chief, who asked him a question.
  19063. If he had known the answer, he probably would have died.  He didn't, and
  19064. lived.  (MWD original)
  19065.  
  19066. 2.26.  Two men are found dead outside of an igloo. (SK original)
  19067.  
  19068.  
  19069. Attributions key:
  19070.  
  19071.    When I know who first told me the current version of a puzzle, I've put
  19072. initials in parentheses after the puzzle statement; this is the key to
  19073. those acknowledgments.  The word "original" following an attribution means
  19074. that, to the best of my knowledge, the cited person invented that puzzle.
  19075. If a given puzzle isn't marked "original" but is attributed, that just
  19076. means that's the first person I heard it from.  I would appreciate it if
  19077. attributions for originals were not removed; however, this list is hereby
  19078. entered into the public domain, so do with it what you wish.
  19079.  
  19080. LA == Laura Almasy               RSB == Ranjit S. Bhatnagar
  19081. CC == Chris Cole                 MC == Matt Crawford
  19082. MWD == Matthew William Daly      KD == Ken Duisenberg
  19083. SD == Sylvia Dutcher             ME == Marguerite Eisenstein
  19084. TF == Thomas Freeman             JH == Joaquin Hartman
  19085. MH == Marcy Hartman              KH == Karl Heuer
  19086. GH == Geoff Hopcraft             DH == David Huddleston
  19087. MI == Mark Isaak                 SJ == Steve Jacquot
  19088. JJ == J!rgen Jensen              KK == Karen Karp
  19089. SK == Shelby Kilmer              KL == Ken Largman
  19090. AL == Andy Latto                 HL == Howard Lazoff
  19091. ML == Merlyn LeRoy               RM == "Reaper Man" (real name unknown)
  19092. TM == Ted McCabe                 JM == Jim Moskowitz
  19093. DM == Damian Mulvena             MN == Jan Mark Noworolski
  19094. PRO == Peter R. Olpe (from his list)
  19095. MP == Martin Pitwood             CR == Charles Renert
  19096. EMS == Ellen M. Sentovich (from her list)
  19097. ES == Eric Stephan               DS == Diana Stiefbold
  19098. ST == Simon Travaglia            DVS == David Van Stone
  19099. RW == Randy Whitaker             MPW == Matthew P Wiener
  19100. SW == Steve Wilson (not sure of name)
  19101.  
  19102. Special thanks to Jim Moskowitz, Karl Heuer, and Mark Brader, for a lot of
  19103. discussion of small but important details and wording.
  19104.  
  19105.  
  19106. Notes and comments:
  19107.  
  19108.    My outtakes list (items removed from this list for various reasons,
  19109. most of which came down to the fact that I didn't like them) is now
  19110. available from the r.p FAQ server.
  19111.  
  19112.    There are many possible wordings for most of the puzzles in this list.
  19113. Most of them have what I consider the best wording of the variants I've
  19114. heard; if you think there's a better way of putting one or more of them,
  19115. or if you don't like my categorization of any of them, or if you have any
  19116. other comments or suggestions, please drop me a note.  If you know others
  19117. not on this list, please send them to me.
  19118.    Of course, in telling a group of players one of these situations, you
  19119. can add or remove details, either to make getting the answer harder or
  19120. easier, or simply to throw in red herrings.  I've made a few specific
  19121. suggestions along these lines in the answer list, available in a separate
  19122. file.  Also in the answer list are variant problem statements and variant
  19123. answers.
  19124.  
  19125. --Jed Hartman
  19126. zorn@apple.com (as of 9/92)
  19127.  
  19128.  
  19129.  
  19130. ==> logic/situation.puzzles.s <==
  19131.                   Answers to Jed's List of Situation Puzzles
  19132.  
  19133.    This is the list of answers to the puzzles in my situation puzzles
  19134. list.  See that list for more details.  This document also contains
  19135. variant setups and answers for some of the puzzles.
  19136.  
  19137. Section 1: "Realistic" situation puzzles.
  19138.  
  19139. 1.1.  A bunch of people are on an ocean voyage in a yacht.  One afternoon,
  19140. they all decide to go swimming, so they put on swimsuits and dive off the
  19141. side into the water.  Unfortunately, they forget to set up a ladder on the
  19142. side of the boat, so there's no way for them to climb back in, and they
  19143. drown.
  19144. 1.1a.  Variant answer: The same situation, except that they set out a
  19145. ladder which is just barely long enough.  When they all dive into the
  19146. water, the boat, without their weight, rises in the water until the ladder
  19147. is just barely out of reach.  (also from Steve Jacquot)
  19148.  
  19149. 1.2.  The room is the ballroom of an ocean liner which sank some time ago.
  19150. The man ran out of air while diving in the wreck.
  19151. 1.2a.  Variant which puts this in section 2: same statement, ending with
  19152. "a large window through which rays are coming."  Answer: the rays are
  19153. manta rays (this version tends to make people assume vampires are
  19154. involved, unless they notice the awkwardness of the phrase involving
  19155. rays).
  19156.  
  19157. 1.3.  The husband killed himself a while ago; it's his ashes in an urn on
  19158. the mantelpiece that the wife looks at.  It's debatable whether this
  19159. belongs in section 2 for double meanings.
  19160.  
  19161. 1.4.  A poor peasant from somewhere in Europe wants desperately to get to
  19162. the U.S.  Not having money for airfare, he stows away in the landing gear
  19163. compartment of a jet.  He dies of hypothermia in mid-flight, and falls out
  19164. when the landing gear compartment opens as the plane makes its final
  19165. approach.
  19166. 1.4a.  Variant: A man is lying drowned in a dead forest.  Answer: He's
  19167. scuba diving when a firefighting plane lands nearby and fills its tanks
  19168. with water, sucking him in with the water.  He runs out of air while the
  19169. plane is in flight; the plane then dumps its load of water, with him in
  19170. it, onto a burning forest.  (from Jim Moskowitz)
  19171.  
  19172. 1.5.  The man is a midget.  He can't reach the upper elevator buttons, but
  19173. he can ask people to push them for him.  He can also push them with his
  19174. umbrella.  I've usually heard this stated with more details: "Every
  19175. morning he wakes up, gets dressed, eats, goes to the elevator..."  Ron
  19176. Carter suggests a nice red herring: the man lives on the 13th floor of the
  19177. building.
  19178.  
  19179. 1.6.  The sisters are Siamese twins.
  19180. 1.6a.  Variant: A man and his brother are in a bar drinking.  They begin to
  19181. argue (as always) and the brother won't get out of the man's face, shouting
  19182. and cursing.  The man, finally fed up, pulls out a pistol and blows his
  19183. brother's brains out.  He sits down to die.  Answer: They are Siamese twins.
  19184. In the original story, the argument started when one complained about the
  19185. other's bad hygiene and bad breath.  The shooter bled to death (from his
  19186. brother's wounds) by the time the police arrived.  (from Randy Whitaker,
  19187. based on a 1987 _Weekly World News_ story)
  19188.  
  19189. 1.7.  The man has hiccups; the bartender scares them away by pulling a
  19190. gun.
  19191.  
  19192. 1.8.  The man used to be blind; he's now returning from an eye operation
  19193. which restored his sight.  He's spent all his money on the operation, so
  19194. when the train (which has no internal lighting) goes through a tunnel he
  19195. at first thinks he's gone blind again and almost decides to kill himself.
  19196. Fortunately, the light of the cigarettes people are smoking convinces him
  19197. that he can still see.
  19198. 1.8a.  Variant: A man dies on a train he does not ordinarily catch.
  19199. Answer: The man (a successful artist) has had an accident in which he
  19200. injured his eyes.  His head is bandaged and he has been warned not to
  19201. remove the bandages under any circumstances lest the condition be
  19202. irreversibly aggravated.  He catches the train home from the hospital and
  19203. cannot resist peeking.  Seeing nothing at all (the same train-in-tunnel
  19204. situation as above obtains, but without the glowing cigarettes this time),
  19205. he assumes he is blinded and kills himself in grief.  I like this version
  19206. a lot, except that it makes much less sense that he'd be traveling alone.
  19207. (from Bernd Wechner)
  19208.  
  19209. 1.9.  The man was in a ship that was wrecked on a desert island.  When
  19210. there was no food left, another passenger brought what he said was abalone
  19211. but was really part of the man's wife (who had died in the wreck).  The
  19212. man suspects something fishy, so when they finally return to civilization,
  19213. he orders abalone, realizes that what he ate before was his wife, and
  19214. kills himself.
  19215. 1.9a.  Variant: same problem statement but with albatross instead of
  19216. abalone.  Answer: In this version, the man was in a lifeboat, with his
  19217. wife, who died.  He hallucinated an albatross landing in the boat which he
  19218. caught and killed and ate; he thought that his wife had been washed
  19219. overboard.  When he actually eats albatross, he discovers that he had
  19220. actually eaten his wife.
  19221. 1.9b.  Variant answer to 1.9a, with a slightly different problem
  19222. statement: the man already knew that he had been eating human flesh.  He
  19223. asks the waiter in the restaurant what kind of soup is available, and the
  19224. waiter responds, "Albatross soup."  Thinking that "albatross soup" means
  19225. "human soup," and sickened by the thought of such a society (place in a
  19226. foreign country if necessary), he kills himself.  (from Mike Neergaard)
  19227.  
  19228. 1.10.  He stood on a block of ice to hang himself.  The fact that there's
  19229. no furniture in the room can be added to the statement, but if it's
  19230. mentioned in conjunction with the puddle of water the answer tends to be
  19231. guessed more easily.
  19232.  
  19233. 1.11.  He stabbed himself with an icicle.
  19234.  
  19235. 1.12.  He jumped out of an airplane, but his parachute failed to open.
  19236. Minor variant wording (from Joe Kincaid): he's on a mountain trail instead
  19237. of in a desert.  Minor variant wording (from Mike Reymond): he's got a
  19238. ring in his hand (it came off of the ripcord).
  19239. 1.12a.  Silly variant: same problem statement, with the addition that one
  19240. of the man's shoelaces is untied.  Answer: He pulled his shoelace instead
  19241. of the ripcord.
  19242. 1.12b.  Variant answer: The man was let loose in the desert with a pack
  19243. full of poisoned food.  He  knows it's poisoned, and doesn't eat it -- he
  19244. dies of hunger.  (from Mike Neergaard)
  19245.  
  19246. 1.13.  He was with several others in a hot air balloon crossing the
  19247. desert.  The balloon was punctured and they began to lose altitude.  They
  19248. tossed all their non-essentials overboard, then their clothing and food,
  19249. but were still going to crash in the middle of the desert.  Finally, they
  19250. drew matches to see who would jump over the side and save the others; this
  19251. man lost.  Minor variant wording: add that the man is nude.
  19252.  
  19253. 1.14.  The radio program is one of the call-up-somebody-and-ask-them-a-
  19254. question contest shows; the announcer gives the phone number of the man's
  19255. bedroom phone as the number he's calling, and a male voice answers.  It's
  19256. been suggested that such shows don't usually give the phone number being
  19257. called; so instead the wife's name could be given as who's being called,
  19258. and there could be appropriate background sounds when the other man
  19259. answers the phone.
  19260.  
  19261. 1.15.  He worked as a DJ at a radio station.  He decided to kill his wife,
  19262. and so he put on a long record and quickly drove home and killed her,
  19263. figuring he had a perfect alibi: he'd been at work.  On the way back he
  19264. turns on his show, only to discover that the record is skipping.
  19265. 1.15a.  Variant: The music stops and the man dies.  Answer: The same,
  19266. except it's a tape breaking instead of a record skipping.  (from Michael
  19267. Killianey)  (See also #1.16, #1.19e, and #1.34a.)
  19268.  
  19269. 1.16.  The woman is a tightrope walker in a circus.  Her act consists of
  19270. walking the rope blindfolded, accompanied by music, without a net.  The
  19271. musician (organist, or calliopist, or pianist, or whatever) is supposed to
  19272. stop playing when she reaches the end of the rope, telling her that it's
  19273. safe to step off onto the platform.  For unknown reasons (but with
  19274. murderous intent), he stops the music early, and she steps off the rope to
  19275. her death.
  19276. 1.16a.  Variant answer: The woman is a character in an opera, who "dies"
  19277. at the end of her song.
  19278. 1.16b.  Variant answer: The "woman" is the dancing figure atop a music
  19279. box, who "dies" when the box runs down.  (Both of the above variants would
  19280. probably require placing this puzzle in section 2 of the list.)
  19281. 1.16c.  Variant: Charlie died when the music stopped.  Answer: Charlie was
  19282. an insect sitting on a chair; the music playing was for the game Musical
  19283. Chairs.  (from Bob Philhower)
  19284. (See also #1.15a, #1.19e, and #1.34a.)
  19285.  
  19286. 1.17.  The man is a blind midget, the shortest one in the circus.  Another
  19287. midget, jealous because he's not as short, has been sawing small pieces
  19288. off of the first one's cane every night, so that every day he thinks he's
  19289. taller.  Since his only income is from being a circus midget, he decides
  19290. to kill himself when he gets too tall.
  19291. 1.17a.  Slightly variant answer: Instead of sawing pieces off of the
  19292. midget's cane, someone has sawed the legs off of his bed.  He wakes up,
  19293. stands up, and thinks he's grown during the night.
  19294. 1.17b.  Variant: A pile of sawdust, no net, a man dies.  Answer: A midget
  19295. is jealous of the clown who walks on stilts.  He saws partway through the
  19296. stilts; the clown walks along and falls and dies when they break.  (from
  19297. Peter R. Olpe)
  19298. 1.17c.  Rough sketch of variant: There were a mirror and a bottle on the
  19299. table, and sawdust on the floor.  He came in and dropped dead.  Answer: He
  19300. was a midget, but he wasn't aware of it, because the table used to be too
  19301. high for him to see his reflection in the mirror, until someone shortened
  19302. its legs.  He was horrified by the discovery, and the shock killed him.
  19303. (vaguely remembered by Ivan A Derzhanski, who adds that this would be best
  19304. used as raw material for some elaboration.  I agree; it's pretty
  19305. implausible as is)
  19306.  
  19307. 1.18.  The man is a lion-tamer, posing for a photo with his lions.  The
  19308. lions react badly to the flash of the camera, and the man can't see
  19309. properly, so he gets mauled.
  19310. 1.18a.  Variant: He couldn't find a chair, so he died.  Answer: He was a
  19311. lion-tamer.  This one is kind of silly, but I like it, and it sounds
  19312. possible to me (though I'm told a whip is more important than a chair to a
  19313. lion-tamer).  (from "Reaper Man," with Karl Heuer wording)
  19314.  
  19315. 1.19.  A blind man enjoys walking near a cliff, and uses the sound of a
  19316. buoy to gauge his distance from the edge.  One day the buoy's anchor rope
  19317. breaks, allowing the buoy to drift away from the shore, and the man walks
  19318. over the edge of the cliff.
  19319. 1.19a.  Variant: A bell rings.  A man dies.  A bell rings.  Answer: A
  19320. blind swimmer sets an alarm clock to tell him when and what direction to
  19321. go to shore.  The first bell is a buoy, which he mistakenly swims to,
  19322. getting tired and drowning.  Then the alarm clock goes off.  In other
  19323. variations, the first bell is a ship's bell, and/or the second bell is a
  19324. hand-bell rung by a friend on shore at a pre-arranged time.
  19325. 1.19b.  Variant answer to 1.19a: The man falls off a belltower, pulling
  19326. the bell-cord (perhaps he was climbing a steeple while hanging onto the
  19327. rope), and dies.  The second bell is one rung at his funeral.  Could also
  19328. be a variant on 1.19 (as suggested by Mike Neergaard): the bell-cord
  19329. breaks when he falls (and there's no second bell involved).
  19330. 1.19c.  Variant answer to 1.19a: The man is a boxer.  The first bell
  19331. signals the start of a round; the second is either the end of the round or
  19332. a funeral bell after he dies during the match.  Could also be a variant on
  19333. 1.19 (as suggested by Mike Neergaard): a boxing match in which the top
  19334. rope breaks, tumbling a boxer to the floor (and he dies of a concussion).
  19335. 1.19d.  Variant: The wind stopped blowing and the man died.  Answer: The
  19336. sole survivor of a shipwreck reached a desert isle.  Unfortunately, he was
  19337. blind.  Luckily, there was a freshwater spring on the island, and he
  19338. rigged the ship's bell (which had drifted to the island also) at the
  19339. spring's location.  The bell rang in the wind, directing him to water.
  19340. When he was becalmed for a week, he could not find water again, and so he
  19341. died of thirst.  (from Peter R. Olpe)
  19342. 1.19e.  Variant: The music stopped and the man died.  Answer: Same as
  19343. 1.19a, but the blind swimmer kept a portable transistor radio on the beach
  19344. instead of a bell.  When the batteries gave out, he got lost and drowned.
  19345. (from Joe Kincaid)  (See also #1.15a, #1.16, and #1.34a.)
  19346.  
  19347. 1.20.  The woman is the assistant to a (circus or sideshow) knife thrower.
  19348. The new shoes have higher heels than she normally wears, so that the
  19349. thrower misjudges his aim and one of his knives kills her during the show.
  19350.  
  19351. 1.21.  Several men were shipwrecked together.  They agreed to survive by
  19352. eating each other a piece at a time.  Each of them in turn gave up an arm,
  19353. but before they got to the last man, they were rescued.  They all demanded
  19354. that the last man live up to his end of the deal.  Instead, he killed a
  19355. bum and sent the bum's arm to the others in a box to "prove" that he had
  19356. fulfilled the bargain.  Later, one of them sees him on the subway, holding
  19357. onto an overhead ring with the arm he supposedly cut off; the other
  19358. realizes that the last man cheated, and kills him.
  19359. 1.21a.  Variant wording: A man sends a package to someone in Europe and
  19360. gets a note back saying "Thank you.  I received it."  Answer: This is just
  19361. a simpler version; the shipwreck situation is the same, and the man
  19362. actually did send his own arm.
  19363. 1.21b.  Variant wording: Two men throw a box off of a cliff.  Answer:
  19364. Exactly the same situation as in 1.21a (one slight variation has a hand in
  19365. the box instead of a whole arm), with the two men being two of the fellow
  19366. passengers who had already lost their arms.
  19367. 1.21c.  Variant wording: A man in a Sherlock Holmes-style cape walks
  19368. into a room, places a box on the table and leaves.  Answer: In this one
  19369. he's wearing the cape either to disguise the fact that he hasn't really
  19370. cut off his arm/hand as required, or else simply in order to hide his
  19371. now-missing limb.  (from Joe Kincaid)
  19372.  
  19373. 1.22.  Both women are white; the one whose house this takes place in is
  19374. single.  A black friend of the other woman, the one who goes into the
  19375. bathroom, was recently killed, reportedly by the KKK.  The woman who goes
  19376. into the bathroom discovers a bloodstained KKK robe in the other's laundry
  19377. hamper, picks up a nail file from the medicine cabinet (or some other
  19378. impromptu weapon), and goes out and kills the other.
  19379. 1.22a.  Variant: A man goes to hang his coat and realises he will die that
  19380. day.  Answer: The man (who is black) has car trouble and is in need of a
  19381. telephone.  He asks at the nearest house and on being invited in goes to
  19382. hang his coat, whereupon he notices the white robes of the Ku Klux Klan in
  19383. the closet.  (from Bernd Wechner)
  19384.  
  19385. 1.23.  He is in a hotel, and is unable to sleep because the man in the
  19386. adjacent room is snoring.  He calls the room next door (from his own
  19387. room number he can easily figure out his neighbor's, and from the room
  19388. number, the telephone number).  The snorer wakes up, answers the phone.
  19389. The first man hangs up without saying anything and goes to sleep before
  19390. the snorer gets back to sleep and starts snoring again.
  19391. 1.23a.  Slightly variant answer: It's a next-door neighbor in an apartment
  19392. building who's snoring, rather than in a hotel.  The caller thus knows his
  19393. neighbor and the phone number.
  19394.  
  19395. 1.24.  It's the man's fiftieth birthday, and in celebration of this he
  19396. plans to kill his wife, then take the money he's embezzled and move on to
  19397. a new life in another state.  His wife takes him out to dinner; afterward,
  19398. on their front step, he kills her.  He opens the door, dragging her body
  19399. in with him, and all the lights suddenly turn on and a group of his
  19400. friends shout "Surprise]"  He kills himself.  (Note that the whole first
  19401. part, including the motive, isn't really necessary; it was just part of
  19402. the original story.)
  19403.  
  19404. 1.25.  Abel is a prince of the island nation that he landed on.  A cruel
  19405. and warlike prince, he waged many land and naval battles along with his
  19406. father the king.  In one naval encounter, their ship sank, the king died,
  19407. and the prince swam to a deserted island where he spent several months
  19408. building a raft or small boat.  In the meantime, a regent was appointed to
  19409. the island nation, and he brought peace and prosperity.  When Prince Abel
  19410. returned to his kingdom, Cain (a native fisherman) realized that the peace
  19411. of the land would only be maintained if Abel did not reascend to his
  19412. throne, and killed the prince (with a piece of driftwood or some other
  19413. impromptu weapon).
  19414.  
  19415. 1.26.  The drinks contain poisoned ice cubes; one man drinks slowly,
  19416. giving them time to melt, while the other drinks quickly and thus doesn't
  19417. get much of the poison.  The fact that they drink at different speeds
  19418. could be added to the statement, possibly along with red herrings such as
  19419. saying that one of the men is big and burly and the other short and thin.
  19420.  
  19421. 1.27.  Joe is a kid who goes trick-or-treating for Halloween.
  19422.  
  19423. 1.28.  He's a smuggler.  On the first cruise, someone brings the
  19424. contraband to his cabin, and he hides it in an air conditioning duct.
  19425. Returning to the U.S., he leaves without the contraband, and so passes
  19426. through customs with no trouble.  On the second trip, he has the same
  19427. cabin on the same ship.  Because it doesn't stop anywhere, he doesn't have
  19428. to go through customs when he returns, so he gets the contraband off
  19429. safely.
  19430.  
  19431. 1.29.  Hans and Fritz do everything right up until they're filling out a
  19432. personal-information form and have to write down their birthdays.  Fritz'
  19433. birthday is, say, July 7, so he writes down 7/7/15.  Hans, however, was
  19434. born on, say, June 20, so he writes down 20/6/18 instead of what an
  19435. American would write, 6/20/18.  Note that this is only a problem because
  19436. they *claim* to be returning Americans; as has been pointed out to me,
  19437. there are lots of other nations which use the same date ordering.
  19438.  
  19439. 1.30.  Another WWII story.  Greg is a German spy.  His "friend" Tim is
  19440. suspicious, so he plays a word-association game with him.  When Tim says
  19441. "The land of the free," Greg responds with "The home of the brave."  Then
  19442. Tim says "The terror of flight," and Greg says "The gloom of the grave."
  19443. Any U.S. citizen knows the first verse of the national anthem, but only a
  19444. spy would have memorized the third verse.  (Why Tim knew the third verse
  19445. is left as an exercise to the reader.)
  19446.  
  19447. 1.31.  The dead man was the driver in a hit-and-run acccident which
  19448. paralyzed its victim.  The victim did manage to get the license plate
  19449. number of the car; now in a wheelchair, he eventually tracked down the
  19450. driver and shot and killed him.
  19451.  
  19452. 1.32.  His home is a houseboat and he has run out of water while on an
  19453. extended cruise.
  19454. 1.32a.  Variant wording: A man dies of thirst in his own home.  This
  19455. version goes more quickly because it gives more information; but it may be
  19456. less likely to annoy people who think the original statement is too vague.
  19457.  
  19458. 1.33.  I'm told this is a true story.  Windows in Paris at that time were
  19459. apparently imperfectly flat; they could act as lenses.  One particularly
  19460. hot day, the sun shining in through such a window caused a woman's
  19461. lingerie (which she was wearing at the time, awaiting her husband's
  19462. return) to catch fire, and eventually the entire house caught and burned.
  19463.  
  19464. 1.34.  He's leaving a hospital after visiting his wife, who's on heavy
  19465. life-support.  When the power goes out, he knows she can't live without
  19466. the life-support systems (he assumes that if the emergency backup
  19467. generator were working, the elevator wouldn't lose power; this aspect
  19468. isn't entirely satisfactory, so in a variant, the scene is at home rather
  19469. than in a hospital).
  19470. 1.34a.  Variant: The music stops and a woman dies.  Answer: The woman is
  19471. confined in an iron lung, and the music is playing on her radio or stereo.
  19472. The power goes out.  (from Randy Whitaker)  (See also #1.15a, #1.16, and
  19473. #1.19e.)
  19474.  
  19475. 1.35.  A large man comes home to the penthouse apartment he shares with
  19476. his beautiful young wife, taking the elevator up from the ground floor.
  19477. He sees signs of lovemaking in the bedroom, and assumes that his wife is
  19478. having an affair; her beau has presumably escaped down the stairs.  The
  19479. husband looks out the French windows and sees a good-looking man just
  19480. leaving the main entrance of the building.  The husband pushes the
  19481. refrigerator out through the window onto the young man below.  The husband
  19482. dies of a heart attack from overexertion; the young man below dies from
  19483. having a refrigerator fall on him; and the wife's boyfriend, who was
  19484. hiding inside the refrigerator, also dies from the fall.
  19485.  
  19486. 1.36.  Let's say "she" is named Suzy, and "they" are named Harry and Jane.
  19487. Harry is an elderly archaeologist who has found a very old skeleton, which
  19488. he's dubbed "Jane" (a la "Lucy").  Suzy is a buyer for a museum; she's
  19489. supposed to make some sort of purchase from Harry, so she invites him to
  19490. have a business dinner with her (at a restaurant).  When she calls to
  19491. invite him, he keeps talking about "Jane," so Suzy assumes that Jane is
  19492. his wife and says to bring her along.  Harry, offended, calls Suzy's boss
  19493. and complains; since Suzy should've known who Jane was, she gets fired.
  19494.  
  19495. 1.37.  The man is delivering a pardon, and the flicker of the lights
  19496. indicates that the person to be pardoned has just been electrocuted.
  19497.  
  19498. 1.38.  The murderer sets the car on a slope above the hot dog stand where
  19499. the victim works.  He then wedges an ice block in the car to keep the
  19500. brake pedal down, and puts the car in neutral, after which he flies to
  19501. another city to avoid suspicion.  It's a warm day; when the ice melts, the
  19502. car rolls down the hill and strikes the hot dog man at his roadside stand,
  19503. killing him.
  19504.  
  19505. 1.39.  There's a car wash on that corner.  On rainy days, the rain reduces
  19506. traction.  On sunny days, water from the car wash has the same effect.  If
  19507. rain is threatening, though, the car wash gets little business and thus
  19508. doesn't make the road wet, so I can take the corner faster.
  19509.  
  19510. 1.40.  The object she throws is a boomerang.  It flies out, loops around,
  19511. and comes back and hits her in the head, killing her.  Boomerangs do not
  19512. often return so close to the point from which they were thrown, but I
  19513. believe it's possible for this to happen.
  19514. 1.40a.  Silly variant answer: She's in a submarine or spacecraft and
  19515. throws a heavy object at the window, which breaks.
  19516.  
  19517. 1.41.  He is a passenger in an airplane and sees the bird get sucked into
  19518. an engine at 20,000 feet.
  19519.  
  19520. 1.42.  They're the remains of a melted snowman.
  19521.  
  19522. 1.43.  One of the brothers (A) confesses to the murder.  At his trial, his
  19523. brother (B) is called as the only defense witness; B immediately
  19524. confesses, in graphic detail, to having committed the crime.  The defense
  19525. lawyer refuses to have the trial stopped, and A is acquitted under the
  19526. "reasonable doubt" clause.  Immediately afterward, B goes on trial for the
  19527. murder; A is called as the only defense witness and HE confesses.  B is
  19528. declared innocent; and though everyone knows that ONE of them did it, how
  19529. can they tell who?  Further, neither can be convicted of perjury until
  19530. it's decided which of them did it...  I don't know if that would actually
  19531. work under our legal system, but someone else who heard the story said
  19532. that his father was on the jury for a VERY similar case in New York some
  19533. years ago.  Mark Brader points out that the brothers might be convicted of
  19534. conspiracy to commit perjury or to obstruct justice, or something of that
  19535. kind.
  19536.  
  19537. 1.44.  He is a mail courier who delivers packages to the different foreign
  19538. embassies in the United States.  The land of an embassy belongs to the
  19539. country of the embassy, not to the United States.
  19540.  
  19541. 1.45.  A man was shot during a robbery in his store one night.  He
  19542. staggered into the back room, where the telephone was, and called home,
  19543. dialing by feel since he hadn't turned on the light.  Once the call went
  19544. through he gasped, "I'm at the store.  I've been shot.  Help]" or words to
  19545. that effect.  He set the phone down to await help, but none came; he'd
  19546. treated the telephone pushbuttons like cash register numbers, when the
  19547. arrangements of the numbers are upside down reflections of each other.
  19548. The stranger he'd dialed had no way to know where "the store" was.
  19549.  
  19550. 1.46.  The dead man was playing Santa Claus, for whatever reason; he
  19551. slipped while coming down the chimney and broke his neck.
  19552. 1.46a.  Variant answer: The dead man WAS Santa Claus.  This moves the
  19553. puzzle to section 2.
  19554.  
  19555. 1.47.  The man was struck by an object thrown from the roof of the Empire
  19556. State Building.  Originally I had the object being a penny, but several
  19557. people suggested that a penny probably wouldn't be enough to penetrate
  19558. someone's skull.  Something aerodynamic and heavier, like a dart, was
  19559. suggested, but I don't know how much mass would be required.
  19560. 1.47a.  Variant: A man is found dead outside a large marble building with
  19561. three holes in him.  Answer: The man was a paleontologist working with the
  19562. Archaeological Research Institute.  He was reviving a triceratops frozen
  19563. in the ice age when it came to life and killed him.  This couldn't
  19564. possibly happen because triceratops didn't exist during the ice age.
  19565. (from Peter R. Olpe)
  19566.  
  19567. 1.48.  The man died from eating a poisoned popsicle.
  19568.  
  19569. 1.49.  The man was a sword swallower in a carnival side-show.  While he
  19570. was practicing, someone tickled his throat with the feather, causing him
  19571. to gag.
  19572.  
  19573. 1.50.  A mosquito bit me, and I swatted it when it later landed on my
  19574. ceiling (so the blood is my own as well as the mosquito's).
  19575.  
  19576. 1.51.  The man is a lighthouse keeper.  He turns off the light in the
  19577. lighthouse and during the night a ship crashes on the rocks.  Seeing this
  19578. the next morning, the man realizes what he's done and commits suicide.
  19579. 1.51a.  Variant, similar to #1.15: The light goes out and a man dies.
  19580. Answer: The lighthouse keeper uses his job as an alibi while he's
  19581. elsewhere committing a crime, but the light goes out and a ship crashes,
  19582. thereby disproving the alibi.  The lighthouse keeper kills himself when he
  19583. realizes his alibi is no good. (From Eric Wang)
  19584. 1.51b.  Variant answer to 1.51a: Someone else's alibi is disproven.  (A
  19585. man commits a heinous crime, claiming as his alibi that he was onboard a
  19586. certain ship.  When he learns that it was wrecked without reaching port
  19587. safely, he realizes that his alibi is disproven and commits suicide to
  19588. avoid being sent to prison.)  (From Eric Wang)
  19589.  
  19590. 1.52.  They were skydiving.  He broke his arm as he jumped from the plane
  19591. by hitting it on the plane door; he couldn't reach his ripcord with his
  19592. other arm.  She pulled the ripcord for him.
  19593. 1.52a.  Sketch of variant answer: The ring was attached to the pin of a
  19594. grenade that he was holding.  Develop a situation from there.
  19595.  
  19596. 1.53.  The man is a travel agent.  He had sold someone two tickets for an
  19597. ocean voyage, one round-trip and one one-way.  The last name of the man
  19598. who bought the tickets is the same as the last name of the woman who
  19599. "fell" overboard and drowned on the same voyage, which is the subject of
  19600. the article he's reading.
  19601.  
  19602. 1.54.  The man is a beekeeper, and the bees attack en masse because they
  19603. don't recognize his fragrance.  Randy adds that this is based on something
  19604. that actually happened to his grandfather, a beekeeper who was severely
  19605. attacked by his bees when he used a new aftershave for the first time in 10
  19606. or 20 years.
  19607.  
  19608. 1.55.  He is a guard / attendant in a leper colony.  The letter (to him)
  19609. tells him that he has contracted the disease.  The key is the cigarette
  19610. burning down between his fingers -- leprosy is fairly unique in killing off
  19611. sensory nerves without destroying motor ability.  (Randy was told this by
  19612. Gary Haas and Chris Englehard)
  19613.  
  19614. 1.56.  The man was a famous artist.  A woman who collected autographs saw
  19615. him dining; after he left the restaurant, she purchased the check that he
  19616. used to pay for the meal from the restaurant manager.  The check was
  19617. therefore never cashed, so the artist never paid for the meal.
  19618.  
  19619. 1.57.  The movie is at a drive-in theatre.
  19620.  
  19621.  
  19622. Section 2: Double meanings, fictional settings, and miscellaneous others.
  19623.  
  19624. 2.1.  The man is a heroin addict, and has contracted AIDS by using an
  19625. infected needle.  In despair, he shoots himself up with an overdose,
  19626. thereby committing suicide.
  19627.  
  19628. 2.2.  The man walks into a casino and goes to the craps table.  He bets
  19629. all the money he owns, and shoots craps.  Since he is now broke, he
  19630. becomes despondent and commits suicide.
  19631.  
  19632. 2.3.  Kids getting their pictures taken with Santa.  I see #2.1, #2.2, and
  19633. #2.3 as different enough from each other to merit separate numbers,
  19634. although they all rely on the same basic gimmick of alternate meanings of
  19635. the word "shoot."
  19636.  
  19637. 2.4.  It's the cabin of an airplane that crashed there because of the
  19638. snowstorm.
  19639. 2.4a.  Variant wording: A cabin, on the side of a mountain, locked from
  19640. the inside, is opened, and 30 people are found dead inside.  They had
  19641. plenty of food and water.  (from Ron Carter)
  19642.  
  19643. 2.5.  He's a priest; he is marrying them to other people, not to himself.
  19644.  
  19645. 2.6.  The "island" is a traffic island.
  19646.  
  19647. 2.7.  A baseball game is going on.  The base-runner sees the catcher
  19648. waiting at home plate with the ball, and so decides to stay at third base
  19649. to avoid being tagged out.
  19650. 2.7a.  Variant: Two men are in a field.  One is wearing a mask.  The other
  19651. man is running towards him to avoid him.  Answer: the same, but the
  19652. catcher isn't right at home plate; the runner is trying to get home before
  19653. the catcher can.  (from Hal Lowery, by way of Chris Riley)  This phrasing
  19654. would allow the puzzle to migrate to section 1, but I don't like it as
  19655. much.
  19656.  
  19657. 2.8.  The man is an astronaut out on a space walk.
  19658.  
  19659. 2.9.  The man was an amateur mechanic, the book is a Volkswagen service
  19660. manual, the beetle is a car, and the pile of bricks is what the car fell
  19661. off of.
  19662.  
  19663. 2.10.  The Eagle landed in the Sea of Tranquility and will likely remain
  19664. there for the foreseeable future.
  19665.  
  19666. 2.11.  It's a wolf pack; they've killed and eaten (most of) the man.
  19667.  
  19668. 2.12.  The dead man is Superman; the rock is Green Kryptonite.  Invent a
  19669. reasonable scenario from there.
  19670.  
  19671. 2.13.  This is a post-holocaust scenario of some kind; for whatever
  19672. reason, the man believes himself to be the last human on earth.  He
  19673. doesn't want to live by himself, so he jumps, just before the telephone
  19674. rings...  (of course, it could be a computer calling, but he has no way of
  19675. knowing).
  19676.  
  19677. 2.14.  The one who looks around sees his own reflection in the window
  19678. (it's dark outside), but not his companion's.  Thus, he realizes the other
  19679. is a vampire, and that he's going to be killed by him.
  19680.  
  19681. 2.15.  The "bicycles" are Bicycle playing cards; the man was cheating at
  19682. cards, and when the extra card was found, he was killed by the other
  19683. players.
  19684. 2.15a.  Variant: There are 53 bees instead of 53 bicycles.  Answer: The
  19685. same (Bee is another brand of playing cards).
  19686. 2.15b.  Variant: There are 51 instead of 53.  Answer: Someone saw the guy
  19687. conceal a card, and proved the deck was defective by turning it up and
  19688. pointing out the missing ace.  Or, the game was bridge, and the others
  19689. noticed the cheating when the deal didn't come out even.  The man had
  19690. palmed an ace during the shuffle and meant to put it in his own hand
  19691. during the deal, but muffed it.  (both answers from Mark Brader)
  19692.  
  19693. 2.16.  A chess game; knight takes pawn.
  19694. 2.16a.  Variant: It's the year 860 A.D., at Camelot.  Two priests are
  19695. sitting in the castle's chapel.  The queen attacks the king.  The two
  19696. priests rise, shake hands, and leave the room.  Answer: The two priests
  19697. are playing chess; one of them just mated by moving his queen.  (from
  19698. Ellen M. Sentovich)
  19699. 2.16b.  Variant: A black leader dies in Africa.  Answer: The black leader
  19700. is a chess king, and the game was played in Africa.  (from Erick
  19701. Brethenoux)
  19702.  
  19703. 2.17.  It's a model train set.
  19704. 2.17a.  Variant: The Orient Express is derailed and a kitten plays nearby.
  19705. Answer: The Orient Express is a model train which has been left running
  19706. unattended.  The kitten has playfully derailed it.  (from Bernd Wechner)
  19707.  
  19708. 2.18.  It's a game of Monopoly.
  19709.  
  19710. 2.19.  The sun is shining; there's no rain.
  19711.  
  19712. 2.20.  It's daytime; the sun is out.
  19713.  
  19714. 2.21.  Alice is a goldfish; Ted is a cat.
  19715. 2.21a.  A very common variant uses the names Romeo and Juliet instead, to
  19716. further mislead audiences.  For example: Romeo is looking down on Juliet's
  19717. dead body, which is on the floor surrounded by water and broken glass.
  19718. (from Adam Carlson)
  19719. 2.21b.  Minor variant: Tom and Jean lay dead in a puddle of water with
  19720. broken pieces of glass and a baseball nearby.  Answer: Tom and Jean are both
  19721. fish; it was a baseball, rather than a cat, that broke their tank.  (from
  19722. Mike Reymond)
  19723.  
  19724. 2.22.  Friday is a horse.
  19725. 2.22a.  Variant with the same basic gimmick: A woman comes home, sees
  19726. Spaghetti on the wall and kills her husband.  Answer: Spaghetti was the
  19727. name of her pet dog.  Her husband had it stuffed and mounted after it made
  19728. a mess on his rug.  (Simon Travaglia original)
  19729.  
  19730. 2.23.  Bruce is a horse.
  19731.  
  19732. 2.24.  Should be done orally; the envelope is an evelope of dye, and she's
  19733. dying some cloth, but it sounds like "opens an envelope and dies" if said
  19734. out loud.
  19735.  
  19736. 2.25.  The native chief asked him, "What is the third baseman's name in
  19737. the Abbot and Costello routine 'Who's on First'?"  The man, who had no
  19738. idea, said "I don't know," the correct answer.  However, he was a major
  19739. smartass, so if he had known the answer he would have pointed out that
  19740. What was the SECOND baseman's name.  The chief, being quite humorless,
  19741. would have executed him on the spot.  This is fairly silly, but I like it
  19742. too much to remove it from the list.
  19743.  
  19744. 2.26.  The men have gone spelunking and have taken an Igloo cooler with
  19745. them so they can have a picnic down in the caves.  They cleverly used dry
  19746. ice to keep their beer cold, not realizing that as the dry ice sublimed
  19747. (went from solid state to vapor state) it would push the lighter oxygen
  19748. out of the cave and they would suffocate.
  19749.  
  19750. ==> logic/smullyan/black.hat.p <==
  19751. Three logicians, A, B, and C, are wearing hats, which they know are either
  19752. black or white but not all white. A can see the hats of B and C; B can see
  19753. the hats of A and C; C is blind.  Each is asked in turn if they know the color
  19754. of their own hat.  The answers are:
  19755.         A: "No."
  19756.         B: "No."
  19757.         C: "Yes."
  19758. What color is C's hat and how does she know?
  19759.  
  19760. ==> logic/smullyan/black.hat.s <==
  19761. A must see at least one black hat, or she would know that her hat is black
  19762. since they are not all white.  B also must see at least one black hat, and
  19763. further, that hat had to be on C, otherwise she would know that her
  19764. hat was black (since she knows A saw at least one black hat).  So C knows
  19765. that her hat is black, without even seeing the others' hats.
  19766.  
  19767. ==> logic/smullyan/fork.three.men.p <==
  19768. Three men stand at a fork in the road.  One fork leads to Someplaceorother;
  19769. the other fork leads to Nowheresville.  One of these people always answers
  19770. the truth to any yes/no question which is asked of him.  The other always
  19771. lies when asked any yes/no question.  The third person randomly lies and
  19772. tells the truth.  Each man is known to the others, but not to you.
  19773. What is the least number of yes/no questions you can ask of these men and
  19774. pick the road to Someplaceorother?
  19775.  
  19776. ==> logic/smullyan/fork.three.men.s <==
  19777. It is clear that you must ask at least two questions, since you might be
  19778. asking the first one of the randomizer and there is nothing you can tell
  19779. from his answers.
  19780.  
  19781. Start by asking A "Is B more likely to tell the truth than C?"
  19782.  
  19783. If he answers "yes", then:
  19784.    If A is truthteller, B is randomizer, C is liar.
  19785.    If A is liar, B is randomizer, C is truthteller.
  19786.    If A is randomizer, C is truthteller or liar.
  19787.  
  19788. If he answers "no", then:
  19789.    If A is truthteller, B is liar, C is randomizer.
  19790.    If A is liar, B is truthteller, C is randomizer.
  19791.    If A is randomizer, B is truthteller or liar.
  19792.  
  19793. In either case, we now know somebody (C or B, respectively) who is either
  19794. a truthteller or liar.  Now, use the technique for finding information from
  19795. a truthteller/liar, viz.:
  19796.  
  19797. You ask him the following question: "If I were to ask a person of the opposite
  19798. type to yourself if the left fork leads to Someplacerother, would he say yes?"
  19799.  
  19800. If the person asked is a truthteller, he will tell you what a liar would
  19801. say, which would be the wrong information.  If the person asked is a liar,
  19802. he will either tell you what a liar would say, or he will lie about what a
  19803. truthteller would say.  In either case, he will report the wrong information.
  19804. If the answer is yes, take the right fork, if no take the left fork.
  19805.  
  19806. ==> logic/smullyan/fork.two.men.p <==
  19807. Two men stand at a fork in the road.  One fork leads to Someplaceorother; the
  19808. other fork leads to Nowheresville.  One of these people always answers the
  19809. truth to any yes/no question which is asked of him.  The other always lies
  19810. when asked any yes/no question.  By asking one yes/no question, can you
  19811. determine the road to Someplaceorother?
  19812.  
  19813. ==> logic/smullyan/fork.two.men.s <==
  19814. The question to ask is: "Will the other person say the right fork leads to
  19815. Someplaceorother?"  If the person asked says yes, then take the left fork,
  19816. else take the right fork.
  19817.  
  19818. If the person asked is the truthteller, then he correctly reports that the
  19819. liar will misinform you about the right fork.  If he is the liar, then he
  19820. lies about what the truthteller will say.  Either way, you should go the
  19821. opposite direction from the way that the person asked says the other person
  19822. will answer.
  19823.  
  19824. The fact that there are two is a red herring - you only need one of
  19825. either type.  You ask him the following question: "If I were to ask a
  19826. person of the opposite type to yourself if the left fork leads to
  19827. Someplacerother, would he say yes?"
  19828.  
  19829. If the person asked is a truthteller, he will tell you what a liar would
  19830. say, which would be the wrong information.  If the person asked is a liar,
  19831. he will either tell you what a liar would say, or he will lie about what a
  19832. truthteller would say.  In either case, he will report the wrong information.
  19833. If the answer is yes, take the right fork, if no take the left fork.
  19834.  
  19835. This solution also removes the problem that the men may not know the
  19836. other's identity.
  19837.  
  19838. It is possible, of course, that the liars are malicious, and they will tell
  19839. the truth if they figure out that you are trying to trick them.
  19840.  
  19841.  
  19842. ==> logic/smullyan/integers.p <==
  19843. Two logicians place cards on their foreheads so that what is written on the
  19844. card is visible only to the other logician.  Consecutive positive integers
  19845. have been written on the cards.  The following conversation ensues:
  19846.     A: "I don't know my number."
  19847.     B: "I don't know my number."
  19848.     A: "I don't know my number."
  19849.     B: "I don't know my number."
  19850.     ... n statements of ignorance later ...
  19851.     A or B: "I know my number."
  19852. What is on the card and how does the logician know it?
  19853.  
  19854. ==> logic/smullyan/integers.s <==
  19855. If A saw 1, she would know that she had 2, and would say so.  Therefore,
  19856. A did not see 1.  A says "I don't know my number."
  19857. If B saw 2, she would know that she had 3, since she knows that A did not see
  19858. 1, so B did not see 1 or 2.  B says "I don't know my number."
  19859. If A saw 3, she would know that she had 4, since she knows that B did not
  19860. see 1 or 2, so A did not see 1, 2 or 3.  A says "I don't know my number."
  19861. If B saw 4, she would know that she had 5, since she knows that A did not
  19862. see 1, 2 or 3, so B did not see 1, 2, 3 or 4.  B says "I don't know my number."
  19863. ... n statements of ignorance later ...
  19864. If X saw n, she would know that she had n + 1, since she knows that ~X did not
  19865. see 1 ... n - 1, so X did see n.  X says "I know my number."
  19866.  
  19867. And the number in n + 1.
  19868.  
  19869. ==> logic/smullyan/liars.et.al.p <==
  19870. Of a group of n men, some always lie, some never lie, and the rest sometimes
  19871. lie.  They each know which is which.  You must determine the identity of each
  19872. man by asking the least number of yes-or-no questions.
  19873.  
  19874. ==> logic/smullyan/liars.et.al.s <==
  19875. The real problem is to isolate the sometimes liars.
  19876.  
  19877. Consider the case of three men:
  19878. Ask man 1: "Does man 2 lie more than 3?"
  19879. If the answer is yes, then man 2 cannot be the sometimes liar.
  19880. Proof by analyzing the cases:
  19881. Case 1: Man 2 is not the sometimes liar.
  19882. Case 2: Man 2 is the sometimes liar, man 1 is the truth teller, and man 3 is
  19883.         the liar.  Then man 1 would not say that man 2 lies more than man 3.
  19884. Case 3: Man 2 is the sometimes liar, man 3 is the truth teller, and man 1 is
  19885.         the liar.  Then man 1 would not say that man 2 lies more than man 3.
  19886. QED.
  19887. Similarly, if the answer is no, then man 3 cannot be the sometimes liar.
  19888. Now ask the symmetric question of whichever man has been eliminated as the
  19889. sometimes liar.  The answer will now allow you to determine the identity
  19890. of the sometimes liar.  To determine the identity of the two remaining men, ask
  19891. some question like "Does 1=1?" which is always true.
  19892.  
  19893. This is not the only way to solve this problem.  You could have asked the
  19894. question which is always true (or false) second, which would now establish
  19895. the identity of either the liar or the truth teller.  Then ask the third
  19896. question of this man to find out which of the other two is the sometimes
  19897. liar.
  19898.  
  19899. This problem requires three questions, whether or not they are yes-or-no
  19900. questions.  In order to identify all three men, you must identify the
  19901. sometimes liar.  You cannot identify the sometimes liar in one question
  19902. since you may be asking it of the sometimes liar, and any answer from him
  19903. conveys no information at all.  Therefore at least two questions are
  19904. necessary to identify the sometimes liar.  Once the sometimes liar is
  19905. identified, you still need one more question at least to identify the
  19906. remaining men.  Therefore, three questions are required.
  19907.  
  19908. Suppose we have two truth-tellers, two liars, and two randomizers.
  19909. The answer is 8.  A proof follows.
  19910.  
  19911. For brevity, "T" means truth-teller, "L" liar, "R" randomizer, "P" predictable
  19912. (either T or L).  Define a _pattern_ to be one of the C(6,2)=15 permutations
  19913. of RRPPPP (each of which has C(4,2)=6 interpretations of the Ps as 2 Ts and 2
  19914. Ls).  For any question Q, let ]Q denote the question "If I were to ask you Q,
  19915. would you answer Yes?".  Note that question ]Q directed toward any P will
  19916. yield a truthful answer to question Q; in other words, a "Yes" answer to ]Q
  19917. means that either Q is true or the respondent is an R, whereas "No" means that
  19918. either Q is false or the respondent is an R.
  19919.  
  19920. Ask #1, ]"Are both Rs in the set {#2, #3, #4}?".  "No" implies that at most
  19921. one of {#2, #3, #4} is an R.  "Yes" implies that at most one of {#2, #5, #6}
  19922. is an R.  Without loss of generality, assume the former.
  19923.  
  19924. Ask #2, ]"Is #3 an R?".  "No" implies that #3 is a P.  "Yes" implies that #4
  19925. is a P.
  19926.  
  19927. Having identified someone as a P, there are at most C(5,2)=10 possible
  19928. patterns, and hence at most 10*6=60 possible results.  We can determine which
  19929. one reflects reality with at most 6 more questions with a binary search.  (At
  19930. each step, bisect the set of possible answers, and ask the question ]"Is the
  19931. correct pattern in the first subset?".)
  19932.  
  19933. Now, let's show that it can't be done in 7.
  19934.  
  19935. After asking your first two questions, renumber if necessary so that the first
  19936. question was directed to #1 and the second to #2.  (If you asked the same
  19937. person twice, you're even worse off than in the analysis below.)  You have no
  19938. way to rule out the possibility that both are Rs, so pattern RRPPPP yields 6
  19939. possibilities.  Of the four patterns RPRPPP RPPRPP RPPPRP RPPPPR, your first
  19940. question gave no information and the second had one bit; so at best you can
  19941. eliminate half of these 4*6 possibilities, leaving 12.  Similarly for the four
  19942. patterns PRRPPP PRPRPP PRPPRP PRPPPR there remain at least 12 possibilities.
  19943. Of the remaining 6 patterns PPRRPP PPRPRP PPRPPR PPPRRP PPPRPR PPPPRR, your
  19944. two bits of information can eliminate 3/4 of the 6*6, leaving 9.  Thus, after
  19945. two questions there are at least 6+12+12+9=39 arrangements that could have
  19946. given the answers you heard; your five remaining questions have only 32
  19947. possible replies, so you can't distinguish them.
  19948.  
  19949. ==> logic/smullyan/painted.heads.p <==
  19950. While three logicians were sleeping under a tree, a malicious child painted
  19951. their heads red.  Upon waking, each logician spies the child's handiwork as
  19952. it applied to the heads of the other two.  Naturally they start laughing.
  19953. Suddenly one falls silent.  Why?
  19954.  
  19955. ==> logic/smullyan/painted.heads.s <==
  19956. The one who fell silent, presumably the quickest of the three, reasoned
  19957. that his head must be painted also.  The argument goes as follows.
  19958. Let's call the quick one Q, and the other two D and S.  Let's assume
  19959. Q's head is untouched.  Then D is laughing because S's head is painted,
  19960. and vice versa.  But eventually, D and S will realize that their head
  19961. must be painted, because the other is laughing.  So they will quit
  19962. laughing as soon as they realize this.  So, Q waits what he thinks is
  19963. a reasonable amount of time for them to figure this out, and when they
  19964. don't stop laughing, his worst fears are confirmed.  He concludes that
  19965. his assumption is invalid and he must be crowned in crimson too.
  19966.  
  19967.  
  19968. ==> logic/smullyan/priest.p <==
  19969. A priest takes confession of all the inhabitants in a small town.  He
  19970. discovers that in N married pairs in the town, one of the pair has
  19971. committed adultery.  Assume that the spouse of each adulterer does not
  19972. know about the infidelity of his or her spouse, but that, since it is
  19973. a small town, everyone knows about everyone else's infidelity.  In
  19974. other words, each spouse of an adulterer thinks there are N - 1
  19975. adulterers, but everyone else thinks there are N adulterers.  The
  19976. priest, who is an Old Testament type, decides that he should do
  19977. something about the situation.  He cannot break the confessional, but
  19978. being an amateur logician of sorts, he hits upon a plan to do God's
  19979. work.  He announces in Mass one Sunday that the spouse of each
  19980. adulterer has the moral obligation to punish his or her adulterous
  19981. spouse by publicly denouncing them in church, and that he will make
  19982. time during his next Sunday service for this, and continue to do so
  19983. until all adulterers have been denounced.  Is the priest correct?  Will
  19984. this result in every adulterer being denounced?
  19985.  
  19986. ==> logic/smullyan/priest.s <==
  19987. Yes.  Let's start with the simple case that N = 1.  The offended spouse
  19988. reasons as follows: the priest knows there is at least one adulterer,
  19989. but I don't know who this person is, and I would if it were anyone
  19990. other than me, so it must be me.  What happens if N = 2?  On the first
  19991. Sunday, the two offended spouses each calmly wait for the other to get
  19992. up and condemn their spouses.  When the other doesn't stand, they
  19993. think:  They do not think that they are a victim.  But if they do not
  19994. think they are victims, then they must think there are no adulterers,
  19995. contrary to what the priest said.  But everyone knows the priest speaks
  19996. with the authority of God, so it is unthinkable that he is mistaken.
  19997. The only remaining possibility is that they think there WAS another
  19998. adulterer, and the only possibility is: MY SPOUSE]  So, they know that
  19999. they too must be victims.  So on the next Sunday, they will get up.
  20000. What if N = 3?  On the first Sunday, each victim waits for the other
  20001. two to get up.  When they do not, they assume that they did not get up
  20002. because they did not know about the other person (in other words, they
  20003. hypothesize that each of the two other victims thought there was only
  20004. one adulterer).  However, each victim reasons, the two will now realize
  20005. that they must be two victims, for the reasons given under the N = 2
  20006. case above.  So they will get up next Sunday.  This excuse lasts until
  20007. the next Sunday, when still no one gets up, and now each victim
  20008. realizes that either the priest was mistaken (unthinkable]) or there
  20009. are really three victims, and I am ONE]  So, on the third Sunday, all
  20010. three get up.  This reasoning can be repeated inductively to show that
  20011. no one will do anything (except use up N - 1 excuses as to why no one
  20012. got up) until the Nth Sunday, when all N victims will arise in unison.
  20013.  
  20014. By the way, the rest of the town, which thinks there are N adulterers,
  20015. is about to conclude that their perfectly innocent spouses have been
  20016. unfaithful too.  This includes the adulterous spouses, who are about to
  20017. conclude that the door swings both ways.  So the priest is playing a
  20018. dangerous game.  A movie plot in there somewhere?
  20019.  
  20020. ==> logic/smullyan/stamps.p <==
  20021. The moderator takes a set of 8 stamps, 4 red and 4 green, known to the
  20022. logicians, and loosely affixes two to the forehead of each logician so that
  20023. each logician can see all the other stamps except those 2 in the moderator's
  20024. pocket and the two on her own head.  He asks them in turn
  20025. if they know the colors of their own stamps:
  20026.   A: "No"
  20027.   B: "No"
  20028.   C: "No"
  20029.   A: "No
  20030.   B: "Yes"
  20031.   What are the colors of her stamps, and what is the situation?
  20032.  
  20033. ==> logic/smullyan/stamps.s <==
  20034. B says: "Suppose I have red-red. A would have said on her
  20035. second turn: 'I see that B has red-red. If I also have red-red, then all
  20036. four reds would be used, and C would have realized that she had green-green.
  20037. But C didn't, so I don't have red-red.  Suppose I have green-green. In that
  20038. case, C would have realized that if she had red-red, I would have seen
  20039. four reds and I would have answered that I had green-green on my first
  20040. turn.  On the other hand, if she also has green-green ▌we assume that
  20041. A can see C; this line is only for completeness¿, then B would have seen
  20042. four greens and she would have answered that she had two reds.  So C would
  20043. have realized that, if I have green-green and B has red-red, and if
  20044. neither of us answered on our first turn, then she must have green-red.
  20045.   "'But she didn't. So I can't have green-green either, and if I can't have
  20046. green-green or red-red, then I must have green-red.'
  20047.   So B continues: "But she (A) didn't say that she had green-red, so
  20048. the supposition that I have red-red must be wrong.  And as my logic applies
  20049. to green-green as well, then I must have green-red."
  20050.   So B had green-red, and we don't know the distribution of the others
  20051. certainly.
  20052.   (Actually, it is possible to take the last step first, and deduce
  20053. that the person who answered YES must have a solution which would work
  20054. if the greens and reds were switched -- red-green.)
  20055.  
  20056. ==> logic/timezone.p <==
  20057. Two people are talking long distance on the phone; one is in an East-
  20058. Coast state, the other is in a West-Coast state.  The first asks the other
  20059. "What time is it?", hears the answer, and says, "That's funny.  It's the
  20060. same time here]"
  20061.  
  20062. ==> logic/timezone.s <==
  20063. One is in Eastern Oregon (in Mountain time), the other in
  20064. Western Florida (in Central time), and it's daylight-savings
  20065. changeover day at 1:30 AM.
  20066.  
  20067. ==> logic/unexpected.p <==
  20068. Swedish civil defense authorities announced that a civil defense drill would
  20069. be held one day the following week, but the actual day would be a surprise.
  20070. However, we can prove by induction that the drill cannot be held.  Clearly,
  20071. they cannot wait until Friday, since everyone will know it will be held that
  20072. day.  But if it cannot be held on Friday, then by induction it cannot be held
  20073. on Thursday, Wednesday, or indeed on any day.
  20074.  
  20075. What is wrong with this proof?
  20076.  
  20077. ==> logic/unexpected.s <==
  20078. This problem has generated a vast literature (see below).  Several
  20079. solutions of the paradox have been proposed, but as with most paradoxes
  20080. there is no consensus on which solution is the "right" one.
  20081.  
  20082. The earliest writers (O'Connor, Cohen, Alexander) see the announcement as
  20083. simply a statement whose utterance refutes itself.  If I tell you that I
  20084. will have a surprise birthday party for you and then tell you all the
  20085. details, including the exact time and place, then I destroy the surprise,
  20086. refuting my statement that the birthday will be a surprise.
  20087.  
  20088. Soon, however, it was noticed that the drill could occur (say on Wednesday),
  20089. and still be a surprise.  Thus the announcement is vindicated instead of
  20090. being refuted.  So a puzzle remains.
  20091.  
  20092. One school of thought (Scriven, Shaw, Medlin, Fitch, Windt) interprets
  20093. the announcement that the drill is unexpected as saying that the date
  20094. of the drill cannot be deduced in advanced.  This begs the question,
  20095. deduced from which premises?  Examination of the inductive argument
  20096. shows that one of the premises used is the announcement itself, and in
  20097. particular the fact that the drill is unexpected.  Thus the word
  20098. "unexpected" is defined circularly.  Shaw and Medlin claim that this
  20099. circularity is illegitimate and is the source of the paradox.  Fitch
  20100. uses Godelian techniques to produce a fully rigorous self-referential
  20101. announcement, and shows that the resulting proposition is
  20102. self-contradictory.  However, none of these authors explain how it can
  20103. be that this illegitimate or self-contradictory announcement
  20104. nevertheless appears to be vindicated when the drill occurs.  In other
  20105. words, what they have shown is that under one interpretation of "surprise"
  20106. the announcement is faulty, but their interpretation does not capture the
  20107. intuition that the drill really is a surprise when it occurs and thus
  20108. they are open to the charge that they have not captured the essence of
  20109. the paradox.
  20110.  
  20111. Another school of thought (Quine, Kaplan and Montague, Binkley,
  20112. Harrison, Wright and Sudbury, McClelland, Chihara, Sorenson) interprets
  20113. "surprise" in terms of "knowing" instead of "deducing."  Quine claims
  20114. that the victims of the drill cannot assert that on the eve of the last
  20115. day they will "know" that the drill will occur on the next day.  This
  20116. blocks the inductive argument from the start, but Quine is not very
  20117. explicit in showing what exactly is wrong with our strong intuition
  20118. that everybody will "know" on the eve of the last day that the drill
  20119. will occur on the following day.  Later writers formalize the paradox
  20120. using modal logic (a logic that attempts to represent propositions
  20121. about knowing and believing) and suggest that various axioms about
  20122. knowing are at fault, e.g., the axiom that if one knows something, then
  20123. one knows that one knows it (the "KK axiom").  Sorenson, however,
  20124. formulates three ingenious variations of the paradox that are
  20125. independent of these doubtful axioms, and suggests instead that the
  20126. problem is that the announcement involves a "blindspot":  a statement
  20127. that is true but which cannot be known by certain individuals even if
  20128. they are presented with the statement.  This idea was foreshadowed by
  20129. O'Beirne and Binkley.  Unfortunately, a full discussion of how this
  20130. blocks the paradox is beyond the scope of this summary.
  20131.  
  20132. Finally, there are two other approaches that deserve mention.  Cargile
  20133. interprets the paradox as a game between ideally rational agents and finds
  20134. fault with the notion that ideally rational agents will arrive at the same
  20135. conclusion independently of the situation they find themselves in.  Olin
  20136. interprets the paradox as an issue about justified belief: on the eve of
  20137. the last day one cannot be justified in believing BOTH that the drill will
  20138. occur on the next day AND that the drill will be a surprise even if both
  20139. statements turn out to be true; hence the argument cannot proceed and the
  20140. drill can be a surprise even on the last day.
  20141.  
  20142. For those who wish to read some of the literature, good papers to start with
  20143. are Bennett-Cargile and both papers of Sorenson.  All of these provide
  20144. overviews of previous work and point out some errors, and so it's helpful to
  20145. read them before reading the original papers.  For further reading on the
  20146. "deducibility" side, Shaw, Medlin and Fitch are good representatives.  Other
  20147. papers that are definitely worth reading are Quine, Binkley, and Olin.
  20148.  
  20149. D. O'Connor, "Pragmatic Paradoxes," Mind 57:358-9, 1948.
  20150. L. Cohen, "Mr. O'Connor's 'Pragmatic Paradoxes,'" Mind 59:85-7, 1950.
  20151. P. Alexander, "Pragmatic Paradoxes," Mind 59:536-8, 1950.
  20152. M. Scriven, "Paradoxical Announcements," Mind 60:403-7, 1951.
  20153. D. O'Connor, "Pragmatic Paradoxes and Fugitive Propositions," Mind 60:536-8,
  20154.  1951
  20155. P. Weiss, "The Prediction Paradox," Mind 61:265ff, 1952.
  20156. W. Quine, "On A So-Called Paradox," Mind 62:65-7, 1953.
  20157. R. Shaw, "The Paradox of the Unexpected Examination," Mind 67:382-4, 1958.
  20158. A. Lyon, "The Prediction Paradox," Mind 68:510-7, 1959.
  20159. D. Kaplan and R. Montague, "A Paradox Regained," Notre Dame J Formal Logic
  20160.  1:79-90, 1960.
  20161. G. Nerlich, "Unexpected Examinations and Unprovable Statements," Mind
  20162.  70:503-13, 1961.
  20163. M. Gardner, "A New Prediction Paradox," Brit J Phil Sci 13:51, 1962.
  20164. K. Popper, "A Comment on the New Prediction Paradox," Brit J Phil Sci 13:51,
  20165.  1962.
  20166. B. Medlin, "The Unexpected Examination," Am Phil Q 1:66-72, 1964.
  20167. F. Fitch, "A Goedelized Formulation of the Prediction Paradox," Am Phil Q
  20168.  1:161-4, 1964.
  20169. R. Sharpe, "The Unexpected Examination," Mind 74:255, 1965.
  20170. J. Chapman & R. Butler, "On Quine's So-Called 'Paradox,'" Mind 74:424-5, 1965.
  20171. J. Bennett and J. Cargile, Reviews, J Symb Logic 30:101-3, 1965.
  20172. J. Schoenberg, "A Note on the Logical Fallacy in the Paradox of the
  20173.  Unexpected Examination," Mind 75:125-7, 1966.
  20174. J. Wright, "The Surprise Exam: Prediction on the Last Day Uncertain," Mind
  20175.  76:115-7, 1967.
  20176. J. Cargile, "The Surprise Test Paradox," J Phil 64:550-63, 1967.
  20177. R. Binkley, "The Surprise Examination in Modal Logic," J Phil 65:127-36,
  20178.  1968.
  20179. C. Harrison, "The Unanticipated Examination in View of Kripke's Semantics
  20180.  for Modal Logic," in Philosophical Logic, J. Davis et al (ed.), Dordrecht,
  20181.  1969.
  20182. P. Windt, "The Liar in the Prediction Paradox," Am Phil Q 10:65-8, 1973.
  20183. A. Ayer, "On a Supposed Antinomy," Mind 82:125-6, 1973.
  20184. M. Edman, "The Prediction Paradox," Theoria 40:166-75, 1974.
  20185. J. McClelland & C. Chihara, "The Surprise Examination Paradox," J Phil Logic
  20186.  4:71-89, 1975.
  20187. C. Wright and A. Sudbury, "The Paradox of the Unexpected Examination,"
  20188.  Aust J Phil 55:41-58, 1977.
  20189. I. Kvart, "The Paradox of the Surprise Examination," Logique et Analyse
  20190.  337-344, 1978.
  20191. R. Sorenson, "Recalcitrant Versions of the Prediction Paradox," Aust J Phil
  20192.  69:355-62, 1982.
  20193. D. Olin, "The Prediction Paradox Resolved," Phil Stud 44:225-33, 1983.
  20194. R. Sorenson, "Conditional Blindspots and the Knowledge Squeeze: A Solution to
  20195.  the Prediction Paradox," Aust J Phil 62:126-35, 1984.
  20196. C. Chihara, "Olin, Quine and the Surprise Examination," Phil Stud 47:191-9,
  20197.  1985.
  20198. R. Kirkham, "The Two Paradoxes of the Unexpected Hanging," Phil Stud
  20199.  49:19-26, 1986.
  20200. D. Olin, "The Prediction Paradox: Resolving Recalcitrant Variations," Aust J
  20201.  Phil 64:181-9, 1986.
  20202. C. Janaway, "Knowing About Surprises: A Supposed Antinomy Revisited," Mind
  20203.  98:391-410, 1989.
  20204.  
  20205.     -- tycchow@math.mit.edu.
  20206.  
  20207. ==> logic/verger.p <==
  20208. A very bright and sunny Day
  20209. The Priest didst to the Verger say:
  20210. "Last Monday met I strangers three
  20211. None of which were known to Thee.
  20212. I ask'd Them of Their Age combin'd
  20213. which amounted twice to Thine]
  20214. A Riddle now will I give Thee:
  20215. Tell Me what Their Ages be]"
  20216.  
  20217. So the Verger ask'd the Priest:
  20218. "Give to Me a Clue at least]"
  20219. "Keep Thy Mind and Ears awake,
  20220. And see what Thou of this can make.
  20221. Their Ages multiplied make plenty,
  20222. Fifty and Ten Dozens Twenty."
  20223.  
  20224. The Verger had a sleepless Night
  20225. To try to get Their Ages right.
  20226. "I almost found the Answer right.
  20227. Please shed on it a little Light."
  20228. "A little Clue I give to Thee,
  20229. I'm older than all Strangers three."
  20230. After but a little While
  20231. The Verger answered with a Smile:
  20232. "Inside my Head has rung a Bell.
  20233. Now I know the answer well]"
  20234.  
  20235.  
  20236. Now, the question is:
  20237.  
  20238. How old i
  20239.  
  20240. ==> logic/verger.s <==
  20241. The puzzler tried to take the test;
  20242. Intriguing rhymes he wished to best.
  20243. But "Fifty and ten dozens twenty"
  20244. made his headache pound aplenty.
  20245. When he finally found some leisure,
  20246. He took to task this witty treasure.
  20247.  
  20248. "The product of the age must be
  20249. Twenty-Four Hundred Fifty]"
  20250. Knowing that, he took its primes,
  20251. permuted them as many times
  20252. as needed, til he found amounts
  20253. equal to, by all accounts,
  20254. twice the Verger's age, so that
  20255. He would have that next day's spat.
  20256.  
  20257. The reason for the lad's confusion
  20258. was due to multiple solution]
  20259. Hence he needed one more clue
  20260. to give the answer back to you]
  20261. Since only one could fit the bill,
  20262. and then confirm the priest's age still,
  20263. the eldest age of each solution
  20264. by one could differ, with no coercion.  <=(Sorry)
  20265.  
  20266. Else, that last clue's revelation
  20267. would not have brought information]
  20268. With two, two, five, seven, and seven,
  20269. construct three ages, another set of seven.
  20270. Two sets of three yield sixty-four,
  20271. Examine them, yet one time more.
  20272. The eldest age of each would be
  20273. forty-nine, and then, fifty]
  20274.  
  20275. With lack of proper rhyme and meter,
  20276. I've tried to be the first completor
  20277. of this poem and a puzzle;
  20278. my poetry, you'd try to muzzle]
  20279. And lest you think my wit is thrifty,
  20280. The answer, of course, must be fifty]
  20281. If dispute, you wish to tender,
  20282. note my addresss, as the sender]
  20283.  
  20284. --
  20285. Kevin Nechodom <knechod@stacc.med.utah.edu>
  20286.  
  20287. ==> logic/weighing/balance.p <==
  20288. You are given N balls and a balance scale and told that
  20289. one ball is slightly heavier or lighter than the other identical
  20290. ones.  The scale lets you put the same number of balls on each side
  20291. and observe which side (if either) is heavier.
  20292.  
  20293. 1.     What's the minimum # of weighings X (and way of doing them)
  20294. that will always find the unique ball and whether it's heavy or light?
  20295.  
  20296. 2.     If you are told the unique ball is, in fact, heavier than the
  20297. others, what's the minimum # of weighings Y to find it?
  20298.  
  20299. ==> logic/weighing/balance.s <==
  20300. Martin Gardner gave a neat solution to this problem.
  20301.  
  20302. Assume that you are allowed N weighings.  Write down the 3^N possible
  20303. length N strings of the symbols '0', '1', and '2'.  Eliminate the three
  20304. such strings that consist of only one symbol repeated N times.
  20305.  
  20306. For each string, find the first symbol that is different from the symbol
  20307. preceeding it.  Consider that pair of symbols.  If that pair is not 01,
  20308. 12, or 20, cross out that string.  In other words, we only allow strings
  20309. of the forms 0*01.*, 1*12.*, or 2*20.* ( using ed(1) regular expressions ).
  20310.  
  20311. You will have (3^N-3)/2 strings left.  This is how many balls you can
  20312. handle in N weighings.
  20313.  
  20314. Perform N weighings as follows:
  20315.  
  20316.         For weighing I, take all the balls that have a 0 in string
  20317.         position I, and weigh them against all the balls that have
  20318.         a 2 in string position I.
  20319.  
  20320.         If the side with the 0's in position I goes down, write down
  20321.         a 0.  If the other side goes down, write down a 2.  Otherwise,
  20322.         write down a 1.
  20323.  
  20324. After the N weighings, you have written down an N symbol string.  If
  20325. your string matches the string on one of the balls, then that is the
  20326. odd ball, and it is heavy.  If none of them match, than change every
  20327. 2 to a 0 in your string, and every 0 to a 2.  You will then have a
  20328. string that matches one of the balls, and that ball is lighter than
  20329. the others.
  20330.  
  20331. Note that if you only have to identify the odd ball, but don't have to
  20332. determine if it is heavy or light, you can handle (3^N-3)/2+1 balls.
  20333. Label the extra ball with a string of all 1's, and use the above
  20334. method.
  20335.  
  20336. Note also that you can handle (3^N-3)/2+1 balls if you *do* have to
  20337. determine whether it is heavy or light, provided you have a single reference
  20338. ball available, which you know has the correct weight. You do this by
  20339. labelling the extra ball with a string of all 2s. This results in it being
  20340. placed on the same side of the scales each time, and in that side of the
  20341. scales having one more ball than the other each time. So you put the
  20342. reference ball on the other side of the scales to the "all 2s" ball on each
  20343. weighing.
  20344.  
  20345. Proving that this works is straightforward, once you notice that the
  20346. method of string construction makes sure that in each position, 1/3
  20347. of the strings have 0, 1/3 have 1, and 1/3 have 2, and that if a
  20348. string occurs, then the string obtained by replacing each 0 with a
  20349. 2 and each 2 with a 0 does not occur.
  20350.  
  20351. ==> logic/weighing/box.p <==
  20352. You have ten boxes; each contains nine balls.  The balls in one box
  20353. weigh 0.9 kg; the rest weigh 1.0 kg.  You have one weighing on a
  20354. scale to find the box containing the light balls.  How do you do it?
  20355.  
  20356. ==> logic/weighing/box.s <==
  20357. Number the boxes 0-9.  Take 0 balls from box 0, 1 ball from box 1, 2
  20358. balls from box 2, etc.  Now weight all those balls and follow this
  20359. table:
  20360.  
  20361. If odd box is   Weight is
  20362. 0               45 kg
  20363. 1               44.9 kg
  20364. 2               44.8 kg
  20365. 3               44.7 kg
  20366. 4               44.6 kg
  20367. 5               44.5 kg
  20368. 6               44.4 kg
  20369. 7               44.3 kg
  20370. 8               44.2 kg
  20371. 9               44.1 kg
  20372.  
  20373. ==> logic/weighing/gummy.bears.p <==
  20374. Real gummy drop bears have a mass of 10 grams, while imitation gummy
  20375. drop bears have a mass of 9 grams.  Spike has 7 cartons of gummy drop bears,
  20376. 4 of which contain real gummy drop bears, the others imitation.
  20377. Using a scale only once and the minimum number of gummy drop bears, how
  20378. can Spike determine which cartons contain real gummy drop bears?
  20379.  
  20380. ==> logic/weighing/gummy.bears.s <==
  20381. Spike used 51 gummy drop bears:  from the 7 boxes he took respectively
  20382. 0, 1, 2, 4, 7, 13, and 24 bears.
  20383.  
  20384. The notion is that each box of imitation bears will subtract its
  20385. number of bears from the total "ideal" weight of 510 grams (1 gram of
  20386. missing weight per bear), so Spike weighs the bears, subtracts the
  20387. result from 510 to obtain a number N, and finds the unique combination
  20388. of 3 numbers from the above list (since there are 3 "imitation" boxes)
  20389. that sum to N.
  20390.  
  20391. The trick is for the sums of all triples selected from the set S of
  20392. numbers of bears to be unique.  To accomplish this, I put numbers into
  20393. S one at a time in ascending order, starting with the obvious choice,
  20394. 0.  (Why is this obvious?  If I'd started with k > 0, then I could
  20395. have improved on the resulting solution by subtracting k from each
  20396. number) Each new number obviously had to be greater than any previous,
  20397. because otherwise sums are not unique, but also the sums it made when
  20398. paired with any previous number had to be distinct from all previous
  20399. pairs (otherwise when this pair is combined with a third number you
  20400. can't distinguish it from the other pair)--except for the last box,
  20401. where we can ignore this point.  And most obviously all the new
  20402. triples had to be distinct from any old triples; it was easy to find
  20403. what the new triples were by adding the newest number to each old sum
  20404. of pairs.
  20405.  
  20406. Now, in case you're curious, the possible weight deficits and their
  20407. unique decompositions are:
  20408.  
  20409. 3       = 0 + 1 + 2
  20410. 5       = 0 + 1 + 4
  20411. 6       = 0 + 2 + 4
  20412. 7       = 1 + 2 + 4
  20413. 8       = 0 + 1 + 7
  20414. 9       = 0 + 2 + 7
  20415. 10      = 1 + 2 + 7
  20416. 11      = 0 + 4 + 7
  20417. 12      = 1 + 4 + 7
  20418. 13      = 2 + 4 + 7
  20419. 14      = 0 + 1 + 13
  20420. 15      = 0 + 2 + 13
  20421. 16      = 1 + 2 + 13
  20422. 17      = 0 + 4 + 13
  20423. 18      = 1 + 4 + 13
  20424. 19      = 2 + 4 + 13
  20425. 20      = 0 + 7 + 13
  20426. 21      = 1 + 7 + 13
  20427. 22      = 2 + 7 + 13
  20428. 24      = 4 + 7 + 13
  20429. 25      = 0 + 1 + 24
  20430. 26      = 0 + 2 + 24
  20431. 27      = 1 + 2 + 24
  20432. 28      = 0 + 4 + 24
  20433. 29      = 1 + 4 + 24
  20434. 30      = 2 + 4 + 24
  20435. 31      = 0 + 7 + 24
  20436. 32      = 1 + 7 + 24
  20437. 33      = 2 + 7 + 24
  20438. 35      = 4 + 7 + 24
  20439. 37      = 0 + 13 + 24
  20440. 38      = 1 + 13 + 24
  20441. 39      = 2 + 13 + 24
  20442. 41      = 4 + 13 + 24
  20443. 44      = 7 + 13 + 24
  20444.  
  20445. Note that there had to be (7 choose 3) distinct values; they end up
  20446. ranging from 3 to 44 inclusive with 7 numbers missing:  4, 23, 34, 36,
  20447. 40, 42, and 43.
  20448.  
  20449. -- David Karr (karr@cs.cornell.edu)
  20450.  
  20451. ==> logic/weighing/weighings.p <==
  20452. Some of the supervisors of Scandalvania's n mints are producing bogus coins.
  20453. It would be easy to determine which mints are producing bogus coins but,
  20454. alas, the only scale in the known world is located in Nastyville,
  20455. which isn't on very friendly terms with Scandalville.  In fact, Nastyville's
  20456. king will only let you use the scale twice.  Your job?  You must determine
  20457. which of the n mints are producing the bogus coins using only two weighings
  20458. and the minimum number of coins (your king is rather parsimonious, to put it
  20459. nicely).  This is a true scale, i.e. it will tell you the weight of whatever
  20460. you put on it.  Good coins are known to have a weight of 1 ounce and it is
  20461. also known that all the bogus mints (if any) produce coins that are
  20462. light or heavy by the same amount.
  20463.  
  20464. Some examples: if n=1 then we only need 1 coin, if n=2 then clearly 2
  20465. coins suffice, one from each mint.
  20466.  
  20467. What are the solutions for n=3,4,5?  What can be said for general n?
  20468.  
  20469. ==> logic/weighing/weighings.s <==
  20470. Oh gracious and wise king, I have solved this problem by first
  20471. simplifying and then expanding.  That is, consider the problem of
  20472. being allowed only a single weighing.  Stop reading right now if you
  20473. want to think about it further.
  20474.  
  20475. There are three possible outcomes for each mint (light, OK, heavy)
  20476. which may be represented as (-1, 0, +1).  Now, let each mint represent
  20477. one place in base 3.  Thus, the first mint is the ones place, the
  20478. second the threes place, the third is the nines place and so on.  The
  20479. number of coins from each mint must equal the place.  That is, we'll
  20480. have 1 coin from mint 1, 3 from mint 2, 9 from mint 3, and, in
  20481. general, 3^(n-1) from mint n.
  20482.  
  20483. By weighing all coins at once, we will get a value between 1 + 3 + 9 +
  20484. ...  and -1 + -3 + -9 + ...  In fact, we notice that that value will
  20485. be unique for any mint outcomes.  Thus, for the one weighing problem,
  20486. we need
  20487.  
  20488. sum for i=1 to n (3^(i-1))
  20489.  
  20490. which evaluates to (3^n - 1)/2
  20491.  
  20492. I'm fairly satisfied that this is a minimum for a single weighing.
  20493. What does a second weighing give us?  Well, we can divide the coins
  20494. into two groups and use the same method.  That is, if we have 5 mints,
  20495. one weighing will be:
  20496.  
  20497. 1 coin from mint 1 + 3 coins from mint 2 + 9 coins from mint 3
  20498.  
  20499. while the other weighing will be:
  20500.  
  20501. 1 coin from mint 4 + 3 coins from mint 5
  20502.  
  20503. It's pretty plain that this gives us a total coinage of:
  20504.  
  20505.    3^(n/2) - 1 for even n and, after some arithmetic agitation:
  20506.    2 * 3^((n-1)/2) - 1 for odd n
  20507.  
  20508. I think the flaw in this solution is that we don't know ahead of time
  20509. the amount by which the coins are off weight.  So if you weigh 1 coin
  20510. from mint 1 together with 3 coins from mint 2 and the result is heavy
  20511. by 3x units, you still don't know whether the bogus coins are from
  20512. mint 3 (heavy by x units) or from mint 1 (heavy by 3x units).  Note
  20513. that we're not given the error amount, only the fact that is is equal
  20514. for all bogus coins.
  20515.  
  20516. Here is my partial solution:
  20517.  
  20518. After considering the above, it would seem that on each of the two
  20519. weighings we must include coins from all of the mints (except for the
  20520. special cases of small n).  So let ai (a sub i) be the number of coins
  20521. from mint i on weighing 1 and bi be the number of coins from mint i on
  20522. weighing 2.  Let the error in the bogus coins have a value x, and let
  20523. ci be a the counterfeit function: ci is 0 if mint i is good, 1
  20524. otherwise.
  20525.  
  20526. Then
  20527.         Sum   ai ci x = delta1          error on weighing 1
  20528.         Sum   bi ci x = delta2          error on weighing 2
  20529.  
  20530. Now the ratio of delta1 to delta2 will be rational regardless of the
  20531. value of x, since x will factor out; let's call this ratio p over q (p
  20532. and q relatively prime).  We would like to choose { ai } and { bi }
  20533. such that for any set of mints J, which will be a subset of { 1 , 2 ,
  20534. ... , n }, that
  20535.  
  20536.         Sum aj  ( = Sum  ai ci ) is relatively prime to Sum bj.
  20537.  
  20538. If this is true then we can determine the error x; it will simply be
  20539. delta1/p, which is equal to  delta2/q.
  20540.  
  20541. If the { ai } have been carefully chosen, we should be able to figure
  20542. out the bogus mints from one of the weighings, provided that
  20543. all subsets ( { { aj } over all J } ) have unique sums.
  20544. This was the strategy proposed above, where is was suggested
  20545. that ai = 3 ** (i-1) ; note that you can use base 2 instead
  20546. of base 3 since all the errors have the same sign.
  20547.  
  20548. Well, for the time being I'm stumped.
  20549.  
  20550. This agrees with the analysis I've been fighting with.  I actually
  20551. came up with a pair of functions that "almost" works.  So that the
  20552. rest of you can save some time (in case you think the way I did):
  20553.     Weighing 1: 1 coin from each mint
  20554.     Weighing 2: 2^(k-1) coins from mint k, for 1...k...n
  20555.         (total 2^n - 1 coins)
  20556.  
  20557. Consider the n mints to be one-bit-each -- bit set -> mint makes bogus
  20558. coins.  Then we can just state that we're trying to discover "K",
  20559. where K is a number whose bit pattern _just_ describes the bogosity of
  20560. each mint.  OK - now, assuming we know 'x', and we only consider the
  20561. *difference* of the weighing from what it should be, for weighing 1,
  20562. the devaiation is just the Hamming weight of K -- that is the number
  20563. of 1-bits in it -- that is, the number of bogosifying mints.  For
  20564. weighing 2, the deviation is just K]  When the nth bit of K is set,
  20565. then that mint contributes just 2^n to the deviation, and so the total
  20566. deviation will just be K.
  20567.  
  20568. So that set me in search of a lemma: given H(x) is the hamming weight
  20569. of x, is f(x) = x / H(x) a 1-1 map integers into rationals?  That is,
  20570. if x/H(x) = y/H(y) can we conclude that x = y?
  20571.  
  20572. The answer (weep) is NO.  The lowest pair I could find are 402/603
  20573. (both give the ratio 100.5).  Boy it sure looked like a good
  20574. conjecture for a while]  Sigh.
  20575.  
  20576.  
  20577. There are two parts to the problem. First let us try to come up with a
  20578. solution to finding the answer in 2 weighings - then worry about using the
  20579. min. number of coins.
  20580. Solutions are for GENERAL n.
  20581.  
  20582. Let N = set of all mints, 1 to n. Card(N) = n.
  20583. Let P = set of all bogus mints. Let Card(P) = p.
  20584.  
  20585. Weighing I: Weigh n coins, 1 from each mint.
  20586.  
  20587. Since each "good" coins weighs one ounce, let delta1 be the error in weighing.
  20588. Since all bogus coins are identical, let delta1 be abs(error).
  20589. If x is the weight by which one bogus coin differs from a good coin,
  20590.  delta1 = p * x.
  20591.  
  20592. Weighing II: The coins to be weighed are composed thusly.
  20593.  
  20594. Let a1 be the number of coins from mint 1, a2 # from mint2 .. and an from
  20595. mint n. All ai's are distinct integers.
  20596.  
  20597. Let A = Set of all ai's.
  20598.  
  20599. Let delta2 = (abs.) error in weighing 2 = x * k
  20600.         where k is the number of coins that are bogus in weighing two.
  20601. Or more formally
  20602.                 k = sigma(ai)
  20603.                  (over all i in P)
  20604.  
  20605. Assuming p is not zero (from Weighing I - in that case go back and get beheaded
  20606. for giving the king BAAAAAD advice),
  20607.         Let ratio = delta1/delta2 = p/k.
  20608.         Let IR = delta2/delta1 = k/p = inverse-ratio (for later proof).
  20609.  
  20610. Let S(i) be the bag of all numbers generated by summing i distinct elements
  20611. from A. Clearly there will be nCi (that n comb. i) elements in S(i).
  20612.  
  20613. ▌A bag is a set that can have the same element occur more than once.¿
  20614.  
  20615. So S(1) = A
  20616. and S(n) will have one element that is the sum of all the elements of A.
  20617.  
  20618. Let R(i) = {x : For-all y in S(i), x = i/y} expressed as p/q (no common
  20619.                                                                 factors).
  20620. (R is a bag too).
  20621.  
  20622. Let R-A = Bag-Union(R(i) for 1>= i >=n). (can include same element twice)
  20623.  
  20624. Choose A, such that all elements of R-A are DISTINCT, i.e. Bag(R-A) = Set(R-A).
  20625.  
  20626. Let the sequence a1, a2, .. an, be an L-sequence if the above property is
  20627. true. Or more simply, A is in L.
  20628.  
  20629. **********************************************************************
  20630. CONJECTURE: The bogus mint problem is solved in two weighings if A is in L.
  20631.  
  20632. Sketchy proof: R(1) = all possible ratios (= delta1/delta2) when p=1.
  20633. R(i) = all possible ratio's when p=i.
  20634.  
  20635. Since all possible combinations of bogus mints are reflected in R, just match
  20636. the actual ratio with the generated table for n.
  20637.  
  20638. ************************************************************************
  20639. A brief example. Say n=3. Skip to next line if you want.
  20640. Let A=(2,3,7).
  20641.  
  20642. p=1 possible ratios = 1/2 1/3 1/7
  20643. p=2 possible ratios = 2/5 2/9 1/5(2/10)
  20644. p=3 possible ratios = 1/4(3/12) (lots of blood in Scandalvania).
  20645.  
  20646. As all outcomes are distinct, and the actual ratio MUST be one of these,
  20647. match it to the answer, and start sharpening the axe.
  20648.  
  20649. Note that the minimum for n=3 is A=(0,1,3)
  20650. possible ratios are
  20651. p=1 infinity (delta2=0),1,1/3
  20652. p=2 2/1,2/3,1/2
  20653. p=3 3/4
  20654.  
  20655. ************************************************************************
  20656.  
  20657. All those with the determination to get this far are saying OK, OK how do we
  20658. get A.
  20659.  
  20660. I propose a solution that will generate A, to give you the answer in two
  20661. weighings, but will not give you the optimal number of coins.
  20662.  
  20663. Let a1=0
  20664.  
  20665. For i>=2 >=n
  20666.  
  20667. ai = i*(a1 + a2 + ... + ai-1) + 1
  20668.  
  20669. *****************************************
  20670. *               i-1                     *
  20671. *       ai = i* ▌Sigma(aj)¿ + 1         *   ****Generator function G*****
  20672. *               j=1                     *
  20673. *****************************************
  20674.  
  20675. If A is L, all RATIO's are unique. Also all inverse-ratio's (1/ratio) are
  20676. unique. I will prove that all inverse-ratio's (or IR's) are unique.
  20677.  
  20678. Let A(k), be the series generated by the first k elements from eqn. G.(above)
  20679.  
  20680. ************************************************************************
  20681.  
  20682. PROOF BY INDUCTION.
  20683.  
  20684. A(1) = {0} is in L.
  20685. A(2) = {0,1} is in L.
  20686.  
  20687. ASSUME A(k) = {0,1, ..., ak} is in L.
  20688.  
  20689. T.P.T. A(k+1) = {0,1, ..., ak, D) is in L where D is generated from G.
  20690.  
  20691. We know that all IR's(inverse ratio's)  from A(k) are distinct.
  20692.  
  20693. Let K = set of all IR's of A(k).
  20694.  
  20695. Since A(k+1) contains A(k), all IR's of A(k) will also be IR's of A(K+1).
  20696.  
  20697. So for all P, such that (k+1) is not in P, we get a distinct IR.
  20698.  
  20699. So consider cases when (k+1) is in P.
  20700.  
  20701. p=1 (i.e. (k+1) = only bogus mint), IR = D
  20702.  
  20703. ______________________________________________________________________
  20704. CONJECTURE: Highest IR for A(k) = max(K) = ak
  20705.  
  20706. Proof:  Since max▌A(k)¿ = ak,
  20707.                 for p'= 1, max IR = ak/1 = ak
  20708.                 for p'= 2, max IR (max sum of 2 ai's)/2
  20709.                            = (ak + ak-1)/2 < ak (as ak>ak-1).
  20710.                 for p'= i max IR sum of largest i elements of A(k)
  20711.                                   --------------------------------
  20712.                                                 i
  20713.                                 < i * ak/i = ak.
  20714.         So max. IR for A(k) is ak.
  20715. ______________________________________________________________________
  20716.  
  20717. D > ak
  20718. So for p=1 IR is distinct.
  20719.  
  20720. Let Xim be the IR formed by choosing i elements from A(k+1).
  20721.         Note: We are choosing D and (i-1) elements from A(k).
  20722.               m is just an index to denote each distinct combination of
  20723.               (i-1) elemnts of A(i).
  20724.  
  20725. ______________________________________________________________________
  20726. CONJECTURE : For p=j, all new IR's Xjm are limited to the range
  20727.                  D/(j-1) > Xjm > D/j.
  20728.  
  20729. Proof:
  20730.         Xjm = (D + {j-1 elements of A(k)})/j
  20731.  
  20732.         Clearly Xjm > D/j.
  20733.  
  20734.         To show: max▌Xjm¿ < D/(j-1)
  20735.  
  20736.         Note: a1 + a2 .. + ak < D/(k+1)
  20737.  
  20738.        max▌Xjm¿ = (D + ak + ak-1 +  ... + a(k-j+1))/j
  20739.                 < (D + D/(k+1))/j
  20740.                 = D (k+2)/(k+1)j
  20741.                 = ▌D/(j-1)¿ * alpha.
  20742.  
  20743.         alpha = (j-1)/(j)  *  (k+2)/(k+1)
  20744.  
  20745.                 Since j <= k, (j-1)/j <= (k-1)/k < (k+1)/(k+2)
  20746.  
  20747.         IMPLIES alpha < 1.
  20748.  
  20749. Conjecture proved.
  20750.  
  20751. ______________________________________________________________________
  20752. CONJECTURE : For a given p, all newly generated IR's are distinct.
  20753.  
  20754. Proof by contradiction:
  20755.  
  20756. Assume this is not so.
  20757.  
  20758. Implies
  20759.         (D + (p-1) elements of A(k))/p
  20760.         = (D + some other (p-1) elements of A(k))/p
  20761.  
  20762. Implies SUM▌(p-1) elements of A(k)¿ = SUM▌ some other (p-1) elements of A(k)¿
  20763.  
  20764. Implies SUM▌(p-1) elements of A(k)¿/(p-1)
  20765.                 = SUM▌some other (p-1) elements¿/(p-1)
  20766.  
  20767. Implies A(k) is NOT in L.
  20768.  
  20769. Contra.
  20770.  
  20771. Hence conjecture.
  20772. ______________________________________________________________________
  20773.  
  20774. CONJECTURE: A(k+1) is in L.
  20775.  
  20776. Since all newly generated IR's are distinct from each other, and all newly generated IR's are greater than previous IR's, A(k+1) is in L.
  20777.  
  20778. ==> logic/zoo.p <==
  20779.  I took some nephews and nieces to the Zoo, and we halted at a cage marked
  20780.  
  20781.                     Tovus Slithius, male and female.
  20782.                   Beregovus Mimsius, male and female.
  20783.                      Rathus Momus, male and female.
  20784.                 Jabberwockius Vulgaris, male and female.
  20785.  
  20786.  The eight animals were asleep in a row, and the children began to guess
  20787.  which was which.  "That one at the end is Mr Tove."  "No, no]  It's Mrs
  20788.  Jabberwock," and so on.  I suggested that they should each write down
  20789.  the names in order from left to right, and offered a prize to the one
  20790.  who got most names right.
  20791.  
  20792.  As the four species were easily distinguished, no mistake would arise in
  20793.  pairing the animals; naturally a child who identified one animal as Mr
  20794.  Tove identified the other animal of the same species as Mrs Tove.
  20795.  
  20796.  The keeper, who consented to judge the lists, scrutinised them carefully.
  20797.  "Here's a queer thing.  I take two of the lists, say, John's and Mary's.
  20798.  The animal which John supposes to be the animal which Mary supposes to be
  20799.  Mr Tove is the animal which Mary supposes to be the animal which John
  20800.  supposes to be Mrs Tove.  It is just the same for every pair of lists,
  20801.  and for all four species.
  20802.  
  20803.  "Curiouser and curiouser]  Each boy supposes Mr Tove to be the animal
  20804.  which he supposes to be Mr Tove; but each girl supposes Mr Tove to be
  20805.  the animal which she supposes to be Mrs Tove.  And similarly for the oth-
  20806.  er animals.  I mean, for instance, that the animal Mary calls Mr Tove
  20807.  is really Mrs Rathe, but the animal she calls Mrs Rathe is really Mrs
  20808.  Tove."
  20809.  
  20810.  "It seems a little involved," I said, "but I suppose it is a remarkable
  20811.  coincidence."
  20812.  
  20813.  "Very remarkable," replied Mr Dodgson (whom I had supposed to be the
  20814.  keeper) "and it could not have happened if you had brought any more
  20815.  children."
  20816.  
  20817.  How many nephews and nieces were there?  Was the winner a boy or a girl?
  20818.  And how many names did the winner get right?   ▌by Sir Arthur Eddington¿
  20819.  
  20820. ==> logic/zoo.s <==
  20821. Given that there is at least one boy and one girl (John and Mary are
  20822. mentioned) then the answer is that there were 3 nephews and 2 nieces,
  20823. the winner was a boy who got 4 right.
  20824.  
  20825. Number the animals 1 through 8, such that the females are even and the
  20826. males are odd, with members of the same species consecutive; i.e.
  20827. 1 is Mr. Tove, 2 Mrs. Tove, etc.
  20828.  
  20829. Then each childs guesses can be represented by a permutation.  I use
  20830. the standard notation of a permutation as a set of orbits.
  20831. For example: (1 3 5)(6 8) means 1 -> 3, 3 -> 5, 5 -> 1, 6 -> 8, 8 -> 6
  20832. and 2,4,7 are unchanged.
  20833.  
  20834. ▌1¿  Let P be any childs guesses.  Then P(mate(i)) = mate(P(i)).
  20835.  
  20836. ▌2¿  If Q is another childs guesses, then ▌P,Q¿ = T, where
  20837. ▌P,Q¿ is the commutator of P and Q (P composed with Q composed with
  20838. P inverse composed with Q inverse) and T is the special permutation
  20839. (1 2) (3 4) (5 6) (7 8) that just swaps each animal with its spouse.
  20840.  
  20841. ▌3¿  If P represents a boy, then P*P = I (I use * for composition, and I
  20842. for
  20843. the identity permutation: (1)(2)(3)(4)(5)(6)(7)(8)
  20844.  
  20845. ▌4¿  If P represents a girl, then P*P = T.
  20846.  
  20847. ▌1¿ and ▌4¿ together mean that all girl's guesses must be of the form:
  20848.     (A B C D) (E F G H) where A and C are mates, as are B & D, E & F
  20849.     G & H.
  20850.  
  20851. So without loss of generality let Mary = (1 3 2 4) (5 7 6 8)
  20852. Without to much effort we see that the only possibilities for other
  20853. girls "compatible" with Mary (I use compatible to mean the relation
  20854. expressed in ▌2¿) are:
  20855.     g1:  (1 5 2 6) (3 8 4 7)
  20856.     g2:  (1 6 2 5) (3 7 4 8)
  20857.     g3:  (1 7 2 8) (3 5 4 6)
  20858.     g4:  (1 8 2 7) (3 6 4 5)
  20859.  
  20860. Note that g1 is incompatible with g2 and g3 is incompatible with g4.
  20861. Thus no 4 of Mary and g1-4 are mutually compatible.  Thus there are at
  20862. most three girls: Mary, g1 and g3 (without loss of generality)
  20863.  
  20864. By ▌1¿ and ▌3¿, each boy must be represented as a product of
  20865. transpostions and/or singletons: e.g. (1 3) (2 4) (5) (6) (7) (8) or
  20866. (1) (2) (3 4) (5 8) (6 7).
  20867.  
  20868. Let J represent John's guesses and consider J(1).
  20869. If J(1) = 1, then J(2) = 2 (by ▌1¿) using ▌2¿ and Mary J(3) = 4, J(4) =
  20870. 3,  and g1 & J => J(5) = 6, J(6) = 5, & g3 & J => J(8) = 7 J(7) = 8
  20871. i.e. J = (1)(2)(3 4)(5 6)(7 8).  But the ▌J,Mary¿ <> T.  In fact, we
  20872. can see that J must have no fixed points, J(i) <> i for all i, since
  20873. there is nothing special about i = 1.
  20874.  
  20875. If J(1) = 2, then we get from Mary that J(3) = 3.  contradiction.
  20876.  
  20877. If J(1) = 3, then J(2) = 4, J(3) = 1, J(4) = 2 (from Mary) =>
  20878.    J(5) = 7, J(6) = 8, J(7) = 5, J(8) = 6 => J = (1 3)(2 4)(5 7)(6 8)
  20879.    (from g1)
  20880. But then J is incompatible with g3.
  20881.  
  20882. A similar analysis shows that J(1) cannot be 4,5,6,7 or 8; i.e. no J
  20883. can be compatible with all three girls.  So without loss of generality,
  20884. throw away g3.
  20885.  
  20886. We have Mary = (1 3 2 4) (5 7 6 8)
  20887.         g1   = (1 5 2 6) (3 8 4 7)
  20888.  
  20889. The following are the only possible boy guesses which are compatible
  20890. with
  20891. both of these:
  20892.  
  20893.   B1: (1)(2)(3 4)(5 6)(7)(8)
  20894.   B2: (1 2)(3)(4)(5)(6)(7 8)
  20895.   B3: (1 3)(2 4)(5 7)(6 8)
  20896.   B4: (1 4)(2 3)(5 8)(6 7)
  20897.   B5: (1 5)(2 6)(3 8)(4 7)
  20898.   B6: (1 6)(2 5)(3 7)(4 8)
  20899.  
  20900. Note that B1 & B2 are incombatible, as are B3 & B4, B5 & B6, so at most
  20901. three
  20902. of them are mutually compatible.  In fact, Mary, g1, B1, B3 and B5 are
  20903. all
  20904. mutually compatible (as are all the other possibilities you can get by
  20905. choosing
  20906. either B1 or B2, B3 or B4, B5 or B6.  So if there are 2 girls there can
  20907. be
  20908. 3 boys, but no more, and we have already eliminated the case of 3 girls
  20909. and
  20910. 1 boy.
  20911.  
  20912. The only other possibility to consider is whether there can be 4 or more
  20913. boys
  20914. and 1 girl.  Suppose there are Mary and 4 boys.  Each boy must map 1 to
  20915. a
  20916. different digit or they would not be mutually compatible.  For example
  20917. if b1
  20918. and b2 both map 1 to 3, then they both map 3 to 1 (since a boy's map
  20919. consists
  20920. of transpositions), so both b1*b2 and b2*b1 map 1 to 1.  Furthermore, b1
  20921. and
  20922. b2 cannot map 1 onto spouses.  For example, if b1(1) = a and b is the
  20923. spouse
  20924. of a, then b1(2) = b.  If b2(1) = b, then b2(2) = a.  Then
  20925. b1*b2(1) = b1(b) = 2 and b2*b1(1) = b2(a) = 2 (again using the fact that
  20926. boys
  20927. are all transpostions).  Thus the four boys must be:
  20928.  
  20929. B1: (1)(2)...    or (1 2)....
  20930. B2: (1 3)...     or (1 4) ...
  20931. B3: (1 5) ...    or (1 6) ...
  20932. B4: (1 7) ...    or (1 8) ...
  20933.  
  20934. Consider B4.  The only permutation of the form (1 7)... which is
  20935. compatible
  20936. with Mary ( (1 3 2 4) (5 7 6 8) ) is:
  20937.  
  20938.    (1 7)(2 8)(3 5)(4 6)
  20939.  
  20940. The only (1 8)... possibility is:
  20941.  
  20942.    (1 8)(2 7)(3 6)(4 5)
  20943.  
  20944. Suppose B4 = (1 7)(2 8)(3 5)(4 6)
  20945.  
  20946. If B3 starts (1 5), it must be (1 5)(2 6)(3 8)(4 7) to be compatible
  20947. with B4.
  20948. This is compatible with Mary also.
  20949.  
  20950. Assuming this and B2 starts with (1 3) we get B2 = (1 3)(2 4)(5 8)(6 7)
  20951. in
  20952. order to be compatible with B4.  But then B2*B3 and B3*B2 moth map 1 to
  20953. 8.
  20954. I.e. no B2 is mutually compatible with B3 & B4.
  20955.  
  20956. Similarly if B2 starts with (1 4) it must be (1 4)(2 3)(5 7)(6 8) to
  20957. work
  20958. with B4, but this doesn't work with B3.
  20959.  
  20960. Likewise B3 starting with (1 6) leads to no possible B2 and the
  20961. identical
  20962. reasoning eliminates B4 = (1 8)...
  20963.  
  20964. So no B4 is possible]
  20965.  
  20966. I.e at most 3 boys are mutually compatiblw with Mary, so 2 girls & 3
  20967. boys is optimal.
  20968.  
  20969. Thus:
  20970.  
  20971. Mary = (1 3 2 4) (5 7 6 8)
  20972. Sue  = (1 5 2 6) (3 8 4 7)
  20973. John = (1)(2)(3 4)(5 6)(7)(8)
  20974. Bob  = (1 3)(2 4)(5 7)(6 8)
  20975. Jim  = (1 5)(2 6)(3 8)(4 7)
  20976.  
  20977. is one optimal solution, with the winner being John (4 right: 1 2 7 & 8)
  20978. ==> physics/balloon.p <==
  20979. A helium-filled balloon is tied to the floor of a car that makes a
  20980. sharp right turn.  Does the balloon tilt while the turn is made?
  20981. If so, which way?  The windows are closed so there is no connection
  20982. with the outside air.
  20983.  
  20984. ==> physics/balloon.s <==
  20985. Because of buoyancy, the helium balloon on the string will want to move
  20986. in the direction opposite the effective gravitational field existing
  20987. in the car.  Thus, when the car turns the corner, the balloon will
  20988. deflect towards the inside of the turn.
  20989.  
  20990. ==> physics/bicycle.p <==
  20991. A boy, a girl and a dog go for a 10 mile walk. The boy and girl can
  20992. walk 2 mph and the dog can trot at 4 mph. They also have bicycle
  20993. which only one of them can use at a time. When riding, the boy and
  20994. girl can travel at 12 mph while the dog can peddle at 16 mph.
  20995. What is the shortest time in which all three can complete the trip?
  20996.  
  20997. ==> physics/bicycle.s <==
  20998. First note that there's no apparent way to benefit from letting either the
  20999. boy or girl ride the bike longer than the other.  Any solution which gets the
  21000. boy there faster, must involve him using the bike (forward) more; similarly
  21001. for the girl.  Thus the bike must go backwards more for it to remain within
  21002. the 10-mile route.  Thus the dog won't make it there in time.  So the solution
  21003. assumes they ride the bike for the same amount of time.
  21004.  
  21005. Also note that there's no apparent way to benefit from letting any of the three
  21006. arrive at the finish ahead of the others.  If they do, they can probably take
  21007. time out to help the others.  So the solution assumes they all finish at the
  21008. same time.
  21009.  
  21010. The boy starts off on the bike, and travels 5.4 miles.  At this
  21011. point, he drops the bike and completes the rest of the trip on foot.  The
  21012. dog eventually reaches the bike, and takes it *backward* .8 miles (so the
  21013. girl gets to it sooner) and then returns to trotting.  Finally, the girl makes
  21014. it to the bike and rides it to the end.  The answer is 2.75 hours.
  21015.  
  21016. The puzzle is in Vasek Chvatal, Linear Programming, W. H. Freeman & Co.
  21017. The generalized problem (n people, 1 bike, different walking and riding speeds)
  21018. is known as "The Bicycle Problem".  A couple references are
  21019.  
  21020. Masuda, S. (1970). "The bicycle problem," University of California, Berkeley:
  21021.   Operations Research Center Technical Report ORC 70-35.
  21022.  
  21023. Chvatal, V. (1983). "On the bicycle problem," Discrete Applied Mathematics 5:
  21024.   pp. 165 - 173.
  21025.  
  21026. As for the linear program which gives the lower bound of 2.75 hours, let
  21027. t▌person, mode, direction¿ by the amount of time "person" (boy, girl or dog)
  21028. is travelling by "mode" (walk or bike) in "direction" (forward or backwards).
  21029. Define Time▌person¿ to be the total time spent by person doing each of these
  21030. four activities. The objective is to minimize the maximum of T▌person¿, for
  21031. person = boy, girl, dog, e.g.
  21032.  
  21033.     minimize T
  21034.     subject to  T >= T▌boy¿, T >= T▌girl¿, T >= T▌dog¿.
  21035.  
  21036. Now just think of all the other linear constraints on the variables t▌x,y,z¿,
  21037. such as everyone has to travel 10 miles, etc. In all, there are 8 contraints
  21038. in 18 variables (including slack variables). Solving this program yields the
  21039. lower bound.
  21040.  
  21041. ==> physics/boy.girl.dog.p <==
  21042. A boy, a girl and a dog are standing together on a long, straight road.
  21043. Simulataneously, they all start walking in the same direction:
  21044. The boy at 4 mph, the girl at 3 mph, and the dog trots back and forth
  21045. between them at 10 mph.  Assume all reversals of direction instantaneous.
  21046. In one hour, where is the dog and in which direction is he facing?
  21047.  
  21048. ==> physics/boy.girl.dog.s <==
  21049. The dog's position and direction are indeterminate, other than that the
  21050. dog must be between the boy and girl (endpoints included).  To see this,
  21051. simply time reverse the problem.  No matter where the dog starts out,
  21052. the three of them wind up together in one hour.
  21053.  
  21054. This argument is not quite adequate.  It is possible to construct problems
  21055. where the orientation changes an infinite number of times initially, but for
  21056. which there can be a definite result.  This would be the case if the positions
  21057. at time t are uniformly continuous in the positions at time s, s small.
  21058.  
  21059. But suppose that at time a the dog is with the girl.  Then the boy is at
  21060. 4a, and the time it takes the dog to reach the boy is a/6, because
  21061. the relative speed is 6 mph.  So the time b at which the dog reaches the
  21062. boy is proportional to a.  A similar argument shows that the time the
  21063. dog next reaches the girl is b + b/13, and is hence proportional to b.
  21064. This makes the position of the dog at time (t > a) a periodic function of
  21065. the logarithm of a, and thus does not approach a limit as a -> 0.
  21066.  
  21067. ==> physics/brick.p <==
  21068. What is the maximum overhang you can create with an infinite supply of bricks?
  21069.  
  21070. ==> physics/brick.s <==
  21071. You can create an infinite overhang.
  21072.  
  21073. Let us reverse the problem: how far can brick 1 be from brick 0?
  21074.  
  21075. Let us assume that the brick is of length 1.
  21076.  
  21077. To determine the place of the center of mass a(n):
  21078. a(1)=1/2
  21079. a(n)=1/n▌(n-1)*a(n-1)+▌a(n-1)+1/2¿¿=a(n-1)+1/(2n)
  21080. Thus
  21081.       n   1        n  1
  21082. a(n)=Sum -- = 1/2 Sum - = 1/2 H(n)
  21083.      m=1 2m       m=1 m
  21084. Needless to say the limit for n->oo of half the Harmonic series is oo.
  21085.  
  21086. ==> physics/cannonball.p <==
  21087. A person in a boat drops a cannonball overboard; does the water level change?
  21088.  
  21089. ==> physics/cannonball.s <==
  21090. The cannonball in the boat displaces an amount of water equal to the MASS
  21091. of the cannonball.  The cannonball in the water displaces an amount of water
  21092. equal to the VOLUME of the cannonball.  Water is unable to support the
  21093. level of salinity it would take to make it as dense as a cannonball, so the
  21094. first amount is definitely more than the second amount, and the water level
  21095. drops.
  21096.  
  21097. ==> physics/dog.p <==
  21098. A body of soldiers form a 50m-by-50m square ABCD on the parade ground.
  21099. In a unit of time, they march forward 50m in formation to take up the
  21100. position DCEF. The army's mascot, a small dog, is standing next to its
  21101.                                        handler at location A. When the
  21102.           B----C----E                  soldiers start marching, the dog
  21103.           !    !    !   forward-->     begins to run around the moving
  21104.           A----D----F                  body in a clockwise direction,
  21105.                                        keeping as close to it as possible.
  21106. When one unit of time has elapsed, the dog has made one complete
  21107. circuit and has got back to its handler, who is now at location D. (We
  21108. can assume the dog runs at a constant speed and does not delay when
  21109. turning the corners.)
  21110.  
  21111. How far does the dog travel?
  21112.  
  21113. ==> physics/dog.s <==
  21114. Let L be the side of the square, 50m, and let D be the distance the
  21115. dog travels.
  21116.  
  21117. Let v1 be the soldiers' marching speed and v2 be the speed of the dog.
  21118. Then v1 = L / (1 time unit) and v2 = v1*D/L.
  21119.  
  21120. Let t1, t2, t3, t4 be the time the dog takes to traverse each side of
  21121. the square, in order.  Find t1 through t4 in terms of L and D and solve
  21122. t1+t2+t3+t4 = 1 time unit.
  21123.  
  21124. While the dog runs along the back edge of the square in time t1, the
  21125. soldiers advance a distance d=t1*v1, so the dog has to cover a distance
  21126. sqrt(L^2 + (t1*v1)^2), which takes a time t1=sqrt(L^2 + (t1*v1)^2)/v2.
  21127. Solving for t1 gives t1=L/sqrt(v2^2 - v1^2).
  21128.  
  21129. The rest of the times are t2 = L/(v2-v1), t3 = t1, and t4 = L/(v2+v1).
  21130.  
  21131. In t1+t2+t3+t4, eliminate v2 by using v2=v1*D/L and eliminate v1 by
  21132. using v1=L/(1 time unit), obtaining
  21133.  
  21134.             2 L (D + sqrt(D^2-L^2)) / (D^2 - L^2) = 1
  21135.  
  21136. which can be turned into
  21137.  
  21138.             D^4 - 4LD^3 - 2L^2D^2 + 4L^3D + 5L^4 = 0
  21139.  
  21140. which has a root D = 4.18113L = 209.056m.
  21141.  
  21142. ==> physics/magnets.p <==
  21143. You have two bars of iron.  One is magnetic, the other is not.  Without
  21144. using any other instrument (thread, filings, other magnets, etc.), find
  21145. out which is which.
  21146.  
  21147. ==> physics/magnets.s <==
  21148. Take the two bars, and put them together like a T, so that one bisects the
  21149. other.
  21150.                        ___________________
  21151.           bar A --->  !___________________!
  21152.                                ! !
  21153.                                ! !
  21154.                                ! !
  21155.                                ! !
  21156.           bar B ------------>  ! !
  21157.                                ! !
  21158.                                ! !
  21159.                                !_!
  21160.  
  21161. If they stick together, then bar B is the magnet.  If they don't, bar A is
  21162. the magnet. (reasoning follows)
  21163.  
  21164. Bar magnets are "dead" in their centers (ie there is no magnetic force,
  21165. since the two poles cance out).  So, if bar A is the magnet, then bar B
  21166. won't stick to its center.
  21167.  
  21168. However, bar magnets are quite "alive" at their edges (ie the magnetic
  21169. force is concentrated).  So, if bar B is the magnet, then bar A will stick
  21170. nicely to its end.
  21171.  
  21172. ==> physics/milk.and.coffee.p <==
  21173. You are just served a hot cup of coffee and want it to be as hot as possible
  21174. when you drink it some number of minutes later.  Do you add milk when you get
  21175. the cup or just before you drink it?
  21176.  
  21177. ==> physics/milk.and.coffee.s <==
  21178. Normalize your temperature scale so that 0 degrees = room temperature.
  21179.  
  21180. Assume that the coffee cools at a rate proportional to the difference
  21181. in temperature, and that the amount of milk is sufficiently small that
  21182. the constant of proportinality is not changed when you add the milk.
  21183.  
  21184. An early calculus homework problem is to compute that the temperature
  21185. of the coffee decays exponentially with time,
  21186.  
  21187. T(t) = exp(-ct) T0,   where T0 = temperature at t=0.
  21188.  
  21189. Let l = exp(-ct), where t is the duration of the experiment.
  21190.  
  21191. Assume that the difference in specific heats of coffee and milk are
  21192. negligible, so that if you add milk at temperature M to coffee at
  21193. temperature C, you get a mix of temperature aM+bC, where a and b
  21194. are constants between 0 and 1, with a+b=1.  (Namely, a = the fraction
  21195. of final volume that is milk, and b = fraction that is coffee.)
  21196.  
  21197. If we let C denote the original coffee temperature and M the milk
  21198. temperature, we see that
  21199.  
  21200. Add milk later: aM + blC
  21201. Add milk now:   l(aM+bC) = laM+blC
  21202.  
  21203. The difference is d=(1-l)aM.  Since l<1 and a>0, we need to worry about
  21204. whether M is positive or not.
  21205.  
  21206. M>0: Warm milk.  So d>0, and adding milk later is better.
  21207. M=0: Room temp.  So d=0, and it doesn't matter.
  21208. M<0: Cold milk.  So d<0, and adding milk now is better.
  21209.  
  21210. Of course, if you wanted to be intuitive, the answer is obvious if you
  21211. assume the coffee is already at room temperature and the milk is
  21212. either scalding hot or subfreezing cold.
  21213.  
  21214. Moral of the story:  Always think of extreme cases when doing these puzzles.
  21215. They are usually the key.
  21216.  
  21217. Oh, by the way, if we are allowed to let the milk stand at room
  21218. temperature, then let r = the corresponding exponential decay constant
  21219. for your milk container.
  21220.  
  21221. Add acclimated milk later:  arM + blC
  21222.  
  21223. We now have lots of cases, depending on whether
  21224.  
  21225. r<l:  The milk pot is larger than your coffee cup.
  21226.       (E.g, it really is a pot.)
  21227. r>l:  The milk pot is smaller than your coffee cup.
  21228.       (E.g., it's one of those tiny single-serving things.)
  21229. M>0:  The milk is warm.
  21230. M<0:  The milk is cold.
  21231.  
  21232. Leaving out the analysis, I compute that you should...
  21233.  
  21234. Add warm milk in large pots LATER.
  21235. Add warm milk in small pots NOW.
  21236. Add cold milk in large pots NOW.
  21237. Add cold milk in small pots LATER.
  21238.  
  21239. Of course, observe that the above summary holds for the case where the
  21240. milk pot is allowed to acclimate; just treat the pot as of infinite
  21241. size.
  21242.  
  21243. ==> physics/mirror.p <==
  21244. Why does a mirror appear to invert the left-right directions, but not up-down?
  21245.  
  21246. ==> physics/mirror.s <==
  21247. Mirrors invert front to back, not left to right.
  21248.  
  21249. The popular misconception of the inversion is caused by the fact that
  21250. a person when looking at another person expects him/her to face her/him,
  21251. so with the left-hand side to the right. When facing oneself (in the
  21252. mirror) one sees an 'uninverted' person.
  21253.  
  21254. See Martin Gardner, ``Hexaflexagons and other mathematical
  21255. diversions,'' University of Chicago Press 1988, Chapter 16.  A letter
  21256. by R.D. Tschigi and J.L. Taylor published in this book states that the
  21257. fundamental reason is: ``Human beings are superficially and grossly
  21258. bilaterally symmetrical, but subjectively and behaviorally they are
  21259. relatively asymmetrical. The very fact that we can distinguish our
  21260. right from our left side implies an asymettry of the perceiving
  21261. system, as noted by Ernst Mach in 1900. We are thus, to a certain
  21262. extent, an asymmetrical mind dwelling in a bilaterally symmetrical
  21263. body, at least with respect to a casual visual inspection of our
  21264. external form.''
  21265.  
  21266. Martin Gardner has also written the book ``The Amidextrous Universe.''
  21267.  
  21268. ==> physics/monkey.p <==
  21269. Hanging over a pulley, there is a rope, with a weight at one end.
  21270. At the other end hangs a monkey of equal weight.  The rope weighs
  21271. 4 ounces per foot.  The combined ages of the monkey and it's mother
  21272. is 4 years.  The weight of the monkey is as many pounds as the mother
  21273. is years old.  The mother is twice as old as the monkey was when the
  21274. mother was half as old as the monkey will be when the monkey is 3 times
  21275. as old as the mother was when she was 3 times as old as the monkey.
  21276.  
  21277. The weight of the rope and the weight is one-half as much again as the
  21278. difference between the weight of the weight and the weight of the weight
  21279. plus the weight of the monkey.
  21280.  
  21281. How long is the rope?
  21282.  
  21283. ==> physics/monkey.s <==
  21284. The most difficult thing about this puzzle, as you probably expected,
  21285. is translating all the convoluted problem statements into equations...
  21286. the solution is pretty trivial after that.  So...
  21287.  
  21288. Let:
  21289.         m represent monkey
  21290.         M represent mother of monkey
  21291.         w represent weight
  21292.         r represent rope
  21293.  
  21294.         W▌x¿ = present weight of x (x is m, M, w, or r)
  21295.         A▌x,t¿ = age of x at time t (x is M or M, t is one of T1 thru T4)
  21296.  
  21297.         T1 = time at which mother is 3 times as old as monkey
  21298.         T2 = time at which monkey is 3 times as old as mother at T1
  21299.         T3 = time at which mother is half as old as monkey at T2
  21300.         T4 = present time
  21301.  
  21302. For the ages, we have:
  21303.  
  21304.         A▌M,T1¿ = 3*A▌m,T1¿
  21305.         A▌m,T2¿ = 3*A▌M,T1¿ = 9*A▌m,T1¿
  21306.         A▌M,T3¿ = A▌m,T2¿/2 = 9*A▌m,T1¿/2
  21307.         A▌m,T3¿ = A▌m,T1¿ + (T3-T1)
  21308.                 = A▌m,T1¿ + (A▌M,T3¿-A▌M,T1¿)
  21309.                 = A▌m,T1¿ + (9*A▌M,T1¿/2 - 3*A▌m,T1¿)
  21310.                 = 5*A▌m,T1¿/2
  21311.         A▌M,T4¿ = 2*A▌m,T3¿ = 5*A▌m,T1¿
  21312.         A▌m,T4¿ = A▌m,T1¿ + (T4-T1)
  21313.                 = A▌m,T1¿ + (A▌M,T4¿-A▌M,T1¿)
  21314.                 = A▌m,T1¿ + (5*A▌m,T1¿ - 3*A▌m,T1¿)
  21315.                 = 3*A▌m,T1¿
  21316.  
  21317. The present ages of monkey and mother sum to 4, so we have
  21318.  
  21319.         A▌m,T4¿   + A▌M,T4¿   = 4
  21320.         3*A▌m,T1¿ + 5*A▌m,T1¿ = 4
  21321.                     8*A▌m,T1¿ = 4
  21322.                       A▌m,T1¿ = 1/2
  21323.  
  21324. Thus:
  21325.         A▌M,T4¿ = 5/2
  21326.         A▌m,T4¿ = 3/2
  21327.  
  21328. Now for the weights, translating everything to ounces:
  21329.  
  21330. Monkey's weight in lbs = mother's age in years, so:
  21331.  
  21332.         W▌m¿ = 16*5/2 = 40
  21333.  
  21334. Weight and monkey are same weight, so:
  21335.  
  21336.         W▌w¿ = W▌m¿ = 40
  21337.  
  21338. The last paragraph in the problem translates into:
  21339.  
  21340.         W▌r¿+W▌w¿ = (3/2)*((W▌w¿+W▌m¿)-W▌w¿)
  21341.         W▌r¿+ 40  = (3/2)*(( 40 + 40 )- 40 )
  21342.         W▌r¿+ 40  = 60
  21343.         W▌r¿      = 20
  21344.  
  21345. The rope weighs 4 ounces per foot, so its length is 5 feet.
  21346.  
  21347. ==> physics/particle.p <==
  21348. What is the longest time that a particle can take in travelling between two
  21349. points if it never increases its acceleration along the way and reaches the
  21350. second point with speed V?
  21351.  
  21352. ==> physics/particle.s <==
  21353. Assumptions:
  21354.  
  21355. 1. x(0) = 0; x(T) = X
  21356.  
  21357. 2. v(0) = 0; v(T) = V
  21358.  
  21359. 3. d(a)/dt <= 0
  21360.  
  21361. Solution:
  21362.  
  21363. a(t) = constant = A = V^2/2X which implies T = 2X/V.
  21364.  
  21365. Proof:
  21366.  
  21367. Consider assumptions as they apply to f(t) = A * t - v(t):
  21368.  
  21369. 1. integral from 0 to T of f = 0
  21370.  
  21371. 2. f(0) = f(T) = 0
  21372.  
  21373. 3. d^2(f)/dt^2 <= 0
  21374.  
  21375. From the mean value theorem, f(t) = 0.  QED.
  21376.  
  21377. ==> physics/pole.in.barn.p <==
  21378. Accelerate a pole of length l to a constant speed of 90% of the speed of
  21379. light (.9c).  Move this pole towards an open barn of length .9l (90%
  21380. the length of the pole).  Then, as soon as the pole is fully inside the
  21381. barn, close the door.  What do you see and what actually happens?
  21382.  
  21383. ==> physics/pole.in.barn.s <==
  21384. What the observer sees depends upon where the observer is, due to
  21385. the finite speed of light.
  21386.  
  21387. For definiteness, assume the forward end of the pole is marked "A" and
  21388. the after end is marked "B".  Let's also assume there is a light source
  21389. inside the barn, and that the pole stops moving as soon as end "B" is
  21390. inside the barn.
  21391.  
  21392. An observer inside the barn next to the door will see the following
  21393. sequence of events:
  21394.  
  21395.         1.  End "A" enters the barn and continues toward the back.
  21396.         2.  End "B" enters the barn and stops in front of the observer.
  21397.         3.  The door closes.
  21398.         4.  End "A" continues moving and penetrates the barn at the far end.
  21399.         5.  End "A" stops outside the barn.
  21400.  
  21401. An observer at the other end of the barn will see:
  21402.  
  21403.         1.  End "A" enters the barn.
  21404.         2.  End "A" passes the observer and penetrates the back of the barn.
  21405.         3.  If the pole has markings on it, the observer will notice the part
  21406.             nearest him has stopped moving.  However, both ends are still
  21407.             moving.
  21408.         4.  End "A" stops moving outside the barn.
  21409.         5.  End "B" continues moving until it enters the barn and then stops.
  21410.         6.  The door closes.
  21411.  
  21412. After the observers have subtracted out the effects of the finite speed
  21413. of light on what they see, both observers will agree on what happened:
  21414. The pole entered the barn; the door closed so that the pole was
  21415. completely contained within the barn; as the pole was being stopped it
  21416. elongated and penetrated the back wall of the barn.
  21417.  
  21418. Things are different if you are riding along with the pole.  The pole
  21419. is never inside the barn since it won't fit.  End A of the pole penetrates
  21420. the rear wall of the barn before the door is closed.
  21421.  
  21422. If the wall of the barn is impenetrable, in all the above scenarios insert
  21423. the wording "End A of the pole explodes" for "End A penetrates the barn."
  21424.  
  21425. ==> physics/resistors.p <==
  21426. What are the resistances between lattices of resistors in the shape of a:
  21427.  
  21428. 1. Cube
  21429.  
  21430. 2. Platonic solid
  21431.  
  21432. 3. Hypercube
  21433.  
  21434. 4. Plane sheet
  21435.  
  21436. 5. Continuous sheet
  21437.  
  21438. ==> physics/resistors.s <==
  21439. 1. Cube
  21440.  
  21441. The key idea is to observe that if you can show that two
  21442. points in a circuit must be at the same potential, then you can
  21443. connect them, and no current will flow through the connection and the
  21444. overall properties of the circuit remain unchanged.  In particular, for
  21445. the cube, there are three resistors leaving the two "connection
  21446. corners".  Since the cube is completely symmetrical with respect to the
  21447. three resistors, the far sides of the resistors may be connected
  21448. together.  And so we end up with:
  21449.  
  21450.         !---WWWWWW---!    !---WWWWWW---!
  21451.         !            !    !            !
  21452.      *--+---WWWWWW---+----+---WWWWWW---+---*
  21453.         !            !    !            !
  21454.         !---WWWWWW---!    !---WWWWWW---!
  21455.  
  21456. 2. Platonic Solids
  21457.  
  21458. Same idea for 8 12 and 20, since you use the symmetry to identify
  21459. equi-potential points.  The tetrahedron is hair more subtle:
  21460.  
  21461.     *---!---WWWWWW---!---*
  21462.         !\          /!
  21463.         W W        W W
  21464.         W  W      W  W
  21465.         W   W    W   W
  21466.         !    \  /    !
  21467.          \    !!     !
  21468.           \    !    /
  21469.            \   W   /
  21470.             \  W  /   <-------
  21471.              \ W /
  21472.               \!/
  21473.                +
  21474.  
  21475. By symmetry, the endpoints of the marked resistor are equi-potential.  Hence
  21476. they can be connected together, and so it becomes a simple:
  21477.  
  21478.     *---+---WWWWW---+----*
  21479.         !           !
  21480.         +-WWW   WWW-+
  21481.         !    !-!    !
  21482.         !-WWW   WWW-!
  21483.  
  21484. 3. Hypercube
  21485.  
  21486. Think of injecting a constant current I into the start vertex.
  21487. It splits (by symmetry) into n equal currents in the n arms; the current of
  21488. I/n then splits into I/n(n-1), which then splits into I/▌n(n-1)(n-1)¿ and so
  21489. on till the halfway point, when these currents start adding up. What is the
  21490. voltage difference between the antipodal points? V = I x R; add up the voltages
  21491. along any of the paths:
  21492. n even:                                                         (n-2)/2
  21493.      V = 2{I/n + I/(n(n-1)) + I/(n(n-1)(n-1)) + ... +  I/(n(n-1)       )}
  21494.  
  21495. n odd:                                                      (n-3)/2
  21496.  V = 2{I/n + I/(n(n-1)) + I/(n(n-1)(n-1)) + ... +  I/(n(n-1)       )}    (n-1)/2
  21497.                                                                + I/(n(n-1)     )
  21498. And R = V/I i.e. replace the Is in the above expression by 1s.
  21499.  
  21500. For the 3-cube: R = 2{1/3} + 1/(3x2) = 5/6 ohm
  21501. For the 4-cube: R = 2{1/4 + 1/(4x3)} = 2/3 ohm
  21502.  
  21503. This formula yields the resistance from root to root of
  21504. two (n-1)-ary trees of height n/2 with their end nodes identified
  21505. (-when n is even; something similar when n is odd).
  21506. Coincidentally, the 4-cube is such an animal and thus the answer
  21507. 2/3 ohms is correct in that case.
  21508. However, it does not provide the solution for n >= 5, as the hypercube
  21509. does not have quite as many edges as were counted in the formula above.
  21510.  
  21511. 4. The Infinite Plane
  21512.  
  21513. For an infinite lattice: First inject a constant current I at a point; figure
  21514. out the current flows (with heavy use of symmetry). Remove that current. Draw
  21515. out a current I from the other point of interest (or inject a negative current)
  21516. and figure out the flows (identical to earlier case, but displaced and in the
  21517. other direction). By the principle of superposition, if you inject a current I
  21518. into point a and take out a current I at point b at the same time, the currents
  21519. in the paths are simply the sum of the currents obtained in the earlier two
  21520. simpler cases. As in the n-cube, find the voltage between the points of
  21521. interest, divide by I and voila']
  21522.  
  21523. As an illustration, in the adjacent points case: we have a current of I/4 in
  21524. each of the four resistors:
  21525.  
  21526.     ^                !
  21527.     !                v
  21528.  <--o-->          -->o<--
  21529.     !                ^
  21530.     v                !
  21531.  (inject)          (take out)
  21532. And adding the currents, we have I/2 in the resistor connecting the two points.
  21533. Therefore V=(1 ohm) x I/2 and effective resistance between the points = 1/2 ohm.
  21534.  
  21535. You can (and showed how to) use symmetry to obtain the equivalent resistance
  21536. of 1/2 between two adjacent nodes; but I doubt that symmetry alone will give you
  21537. even the equivalent resistance of 2/pi between two diagonally adjacent nodes.
  21538. ▌More generally, the equivalent resistance between two nodes k diagonal units
  21539. apart is (2/pi)(1+1/3+1/5+...+1/(2k-1)); that, plus symmetry and the known
  21540. equivalent resistance between two adjacent nodes, is sufficient to derive all
  21541. equivalent resistances in the lattice.
  21542.  
  21543. 5. Continuous sheet
  21544.  
  21545. Doesn't the resistance diverge in that case?  The problem is that you can't
  21546. inject current at a point.
  21547.  
  21548. cf. "Random Walks and Electric Networks", by Doyle and Snell, published by the
  21549. Mathematical Association of America.
  21550.  
  21551. ==> physics/sail.p <==
  21552. A sailor is in a sailboat on a river.  The water (current) is flowing
  21553. downriver at a velocity of 3 knots with respect to the land.  The wind
  21554. (air velocity) is zero, with respect to the land.  The sailor wants
  21555. to proceed downriver as quickly as possible, maximizing his downstream
  21556. speed with respect to the land.
  21557.  
  21558. Should he raise the sail, or not?
  21559.  
  21560. ==> physics/sail.s <==
  21561. Depends on the sail.  If the boat is square-rigged, then not, since
  21562. raising the sail will simply increase the air resistance.
  21563.  
  21564. If the sailor has a fore-and-aft rig, then he should, since he can then
  21565. tack into the wind.  (Imagine the boat in still water with a 3-knot head
  21566. wind).
  21567.  
  21568. ==> physics/skid.p <==
  21569. What is the fastest way to make a 90 degree turn on a slippery road?
  21570.  
  21571. ==> physics/skid.s <==
  21572. For higher speeds (measured at a small distance from the point of initiation
  21573. of a sharp turn) the fastest way round is to "outside loop" - that is, steer
  21574. away from the curve, and do a kidding 270.
  21575.  
  21576. This technique is taught in advanced driving schools.
  21577.  
  21578. References:
  21579.  
  21580. M. Freeman and P. Palffy, American Journal of Physics, vol 50, p. 1098, 1982.
  21581. P. Palffy and Unruh, American Journal of Physics, vol 49, p. 685, 1981.
  21582.  
  21583. ==> physics/spheres.p <==
  21584. Two spheres are the same size and weight, but one is hollow.  They are
  21585. made of uniform material, though of course not the same material.  Without
  21586. a minimum of apparatus, how can I tell which is hollow?
  21587.  
  21588. ==> physics/spheres.s <==
  21589. Since the balls have equal diameter and equal mass, their volume and
  21590. density are also equal.  However, the mass distribution is not equal,
  21591. so they will have different moments of inertia - the hollow sphere has
  21592. its mass concentrated at the outer edge, so its moment of inertia will
  21593. be greater than the solid sphere.  Applying a known torque and observing
  21594. which sphere has the largest angular acceleration will determine which
  21595. is which.  An easy way to do this is to "race" the spheres down an
  21596. inclined plane with enough friction to prevent the spheres from sliding.
  21597. Then, by conservation of energy:
  21598.  
  21599.          mgh = 1/2 mv^2 + 1/2 Iw^2
  21600.  
  21601. Since the spheres are rolling without sliding, there is a relationship
  21602. between velocity and angular velocity:
  21603.  
  21604.         w = v / r
  21605.  
  21606. so
  21607.  
  21608.          mgh = 1/2 mv^2 + 1/2 I (v^2 / r^2) = 1/2 (m + I/r^2) v^2
  21609.  
  21610. and
  21611.  
  21612.          v^2 = 2mgh / (m + I / r^2)
  21613.  
  21614. From this we can see that the sphere with larger moment of inertia (I) will
  21615. have a smaller velocity when rolled from the same height, if mass and radius
  21616. are equal with the other sphere.  Thus the solid sphere will roll faster.
  21617.  
  21618. ==> physics/wind.p <==
  21619. Is a round-trip by airplane longer or shorter if there is wind blowing?
  21620.  
  21621. ==> physics/wind.s <==
  21622. It will take longer, by the ratio (s^2)/(s^2 - w^2) where s is
  21623. the plane's speed, and w is the wind speed.  The stronger the wind the
  21624. longer it will take, up until the wind speed equals the planes speed, at
  21625. which point the plane will never finish the trip.
  21626.  
  21627. Math:
  21628.         s = plane's speed
  21629.         w = wind speed
  21630.         d = distance in one direction
  21631.  
  21632.         d / (s + w)     = time to complete leg flying with the wind
  21633.         d / (s - w)     = time to complete leg flying against the wind
  21634.         d / (s + w) + d / (s - w)       = round trip time
  21635.  
  21636.         d / (s + w) + d / (s - w)       = ratio of flying with wind to
  21637.         -------------------------         flying with no wind (bottom of
  21638.            d / s + d / s                  equation is top with w = 0)
  21639.  
  21640.         this simplifies to s^2 / (s^2 - w^2).
  21641.  
  21642. ==> probability/amoeba.p <==
  21643. A jar begins with one amoeba.  Every minute, every amoeba
  21644. turns into 0, 1, 2, or 3 amoebae with probability 25%
  21645. for each case ( dies, does nothing, splits into 2, or splits
  21646. into 3).  What is the probability that the amoeba population
  21647. eventually dies out?
  21648.  
  21649. ==> probability/amoeba.s <==
  21650. If p is the probability that a single amoeba's descendants will die
  21651. out eventually, the probability that N amoebas' descendents will all
  21652. die out eventually must be p^N, since each amoeba is independent of
  21653. every other amoeba.  Also, the probability that a single amoeba's
  21654. descendants will die out must be independent of time when averaged
  21655. over all the possibilities.  At t=0, the probability is p, at t=1 the
  21656. probability is 0.25(p^0+p^1+p^2+p^3), and these probabilities must be
  21657. equal.  Extinction probability p is a root of f(p)=p.  In this case,
  21658.  p = sqrt(2)-1.
  21659.  
  21660. The generating function for the sequence P(n,i), which gives the
  21661. probability of i amoebas after n minutes, is f^n(x), where f^n(x) ==
  21662. f^(n-1) ( f(x) ), f^0(x) == x .  That is, f^n is the nth composition
  21663. of f with itself.
  21664.  
  21665. Then f^n(0) gives the probability of 0 amoebas after n minutes, since
  21666. f^n(0) = P(n,0). We then note that:
  21667.  
  21668.         f^(n+1)(x) = ( 1 + f^n(x) + (f^n(x))^2 + (f^n(x))^3 )/4
  21669.  
  21670. so that if f^(n+1)(0) -> f^n(0) we can solve the equation.
  21671.  
  21672. The generating function also gives an expression for the expectation
  21673. value of the number of amoebas after n minutes. This is d/dx(f^n(x))
  21674. evaluated at x=1. Using the chain rule we get f'(f^(n-1)(x))*d/dx(f^(n-1)(x))
  21675. and since f'(1) = 1.5  and f(1) = 1, we see that the result is just
  21676. 1.5^n, as might be expected.
  21677.  
  21678. ==> probability/apriori.p <==
  21679. An urn contains one hundred white and black balls.  You sample one hundred
  21680. balls with replacement and they are all white.  What is the probability
  21681. that all the balls are white?
  21682.  
  21683. ==> probability/apriori.s <==
  21684. This question cannot be answered with the information given.
  21685.  
  21686. In general, the following formula gives the conditional probability
  21687. that all the balls are white given you have sampled one hundred balls
  21688. and they are all white:
  21689.  
  21690.         P(100 white ! 100 white samples) =
  21691.  
  21692.                         P(100 white samples ! 100 white) * P(100 white)
  21693.                 -----------------------------------------------------------
  21694.                 sum(i=0 to 100) P(100 white samples ! i white) * P(i white)
  21695.  
  21696. The probabilities P(i white) are needed to compute this formula.  This
  21697. does not seem helpful, since one of these (P(100 white)) is just what we
  21698. are trying to compute.  However, the following argument can be made:
  21699. Before the experiment, all possible numbers of white balls from zero to
  21700. one hundred are equally likely, so P(i white) = 1/101.  Therefore, the
  21701. odds that all 100 balls are white given 100 white samples is:
  21702.  
  21703.         P(100 white ! 100 white samples) =
  21704.  
  21705.                 1 / ( sum(i=0 to 100) (i/100)^100 ) =
  21706.  
  21707.                 63.6%
  21708.  
  21709. This argument is fallacious, however, since we cannot assume that the urn
  21710. was prepared so that all possible numbers of white balls from zero to one
  21711. hundred are equally likely.  In general, we need to know the P(i white)
  21712. in order to calculate the P(100 white ! 100 white samples).  Without this
  21713. information, we cannot determine the answer.
  21714.  
  21715. This leads to a general "problem": our judgments about the relative
  21716. likelihood of things is based on past experience.  Each experience allows
  21717. us to adjust our likelihood judgment, based on prior probabilities.  This
  21718. is called Bayesian inference.  However, if the prior probabilities are not
  21719. known, then neither are the derived probabilities.  But how are the prior
  21720. probabilities determined?  For example, if we are brains in the vat of a
  21721. diabolical scientist, all of our prior experiences are illusions, and
  21722. therefore all of our prior probabilities are wrong.
  21723.  
  21724. All of our probability judgments indeed depend upon the assumption that
  21725. we are not brains in a vat.  If this assumption is wrong, all bets are
  21726. off.
  21727.  
  21728. ==> probability/cab.p <==
  21729. A cab was involved in a hit and run accident at night.  Two cab companies,
  21730. the Green and the Blue, operate in the city.  Here is some data:
  21731.  
  21732.         a)  Although the two companies are equal in size, 85% of cab
  21733. accidents in the city involve Green cabs and 15% involve Blue cabs.
  21734.  
  21735.         b)  A witness identified the cab in this particular accident as Blue.
  21736. The court tested the reliability of the witness under the same circumstances
  21737. that existed on the night of the accident and concluded that the witness
  21738. correctly identified each one of the two colors 80% of the time and failed
  21739. 20% of the time.
  21740.  
  21741. What is the probability that the cab involved in the accident was
  21742. Blue rather than Green?
  21743.  
  21744. If it looks like an obvious problem in statistics, then consider the
  21745. following argument:
  21746.  
  21747. The probability that the color of the cab was Blue is 80%]  After all,
  21748. the witness is correct 80% of the time, and this time he said it was Blue]
  21749.  
  21750. What else need be considered?  Nothing, right?
  21751.  
  21752. If we look at Bayes theorem (pretty basic statistical theorem) we
  21753. should get a much lower probability.  But why should we consider statistical
  21754. theorems when the problem appears so clear cut?  Should we just accept the
  21755. 80% figure as correct?
  21756.  
  21757. ==> probability/cab.s <==
  21758. The police tests don't apply directly, because according to the
  21759. wording, the witness, given any mix of cabs, would get the right
  21760. answer 80% of the time.  Thus given a mix of 85% green and 15% blue
  21761. cabs, he will say 20% of the green cabs and 80% of the blue cabs are
  21762. blue.  That's 20% of 85% plus 80% of 15%, or 17%+12% = 29% of all the
  21763. cabs that the witness will say are blue.  Of those, only 12/29 are
  21764. actually blue.  Thus P(cab is blue ! witness claims blue) = 12/29.
  21765. That's just a little over 40%.
  21766.  
  21767. Think of it this way... suppose you had a robot watching parts on a
  21768. conveyor belt to spot defective parts, and suppose the robot made a
  21769. correct determination only 50% of the time (I know, you should
  21770. probably get rid of the robot...).  If one out of a billion parts are
  21771. defective, then to a very good approximation you'd expect half your
  21772. parts to be rejected by the robot.  That's 500 million per billion.
  21773. But you wouldn't expect more than one of those to be genuinely
  21774. defective.  So given the mix of parts, a lot more than 50% of the
  21775. REJECTED parts will be rejected by mistake (even though 50% of ALL the
  21776. parts are correctly identified, and in particular, 50% of the
  21777. defective parts are rejected).
  21778.  
  21779. When the biases get so enormous, things starts getting quite a bit
  21780. more in line with intuition.
  21781.  
  21782. For a related real-life example of probability in the courtroom see
  21783. People v. Collins, 68 Cal 2d319 (1968).
  21784.  
  21785. ==> probability/coincidence.p <==
  21786. Name some amazing coincidences.
  21787.  
  21788. ==> probability/coincidence.s <==
  21789. The answer to the question, "Who wrote the Bible," is, of
  21790. course, Shakespeare. The King James Version was published in
  21791. 1611. Shakespeare was 46 years old then (he turned 47 later in
  21792. the year). Look up Psalm 46. Count 46 words from the beginning of
  21793. the Psalm. You will find the word "Shake." Count 46 words from
  21794. the end of the Psalm. You will find the word "Spear." An obvious
  21795. coded message. QED.
  21796.  
  21797. How many inches in the pole-to-pole diameter of the Earth?  The
  21798. answer is almost exactly 500,000,000 inches.  Proof that the inch
  21799. was defined by spacemen.
  21800.  
  21801.  
  21802. ==> probability/coupon.p <==
  21803. There is a free gift in my breakfast cereal. The manufacturers say
  21804. that the gift comes in four different colours, and encourage one to
  21805. collect all four (& so eat lots of their cereal). Assuming there is
  21806. an equal chance of getting any one of the colours,  what is the
  21807. expected number of packets I must plough through to get all four?
  21808. Can you generalise to n colours and/or unequal probabilities?
  21809.  
  21810. ==> probability/coupon.s <==
  21811. The problem is well known under the name of "COUPON COLLECTOR PROBLEM".
  21812. The answer for n equally likely coupons is
  21813. (1)             C(n)=n*H(n)    with H(n)=1+1/2+1/3+...+1/n.
  21814. In the unequal probabilities case, with p_i the probability of coupon i,
  21815. it becomes
  21816. (2)             C(n)=int_0^infty ▌1-prod_{i=1}^n (1-exp(-p_i*t))¿ dt
  21817. which reduces to (1) when p_i=1/n (An easy exercise).
  21818. In the equal probabilities case  C(n)~n log(n). For a Zipf law,
  21819. from (2), we have C(n)~n log^2(n).
  21820.  
  21821. A related problem is the "BIRTHDAY PARADOX" familiar to people
  21822. interested in hashing algorithms: With a party of 24 persons,
  21823. you are likely (i.e. with probability >50%) to find two with
  21824. the same birthday. The non equiprobable case was solved by:
  21825.         M. Klamkin and D. Newman, Extensions of the birthday
  21826.         surprise, J. Comb. Th. 3 (1967), 279-282.
  21827.  
  21828. ==> probability/darts.p <==
  21829. Peter throws two darts at a dartboard, aiming for the center.  The
  21830. second dart lands farther from the center than the first.  If Peter now
  21831. throws another dart at the board, aiming for the center, what is the
  21832. probability that this third throw is also worse (i.e., farther from
  21833. the center) than his first?  Assume Peter's skilfulness is constant.
  21834.  
  21835. ==> probability/darts.s <==
  21836. Since the three darts are thrown independently,
  21837. they each have a 1/3 chance of being the best throw.  As long as the
  21838. third dart is not the best throw, it will be worse than the first dart.
  21839. Therefore the answer is 2/3.
  21840.  
  21841. Ranking the three darts' results from A (best) to C
  21842. (worst), there are, a priori, six equiprobable outcomes.
  21843.  
  21844. possibility #   1       2       3       4       5       6
  21845. 1st throw       A       A       B       B       C       C
  21846. 2nd throw       B       C       A       C       A       B
  21847. 3rd throw       C       B       C       A       B       A
  21848.  
  21849. The information from the first two throws shows us that the first
  21850. throw will not be the worst, nor the second throw the best.  Thus
  21851. possibilities 3, 5 and 6 are eliminated, leaving three equiprobable
  21852. cases, 1, 2 and 4.  Of these, 1 and 2 have the third throw worse than
  21853. the first; 4 does not.  Again the answer is 2/3.
  21854.  
  21855. ==> probability/flips.p <==
  21856. Consider a run of coin tosses: HHTHTTHTTTHTTTTHHHTHHHHHTHTTHT
  21857.  
  21858. Define a success as a run of one H or T (as in THT or HTH).  Use two
  21859. different methods of sampling.  The first method would consist of
  21860. sampling the above data by taking 7 groups of three.  This translates
  21861. into the following sequences HHT, HTT, HTT, THT, TTT, HHH, and THH.
  21862. In this sample there was one success, THT.  The second method of
  21863. sampling could be gotten by taking the groups in a serial sequence.
  21864. Another way of explaining the method would be to take the tosses 1, 2,
  21865. and 3 as the first set then tosses 2, 3, and 4 as the second set and
  21866. so on to produce seven samples.  The samples would be HHT, HTH, THT,
  21867. HTT TTH, THT, and HTT, thus giving two success, HTH and THT.  With
  21868. these two methods what would be the expected value and the standard
  21869. deviation for both methods?
  21870.  
  21871. ==> probability/flips.s <==
  21872. Case 1:
  21873.  
  21874. Let us start with a simple case: Define success as T and consider
  21875. sequences of length 1.  In this case, the two sampling techniques are
  21876. equivalent.  Assuming that we are examining a truly random sequence of
  21877. T and H.  Thus if n groups of single sequences are considered or a
  21878. sequence of length n is considered we will have the following
  21879. statistics on the number of successes:
  21880.  
  21881.         Mean = n/2,  and  standard deviation (sd) = square_root(n)/2
  21882.  
  21883.  
  21884. Case 2:
  21885.  
  21886. Define success as HT or TH: Here the two sampling techniques need to
  21887. be distinguished:
  21888.  
  21889. Sampling 1:  Take "n" groups of two:  Here probability of getting success in
  21890. any group is 1/2 (TH and HT out of 4 possible cases). So the mean and the
  21891. standard deviation is same as described in case 1.
  21892.  
  21893. Sampling 2: Generate a sequence of length "n".  It is easy to show
  21894. that (n-1) samples are generated.  The number of successes in this
  21895. sequence is same as the number of T to H and H to T transitions.  This
  21896. problem is solved in John P. Robinson, Transition Count and Syndrome
  21897. are Uncorrelated, IEEE Transactions on Information Theory, Jan 1988.
  21898. I will just state the results:
  21899.  
  21900.   mean = (n-1)/2  and standard deviation = square_root(n-1)/2.
  21901.  
  21902. In fact, if you want "n" samples in this case you need to generate
  21903. length (n+1) sequence.  Then the new mean and standard deviation are
  21904. the same as described in Sampling 1. (replace n by n+1).  The
  21905. advantage in this sampling (i.e., sampling 2) is that you need only
  21906. generate a sequence length of (n+1) to obtain n sample points as
  21907. opposed to 2n (n groups of 2) in Sampling 1.
  21908.  
  21909. OBSERVATION:  The statistics has not changed.
  21910.  
  21911.  
  21912. Case 3:
  21913.  
  21914. Define success as THT and HTH.
  21915.  
  21916. Sampling 1: This is a simple situation.  Let us assume "n" samples
  21917. need to be generated.  Therefore, "n" groups of three are generated.
  21918. The probability of a group being successful is 1/4 (THT and HTH out of
  21919. 8 cases).  Therefore mean and standard deviation are:
  21920.  
  21921.   mean= n/4 and sd= square_root(7n)/4
  21922.  
  21923. Sampling 2: This is not a simple situation.  Let us generate a random
  21924. sequence of length "n".  There will be (n-2) samples in this case.
  21925. Use an approach similar to that followed in the Jan 88 IEEE paper.  I
  21926. will just state the results.  First we define a function or enumerator
  21927. P(n,k) as the number of length "n" sequences that generate "k"
  21928. successes.  For example,
  21929.  
  21930.      P(4,1)= 4  (HHTH, HTHH, TTHT, and THTT are 4 possible length 4 sequences).
  21931.  
  21932. I derived two generating functions g(x) and h(x) in order to enumerate
  21933. P(n,k), they are compactly represented by the following matrix
  21934. polynomial.
  21935.  
  21936.  
  21937.             _    _      _     _           _   _
  21938.            ! g(x) !    ! 1   1 ! (n-3)   !  4  !
  21939.            !      ! =  !       !         !     !
  21940.            ! h(x) !    ! 1   x !         !2+2x !
  21941.            !_    _!    !_     _!         !_   _!
  21942.  
  21943. The above is expressed as matrix generating function.  It can be shown
  21944. that P(n,k) is the coefficient of the x^k in the polynomial
  21945. (g(x)+h(x)).
  21946.  
  21947. For example, if n=4 we get (g(x)+h(x)) from the matrix generating
  21948. function as (10+4x+2x^2).  Clearly, P(4,1) (coefficient of x) is 4 and
  21949. P(4,2)=2 ( There are two such sequences THTH, and HTHT).
  21950.  
  21951. We can show that
  21952.  
  21953.    mean(k) = (n-2)/4 and sd= square_root(5n-12)/4
  21954.  
  21955. If we want to compare Sampling 1 with Sampling 2 then in Sampling 2 we
  21956. need to generate "n" samples. This can be done by using sequences of length
  21957. (n+2).  Then our new statistics would be
  21958.  
  21959.    mean = n/4 (same as that in sampling 1)
  21960.  
  21961.    sd = square_root(5n-2)/4 (not the same as in sampling 1)
  21962.  
  21963.     sd in sampling 2 < sd in sampling 1.
  21964.  
  21965. This difference can be explained by the fact that in serial sampling
  21966. there is dependence on the past state.  For example, if the past
  21967. sample was TTT then we know that the next sample can't be a success.
  21968. This was not the case in Case 1 or Case 2 (transition count). For
  21969. example, in case 2 success was independent of previous sample. That is
  21970. if my past sample was TT then my next sample can be TT or TH. The
  21971. probability of success in Case 1 and Case 2 is not influenced by past
  21972. history).
  21973.  
  21974. Similar approach can be followed for higher dimensional cases.
  21975.  
  21976. Here's a C program I had lying around that is relevant to the
  21977. current discussion of coin-flipping.  The algorithm is N^3 (for N flips)
  21978. but it could certainly be improved.  Compile with
  21979.  
  21980.         cc -o flip flip.c -lm
  21981.  
  21982. -- Guy
  21983.  
  21984. _________________ Cut here ___________________
  21985.  
  21986. #include <stdio.h>
  21987. #include <math.h>
  21988.  
  21989. char *progname;              /* Program name */
  21990.  
  21991. #define NOT(c) (('H' + 'T') - (c))
  21992.  
  21993.  
  21994. /* flip.c -- a program to compute the expected waiting time for a sequence
  21995.              of coin flips, or the probabilty that one sequence
  21996.              of coin flips will occur before another.
  21997.  
  21998.     Guy Jacobson, 11/1/90
  21999. */
  22000.  
  22001. main (ac, av) int ac; char **av;
  22002. {
  22003.     char *f1, *f2, *parseflips ();
  22004.     double compute ();
  22005.  
  22006.     progname = av▌0¿;
  22007.  
  22008.     if (ac == 2) {
  22009.         f1 = parseflips (av▌1¿);
  22010.         printf ("Expected number of flips until %s = %.1f\n",
  22011.                 f1, compute (f1, NULL));
  22012.     }
  22013.     else if (ac == 3) {
  22014.  
  22015.         f1 = parseflips (av▌1¿);
  22016.         f2 = parseflips (av▌2¿);
  22017.  
  22018.         if (strcmp (f1, f2) == 0) {
  22019.             printf ("Can't use the same flip sequence.\n");
  22020.             exit (1);
  22021.         }
  22022.         printf ("Probability of flipping %s before %s = %.1f%%\n",
  22023.                 av▌1¿, av▌2¿, compute (f1, f2) * 100.0);
  22024.     }
  22025.     else
  22026.       usage ();
  22027. }
  22028.  
  22029. char *parseflips (s) char *s;
  22030. {
  22031.     char *f = s;
  22032.  
  22033.     while (*s)
  22034.       if (*s == 'H' !! *s == 'h')
  22035.         *s++ = 'H';
  22036.       else if (*s == 'T' !! *s == 't')
  22037.         *s++ = 'T';
  22038.       else
  22039.         usage ();
  22040.  
  22041.     return f;
  22042. }
  22043.  
  22044. usage ()
  22045. {
  22046.     printf ("usage: %s {HT}^n\n", progname);
  22047.     printf ("\tto get the expected waiting time, or\n");
  22048.     printf ("usage: %s s1 s2\n\t(where s1, s2 in {HT}^n for some fixed n)\n",
  22049.             progname);
  22050.     printf ("\tto get the probability that s1 will occur before s2\n");
  22051.     exit (1);
  22052. }
  22053.  
  22054. /*
  22055.   compute --  if f2 is non-null, compute the probability that flip
  22056.               sequence f1 will occur before f2.  With null f2, compute
  22057.               the expected waiting time until f1 is flipped
  22058.  
  22059.   technique:
  22060.  
  22061.     Build a DFA to recognize (H+T)*f1 ▌or (H+T)*(f1+f2) when f2
  22062.     is non-null¿.  Randomly flipping coins is a Markov process on the
  22063.     graph of this DFA.  We can solve for the probability that f1 precedes
  22064.     f2 or the expected waiting time for f1 by setting up a linear system
  22065.     of equations relating the values of these unknowns starting from each
  22066.     state of the DFA.  Solve this linear system by Gaussian Elimination.
  22067. */
  22068.  
  22069. typedef struct state {
  22070.     char *s;                /* pointer to substring string matched */
  22071.     int len;                /* length of substring matched */
  22072.     int backup;             /* number of one of the two next states */
  22073. } state;
  22074.  
  22075. double compute (f1, f2) char *f1, *f2;
  22076. {
  22077.     double solvex0 ();
  22078.     int i, j, n1, n;
  22079.  
  22080.     state *dfa;
  22081.     int nstates;
  22082.  
  22083.     char *malloc ();
  22084.  
  22085.     n = n1 = strlen (f1);
  22086.     if (f2)
  22087.         n += strlen (f2); /* n + 1 states in the DFA */
  22088.  
  22089.     dfa = (state *) malloc ((unsigned) ((n + 1) * sizeof (state)));
  22090.  
  22091.     if (]dfa) {
  22092.         printf ("Ouch, out of memory]\n");
  22093.         exit (1);
  22094.     }
  22095.  
  22096.     /* set up the backbone of the DFA */
  22097.  
  22098.     for (i = 0; i <= n; i++) {
  22099.         dfa▌i¿.s = (i <= n1) ? f1 : f2;
  22100.         dfa▌i¿.len = (i <= n1) ? i : i - n1;
  22101.     }
  22102.  
  22103.     /* for i not a final state, one next state of i is simply
  22104.        i+1 (this corresponds to another matching character of dfs▌i¿.s
  22105.        The other next state (the backup state) is now computed.
  22106.        It is the state whose substring matches the longest suffix
  22107.        with the last character changed */
  22108.  
  22109.     for (i = 0; i <= n; i++) {
  22110.         dfa▌i¿.backup = 0;
  22111.         for (j = 1; j <= n; j++)
  22112.           if ((dfa▌j¿.len > dfa▌dfa▌i¿.backup¿.len)
  22113.               && dfa▌i¿.s▌dfa▌i¿.len¿ == NOT (dfa▌j¿.s▌dfa▌j¿.len - 1¿)
  22114.               && strncmp (dfa▌j¿.s, dfa▌i¿.s + dfa▌i¿.len - dfa▌j¿.len + 1,
  22115.                           dfa▌j¿.len - 1) == 0)
  22116.             dfa▌i¿.backup = j;
  22117.     }
  22118.  
  22119.     /* our dfa has n + 1 states, so build a system n + 1 equations
  22120.        in n + 1 unknowns */
  22121.  
  22122.     eqsystem (n + 1);
  22123.  
  22124.     for (i = 0; i < n; i++)
  22125.       if (i == n1)
  22126.         equation (1.0, n1, 0.0, 0, 0.0, 0, -1.0);
  22127.       else
  22128.         equation (1.0, i, -0.5, i + 1, -0.5, dfa▌i¿.backup, f2 ? 0.0 : -1.0);
  22129.     equation (1.0, n, 0.0, 0, 0.0, 0, 0.0);
  22130.  
  22131.     free (dfa);
  22132.  
  22133.     return solvex0 ();
  22134. }
  22135.  
  22136.  
  22137. /* a simple gaussian elimination equation solver */
  22138.  
  22139. double *m, **M;
  22140. int rank;
  22141. int neq = 0;
  22142.  
  22143. /* create an n by n system of linear equations.  allocate space
  22144.    for the matrix m, filled with zeroes and the dope vector M */
  22145.  
  22146. eqsystem (n) int n;
  22147. {
  22148.     char *calloc ();
  22149.     int i;
  22150.  
  22151.     m = (double *) calloc (n * (n + 1), sizeof (double));
  22152.     M = (double **) calloc (n, sizeof (double *));
  22153.  
  22154.     if (]m !! ]M) {
  22155.         printf ("Ouch, out of memory]\n");
  22156.         exit (1);
  22157.     }
  22158.  
  22159.     for (i = 0; i < n; i++)
  22160.       M▌i¿ = &m▌i * (n + 1)¿;
  22161.     rank = n;
  22162.     neq = 0;
  22163. }
  22164.  
  22165. /* add a new equation a * x_na + b * x_nb + c * x_nc + d = 0.0
  22166.    (note that na, nb, and nc are not necessarily all distinct.) */
  22167.  
  22168. equation (a, na, b, nb, c, nc, d) double a, b, c, d; int na, nb, nc;
  22169. {
  22170.     double *eq = M▌neq++¿; /* each row is an equation */
  22171.     eq▌na + 1¿ += a;
  22172.     eq▌nb + 1¿ += b;
  22173.     eq▌nc + 1¿ += c;
  22174.     eq▌0¿ = d;             /* column zero holds the constant term */
  22175. }
  22176.  
  22177. /* solve for the value of variable x_0.  This will go nuts if
  22178.    therer are errors (for example, if m is singular) */
  22179.  
  22180. double solvex0 ()
  22181. {
  22182.     register i, j, jmax, k;
  22183.     register double  max, val;
  22184.     register double *maxrow, *row;
  22185.  
  22186.  
  22187.     for (i = rank; i > 0; --i) {      /* for each variable */
  22188.  
  22189.         /* find pivot element--largest value in ith column*/
  22190.         max = 0.0;
  22191.         for (j = 0; j < i; j++)
  22192.             if (fabs (M▌j¿▌i¿) > fabs (max)) {
  22193.                 max = M▌j¿▌i¿;
  22194.                 jmax = j;
  22195.             }
  22196.         /* swap pivot row with last row using dope vectors */
  22197.         maxrow = M▌jmax¿;
  22198.         M▌jmax¿ = M▌i - 1¿;
  22199.         M▌i - 1¿ = maxrow;
  22200.  
  22201.         /* normalize pivot row */
  22202.         max = 1.0 / max;
  22203.         for (k = 0; k <= i; k++)
  22204.           maxrow▌k¿ *= max;
  22205.  
  22206.         /* now eliminate variable i by subtracting multiples of pivot row */
  22207.         for (j = 0; j < i - 1; j++) {
  22208.             row = M▌j¿;
  22209.             if (val = row▌i¿)              /* if variable i is in this eq */
  22210.                 for (k = 0; k <= i; k++)
  22211.                   row▌k¿ -= maxrow▌k¿ * val;
  22212.         }
  22213.     }
  22214.  
  22215.     /* the value of x0 is now in constant column of first row
  22216.        we only need x0, so no need to back-substitute */
  22217.  
  22218.     val = -M▌0¿▌0¿;
  22219.  
  22220.     free (M);
  22221.     free (m);
  22222.  
  22223.     return val;
  22224. }
  22225.  
  22226. _________________________________________________________________
  22227. Guy Jacobson   (201) 582-6558              AT&T Bell Laboratories
  22228.         uucp:  {att,ucbvax}]ulysses]guy    600 Mountain Avenue
  22229.     internet:  guy@ulysses.att.com         Murray Hill NJ, 07974
  22230.  
  22231.  
  22232.  
  22233. ==> probability/flush.p <==
  22234. Which set contains more flushes than the set of all possible hands?
  22235. (1) Hands whose first card is an ace
  22236. (2) Hands whose first card is the ace of spades
  22237. (3) Hands with at least one ace
  22238. (4) Hands with the ace of spades
  22239.  
  22240. ==> probability/flush.s <==
  22241. An arbitrary hand can have two aces but a flush hand can't.  The average
  22242. number of aces that appear in flush hands is the same as the average
  22243. number of aces in arbitrary hands, but the aces are spread out more
  22244. evenly for the flush hands, so set #3 contains a higher fraction of flushs.
  22245.  
  22246. Aces of spades, on the other hand, are spread out the same way over possible
  22247. hands as they are over flush hands, since there is only one of them in the deck.
  22248. Whether or not a hand is flush is based solely on a comparison between
  22249. different cards in the hand, so looking at just one card is necessarily
  22250. uninformative.  So the other sets contain the same fraction of flushes
  22251. as the set of all possible hands.
  22252.  
  22253. ==> probability/hospital.p <==
  22254. A town has two hospitals, one big and one small.  Every day the big
  22255. hospital delivers 1000 babies and the small hospital delivers 100
  22256. babies.  There's a 50/50 chance of male or female on each birth.
  22257. Which hospital has a better chance of having the same number of boys
  22258. as girls?
  22259.  
  22260. ==> probability/hospital.s <==
  22261. If there are 2N babies born, then the probability of an even split is
  22262.  
  22263. (2N choose N) / (2 ** 2N) ,
  22264.  
  22265. where (2N choose N) = (2N)] / (N] * N]) .
  22266.  
  22267. This is a DECREASING function.
  22268.  
  22269. Think about it. If there are two babies born, then the probability of a
  22270. split is 1/2 (just have the second baby be different from the first).
  22271. With 2N babies, If there is a N,N-1 split in the first 2N-1, then there
  22272. is a 1/2 chance of the last baby making it an even split.  Otherwise
  22273. there can be no even split.  Therefore the probability is less than 1/2
  22274. overall for an even split.
  22275.  
  22276. As N goes to infinity the probability of an even split approaches zero
  22277. (although it is still the most likely event).
  22278.  
  22279. ==> probability/icos.p <==
  22280. The "house" rolls two 20-sided dice and the "player" rolls one
  22281. 20-sided die.  If the player rolls a number on his die between the
  22282. two numbers the house rolled, then the player wins.  Otherwise, the
  22283. house wins (including ties).  What are the probabilities of the player
  22284. winning?
  22285.  
  22286. ==> probability/icos.s <==
  22287. It is easily seen that if any two of the three dice agree that the
  22288. house wins.  The probability that this does not happen is 19*18/(20*20).
  22289. If the three numbers are different, the probability of winning is 1/3.
  22290. So the chance of winning is 19*18/(20*20*3) = 3*19/200 = 57/200.
  22291.  
  22292. ==> probability/intervals.p <==
  22293. Given two random points x and y on the interval 0..1, what is the average
  22294. size of the smallest of the three resulting intervals?
  22295.  
  22296. ==> probability/intervals.s <==
  22297. You could make a graph of the size of the
  22298. smallest interval versus x and y.  We know that the height of the
  22299. graph is 0 along all the edges of the unit square where it is defined
  22300. and on the line x=y.  It achieves its maximum of 1/3 at (1/3,2/3) and
  22301. (2/3,1/3).  Assuming the graph has no curves or bizzare jags, the
  22302. volume under the graph must be 1/9 and so the average height must also
  22303. be 1/9.
  22304.  
  22305. ==> probability/lights.p <==
  22306. Waldo and Basil are exactly m blocks west and n blocks north from Central Park,
  22307. and always go with the green light until they run out of options.  Assuming
  22308. that the probability of the light being green is 1/2 in each direction and
  22309. that if the light is green in one direction it is red in the other, find the
  22310. expected number of red lights that Waldo and Basil will encounter.
  22311.  
  22312. ==> probability/lights.s <==
  22313. Let E(m,n) be this number, and let (x)C(y) = x]/(y] (x-y)]).  A model
  22314. for this problem is the following nxm grid:
  22315.  
  22316.      ^         B---+---+---+ ... +---+---+---+ (m,0)
  22317.      !         !   !   !   !     !   !   !   !
  22318.      N         +---+---+---+ ... +---+---+---+ (m,1)
  22319. <--W + E-->    :   :   :   :     :   :   :   :
  22320.      S         +---+---+---+ ... +---+---+---+ (m,n-1)
  22321.      !         !   !   !   !     !   !   !   !
  22322.      v         +---+---+---+ ... +---+---+---E (m,n)
  22323.  
  22324. where each + represents a traffic light.  We can consider each
  22325. traffic light to be a direction pointer, with an equal chance of
  22326. pointing either east or south.
  22327.  
  22328. IMHO, the best way to approach this problem is to ask:  what is the
  22329. probability that edge-light (x,y) will be the first red edge-light
  22330. that the pedestrian encounters?  This is easy to answer; since the
  22331. only way to reach (x,y) is by going south x times and east y times,
  22332. in any order, we see that there are (x+y)C(x) possible paths from
  22333. (0,0) to (x,y).  Since each of these has probability (1/2)^(x+y+1)
  22334. of occuring, we see that the the probability we are looking for is
  22335. (1/2)^(x+y+1)*(x+y)C(x).  Multiplying this by the expected number
  22336. of red lights that will be encountered from that point, (n-k+1)/2,
  22337. we see that
  22338.  
  22339.             m-1
  22340.            -----
  22341.            \
  22342. E(m,n) =    >    ( 1/2 )^(n+k+1) * (n+k)C(n) * (m-k+1)/2
  22343.            /
  22344.            -----
  22345.             k=0
  22346.  
  22347.             n-1
  22348.            -----
  22349.            \
  22350.       +     >    ( 1/2 )^(m+k+1) * (m+k)C(m) * (n-k+1)/2 .
  22351.            /
  22352.            -----
  22353.             k=0
  22354.  
  22355.  
  22356. Are we done?  No]  Putting on our Captain Clever Cap, we define
  22357.  
  22358.             n-1
  22359.            -----
  22360.            \
  22361. f(m,n) =    >    ( 1/2 )^k * (m+k)C(m) * k
  22362.            /
  22363.            -----
  22364.             k=0
  22365.  
  22366. and
  22367.  
  22368.             n-1
  22369.            -----
  22370.            \
  22371. g(m,n) =    >    ( 1/2 )^k * (m+k)C(m) .
  22372.            /
  22373.            -----
  22374.             k=0
  22375.  
  22376. Now, we know that
  22377.  
  22378.              n
  22379.            -----
  22380.            \
  22381. f(m,n)/2 =  >    ( 1/2 )^k * (m+k-1)C(m) * (k-1)
  22382.            /
  22383.            -----
  22384.             k=1
  22385.  
  22386. and since f(m,n)/2 = f(m,n) - f(m,n)/2, we get that
  22387.  
  22388.             n-1
  22389.            -----
  22390.            \
  22391. f(m,n)/2 =  >    ( 1/2 )^k * ( (m+k)C(m) * k - (m+k-1)C(m) * (k-1) )
  22392.            /
  22393.            -----
  22394.             k=1
  22395.  
  22396. - (1/2)^n * (m+n-1)C(m) * (n-1)
  22397.  
  22398.             n-2
  22399.            -----
  22400.            \
  22401.  =          >    ( 1/2 )^(k+1) * (m+k)C(m) * (m+1)
  22402.            /
  22403.            -----
  22404.             k=0
  22405.  
  22406. - (1/2)^n * (m+n-1)C(m) * (n-1)
  22407.  
  22408. = (m+1)/2 * (g(m,n) - (1/2)^(n-1)*(m+n-1)C(m)) - (1/2)^n*(m+n-1)C(m)*(n-1)
  22409.  
  22410. therefore
  22411.  
  22412. f(m,n) = (m+1) * g(m,n) - (n+m) * (1/2)^(n-1) * (m+n-1)C(m) .
  22413.  
  22414.  
  22415. Now, E(m,n) = (n+1) * (1/2)^(m+2) * g(m,n) - (1/2)^(m+2) * f(m,n)
  22416.  
  22417. + (m+1) * (1/2)^(n+2) * g(n,m) - (1/2)^(n+2) * f(n,m)
  22418.  
  22419. = (m+n) * (1/2)^(n+m+1) * (m+n)C(m) + (m-n) * (1/2)^(n+2) * g(n,m)
  22420.  
  22421. + (n-m) * (1/2)^(m+2) * g(m,n) .
  22422.  
  22423.  
  22424. Setting m=n in this formula, we see that
  22425.  
  22426.            E(n,n) = n * (1/2)^(2n) * (2n)C(n),
  22427.  
  22428. and applying Stirling's theorem we get the beautiful asymptotic formula
  22429.  
  22430.                   E(n,n) ~ sqrt(n/pi).
  22431.  
  22432. ==> probability/lottery.p <==
  22433. There n tickets in the lottery, k winners and m allowing you to pick another
  22434. ticket. The problem is to determine the probability of winning the lottery
  22435. when you start by picking 1 (one) ticket.
  22436.  
  22437. A lottery has N balls in all, and you as a player can choose m numbers
  22438. on each card, and the lottery authorities then choose n balls, define
  22439. L(N,n,m,k) as the minimum number of cards you must purchase to ensure that
  22440. at least one of your cards will have at least k numbers in common with the
  22441. balls chosen in the lottery.
  22442.  
  22443. ==> probability/lottery.s <==
  22444. This relates to the problem of rolling a random number
  22445. from 1 to 17 given a 20 sided die.  You simply mark the numbers 18,
  22446. 19, and 20 as "roll again".
  22447.  
  22448. The probability of winning is, of course, k/(n-m) as for every k cases
  22449. in which you get x "draw again"'s before winning, you get n-m-k similar
  22450. cases where you get x "draw again"'s before losing.  The example using
  22451. dice makes it obvious, however.
  22452.  
  22453. L(N,k,n,k) >= Ceiling((N-choose-k)/(n-choose-k)*
  22454.                    (n-1-choose-k-1)/(N-1-choose-k-1)*L(N-1,k-1,n-1,k-1))
  22455.             = Ceiling(N/n*L(N-1,k-1,n-1,k-1))
  22456.  
  22457. The correct way to see this is as follows:  Suppose you have an
  22458. (N,k,n,k) system of cards.  Look at all of the cards that contain the
  22459. number 1.  They cover all ball sets that contain 1, and therefore these
  22460. cards become an (N-1,k-1,n-1,k-1) covering upon deletion of the number
  22461. 1.  Therefore the number 1 must appear at least L(N-1,k-1,n-1,k-1).
  22462. The same is true of all of the other numbers.  There are N of them but
  22463. n appear on each card.  Thus we obtain the bound.
  22464.  
  22465. ==> probability/particle.in.box.p <==
  22466. A particle is bouncing randomly in a two-dimensional box.  How far does it
  22467. travel between bounces, on avergae?
  22468.  
  22469. Suppose the particle is initially at some random position in the box and is
  22470. traveling in a straight line in a random direction and rebounds normally
  22471. at the edges.
  22472.  
  22473. ==> probability/particle.in.box.s <==
  22474. Let theta be the angle of the point's initial vector.  After traveling a
  22475. distance r, the point has moved r*cos(theta) horizontally and r*sin(theta)
  22476. vertically, and thus has struck r*(sin(theta)+cos(theta))+O(1) walls.  Hence
  22477. the average distance between walls will be 1/(sin(theta)+cos(theta)).  We now
  22478. average this over all angles theta:
  22479.         2/pi * intg from theta=0 to pi/2 (1/(sin(theta)+cos(theta))) dtheta
  22480. which (in a computation which is left as an exercise) reduces to
  22481.         2*sqrt(2)*ln(1+sqrt(2))/pi = 0.793515.
  22482.  
  22483. ==> probability/pi.p <==
  22484. Are the digits of pi random (i.e., can you make money betting on them)?
  22485.  
  22486. ==> probability/pi.s <==
  22487. No, the digits of pi are not truly random, therefore you can win money
  22488. playing against a supercomputer that can calculate the digits of pi far
  22489. beyond what we are currently capable of doing.  This computer selects a
  22490. position in the decimal expansion of pi -- say, at 10^100.  Your job is
  22491. to guess what the next digit or digit sequence is.  Specifically, you
  22492. have one dollar to bet.  A bet on the next digit, if correct, returns
  22493. 10 times the amount bet; a bet on the next two digits returns 100 times
  22494. the amount bet, and so on.  (The dollar may be divided in any fashion,
  22495. so we could bet 1/3 or 1/10000 of a dollar.)  You may place bets in any
  22496. combination.  The computer will tell you the position number, let you
  22497. examine the digits up to that point, and calculate statistics for you.
  22498.  
  22499. It is easy to set up strategies that might win, if the supercomputer
  22500. doesn't know your strategy.  For example, "Always bet on 7" might win,
  22501. if you are lucky.  Also, it is easy to set up bets that will always
  22502. return a dollar.  For example, if you bet a penny on every two-digit
  22503. sequence, you are sure to win back your dollar.  Also, there are
  22504. strategies that might be winning, but we can't prove it.  For example,
  22505. it may be that a certain sequence of digits never occurs in pi, but we
  22506. have no way of proving this.
  22507.  
  22508. The problem is to find a strategy that you can prove will always win
  22509. back more than a dollar.
  22510.  
  22511. The assumption that the position is beyond the reach of calculation
  22512. means that we must rely on general facts we know about the sequence of
  22513. digits of pi, which is practically nil.  But it is not completely nil,
  22514. and the challenge is to find a strategy that will always win money.
  22515.  
  22516. A theorem of Mahler (1953) states that for all integers p, q > 1,
  22517.  
  22518.                 -42
  22519.   !pi - p/q! > q
  22520.  
  22521. This says that pi cannot have a rational approximation that is
  22522. extremely tight.
  22523.  
  22524. Now suppose that the computer picks position N.  I know that the next
  22525. 41 * N digits cannot be all zero.   For if they were, then the first N
  22526. digits, treated as a fraction with denominator 10^N, satisfies:
  22527.  
  22528.   !pi - p / 10^N!  <  10^(-42 N)
  22529.  
  22530. which contradicts Mahler's theorem.
  22531.  
  22532. So, I split my dollar into 10^(41N) - 1 equal parts, and bet on each of
  22533. the sequences of 41N digits, except the one with all zeroes.  One of
  22534. the bets is sure to win, so my total profit is about 10(^-41N) of a
  22535. dollar]
  22536.  
  22537. This strategy can be improved a number of ways, such as looking for
  22538. other repeating patterns, or improvements to the bound of 42 -- but the
  22539. earnings are so pathetic, it hardly seems worth the effort.
  22540.  
  22541. Are there other winning strategies, not based on Mahler's theorem?  I
  22542. believe there are algorithms that generate 2N binary digits of pi,
  22543. where the computations are separate for each block of N digits.  Maybe
  22544. from something like this, we can find a simple subsequence of the
  22545. binary digits of pi which is always zero, or which has some simple
  22546. pattern.
  22547.  
  22548. ==> probability/random.walk.p <==
  22549. Waldo has lost his car keys]  He's not using a very efficient search;
  22550. in fact, he's doing a random walk.  He starts at 0, and moves 1 unit
  22551. to the left or right, with equal probability.  On the next step, he
  22552. moves 2 units to the left or right, again with equal probability.  For
  22553. subsequent turns he follows the pattern 1, 2, 1, etc.
  22554.  
  22555. His keys, in truth, were right under his nose at point 0.  Assuming
  22556. that he'll spot them the next time he sees them, what is the
  22557. probability that poor Waldo will eventually return to 0?
  22558.  
  22559. ==> probability/random.walk.s <==
  22560. I can show the probability that Waldo returns to 0 is 1.  Waldo's
  22561. wanderings map to an integer grid in the plane as follows.  Let
  22562. (X_t,Y_t) be the cumulative sums of the length 1 and length 2 steps
  22563. respectively taken by Waldo through time t.  By looking only at even t,
  22564. we get the ordinary random walk in the plane, which returns to the
  22565. origin (0,0) with probability 1.  In fact, landing at (2n, n) for any n
  22566. will land Waldo on top of his keys too.  There's no need to look at odd
  22567. t.
  22568.  
  22569. Similar considerations apply for step sizes of arbitrary (fixed) size.
  22570.  
  22571. ==> probability/reactor.p <==
  22572. There is a reactor in which a reaction is to take place. This reaction
  22573. stops if an electron is present in the reactor. The reaction is started
  22574. with 18 positrons; the idea being that one of these positrons would
  22575. combine with any incoming electron (thus destroying both). Every second,
  22576. exactly one particle enters the reactor. The probablity that this particle
  22577. is an electron is 0.49 and that it is a positron is 0.51.
  22578.  
  22579. What is the probablity that the reaction would go on for ever??
  22580.  
  22581. Note: Once the reaction stops, it cannot restart.
  22582.  
  22583. ==> probability/reactor.s <==
  22584. Let P(n) be the probability that, starting with n positrons, the
  22585. reaction goes on forever.  Clearly P'(n+1)=P'(0)*P'(n), where the
  22586. ' indicates probabilistic complementation; also note that
  22587. P'(n) = .51*P'(n+1) + .49*P'(n-1).  Hence we have that P(1)=(P'(0))^2
  22588. and that P'(0) = .51*P'(1) ==> P'(0) equals 1 or 49/51.  We thus get
  22589. that either P'(18) = 1 or (49/51)^19 ==> P(18) = 0 or 1 - (49/51)^19.
  22590.  
  22591. The answer is indeed the latter.  A standard result in random walks
  22592. (which can be easily derived using Markov chains) yields that if p>1/2
  22593. then the probability of reaching the absorbing state at +infinity
  22594. as opposed to the absorbing state at -1 is 1-r^(-i), where r=p/(1-p)
  22595. (p is the probability of moving from state n to state n-1, in our
  22596. case .49) and i equals the starting location + 1.  Therefore we have
  22597. that P(18) = 1-(.49/.51)^19.
  22598.  
  22599. ==> probability/roulette.p <==
  22600. You are in a game of Russian roulette, but this time the gun (a 6
  22601. shooter revolver) has three bullets _in_a_row_ in three of the
  22602. chambers.  The barrel is spun only once.  Each player then points the
  22603. gun at his (her) head and pulls the trigger.  If he (she) is still
  22604. alive, the gun is passed to the other player who then points it at his
  22605. (her) own head and pulls the trigger.  The game stops when one player
  22606. dies.
  22607.  
  22608. Now to the point:  would you rather be first or second to shoot?
  22609.  
  22610. ==> probability/roulette.s <==
  22611. All you need to consider are the six possible bullet configurations
  22612.  
  22613.     B B B E E E         -> player 1 dies
  22614.     E B B B E E         -> player 2 dies
  22615.     E E B B B E         -> player 1 dies
  22616.     E E E B B B         -> player 2 dies
  22617.     B E E E B B         -> player 1 dies
  22618.     B B E E E B         -> player 1 dies
  22619.  
  22620. One therefore has a 2/3 probability of winning (and a 1/3 probability of
  22621. dying) by shooting second.  I for one would prefer this option.
  22622.  
  22623. ==> probability/unfair.p <==
  22624. Generate even odds from an unfair coin.  For example, if you
  22625. thought a coin was biased toward heads, how could you get the
  22626. equivalent of a fair coin with several tosses of the unfair coin?
  22627.  
  22628. ==> probability/unfair.s <==
  22629. Toss twice.  If both tosses give the same result, repeat this process
  22630. (throw out the two tosses and start again).  Otherwise, take the first
  22631. of the two results.
  22632.  
  22633. ==> series/series.01.p <==
  22634. M, N, B, D, P ?
  22635.  
  22636. ==> series/series.01.s <==
  22637. T.  If you say the sounds these letters make out loud, you
  22638. will see that the next letter is T.
  22639.  
  22640. ==> series/series.02.p <==
  22641. H, H, L, B, B, C, N, O, F ?
  22642.  
  22643. ==> series/series.02.s <==
  22644. Answer 1:  N, N, M, A  The symbols for the elements.
  22645. Answer 2:  N, S, M, A  The names of the elements.
  22646.  
  22647. ==> series/series.03.p <==
  22648. W, A, J, M, M, A, J?
  22649.  
  22650. ==> series/series.03.s <==
  22651. J, V, H, T, P, T, F, P, B, L.  Presidents.
  22652.  
  22653. ==> series/series.03a.p <==
  22654. G, J, T, J, J, J, A, M, W, J, J, Z, M, F, J, ?
  22655.  
  22656.  
  22657. ==> series/series.03a.s <==
  22658. G, J, T, J, J, J, A, M, W, J, J, Z, M, F, J, A.  Presidents' first names.
  22659.  
  22660. ==> series/series.03b.p <==
  22661. A, J, B, C, G, T, C, V, J, T, D, F, K, B, H, ?
  22662.  
  22663.  
  22664. ==> series/series.03b.s <==
  22665. A, J, B, C, G, T, C, V, J, T, D, F, K, B, H, J.  Vice Presidents.
  22666.  
  22667. ==> series/series.03c.p <==
  22668. M, A, M, D, E, L, R, H, ?
  22669.  
  22670.  
  22671. ==> series/series.03c.s <==
  22672. M, A, M, D, E, L, R, H, A.  Presidents' wives' first names.
  22673.  
  22674. ==> series/series.04.p <==
  22675. A, E, H, I, K, L, ?
  22676.  
  22677. ==> series/series.04.s <==
  22678. M, N, O, P, U, W.  Letters in the Hawaiian alphabet.
  22679.  
  22680. ==> series/series.05.p <==
  22681. A B C D E F G H?
  22682.  
  22683. ==> series/series.05.s <==
  22684. M.  The names of the cross-streets travelling west on (say) Commonwealth
  22685. Avenue from Boston Garden: Arlington, Berkeley, Clarendon, Dartmouth,
  22686. Exeter, Fairfield, Gloucester, Hereford, Massachusetts Ave.
  22687.  
  22688. ==> series/series.06.p <==
  22689. Z, O, T, T, F, F, S, S, E, N?
  22690.  
  22691. ==> series/series.06.s <==
  22692. T.  The name of the integers starting with zero.
  22693.  
  22694. ==> series/series.06a.p <==
  22695. F, S, T, F, F, S, ?
  22696.  
  22697. ==> series/series.06a.s <==
  22698. The words "first", "second", "third", etc.  The same as the previous from this
  22699. point on.
  22700.  
  22701. ==> series/series.07.p <==
  22702. 1, 1 1, 2 1, 1 2 1 1, ...
  22703.  
  22704. What is the pattern and asymptotics of this series?
  22705.  
  22706. ==> series/series.07.s <==
  22707. Each line is derived from the last by the transformation (for example)
  22708.  
  22709. ... z z z x x y y y ... ->
  22710. ... 3 z 2 x 3 y ...
  22711.  
  22712. John Horton Conway analyzed this in "The Weird and Wonderful Chemistry
  22713. of Audioactive Decay" (T M Cover & B Gopinath (eds) OPEN PROBLEMS IN
  22714. COMMUNICATION AND COMPUTATION, Springer-Verlag (1987)).  You can also
  22715. find his most complete FRACTRAN paper in this collection.
  22716.  
  22717. First, he points out that under this sequence, you frequently get
  22718. adjacent subsequences XY which cannot influence each other in any
  22719. future derivation of the sequence rule.  The smallest such are
  22720. called "atoms" or "elements".  As Conway claims to have proved,
  22721. there are 92 atoms which show up eventually in every sequence, no
  22722. matter what the starting value (besides <> and <22>), and always in
  22723. the same non-zero limiting proportions.
  22724.  
  22725. Conway named them after some other list of 92 atoms.  As a puzzle,
  22726. see if you can recreate the list from the following, in decreasing
  22727. atomic number:
  22728.  
  22729. U Pa Th Ac Ra Fr Rn Ho.AT Po Bi Pm.PB Tl Hg Au Pt Ir Os Re Ge.Ca.W Ta
  22730. HF.Pa.H.Ca.W Lu Yb Tm ER.Ca.Co HO.Pm Dy Tb Ho.GD EU.Ca.Co Sm PM.Ca.Zn
  22731. Nd Pr Ce LA.H.Ca.Co Ba Cs Xe I Ho.TE Eu.Ca.SB Pm.SN In Cd Ag Pd Rh
  22732. Ho.RU Eu.Ca.TC Mo Nb Er.ZR Y.H.Ca.Tc SR.U Rb Kr Br Se As GE.Na Ho.GA
  22733. Eu.Ca.Ac.H.Ca.ZN Cu Ni Zn.CO Fe Mn CR.Si V Ti Sc Ho.Pa.H.CA.Co K Ar
  22734. Cl S P Ho.SI Al Mg Pm.NA Ne F O N C B Be Ge.Ca.LI He Hf.Pa.H.Ca.Li
  22735.  
  22736. Uranium is 3, Protactinium is 13, etc.  Rn => Ho.AT means the following:
  22737. Radon forms a string that consists of two atoms, Holmium on the left,
  22738. and Astatine on the right.  I capitalize the symbol for At to remind
  22739. you that Astatine, and not Holmium, is one less than Radon in atomic
  22740. number.  As a check, against you or me making a mistake, Hf is 111xx,
  22741. Nd is 111xxx, In and Ni are 111xxxxx, K is 111x, and H is 22.
  22742.  
  22743. Next see if you can at least prove that any atom other than Hydrogen,
  22744. eventually (and always thereafter) forms strings containing all 92 atoms.
  22745.  
  22746. The grand Conway theorem here is that every string eventually forms (within
  22747. a universal time limit) strings containing all the 92 atoms in certain
  22748. specific non-zero limiting proportions, and that digits N greater than 3
  22749. are eventually restricted to one of two atomic patterns (ie, abc...N and
  22750. def...N for some {1,2,3} sequences abc... and def...), which Conway calls
  22751. isotopes of Np and Pu.  (For N=2, these are He and Li), and that these
  22752. transuranic atoms have a zero limiting proportion.
  22753.  
  22754. The longest lived exotic element is Methuselum (2233322211N) which takes
  22755. about 25 applications to reduce to the periodic table.
  22756.  
  22757. -Matthew P Wiener (weemba@libra.wistar.upenn.edu)
  22758.  
  22759. Conway gives many results on the ultimate behavior of strings under
  22760. this transformation: for example, taking the sequence derived from 1
  22761. (or any other string except 2 2), the limit of the ratio of length of
  22762. the (n+1)th term to the length of the nth term as n->infinity is a
  22763. fixed constant, namely
  22764.  
  22765.             1.30357726903429639125709911215255189073070250465940...
  22766.  
  22767. This number is from Ilan Vardi, "Computational Recreations in Mathematica",
  22768. Addison Wesley 1991, page 13.
  22769.  
  22770. Another sequence that is related but not nearly as interesting is:
  22771.  
  22772. 1, 11, 21, 1112, 3112, 211213, 312213, 212223, 114213, 31121314, 41122314,
  22773.     31221324, 21322314,
  22774.  
  22775. and 21322314 generates itself, so we have a cycle.
  22776.  
  22777. ==> series/series.08a.p <==
  22778. G, L, M, B, C, L, M, C, F, S, ?
  22779.  
  22780. ==> series/series.08a.s <==
  22781. Army officer ranks, descending.
  22782.  
  22783. ==> series/series.08b.p <==
  22784. A, V, R, R, C, C, L, L, L, E, ?
  22785.  
  22786. ==> series/series.08b.s <==
  22787. Navy officer ranks, descending.
  22788.  
  22789. ==> series/series.09a.p <==
  22790. S, M, S, S, S, C, P, P, P, ?
  22791.  
  22792. ==> series/series.09a.s <==
  22793. Army non-comm ranks, descending.
  22794.  
  22795. ==> series/series.09b.p <==
  22796. M, S, C, P, P, P, S, S, S, ?
  22797.  
  22798. ==> series/series.09b.s <==
  22799. Navy non-comm ranks, descending.
  22800.  
  22801. ==> series/series.10.p <==
  22802. D, P, N, G, C, M, M, S, ?
  22803.  
  22804. ==> series/series.10.s <==
  22805. N, V, N, N, R.  States in Constitution ratification order.
  22806.  
  22807. ==> series/series.11.p <==
  22808. R O Y G B ?
  22809.  
  22810. ==> series/series.11.s <==
  22811. V.  Colors.
  22812.  
  22813. ==> series/series.12.p <==
  22814. A, T, G, C, L, ?
  22815.  
  22816. ==> series/series.12.s <==
  22817. V, L, S, S, C, A, P.  Zodiacal signs.
  22818.  
  22819. ==> series/series.13.p <==
  22820. M, V, E, M, J, S, ?
  22821.  
  22822. ==> series/series.13.s <==
  22823. U, N, P.  Names of the Planets.
  22824.  
  22825. ==> series/series.14.p <==
  22826. A, B, D, O, P, ?
  22827.  
  22828. ==> series/series.14.s <==
  22829. Q, R.  Only letters with an inside as printed.
  22830.  
  22831. ==> series/series.14a.p <==
  22832. A, B, D, E, G, O, P, ?
  22833.  
  22834. ==> series/series.14a.s <==
  22835. Q.  Letters with cursive insides.
  22836.  
  22837. ==> series/series.15.p <==
  22838. A, E, F, H, I, ?
  22839.  
  22840. ==> series/series.15.s <==
  22841. L, M, N, O, S, U.  Letters whose English names start with vowels.
  22842.  
  22843. ==> series/series.16.p <==
  22844. A, B, C, D, E, F, G, H, I, J, K, L, M, N, O, P, Q, R, S, T, U, V, X, Y?
  22845.  
  22846. ==> series/series.16.s <==
  22847. Z.  Letters whose English names have one syllable.
  22848.  
  22849. ==> series/series.17.p <==
  22850. T, P, O, F, O, F, N, T, S, F, T, F, E, N, S, N?
  22851.  
  22852. ==> series/series.17.s <==
  22853. T, T, T, E, T, E.  Digits of Pi.
  22854.  
  22855. ==> series/series.18.p <==
  22856. 10, 11, 12, 13, 14, 15, 16, 17, 20, 22, 24, ___ , 100, 121, 10000
  22857.  
  22858. ==> series/series.18.s <==
  22859.  10, 11, 12, 13, 14, 15, 16, 17, 20, 22, 24, 31 , 100, 121, 10000
  22860.  
  22861. Sixteen in base n for n=16, 15, ..., 2.
  22862.  
  22863. ==> series/series.19.p <==
  22864. 1 01 01011 0101101011011 0101101011011010110101101101011011 etc.
  22865.  
  22866. Each string is formed from the previous string by substituting '01' for '1'
  22867. and '011' for '0' simultaneously at each occurance.
  22868. Notice that each string is an initial substring of the previous string so
  22869. that we may consider them all as initial substrings of an infinite string.
  22870. The puzzle then is, given n, determine if the nth digit is 0 or 1 without
  22871. having to construct all the previous digits.  That is, give a non-recursive
  22872. formula for the nth digit.
  22873.  
  22874. ==> series/series.19.s <==
  22875. Let G equal the limit string generated by the above process and define
  22876. the string F by
  22877.  
  22878. F▌0¿ = "0",
  22879. F▌n¿ = "1" if n = floor(phi*m) for some positive integer m,
  22880. F▌n¿ = "0" if n = floor(phi^2*m) for some positive integer m,
  22881.  
  22882. where floor(x) is the greatest integer =< x and phi = (1 + \/5)/2;
  22883. I claim that F = G.
  22884.  
  22885.  
  22886. I will try to motivate my solution.  Let g▌0¿="0" and define g▌n+1¿
  22887. to be the string that results from replacing "0" in g▌n¿ with "01"
  22888. and "1" with "011"; furthermore, let s(n) and t(n) be the number of
  22889. "0"'s and "1"'s in g▌n¿, respectively.  Note that we have the
  22890. following recursive formulas : s(n+1) = s(n) + t(n) and t(n+1) =
  22891. s(n) + 2t(n).  I claim that s(n) = Fib(2n-1) and t(n) = Fib(2n),
  22892. where Fib(m) is the mth Fibonacci number (defined by Fib(-1) = 1,
  22893. Fib(0) = 0, Fib(n+1) = Fib(n) + Fib(n-1) for n>=0); this is easily
  22894. established by induction.  Now noting that Fib(2n)/Fib(2n-1) -> phi
  22895. as n -> infinity, we see that if the density of the "0"'s and "1"'s
  22896. exists, they must be be 1/phi^2 and 1/phi, respectively.  What is
  22897. the simplest generating sequence which has this property?  Answer:
  22898. the one given above.
  22899.  
  22900.  
  22901. Proof:  We start with
  22902.  
  22903. Beatty's Theorem: if a and b are positive irrational numbers such
  22904. that 1/a + 1/b = 1, then every positive integer has a representation
  22905. of the form floor(am) or floor(bm) (m a positive integer), and this
  22906. representation is unique.
  22907.  
  22908. This shows that F is well-defined.  I now claim that
  22909.  
  22910. Lemma: If S(n) and T(n) (yes, two more functions; apparently today's
  22911. the day that functions have their picnic) represent the number of
  22912. "0"'s and "1"'s in the initial string of F of length n, then S(n)
  22913. = ceil(n/phi^2) and T(n) = floor(n/phi) (ceil(x) is the smallest
  22914. integer >= x).
  22915.  
  22916. Proof of lemma: using the identity phi^2 = phi + 1 we see that S(n)
  22917. + T(n) = n, hence for a given n either S(n) = S(n-1) + 1 or T(n) =
  22918. T(n-1) + 1.  Now note that if F▌n-1¿="1" ==> n-1 = floor(phi*m) for
  22919. some positive integer m and since phi*m-1 < floor(phi*m) < phi*m ==>
  22920. m-1/phi < (n-1)/phi < m ==> T(n) = T(n-1) + 1.   To finish, note that
  22921. if F▌n-1¿="0" ==> n-1 = floor(phi^2*m) for some positive integer m
  22922. and since phi^2*m-1 < floor(phi^2*m) < phi^2*m ==> m-1/phi^2 <
  22923. (n-1)/phi^2 < m ==> S(n) = S(n-1) + 1.  Q.E.D.
  22924.  
  22925. I will now show that F is invariant under the operation of replacing
  22926. "0" with "01" and "1" with "011"; it will then follow that F=G.
  22927. Note that this is equivalent to showing that F▌2S(n) + 3T(n)¿
  22928. = "0", F▌2S(n) + 3T(n) + 1¿ = "1", and that if n = ▌phi*m¿ for some
  22929. positive integer m, then F▌2S(n) + 3T(n) + 2¿ = "1".  One could
  22930. waste hours trying to prove some fiendish identities; watch how
  22931. I sidestep this trap.  For the first part, note that by the above
  22932. lemma F▌2S(n) + 3T(n)¿ = F▌2*ceil(n/phi^2) + 3*floor(n/phi)¿ =
  22933. F▌2n + floor(n/phi)¿ = F▌2n + floor(n*phi-n)¿ = F▌floor(phi*n+n)¿
  22934. = F▌floor(phi^2*n)¿ ==> F▌2S(n) + 3T(n)¿ = "0".  For the second, it
  22935. is easy to see that since phi^2>2, if F▌m¿="0" ==> F▌m¿="1" hence
  22936. the first part implies the second part.  Finally, note that if n =
  22937. ▌phi*m¿ for some positive integer m, then F▌2S(n) + 3T(n) + 3¿ =
  22938. F▌2S(n+1) + 3T(n+1)¿ = "0", hence by the same reasoning as above
  22939. F▌2S(n) + 3T(n) + 2¿ = "1".
  22940.  
  22941. Q.E.D.
  22942.  
  22943.     -- clong@remus.rutgers.edu (Chris Long)
  22944.  
  22945. ==> series/series.20.p <==
  22946. 1 2 5 16 64 312 1812 12288
  22947.  
  22948. ==> series/series.20.s <==
  22949. ANSWER: 95616
  22950.         The sum of factorial(k)*factorial(n-k) for k=0,...,n.
  22951.  
  22952. ==> series/series.21.p <==
  22953. 5, 6, 5, 6, 5, 5, 7, 5, ?
  22954.  
  22955. ==> series/series.21.s <==
  22956. The number of letters in the ordinal numbers.
  22957.  
  22958. First   5
  22959. Second  6
  22960. Third   5
  22961. Fourth  6
  22962. Fifth   5
  22963. Sixth   5
  22964. Seventh 7
  22965. Eighth  6
  22966. Ninth   5
  22967. etc.
  22968.  
  22969. ==> series/series.22.p <==
  22970. 3 1 1 0 3 7 5 5 2 ?
  22971.  
  22972. ==> series/series.22.s <==
  22973. ANSWER: 4
  22974.         The digits of pi expressed in base eight.
  22975.  
  22976. ==> series/series.23.p <==
  22977. 22 22 30 13 13 16 16 28 28 11 ?
  22978.  
  22979. ==> series/series.23.s <==
  22980. ANSWER: 15
  22981.         The birthdays of the Presidents of the United States.
  22982.  
  22983.  
  22984. ==> series/series.24.p <==
  22985. What is the next letter in the sequence: W, I, T, N, L, I, T?
  22986.  
  22987. ==> series/series.24.s <==
  22988. S.  First letters of words in question.
  22989.  
  22990. ==> series/series.25.p <==
  22991. 1 3 4 9 10 12 13 27 28 30 31 36 37 39 40 ?
  22992.  
  22993. ==> series/series.25.s <==
  22994. 1 3 4 9 10 12 13 27 28 30 31 36 37 39 40 ...
  22995. i in binary, treated as a base 3 number and converted to decimal.
  22996.  
  22997. ==> series/series.26.p <==
  22998. 1 3 2 6 7 5 4 12 13 15 14 10 11 9 8 24 25 27 26 ?
  22999.  
  23000. ==> series/series.26.s <==
  23001. 1 3 2 6 7 5 4 12 13 15 14 10 11 9 8 24 25 27 26 ...
  23002. Take i in binary, for each 1 bit (in i, not changed) flip the next bit.
  23003. This can also be phrased in reversing sequences of numbers.
  23004. More simply, just the integers in reflective-Gray-code order.
  23005.  
  23006. ==> series/series.27.p <==
  23007. 0 1 1 2 1 2 1 3 2 2 1 3 1 2 2 4 1 3 1 3 2 2 1 4 2 ?
  23008.  
  23009. ==> series/series.27.s <==
  23010. 0 1 1 2 1 2 1 3 2 2 1 3 1 2 2 4 1 3 1 3 2 2 1 4 2 ...
  23011. Number of factors in prime factorization of i.
  23012.  
  23013. ==> series/series.28.p <==
  23014. 0 2 3 4 5 5 7 6 6 7 11 7 13 9 8 8 17 8 19 9 10 13 23 9 10 ?
  23015.  
  23016. ==> series/series.28.s <==
  23017. 0 2 3 4 5 5 7 6 6 7 11 7 13 9 8 8 17 8 19 9 10 13 23 9 10 ...
  23018. Sum of factors in prime factorization of i.
  23019.  
  23020. ==> series/series.29.p <==
  23021. 1 1 2 1 2 2 3 1 2 2 3 2 3 3 4 1 2 2 3 2 3 3 4 2 3 3 4 3 4 ?
  23022.  
  23023. ==> series/series.29.s <==
  23024. 1 1 2 1 2 2 3 1 2 2 3 2 3 3 4 1 2 2 3 2 3 3 4 2 3 3 4 3 4 ...
  23025. The number of 1s in the binary expansion of n.
  23026.  
  23027. ==> series/series.30.p <==
  23028. I I T Y W I M W Y B M A D
  23029.  
  23030. ==> series/series.30.s <==
  23031. ? (first letters of "If I tell you what it means will you buy me a drink?")
  23032.  
  23033. ==> series/series.31.p <==
  23034. 6 2 5 5 4 5 6 3 7
  23035.  
  23036. ==> series/series.31.s <==
  23037. 6. The number of segments on a standard calculator display it takes
  23038. to represent the digits starting with 0.
  23039.   _       _   _       _   _   _   _   _
  23040.  ! !   !  _!  _! !_! !_  !_    ! !_! !_!
  23041.  !_!   ! !_   _!   !  _! !_!   ! !_!  _!
  23042.  
  23043. ==> series/series.32.p <==
  23044. 0 1 1 0 1 0 0 1 1 0 0 1 0 1 1 0 1
  23045.  
  23046. ==> series/series.32.s <==
  23047. 0 -> 1   01 -> 10   0110 -> 1001   01101001 ->  10010110
  23048. Recursively append the inverse.
  23049.  
  23050. This sequence is known as the Morse-Thue sequence.  It can be defined
  23051. non-recursively as the nth term is the mod 2 count of 1s in n written
  23052. in binary:
  23053.         0->0 1->1 10->1 11->0 100->1 101->0 110->0 111->1 etc.
  23054.  
  23055. Reference:
  23056.     Dekking, et. al., "Folds] I,II,III"
  23057.     The Mathematical Intelligencer, v4,#3,#4,#4.
  23058.  
  23059. ==> series/series.33.p <==
  23060. 2 12 360 75600
  23061.  
  23062. ==> series/series.33.s <==
  23063. 2         = 2^1
  23064. 12        = 2^2 * 3^1
  23065. 360       = 2^3 * 3^2 * 5^1
  23066. 75600     = 2^4 * 3^3 * 5^2 * 7^1
  23067. 174636000 = 2^5 * 3^4 * 5^3 * 7^2 * 11^1
  23068.  
  23069. ==> series/series.34.p <==
  23070. 3 5 4 4 3 5 5 4 3
  23071.  
  23072. ==> series/series.34.s <==
  23073. The number of letters in the English words for the counting numbers.
  23074.  
  23075. ==> series/series.35.p <==
  23076. 1 2 3 2 1 2 3 4 2 1 2 3 4 2 2 3
  23077.  
  23078. ==> series/series.35.s <==
  23079. The number of letters in the Roman numeral representation of the numbers.
  23080.  
  23081. ==> trivia/area.codes.p <==
  23082. When looking at a map of the distribution of telephone area codes
  23083. for North America, it appears that they are randomly distributed.
  23084. I am doubtful that this is the case, however.  Does anyone know
  23085. how the area codes were/are chosen?
  23086.  
  23087. ==> trivia/area.codes.s <==
  23088. Originally, back in the middle 1950's when direct dialing of long
  23089. distance calls first became possible, the idea was to assign area codes
  23090. with the 'shortest' dialing time required to the larger cities.
  23091.  
  23092. Touch tone dialing was very rare. Most dialed calls were with 'rotary'
  23093. dials.  Area codes like 212, 213, 312 and 313 took very little time to
  23094. dial (while waiting for the dial to return to normal) as opposed, for
  23095. example, to 809, 908, 709, etc ...
  23096.  
  23097. So the 'quickest to dial' area codes were assigned to the places which
  23098. would probably receive the most direct dialed calls, i.e. New York City
  23099. got 212, Chicago got 312, Los Angeles got 213, etc ... Washington, DC got
  23100. 202, which is a little longer to dial than 212, but much shorter than
  23101. others.
  23102.  
  23103. In order of size and estimated amount of telephone traffic, the numbers
  23104. got larger:  San Fransisco got 415, which is sort of in the middle, and
  23105. Miami got 305, etc.  At the other end of the spectrum came places like
  23106. Hawaii (it only got statehood as of about 1958) with 808,  Puerto Rico
  23107. with 809, Newfounland with 709, etc.
  23108.  
  23109. The original (and still in use until about 1993) plan is that area codes
  23110. have a certain construction to the numbers:
  23111.  
  23112. The first digit will be 2 through 9.
  23113. The second digit will always be 0 or 1.
  23114. The third digit will be 1 through 9.
  23115.  
  23116. Three digit numbers with two zeros will be special codes, ie. 700, 800 or
  23117. 900.  Three digit numbers with two ones are for special local codes,
  23118. i.e. 411 for local directory assistance, 611 for repairs, etc.
  23119.  
  23120. Three digit codes ending in '10', i.e. 410, 510, 610, 710, 810, 910 were
  23121. 'area codes' for the AT&T (and later on Western Union) TWX network. This
  23122. rule has been mostly abolished, however 610 is still Canadian TWX, and
  23123. 910 is still used by Western Union TWX. Gradually the '10' codes are
  23124. being converted to regular area codes.
  23125.  
  23126. We are running out of possible combinations of numbers using the above
  23127. rules, and it is estimated that beginning in 1993-94, area codes will
  23128. begin looking like regular telephone prefix codes, with numbers other than
  23129. 0 or 1 as the second digit.
  23130.  
  23131. I hope this gives you a basic  idea. There were other rules at one time
  23132. such as not having an area code with zero in the second digit in the same
  23133. state as a code with one in the second digit, etc .. but after the initial
  23134. assignment of numbers back almost forty years ago, some of those rules
  23135. were dropped when it became apparent they were not flexible enough.
  23136.  
  23137.  
  23138. Patrick Townson
  23139. TELECOM Digest Moderator
  23140.  
  23141. --
  23142. Patrick Townson
  23143.   patrick@chinet.chi.il.us / ptownson@eecs.nwu.edu / US Mail: 60690-1570
  23144.   FIDO: 115/743 / AT&T Mail: 529-6378 (]ptownson) /  MCI Mail: 222-4956
  23145.  
  23146.  
  23147.  
  23148.  
  23149. ==> trivia/eskimo.snow.p <==
  23150. How many words do the Eskimo have for snow?
  23151.  
  23152. ==> trivia/eskimo.snow.s <==
  23153. Couple of weeks ago, someone named D.K. Holm in the Boston Phoenix came up
  23154. with the list, drawn from the Inupiat Eskimo Dictionary by Webster and
  23155. Zibell, and from Thibert's English-Eskimo Eskimo-English Dictionary.
  23156.  
  23157. The words may remind you of generated passwords.
  23158.  
  23159. Eskimo      English                 Eskimo       English
  23160. ---------------------------------+----------------------------
  23161. apun        snow                 !  pukak        sugar snow
  23162. apingaut    first snowfall       !  pokaktok     salt-like snow
  23163. aput        spread-out snow      !  miulik       sleet
  23164. kanik       frost                !  massak       snow mixed with water
  23165. kanigruak   frost on a           !  auksalak     melting snow
  23166.             living surface       !  aniuk        snow for melting
  23167. ayak        snow on clothes      !               into water
  23168. kannik      snowflake            !  akillukkak   soft snow
  23169. nutagak     powder snow          !  milik        very soft snow
  23170. aniu        packed snow          !  mitailak     soft snow covering an
  23171. aniuvak     snowbank             !               opening in an ice floe
  23172. natigvik    snowdrift            !  sillik       hard, crusty snow
  23173. kimaugruk   snowdrift that       !  kiksrukak    glazed snow in a thaw
  23174.             blocks something     !  mauya        snow that can be
  23175. perksertok  drifting snow        !               broken through
  23176. akelrorak   newly drifting snow  !  katiksunik   light snow
  23177. mavsa       snowdrift overhead   !  katiksugnik  light snow deep enough
  23178.             and about to fall    !               for walking
  23179. kaiyuglak   rippled surface      !  apuuak       snow patch
  23180.             of snow              !  sisuuk       avalanche
  23181.  
  23182.                                     =*=
  23183.  
  23184. ==> trivia/federal.reserve.p <==
  23185. What is the pattern to this list:
  23186. Boston, MA
  23187. New York, NY
  23188. Philadelphia, PA
  23189. Cleveland, OH
  23190. Richmond, VA
  23191. Atlanta, GA
  23192. Chicago, IL
  23193. St. Louis, MO
  23194. Minneapolis, MN
  23195. Kansas City, MO
  23196. Dallas, TX
  23197. San Francisco, CA
  23198.  
  23199. ==> trivia/federal.reserve.s <==
  23200. Each of the cities is a location for a Federal Reserve.  The cities
  23201. are listed in alphabetical order based on the letter that represents each
  23202. city on a dollar bill.
  23203.  
  23204. ==> trivia/jokes.self-referential.p <==
  23205. What are some self-referential jokes?
  23206.  
  23207. ==> trivia/jokes.self-referential.s <==
  23208. Q: What is alive, green, lives all over the world, and has seventeen legs?
  23209. A: Grass.  I lied about the legs.
  23210.  
  23211. The two rules for success are:
  23212. 1. Never tell them everything you know.
  23213.  
  23214.  
  23215.